KEMBAR78
Solution | PDF | Equations | Numbers
0% found this document useful (0 votes)
12 views244 pages

Solution

Uploaded by

deepuxcbse
Copyright
© © All Rights Reserved
We take content rights seriously. If you suspect this is your content, claim it here.
Available Formats
Download as PDF, TXT or read online on Scribd
0% found this document useful (0 votes)
12 views244 pages

Solution

Uploaded by

deepuxcbse
Copyright
© © All Rights Reserved
We take content rights seriously. If you suspect this is your content, claim it here.
Available Formats
Download as PDF, TXT or read online on Scribd
You are on page 1/ 244

Contents

Chapter
Chapter Name Page Number
Number

1 Sets 1A.1 - 1A.11

2 Linear Inequalities 2A.1 - 2A.16

3 Relations and Functions 3A.1 - 3A.14

4 Trigonometry 4A.1 - 4A.21

5 Complex Numbers 5A.1 - 5A.17

6 Straight Lines 6A.1 - 6A.30

7 Conic Sections 7A.1 - 7A.22

8 Introduction to Three-Dimensional Geometry 8A.1 - 8A.10

9 Probability 9A.1 - 9A.16

10 Sequences and Series 10A.1 - 10A.24

11 Permutations and Combinations 11A.1 - 11A.14

12 Binomial Theorem 12A.1 - 12A.10

13 Limits and Derivatives 13A.1 - 13A.19

14 Statistics 14A.1 – 14A.19


SETS

HINTS AND SOLUTIONS

1. Ans (B) The set of natural numbers which divide 42, is


The given equation can be written as represented by {1,2,3,6,7,14,21,42}.
x 2 + 2x − x − 2 = 0 8. Ans (C)
or (x − 1)(x + 2) = 0 The required numbers are −2, −1,0,1,2. So, the
i.e., x = 1, −2 given set can be represented as {−2, −1,0,1,2}.
∴ The solution set of the given equation can be 9. Ans (D)
written in roster form as {1, −2}. 2x is an even number ∀x ∈ N.
2. Ans (C) 3x is an odd number ∀x ∈ N.
x 4 − 5x 2 + 6 = 0 2x + 3x is an odd natural number (as sum of even
⇒ x 4 − 3x 2 − 2x 2 + 6 = 0 and odd numbers is an odd number)
⇒ (x 2 2
− 3)(x − 2) = 0 ∴ The given set-in roster form is {5,13,35, … }.
⇒ x = √2, −√2, √3, −√3 Hence, A is false but R is true.
So, the given set-in roster form is 10. Ans (B)

{−√3, −√2, √2, √3}. 4x + 9 < 30

3. Ans (A) ⇒ 4x < 21

The two-digit natural numbers, sum of whose ⇒ x < 5.25

digits is 8, are 17,26,35,44,53,62,71,80. ⇒ x = 1,2,3,4,5

So, the given set in the roster form is ∴ Required set = {1,2,3,4,5}

{17,26,35,44,53,62,71,80}. 11. Ans (A)

4. Ans (C) If A ⊂ B and A ≠ B, then A is called a proper

The given set in the set-builder form is {x: x = 2n , subset of B and B is called a super set of A.

where x ∈ N and 12. Ans (A)

1 ≤ n ≤ 5} The set of negative real numbers is denoted by

5. Ans (B) (−∞, 0).

We see that each member in the given set is a 13. Ans (A)

multiple of 3 and. Hence, the set-builder form of Since, every element of A is in B, so A ⊂ B.

the given set is ∴ A is true. Also, R is true and a correct

{x: x = 3n, where n ∈ N }. explanation of A.

6. Ans (D) 14. Ans (A)

The set {5,25,125,625} in set-builder form is Every natural number is an integer and every

{x: x = 5n , where x ∈ N and 1 ≤ n ≤ 4}. integer is a rational number. Every rational

The set {2,4,6, … } in the set-builder form is {x: x = number is a real number.

2n, where x ∈ N} or {x: x is an even natural 15. Ans (B)

number}. 1 and {2} are the element of {1, {2}}

7. Ans (C) So, the subsets of the set {1, {2}} are ϕ, {1}, {{2}}

Factors of 42 are 1, 2, 3, 6, 7, 14, 21, 42. and {1, {2}}.


Hence, A is true.
MATHEMATICS Page | 1A. 1
SETS

We know, total number of proper subsets of a set ∴ (C − B) ∩ (D − B) = {10}


n
containing n elements is 2 − 1. 23. Ans (A)
Hence, R is true. But R is not the correct It is clear from the Venn diagram
explanation of A. A − B = A − (A ∩ B)
16. Ans (C) II. Also, it is clear from above diagram
A = {1,3,5} A = (A ∩ B) ∪ (A − B)
B = {x: x is an odd natural number less than 6} =
{1,3,5}
Since, every element of A is in B, so A ⊂ B.
Every element of B is in A, so B ⊂ A.
Then, A = B.
24. Ans (C)
17. Ans (B)
We have, A ⊂ B
If a set having n elements, then its no. of proper
∴ A ∩ B = A ⇒ n(A ∩ B) = n(A) …. (i)
subsets = 2n -1
And A ∪ B =B⇒ n(A ∪ B) = n(B)
∴ No. of proper subsets of a set having (n + 1)
25. Ans (A)
elements = 2n+1 − 1.
A ∪ B = {1,2,3,4,5,6}
18. Ans (B)
(A ∪ B) ∩ C = {3,4,6}
The number of subsets of A = 26 .
26. Ans (C)
19. Ans (C)
A ∪ D = {3,5,7,9,11,15,17}
n(A) = 4 (given)
B ∪ C = {7,9,11,13,15}
∴ Total no. of subsets of A = 24 = 16
(A ∪ D) ∩ (B ∪ C) = {7,9,11,15}
∴ No. of proper subsets = 16 − 1 = 15
27. Ans (B)
20. Ans (C)
A = {1,2,3,4,5,6}, B = {2,4,6,8}
Since, A ⊂ B and B ⊂ C
A − B = {1,3,5} and B − A = {8}
⇒ A ⊂ C. . . (i)
∴A−B≠ B−A
Given, C ⊂ A....(ii)
28. Ans (B)
From Eqs. (i) and (ii), we get
C − D = {2,4,6,8,12,14,16}
A=C
D − C = {5,15,20}
21. Ans (C)
∴ (C − D) ∪ (D − C)
If A ⊂ B and B ⊂ C, then these sets is represented
= {2,4,5,6,8,12,14,15,16,20}
in Venn diagram as
29. Ans (D)
Clearly, A ∪ B = B and B ∩ C = B
I. A ∩ B = {7,9,11}, B ∩ C = {11,13}
Hence, A ∪ B = B ∩ C
∴A∩B≠B∩C
II. A ∩ C = {11}, B ∩ D = {} or ϕ
∴A∩C≠B∩D
III. B ∪ C = {7,9,11,13,15}
A ∩ (B ∪ C) = {7,9,11}
22. Ans (C)
B ∪ D = {7,9,11,13,15,17}
C − B = {2,6,10,14}
A ∩ (B ∪ D) = {7,9,11}
D − B = {5,10,15}
∴ A ∩ (B ∪ C) = A ∩ (B ∪ D)

MATHEMATICS Page | 1A. 2


SETS

IV. A ∩ D = ϕ 36. Ans (C)


A ∪ B = {3,5,7,9,11,13} Given, n(A) = 3 and n(B) = 6
B ∪ C = {7,9,11,13,15} ∴ min. n(A ∪ B) = max{n(A), n(B)} = n(B) = 6
(A ∪ B) ∩ (B ∪ C) = {7,9,11,13} 37. Ans (C)
∴ A ∩ D ≠ (A ∪ B) ∩ (B ∪ C) A = {1, −1}, B = {1}.
30. Ans (B) ∴ A ∪ B = {1, −1} = A.
A = {1,2,3,4,5} 38. Ans (B)
B = {x: x 2 − 5x + 6 = 0} = {2,3} If An is the set of first n prime numbers, then
2
C = {x: x = 4 or x = 1} = {−2,1,2} ⋃10
n=2 A n is the set of first 10 prime numbers i.e.,

∴ A ∩ (B ∩ C) = {2} ≠ ϕ {2,3,5,7,11,13,17,19,23,29}.
31. Ans (C) 39. Ans (B)
X ∩ Y ∩ Z = L.C.M. of (x, 2x, 5x) The smallest set A = {3,5,9}, such that
= 10x, ∀x ∈ N A ∪ {1,2} = {1,2,3,5,9}
32. Ans (B) 40. Ans (C)
In Venn diagram, difference of two sets A and B I. Let, A = {1,2,3,4}
can be represented as B = {x: x is a natural number and
4 ≤ x ≤ 6}
= {4,5,6}
A ∩ B = {4}
Hence, A and B are not disjoint.
II. Let, P = {a, e, i, o, u}
Q = {c, d, e, f}
Hence, Statement I is false. P ∩ Q = {e}
Now, it is clear from the diagram Hence, P and Q are not disjoint.
A − B, A ∩ B and B − A are mutually disjoint sets. III. Let, X = {x: x is an even integer }
Hence, Statement II is true. = {… − 6, −4, −2,2,4,6, … }
33. Ans (B) and Y = {x: x is an odd integer }
Using the set-builder form, we can write the = {… − 5, −3, −1,1,3,5, … }
definition of difference as ∴X∩Y=ϕ
A − B = {x: x ∈ A and x ∉ B} Hence, X and Y are disjoint.
34. Ans (D) 41. Ans (B)
aN = {ax: x ∈ N},3N = {3x: x ∈ N} We know, A − (A − B) = A ∩ B (refer Venn
= {x: x is a positive multiple of 3} diagram)
7N = {7x: x ∈ N}
= {x: x is a positive multiple of 7}
3N ∩ 7N = {x: x is a positive multiple of 21}
= {21x: x ∈ N} = 21N
35. Ans (D)
B − C = {3,4,5} − {4,5,7,8} = {3}
∴ Statement I is false.
A − (B − C) = {1,2,3,4} − {3}
Now, B ′ = {1,2,3,4,5,9,10}
= {1,2,4}
MATHEMATICS Page | 1A. 3
SETS

A ∩ B ′ = {1,5} = A = ϕ ∩ B′ = ϕ
A′ ∩ B ′ ∴ Statement II is true. 48. Ans (C)
42. Ans (C) n[(A ∩ B)′ ∩ A)
Now, A − (A − B) = A − (A ∩ B c ) = n[(A′ ∪ B ′ ) ∩ A]
= A ∩ (A ∩ B c )c [by De Morgan's law]
=A∩ (Ac ∪ B) = n[(A′ ∩ A) ∪ (B ′ ∩ A)]
= (A ∩ Ac ) ∪ (A ∩ B) [Distributive law]
=A∩B = n(ϕ ∪ (A ∩ B ′ )) = n(A ∩ B ′ )
Alternatively, refer venn diagram. = n(A − B) = n(A) − n(A ∩ B)
43. Ans (A) = 8 − 2 = 6.
′ (A′
(A ∪ B) ∪ ∩ B) 49. Ans (B)
= (A′ ∩B ∪ ′) (A′ ∩ B) Clearly, A′ = {1,4,5,6}, B ′ = {1,2,6}.
[by De-Morgan's law] Hence, A′ ∩ B ′ = {1,6}.
= A′ ∩ (B ′ ∪ B) by distribution of ∪ over ∩) Also, A ∪ B = {2,3,4,5}, so that
′ ′
=A ∩U= A (A ∪ B)′ = {1,6} = A′ ∩ B ′
44. Ans (A) It can be shown that the above result is true in
′ )′ (A′ ′
Consider (A ∪ B ∩ ∪ B) general. If A and B are any two subsets of the
= (A′ ∩ B) ∩ (A ∩ B ′)
universal set U, then (A ∪ B)′ = A′ ∩ B ′
= (B − A) ∩ (A − B) = ϕ Similarly, (A ∩ B)′ = A′ ∪ B ′
45. Ans (D) Alternatively, refer De Morgan's law
A = {2,4,6,8, … } 50. Ans (C)
B = {1,3,5,7, … } B ∪ C = {2,3,4,6,7,8}
A∩B=ϕ (B ∪ C)′ = U − (B ∪ C) = {1,5,9,10}
(A ∩ B)′ = U C − A = {4,8}
46. Ans (D) (C − A)′ = {1,2,3,5,6,7,9,10}
A ∪ C = {1,2,3,4,5,6} 51. Ans (C)
(A ∪ C) = U − (A ∪ C) = {7,8,9} Since A ⊆ B,
A ∪ B = {1,2,3,4,6,8} ∴A∪B=B
(A ∪ B) = U − (A ∪ B) = {5,7,9} So, n(A ∪ B) = n(B) = 6
′ ′
∴ (A ∪ C) ≠ (A ∪ B) 52. Ans (D)
We know, (A′ )′ =A The shaded region represents
B − C = {2,8} (P ∩ Q) ∪ (P ∩ R)

(B − C) = U − (B − C) 53. Ans (C)
= {1,3,4,5,6,7,9} From the given we have in interval notation A =
′ ′
B − C = (U − B) − (U − C) (0,3) and B = [1,5]
= {1,3,5,7,9} − {1,2,7,8,9} Clearly A − B = (0,1)
= {3,5} = {x ∈ R: 0 < x < 1}
′ ′ ′
∴ (B − C) ≠ B − C and B − A = [3,5]
47. Ans (C) = {x ∈ R: 3 ≤ x ≤ 5}
A ∩ (A ∪ B) = A ∩ (A′ ∩ B ′ )

∴ AΔB = (A − B) ∪ (B − A)
′) ′
= (A ∩ A ∩ B = (0,1) ∪ [3,5]
MATHEMATICS Page | 1A. 4
SETS

= {x ∈ R: 0 < x < 1 or 3 ≤ x ≤ 5} Let A = {θ: sin θ = tan θ}


54. Ans (D) And B = {θ: cos θ = 1}
sin θ
It is clear from the figure that set A ∪ C is not Now, A = {θ: sin θ = }
cos θ
shaded and set B is shaded other than A ∪ C, i.e., = {θ: sin θ(cos θ − 1) = 0}
= {θ = 0, π, 2π, 3π … }
B − (A ∪ C)
For B: cos θ = 1 ⇒ θ = 0,2π, 4π … .
55. Ans (A)
This shows that A is not contained in B. i.e. A ⊄ B.
As given: S = the set of all triangles
But B ⊂ A
P = the set of all isosceles triangles
65. (A)
Q = the set of all equilateral triangles
X = {(1 + 7)n − 7n − 1} = {0,49,490, … . . }
R = the set of all right-angled triangles is divisible by 72
∴ P ∩ Q represents the set of isosceles triangles Y = {49(n − 1): n ∈ N}
= 49{0,1,2 … … . . }
and R − P represents the set of non-isosceles
66. (A)
right-angled triangles.
A⊂B
A−B= ϕ
56. (B) 67. (C)
Clearly 1 ∈ A (A ∪ B) − (A ∩ B) = (A − B) ∪ (B − A)
57. (C)
A⊂B
⇒ A∩B= A
58. (B)
A ∪ B = A ∪ C and A ∩ B = A ∩ C
⇒B=C
59. (B)
The number of subsets contains 2,3&5 is 68. (D)
3
2 =8 N6 ∩ N8 = 24N = N24
60. (D) 69. (B)
D = {x: x 2 = 0, x is an integer } 10
= {0} ≠ ϕ ⋃ An = {2,3,5,7,11,13, 17,19,23,29}
61. (B) n=2

p( A) = {ϕ, {1}, {{2,3}}, {1, {2,3}}} 70. (C)


n(p( A)) = 4 A: y 2 = x
62. (C) B: y = |x|
p(A) = {ϕ, {ϕ}, {{ϕ}}, A}
63. (B)
A = {x | |x| < 3, x ∈ z}
−3 < x < 3
∴ A = {−2, −1,0,1,2}
R = {(x, y)/y = |x|, x ≠ 1}
R = {(−2,2), (0,0), (1,1), (2,2)}
Therefore, the number of elements in the power 71. (A)
set of R, 24 = 16 n(A) = 5 and A ⊂ B
64. (B) n(AΔB) = n(A ∪ B) − n(A ∩ B)
n(A ∪ B) − n(A) ⩾ 1
MATHEMATICS Page | 1A. 5
SETS

[∵ if A ⊂ B then A ∩ B = A] ⇔ sin θ + cos θ = √2 sin θ


minimum is 1 ∴P=Q
72. (B) 81. (B)
10 An = A3 = {2,3,5} A − (B ∩ C)
n=3

73. (A) 82. (C)


(A − B) ∪ (B − A) ∪ A ∩ B = A ∪ B A − B ≠ A − BC

74. (C)
83. (C)
A = {1,3,5,15}
B = {2,3,5,7} A = {x ∈ R: |x| < 2}
C = {2,4,6,8} B = {x ∈ R: |x − 2| ⩾ 3}
A ∪ C = {1,2,3,4,5,6,8,15} |x| < 2
(A ∪ C) ∩ B = {2,3,5} ⇒ −2 < x < 2
∴ A = (−2,2)
75. (A) |x − 2| ⩾ 3
A ∪ B = {2,3,4,5,8,10,12} ⇒x−2⩾3
or x − 2 ⩽ −3
A ∪ C = {2,3,4,5,6,8,10,12,14}
x⩾5
(A ∪ B) ∩ (A ∪ C) = {2,3,4,5,8,10,12} or x ⩽ −1
76. (D) B = (−∞, −1] ∪ [5, ∞)
A ∪ B = ℝ − [2,5)
S − 1A − B = {7,10} A ∩ B = (−2, −1]
S − 2A − B = {x: x ∈ A, x ∉ B} B − A = (−∞, −2] ∪ [5, ∞)
77. (B) = ℝ − (−2,5)
A − B = [−1,2)
A−B = A − (A ∩ B)
= (0,3) − [1,3)
= (0,1)
78. (A)
X∪Y=X
84. (B)
Y⊂X
Maximum number of elements in A ∩ B
both correct and II Explains I
= minimum of {n(A), n(B)}
= 22
79. (C) 85. (B)
A = {1,2}, B = {3,4,5}, C = {2,3,4,5} S = {3,6,9 … . ,99}
n(S) = 33
(A ∪ B) = (A ∪ C)
P = {2,3,5,7,11,13,17,19}
But B ≠ C n(P) = 8
80. (C) ∴ n(S) + n(P) = 41

sin θ − cos θ = √2 cos θ 86. (B)


n(A′ ∪ B ′ ) = n[(A ∩ B)′ ]
⇔ sin θ = (√2 + 1) cos θ
= n(U) − n(A ∩ B)
⇔ (√2 − 1) sin θ = cos θ = 700 − 100
= 600
MATHEMATICS Page | 1A. 6
SETS

87. (B) A1 ∩ A2 ⊂ A3 ⊂ ⋯ … ⊂ A70


n(A ∩ B) ⩽ m {n(A), n(B)} ⇒ A1 ∩ A2∩ A3 ∩ … … ∩ A70 = A1
n(A ∩ B) ⩽ 32 70

88. (A) ⋂ Ai = A10


i=10
n(A ∪ B) ⩽ n(A) + n(B)
70
n(A ∪ B) ⩽ 40
n (⋂ Ai ) = n(A10 )
89. (C) i=10

n(A ∪ B) ⩾ maximum {n(A), n(B)} =10 − 3 = 7


90. (A) 96. (B)
Maximum {n(A), n(B)} ⩽ n(A ∪ B) ⩽ n(U) A1 ⊂ A2 ⊂ A3 ⊂ ⋯ … ⊂ A79
∴ 43 < n(A ∪ B) ⩽ 60 ⇒ A1 ∩ A2 ∩ A3 … … ∩ A79 = A
91. (D) and A1 ∪ A2 ∪ A3 ∪ … … ∪ A79 = A79
79
n(A ∩ B) ⩽ min{n(A), n(B)}
n (⋃ Ai) = A79 = 79 + 2 = 81
92. (D) i=1
2 2
A: x + y = 25 represent a circle with centre at 97. (D)
origin and radius 5. A = {0,1,2, … … … … … . … }
x2 y2 I = {… … − 3, −2, −1,0,1,2 … … … . . }
B: + = 1 represent a ellipse with centre at B = {0, −1, −2, −3, … … … … }
144 16

origin and foci at origin. E = {2,4,6,8 … … … . . }


P = {2,3,5,7,11,13 … … … }
AΔB = (A − B) ∪ (B − A)
= {1,2, … … } ∪ {−1, −2 … … }
= I − {0}
98. (C)
(A′ ∩ B ′ ∩ C) ∪ (B ∩ C) ∪ (A ∩ C)
(A′ ∩ B ′ ) ∪ B ∪ A) ∩ C
=X∩C=C
93. (A) 99. (C)

A∩B=ϕ n(A)=0, n(B)=20 = 1 ⇒ n(C) = 21 = 2


n(A ∩ B) = 0
100. (B)
A = {−1,1}
B = {−1,1, i, −i}
A−B=ϕ
B − A = {i, −i}
AΔB = (A − B) ∪ (B − A)
94. (C) = {i, −i}
A1 ⊂ A2 ⊂ A3 ⊂ ⋯ … ⊂ A100 and n(Ai ) = i + 2 101. (A)
n(A1 ) = 1 + 2 = 3, n(A2 ) = 2 + 2 = 4, A, B ⊂ S
n(A3 ) = 3 + 2 = 5 … … . n(A100 ) ⇒ A∪B⊂ S
= 100 + 3 = 103 102. (C)
100
If elements are not repeated, then number of
A = ⋂ Ai
i=3
elements in
95. (B) A1 ∪ A2 ∪ A3 … ,∪ A30 is 30 × 5.
MATHEMATICS Page | 1A. 7
SETS
30×5
But each element is used 10 times, so S = =
10

15
If elements in B1 , B2 , … , Bn are not repeated, then
total number of elements is 3n but each element is
3n 3n
repeated 9 times, so S = ⇒ 15 = ⇒ n = 45
9 9

103. (B)
2m = 112 + 2n → Verify options 108. (A)
104. (B) By substituting the values of n, we get;
(A ∩ B′)′) ∪ (B ∩ C) = (A′ ∪ B) ∪ (B ∩ C) X = {0, 49, 490, … }, Y = {0, 49, 98, … . }
[ By Demorgan’s Law] ∴ X ⊂ Y.
= A′ ∪ B 109. (C)
1
105. (D) The graph of y = and y = −x do not intersect.
x
Consider the following Venn diagram with 110. (A)
universal set U as the set of all the quadrilaterals in From the Venn diagram, it is clear that: A ∩
plane. (A ∪ B) = A

111. (B)
A′ ∪ [(A ∪ B) ∩ B′)] = A′ ∪ [(A ∩ B′) ∪ (B ∩ B′)]

Clearly, F2 ⊂ F1 , F3 ⊂ F1 , F4 ⊂ F1 = A′ ∪ [(A ∩ B ′ ) ∪ ∅]

∴ F1 = F1 ∪ F2 ∪ F3 ∪ F4 = A′ ∪ (A ∩ B ′ ) = (A′ ∪ A) ∩ (A′ ∪ B ′ )

106. (C) = A′ ∪ B ′ = (A ∩ B)′ [A ∩ B = ∅]



Sets S, T and C can be represented in Venn diagram =∅ =N

as given below: 112. (D)


Here, S =
{x | x is a positive multiple of 3 less than 100}
So, n(s) = 33.
Now,
P = {x | x is a prime number less than 20}.
Therefore, n(P) = 8.
From the diagram it is clear that
Hence, n(S) + n(P) = 33 + 8 = 41
S∪T∪C=S
113. (C)
107. (D)
X ∩ (X ∪ Y)′ = X ∩ (X ′ ∩ Y ′ )
Here, R be the set of points inside a rectangle of
= (X ∩ X ′ ) ∩ Y ′ = ∅ ∩ Y ′ = ∅
sides a and b. So,
114. (A)
R = {(x, y): 0 < x < a, and 0 < y < b}
We have, |2x + 3| < 7
⇒ − 7 < 2x + 3 < 7 ⇒ −7 − 3 < 2x < 7 − 3
⇒ − 10 < 2x < 4 ⇒ −5 < x < 2

MATHEMATICS Page | 1A. 8


SETS

⇒0<x+5<7 2. (A)
115. (D) The power set of a set containing n elements has 2n
Since x = −1 and x = 1 are roots elements.
∴ (x + 1) and (x − 1) are factors of equation. Clearly, 2n cannot be equal to 26
∴ (x + 1)(x − 1) = x 2 − 1 = 0 is required 3. (C)
equation. We have, A ⊃ B ⊃ C
∴ Set builder form of given set A is ∴ A ∪ B ∪ C = A and A ∩ B ∩ C = C
A = {x: x is a root of the equation x 2 = 1} ⇒ (A ∪ B ∪ C) − (A ∩ B ∩ C)
116. (A) = A−C
Given, A ⊂ B ⇒ A ∩ B = A 4. (C)
∴ n(A ∪ B) = n(A) + n(B) − n(A ∩ B) A ∩ B = {x: x a multiple of 3}
= n(A) + n(B) − n(A) = n(B) and {x: x is a multiple of 5}
117. (B) = {x: x is a multiple of 15}
We have, n(A) = 6 = {15, 30, 45, … … … . }
Total number of subsets of A = 26 5. (C)
∴ Number of subsets of A which contains atleast We have, A ⊂ B and B ⊂ C
6
two elements = 2 − 7 ∴ A ∪ B = B and B ∩ C = B
[∵ Subsets of A having none or 1 element are ⇒ A∪B= B∩C
= 64 − 7 = 57 6. (D)
118. Ans (D) We have,
2
x + 1 ≠ 0, ∀x ∈ R A ∩ (A ∩ B)c = A ∩ (Ac ∪ B c )
⇒ A ∩ (A ∩ B)c
119. Ans (C)
= (A ∩ Ac ) ∪ (A ∩ B c )
𝑚 𝑛
2 = 56 + 2
⇒ A ∩ (A ∩ B)c = ϕ ∪ (A ∩ B c )
120. Ans (B)
= A ∩ Bc
𝐴 = {−3, 3}, 𝐵 = {2,3,4}, 𝐶 = {2,3}
7. (C)
𝑁𝑜𝑤 𝑒𝑣𝑎𝑙𝑢𝑎𝑡𝑒 (𝐵 − 𝐶) ∪ 𝐴
It is given that
121. Ans (D)
A1 ⊂ A2 ⊂ A3 … ⊂ A99
• Use TSR min [n(A ∪ B)] = min {n(A), n(B)} 999

• max[n(A ∪ B)] = n (A) + n(B) ⋃ Ai = A99


i=1
• max[n(A ∩ B)] = min{n(A), n(B)} 99

⇒ n (⋃ Ai ) = n(A99 ) = 99 + 1 = 100
i=1

MOCK TEST SOLUTIONS: 8. (D)

1. (A) X ∩ Y = {1, 2, 4, 5, 8, 10, 20, 25, 40, 50, 100, 200}

It is given that A is a proper subset of B ∴ n(X ∩ Y) = 12

∴ A − B = ϕ ⇒ n(A − B) = 0 9. (A)

We have, n(A) = 5. So, minimum number of We have,

elements in B is 6 b N = {b x|x ∈ N} = Set of positive integral


Hence, the minimum possible value of n(A Δ B) is multiples of b

n(B) − n(A) = 6 − 5 = 1

MATHEMATICS Page | 1A. 9


SETS

c N = {c x|x ∈ N} = Set positive integral multiples 16. (C)


of c A ∩ B = {2, 4}
∴ bN ∩ cN = Set of positive integral multiples of bc {A ∩ B} ⊆ {1, 2, 4}, {3, 2, 4}, {6, 2, 4}, {1, 3, 2, 4},
⇒ bN ∩ cN = bc N {1, 6, 2, 4}, {6, 3, 2, 4}, {2, 4}, {1, 3, 6, 2, 4} ⊆ A ∪ B
[∵ b and c are prime] ⇒ n(C) = 8
Hence, d = bc 17. (B)
10. (A) A ∩ ϕ = ϕ is true.
We have, 18. (D)
2
7n + 3n + 3 3 Clearly, S ⊂ R
p= ⇒ p = 7n + 3 +
n n ∴ S ∪ R = R and S ∩ R = S
It is given that n ∈ N and p is prime. Therefore, n =
⇒ (S ∩ R) − (S ∩ R) = Set of rectangles which are
1
not squares
∴ n(A) = 1
19. (C)
11. (B) 4
We have, y = and x 2 + y 2 = 8
x
Required number of subsets is equal to the number
Solving these two equations, we have
of subsets containing 2 and any number of
16
elements from the remaining elements 1 and 4 x2 + = 8 ⇒ (x 2 − 4) = 0 ⇒ x = ±2
x2
So, required number of elements = 22 = 4 4
Substituting x = ±2 in y = , we get y = ±2
x
12. (C)
Thus, the two curves intersect at two points only
According to question,
(2, 2) and (−2, 2). Hence, A ∩ B contains just two
2m − 2n = 48
points
This is possible only if m = 6 and n = 4.
13. (B)
20. (D)
Given, A ∩ X = B ∩ X = ϕ
Given figure clearly represents
⇒ A and X, B and X are disjoint sets.
(A − B) ∪ (B − A)
Also, A ∪ X = B ∪ X ⇒ A = B
21. (B)
14. (A)
Clearly,
From Venn-Euler’s Diagram it is clear that
A2 ⊂ A3 ⊂ A4 ⊂ ⋯ ⊂ A10
10

∴ ⋃ An = A10 = {2,3,5,7,11,13,17,19,23,29}
n=2

22. (A)
The set of negative real numbers is denoted by
(A ∪ B) ∪′ (A′ ′
∩ B) = A (−∞, 0).
15. (C) 23. (C)
5 4 3 2
U = {x: x + 6x + 11x − 6x = 0}
sin θ − cos θ = √2 cos θ
= {0, 1, 2, 3}
⇔ sin θ = (√2 + 1) cos θ
A = {x: x 2 − 5x + 6 = 0} = {2, 3}
⇔ (√2 − 1) sin θ = cos θ
And B = {x: x 2 − 3x + 2 = 0} = {2, 1}
⇔ sin θ + cos θ = √2 sin θ
∴ (A ∩ B)′ = U − (A ∩ B)
∴P=Q
= {0, 1, 2, 3} − {2} = {0, 1, 3}
24. (A)
MATHEMATICS Page | 1A. 10
SETS

A, B ⊂ S A′ ∩ B ′ ∴ Statement II is true.
⇒ A∪B⊂ S 29. Ans (C)
Now, A − (A − B) = A − (A ∩ B c )
25. (C) = A ∩ (A ∩ B c )c
p(A) = {ϕ, {ϕ}, {{ϕ}}, A} = A ∩ (Ac ∪ B)
26. (B) = (A ∩ Ac ) ∪ (A ∩ B)
A = {x | |x| < 3, x ∈ z} =A∩B
−3 < x < 3 Alternatively, refer venn diagram.
∴ A = {−2, −1,0,1,2}
R = {(x, y)/y = |x|, x ≠ 1} 30. Ans (A)
R = {(−2,2), (0,0), (1,1), (2,2)}
{(A − B)  (B − C)  (C − A)}
Therefore, the number of elements in the power
=ABC=U
set of R, 24 = 16
27. Ans (C)
I. Let, A = {1,2,3,4}
B = {x: x is a natural number and
4 ≤ x ≤ 6}
= {4,5,6}
A ∩ B = {4}
Hence, A and B are not disjoint.
II. Let, P = {a, e, i, o, u}
Q = {c, d, e, f}
P ∩ Q = {e}
Hence, P and Q are not disjoint.
III. Let, X = {x: x is an even integer }
= {… − 6, −4, −2,2,4,6, … }
and Y = {x: x is an odd integer }
= {… − 5, −3, −1,1,3,5, … }
∴X∩Y=ϕ
Hence, X and Y are disjoint.
28. Ans (B)
We know, A − (A − B) = A ∩ B (refer Venn
diagram)

∴ Statement I is false.
Now, B ′ = {1,2,3,4,5,9,10}
A ∩ B ′ = {1,5} = A

MATHEMATICS Page | 1A. 11


LINEAR INEQUALITIES

HINTS AND SOLUTIONS

1. Ans (D) ⇒ 3x + 6 > 2 − x


We have, 3x − 3 ≤ 2x − 6 ⇒ 3x + x > 2 − 6
3x − 2x ≤ −6 + 3 ⇒ x ≤ −3 ⇒ 4x > −4
∴ Solution set is (−∞, −3 ]. Dividing both sides by 4,
−4
2. Ans (A) x> ⇒ x > −1… (ii)
4
We have, −3x + 17 < −13
⇒ Draw the graph of inequalities (i) and (ii) on
−3x < −13 − 17
the number line.
⇒ −3x < −30 ⇒ x > 10 Hence, solution set of the inequalities are real
⇒ x ∈ (10, ∞)
numbers, x lying between -1 and 7
3. Ans (B) i.e., −1 < x < 7
3(x−2) 5(2−x)
I. We have, ≤
5 3

3x − 6 10 − 5x
⇒ ≤
5 3
⇒ 9x − 18 ≤ 50 − 25x
⇒ 9x + 25x ≤ 50 + 18 ∴ Solution set is (−1,7).
68 5. Ans (A)
⇒ 34x ≤ 68 ⇒ x ≤ ⇒x≤2
34 x (5x−2) (7x−3)
We have, < −
∴ Solution set is (−∞, 2]. 4 3 5
x 5(5x−2)−3(7x−3)
1 3x 1 ⇒ <
II. We have, ( + 4) ≥ (x − 6) 4 15
2 5 3

1 3x 4 1
⇒ ( + ) ≥ (x − 6)
2 5 1 3
Taking LCM in LHS,
1 3x + 20 1 ∴ Solution set is (4, ∞).
( ) ≥ (x − 6)
2 5 3 ⇒ 15x < 4[(25x − 10) − (21x − 9)]
3x + 20 x − 6
⇒ ≥ ⇒ 15x < 4[(25x − 10 − 21x + 9]
10 3
⇒ 15x < 4[4x − 1]
⇒ 3(3x + 20) ≥ 10(x − 6)
⇒ 15x < 16x − 4
⇒ 9x + 60 ≥ 10x − 60
⇒ 15x − 16x < −4 ⇒ −x < −4
⇒ 9x − 10x ≥ −60 − 60
6. Ans (A)
⇒ −x ≥ −120
x x
⇒ x ≤ 120 I. We have, x + + < 11
2 3

∴ Solution set is (−∞, 120). x x x


⇒ + + < 11
1 2 3
4. Ans (C)
6x + 3x + 2x 11x
Given inequalities are 2(x − 1) < x + 5 and 3(x + ⇒ < 11 ⇒ < 11
6 6
2) > 2 − x ⇒x<6
Now, 2x − 2 < x + 5 ∴ Solution set is (−∞, 6).
x x
⇒ 2x − x < 5 + 2 ⇒ x < 7 … (i) II. Again, we have, > + 1
3 2
and 3(x + 2) > 2 − x
MATHEMATICS Page |2A. 1
LINEAR INEQUALITIES

∴ Solution set is (−∞, −6). Hence, there exists only one pair of even integers
x
Transferring the term to LHS, (10,12).
2
x x 11. Ans (B)
⇒ − >1
3 2 We have, Profit = Revenue - Cost
2x − 3x −x
⇒ >1⇒ >1 = (60x + 2000) − (20x + 4000)
6 6
= 40x − 2000
Multiplying both sides by 6 ,
−x To earn some profit, 40x − 2000 > 0
6× >1×6
6 ⇒ x > 50
−x > 6 ⇒ x < −6 Hence, the manufacturer must sell more than 50
7. Ans (A) items to realize some profit.
We have, 4x + 3 < 5x + 7 12. Ans (B)
⇒ 4x − 5x < 7 − 3 ⇒ −x < 4 We have, 5x − 3 < 7
⇒ x > −4 Adding 3 on both sides,
∴ Solution set is (−4, ∞) 5x − 3 + 3 < 7 + 3
8. Ans (B) ⇒ 5x < 10
We have, 4x + 3 < 6x + 7. Dividing both sides by 5,
or 4x − 6x < 7 − 3 5x 10
< ⇒x<2
or −2x < 4 5 5
or x > −2 I. When x is an integer, the solution of the given

Hence, the solution set is (−2, ∞). inequality is {… , −1,0,1}.

9. Ans (A) II. When x is a real number, the solution of given

We have, 3x + 8 > 2 inequality is (−∞, 2) i.e., all the numbers lying

3x + 8 − 8 > 2 − 8 between −∞ and 2 but ∞ and 2 are not included

⇒ 3x > −6 as x < 2.

Dividing by 3 on both sides, 13. Ans (C)

3x −6 I. Given, x ≥ −3
⇒ > ⇒ x > −2
3 3 Adding 5 on both sides,
(i) When x is an integer, the solution of the given x + 5 ≥ −3 + 5
inequality is {−1,0,1,2 … }. ⇒ x+5≥ 2
(ii) When x is a real number, the solution of the II. We have, −x ≤ −4
given inequality is (−2, ∞). i.e., all the numbers 2x ≥ 8
lying between -2 and ∞ but -2 and ∞ are not 1
III. Given, <0
x−2
included.
⇒ x−2< 0
10. Ans (A)
⇒x<2
Let x be the smaller of two consecutive even
14. Ans (A)
positive integers. Then, the other even integer is
Let x be the marks obtained by student in the
x + 2.
annual examination. Then,
Given, x > 8, x + 2 > 8 and x + x + 2 < 25
62 + 48 + x
⇒ x > 8 and 2x + 2 < 25 ≥ 60
3
⇒ x > 8 and 2x < 23 or 110 + x ≥ 180
23
⇒ x > 8 and x < ⇒ x = 10 or x ≥ 70
2

MATHEMATICS Page |2A. 2


LINEAR INEQUALITIES

Thus, the student must obtain a minimum of 70 −6 −6x 0


≥ >
marks to get an average of at least 60 marks. −6 −6 −6
⇒1≥x>0
15. Ans (A)
⇒0<x≤1
We have, 7x + 3 < 5x + 9
∴ Solution set is (0,1].
⇒ 2x < 6 ⇒ x < 3
18. Ans (B)
⇒ x ∈ (−∞, 3)
We have, −8 < 5x − 3 < 7
⇒ − 5 ≤ 5x < 10
16. Ans (D) ⇒ − 1 ≤ x < 2 ⇒ x ∈ [−1,2)
I. The given inequality 19. Ans (C)
2 ≤ 3x − 4 ≤ 5 We have,
⇒ 2 + 4 ≤ 3x ≤ 5 + 4 MA
IQ = × 100
CA
⇒ 6 ≤ 3x ≤ 9
MA
Dividing by 3 in each term, ⇒ IQ = × 100(∵ CA = 12yr)
12
6 3x 9 25
≤ ≤ = MA
3 3 3 3
⇒2≤x≤3 Given, 80 ≤ 1Q ≤ 140
∴ Solution set is [2,3]. 25
⇒ 80 ≤ MA ≤ 140
II. The given inequality 3
3x ⇒ 240 ≤ 25MA ≤ 420
−12 < 4 − ≤2
−5 240 420
⇒ ≤ MA ≤
3x 25 25
⇒ −12 < 4 + ≤2
5 ⇒ 9.6 ≤ MA ≤ 16.8
Adding ( -4) to each term, 20. Ans (C)
3x Given, 3 < 3t − 18 ≤ 18
−12 − 4 < 4 + − 4 ≤ 2 − 4
5
Adding 18 to each term,
3x
⇒ −16 < ≤ −2 3 + 18 < 3t − 18 + 18 ≤ 18 + 18
5
5
Multiplying by to each term, ⇒ 21 ≤ 3t ≤ 36
3
Dividing by 3 to each term,
5 3x 5 5
−16 × < × ≤ −2 × 21 3t 36
3 5 3 3 ≤ ≤
80 10 3 3 3
⇒− <x≤− ⇒ 7 ≤ t ≤ 12
3 3
∴ Solution set is Adding 1 to each term,
80 10 7 + 1 ≤ t + 1 ≤ 12 + 1
(− ,− ]
3 3 ⇒ 8 ≤ t + 1 ≤ 13
17. Ans (B)
21. Ans (B)
The given inequality
Given inequality is
6 ≤ −3(2x − 4) < 12
−11 ≤ 4x − 3 ≤ 13
6 ≤ −6x + 12 < 12
⇒ 3 − 11 ≤ 4x ≤ 13 + 3
Adding ( -12) to each term,
⇒ −8 ≤ 4x ≤ 16 ⇒ −2 ≤ x ≤ 4
6 − 12 ≤ −6x + 12 − 12 < 12 − 12
∴ x ∈ [−2,4]
⇒ −6 ≤ −6x < 0
22. Ans (B)
Dividing by (−6) to each term,
MATHEMATICS Page |2A. 3
LINEAR INEQUALITIES

It is given that 30 < C < 35. 27. Ans (C)


5 I. If x > 0 and y > 0, then xy > 0 and if xy < 0,
Putting C = (F − 32),
9
5 then one of them must less than zero.
we get 30 < (F − 32) < 35
9
II. If xy > 0, either both are positive or both are
9 9
or (30) < (F − 32) < × (35) negative. i.e., x > 0, y > 0 or x < 0, y < 0
5 5

or 54 < (F − 32) < 63 III. |x| < 5


or 86 < F < 95. ⇒ −5 < x < 5
Thus, the required range of temperature is (∵ |x| < a ⇒ −a < x < a)
∘ ∘
between 86 F and 95 F. ⇒ x ∈ (−5,5)
23. Ans (A) 28. Ans (B)
5−3x
We have, −5 ≤ ≤8 We have 1 ≤ |x − 2| ≤ 3.
2

or −10 ≤ 5 − 3x ≤ 16 ⇒ |x − 2| ≥ 1 and |x − 2| ≤ 3

or −15 ≤ −3x ≤ 11 ⇒ (x − 2 ≤ −1 or x − 2 ≥ 1)
11 and (−3 ≤ x − 2 ≤ 3)
or 5 ≥ x ≥ −
3
−11
⇒ (x ≤ 1 or x ≥ 3) and
or ≤x≤5
3 (−1 ≤ x ≤ 5)
−11
∴ x∈[ , 5] ⇒ x ∈ (−∞, 1] ∪ [3, ∞)
3
and x ∈ [−1,5]
24. Ans (C)
Combining the solutions of two
The given inequality
inequalities, we have
7x
−3 ≤ 4 − ≤ 18 x ∈ [−1,1] ∪ [3,5]
2
Adding ( -4 ) to each term, 29. Ans (C)
7x |3x + 2| < 1 ⇔ −1 < 3x + 2 < 1
−3 − 4 ≤ 4 − − 4 ≤ 18 − 4
2 ⇔ −3 < 3x < −1
−7x 1
⇒ −7 ≤ ≤ 14 ⇔ −1 <x <− .
2 3
−2
Multiplying by ( ) to each term, 30. Ans (D)
7

2 7x 2 |3 − x| = 3 − x is true only
−7 × (− ) ≥ − × (− )
7 2 7 when 3 − x ≥ 0
2 ⇒ x ≤ 3.
≥ 14 × (− )
7
31. Ans (B)
⇒ 2 ≥ x ≥ −4 ⇒ −4 ≤ x ≤ 2
(x − 1)2 is always positive except when x = 1
∴ Solution set is [−4,2].
(and then it is 0 )
25. Ans (D)
∴ Solution is when x + 4 < 0
|3x − 5| ≤ 2
and x ≠ 1
⇒ −2 ≤ 3x − 5 ≤ 2
i.e. x < −4, x ≠ 1 ∴ x ∈ (−∞, −4).
⇒ 3 ≤ 3x ≤ 7
32. Ans (C)
7
⇒1≤x≤ We have, |x + 3| ≥ 10
3
26. Ans (C) ⇒ x + 3 ≤ −10 or x + 3 ≥ 10

−9 < x ≤ 6 (∵ |x| ≥ a ⇒ x < −a or x > a)

⇒ |x| ∈ [0,9) ⇒ x + 3 − 3 ≤ −10 − 3 or

MATHEMATICS Page |2A. 4


LINEAR INEQUALITIES

x + 3 − 3 ≥ 10 − 3 39. Ans (B)


x−2
⇒ x ≤ −13 or x ≥ 7 We have >2
x+5
⇒ x ∈ (−∞, −13] ∪ [7, ∞) x−2
⇒ −2>0
33. Ans (C) x+5
1 −(x+12) x+12
Let us first solve |3x + 1| < ⇒ >0⇒ <0
3 x+5 x+5

1 1 i.e. x ∈ (−12, −5)


⇔ − < 3x + 1 <
3 3 40. Ans (A)
4 2
⇔ − < 3x < − We have, 2 ≤ |x − 3| < 4
3 3
4 2 Case I If x < 3, then
⇔ − <x<− .
9 9
2 ≤ |x − 3| < 4
Also, 0 < |3x + 1| is satisfied by each x except
⇒ 2 ≤ −(x − 3) < 4
when 3x + 1 = 0.
⇒ 2 ≤ −x + 3 < 4
1
i.e. x = − .
3 Subtracting 3 from both sides,
4 2 1
∴ Solution is (− , − ) − {− }. −1 ≤ −x < 1
9 9 3

34. Ans (B) Multiplying (−1) on both sides,

We have, |x| < 3 −1 < x ≤ 1

−3 < x < 3 ⇒ x ∈ (−1,1]

(∵ |x| < a ⇒ −a < x < a) Case II If x > 3, then

35. Ans (B) 2 ≤ |x − 3| < 4

2x − 1 = |x + 7| ⇒2≤x−3<4

x + 7, if x ≥ −7 Adding 3 on both sides,


={
−(x + 7), if x < −7 ⇒5≤x<7
∴ If x ≥ −7,2x − 1 = x + 7 ⇒ x = 8 Hence, the solution set of given inequality is x ∈
If x < −7,2x − 1 = −(x + 7) (−1,1] ∪ [5,7)
⇒ 3x = −6 41. Ans (C)
⇒ x = −2, Not possible. x−7
We have, >2
x+3
36. Ans (B) x−7 x+13
|x−2|
⇒ −2>0⇒ <0
x+3 x+3
Since ≥ 0 and |x − 2| ≥ 0,
x−2
⇒ x ∈ (−13, −3)
⇒ x − 2 > 0.
42. Ans (D)
37. Ans (B)
|3x − 2| ≥ 1
(i) |x − 1| ≤ 2 ⇒ −2 ≤ (x − 1) ≤ 2
⇒ 3x − 2 ≤ −1 or 3x − 2 ≥ 1
⇒ −1 ≤ x ≤ 3i.e., [−1,3]
⇒ 3x ≤ 1 or 3x ≥ 3
(ii) |3x − 7| > 2 1
⇒ x ≤ or x ≥ 1
⇒ 3x − 7 < −2 or 3x − 7 > 2 3

5 Hence, required solution is


⇒ x < or x > 3
3 1
(−∞, ] ∪ [1, ∞).
38. Ans (A) 3

x+4 x+4 43. Ans (D)


We have, < 2⇒ −2<0
x−3 x−3
|x 2 − 10| ≤ 6
−x+10 x−10
⇒ < 0⇒ >0
x−3 x−3 ⇔ − 6 ≤ x 2 − 10 ≤ 6
⇒ x ∈ (−∞, 3) ∪ (10, ∞) ⇔ 4 < x 2 ≤ 16

MATHEMATICS Page |2A. 5


LINEAR INEQUALITIES

⇔ 2 ≤ |x| ≤ 4 From (1) and (2), common values of x are (-1,6].


∴ either 2 ≤ x ≤ 4 or −4 ≤ x ≤ −2 49. Ans(B)
∴ x ∈ [−4, −2] ∪ [2,4] x 2 − 3x + 2 > 0 ⇒(x-1)(x-2)>0
44. Ans (A) ⇒ xϵ (−∞, 1) ∪ (2, ∞)
We have. and x 2 − 3x − 4 ≤ 0 ⇒(x+1)(x-4)≤0
|x − 2| ≤ 1 ⇒ xϵ[−1,4]
⇒ −1 ≤ x − 2 ≤ 1 By clubbing both cases we get (refer the fig)
⇒ −1 + 2 ≤ x − 2 + 2 ≤ 1 + 2
⇒1≤x≤3
∴ x ∈ [1,3]
45. Ans (B)
xϵ [-1,1)∪(2,4]
Given in-equations are
50. Ans(B)
−17 ≤ 3x + 10 ≤ −2
x 2 + 2ax + (10 − 3a) > 0
∴ −27 ≤ 3x ≤ −12
WKT, AX 2 + BX + C > 0 for all X ϵ R if A>0 & D<0
∴ −9 ≤ x ≤ −4 … (i)
Here A>0 , D<0
−22 ≤ 5x + 13 ≤ 3
i.e., B 2 − 4AC<0
∴ −35 ≤ 5x ≤ −10
⇒ (2a)2 − 4(1)(10 − 3a)<0
∴ −7 ≤ x ≤ −2 … (ii)
⇒4a2 -40+12a<0
−19 ≤ 2x − 9 ≤ −3
(a+5) (a-2) <0
∴ −10 ≤ 2x ≤ 6
i.e., critical points for wavy curve are a=-5 & a=2
∴ −5 ≤ x ≤ 3 …. (iii)
The common range of values that satisfies all the
in-equations is [−5, −4]
46. Ans (C)
x−7 x−7 So aϵ (-5,2) ⇒-5<a<2
>2⇒ −2 >0
x+3 x+3
51. Ans(C)
−x − 13 x + 13
⇒ >0⇒ <0 4
x+3 x+3 Given, − 5 < 0
x
⇒ −13 < x < −3 4−5x
= <0
x
47. Ans (C)
5x−4
⇒ > 0 ⇒ critical points for wavy curve are
|x − 2| < 3 x

⇒ −3 < x − 2 < 3 x=4/5 & x =0

⇒ −1 < x < 5
48. Ans (B)
Given −3 < 2x − 1 < 19
⇒ −2 < 2x < 20
4
So, xϵ(−∞, 0) ∪ ( , ∞)
⇒ −1 < x < 10 … (1) 5

2x+3 52. Ans(C)


Also given −1 ≤ ≤3
5
4x + 3 < 6x + 7
⇒ −5 ≤ 2x + 3 ≤ 15
-4<2x
⇒ −8 ≤ 2x ≤ 12
-2<x
⇒ −4 ≤ x ≤ 6. … (2)
Since x>-2 & xϵ Z
MATHEMATICS Page |2A. 6
LINEAR INEQUALITIES

So x= {-1,0,1,2,3……..} x2 + 7 x2 + 7
⇒ + 1 ⩽ 0( or ) −1⩾0
53. Ans(A) 8x 8x
x 2 + 8x + 7 x 2 − 8x + 7
Refer the fig, (x + 5)(x − 2) ≤ 0 and x ≠ 0 ⇒ ⩽ 0( or ) ⩾0
8x 8x
⇒ critical points for wavy curve are
⇒ x(x + 1)(x + 7) ⩽ 0 → (1)( or )
x=-5 & x=2
x(x − 1)(x − 7) > 0, x ≠ 0 → (2)
Draw all the critical points of (1) &(2) in number
line to apply wavy curve method

So, x ∈ [−5,2)
54. Ans(B)
2x − 1
−2 < <2
x−1
2x − 1 2x − 1
+ 2 > 0 and −2 <0
x−1 x−1
4x − 3 1
> 0 and <0
x−1 x−1
(4x − 3)(x − 1) > 0 and x − 1 < 0
So Set is (−∞, −7] ∪ [−1,0) ∪ (0,1] ∪ [7, ∞)
3
x ∈ (−∞, ) ∪ (1, ∞)&x < 1 58. Ans(B)
4
1 1
3 x+ > 2 ( or ) x + < −2
⇒ x ∈ (−∞, ) x x
4
⇒ x ≠ 1,0, −1
55. Ans(B)
∴ x ∈ ℝ − {−1,0,1}
2(3x − 4) ⩾ x + 1 − 4
59. Ans(B)
6x − 8 ⩾ x − 3
−9 ⩽ x + 2 ⩽ 9
5x ≥ 5 ⇒ x ⩾ 1
−11 ⩽ x ⩽ 7
56. Ans(A)
|x − 4| x ∈ [−11,7]
< 0, x ≠ 4
x−4 60. Ans(A)
(i)x − 4 < 0 7
x ∈ (−∞, )
−(x − 4) 2
<0 61. Ans(A)
(x − 4)
−1 < 0( true ) (i)−5 < x − 4 < 5 ⇒ −1 < x < 9
∴x<4 (ii)2x + 5 < −7 or 2x + 5 > 7
(ii)x − 4 > 0 ⇒ x < −6 or x > 1 − (2)
x−4 from (i) & (ii) x ∈ (1,9)
( )<0
x−4 62. Ans(C)
1 < 0( not true )
(x − 3)(x + 4) > 0
∴ only x < 4
⇒ x ∈ (−∞, −4) ∪ (3, ∞)
∴ x ∈ (−∞, 4)
63. Ans(A)
57. Ans(B)
5x − 6 ⩽ −8 or 5x − 6 ⩾ 8
x2 + 7
| |⩾1 5x ⩽ −2 or 5x ⩾ 14
8x −2 14
x⩽ or x ⩾
x2 + 7 x2 + 7 5 5
⇒ ⩽ −1( or ) ⩾1 −2 14
8x 8x (−∞, ] ∪ [ , ∞)
5 5
MATHEMATICS Page |2A. 7
LINEAR INEQUALITIES

64. Ans(B) (x − 1)(1 + x)(x + 2)2 > 0, x ≠ 1, −2, −1


2x − 1 (x − 1)(1 + x) > 0
−2 < <2
x−1 x ∈ (−∞, −1) ∪ (1, ∞), x ≠ 1, −2, −1
2x−1 2x−1
+ 2 > 0 and −2<0 x ∈ (−∞, −2) ∪ (−2, −1) ∪ (1, ∞)
x−1 x−1
4x − 3 1 71. Ans(B)
> 0& <0
x−1 x−1
x − 1 ⩾ −(x − 3)
(4x − 3)(x − 1) > 0 and x − 1 < 0
x − 1 ⩾ −x + 3
x ∈ (−∞, 3/4) ∪ (1, ∞) and x < 1
2x ⩾ 4
∴ x ∈ (−∞, 3/4)
x⩾2
65. Ans(A)
72. Ans(B)
Conceptual
6 + x − x2 > 0
66. Ans(B)
x2 − x − 6 < 0
Conceptual
(x + 2)(x − 3) < 0
67. Ans(A)
−2 < x < 3
5x + 8
−2⩽0 73. Ans(D)
4−x
5x + 8 − 8 + 2x [x]2 − 5[x] + 6 = 0
⩽0
4−x [x]2 − 2[x] − 3[x] + 6 = 0
7x
⩽0 ([x] − 2)([x] − 3) = 0
4−x
[x] = 2 or [x] = 3
x(4 − x) ⩽ 0, 4 − x ≠ 0
x ∈ [2,4)
x(x − 4) ⩾ 0 x ≠ 4
74. Ans (D)
x ⩽ 0 or x ⩾ 4 x ≠ 4
x 1 x 1
(−∞, 0] ∪ (4, ∞) ≤ ⇒ − ≤0
x+2 x x+2 x
68. Ans(D) x2 − x − 2
⇒ ≤0
− 27 < 4x − 3 < 27 x(x + 2)
−27 + 3 < 4x < 27 + 3 (x − 2)(x + 1)
⇒ ≤0
−24 < 4x < 30 x(x + 2)
15 Now, critical points to apply wavy curve method, x
−6 < x <
2 = – 2, –1, 0 and 2
15
(−6, )
2
69. Ans(C)
x 1
− ⩽0
x−3 x
 Solution set is (–2, –1]  (0, 2]
x2 − x + 3
⩽ 0, x ≠ 0, x ≠ 3
x(x − 3) 75. Ans (B)
x2 − x + 3 ⩾ 0 x ∈ R 1−2x−3x2 −(3x2 +2x−1)
> 0⇒ >0
3x−x2 −5 −(x2 −3x+5)
x(x − 3) < 0, x ≠ 0,3
3x2 +2x−1
⇒ > 0 ….. (1)
0<x<3 x2 −3x+5

(0,3) Now, for x2 – 3x + 5, we have,

70. Ans(B) B2 – 4AC = 9 – 20 < 0, A = 1 > 0

(x − 1)(x + 2)2 Thus, x2 – 3x + 5 > 0 for all x.


>0
(1 + x)  (1)  3x2 + 2x – 1 > 0
MATHEMATICS Page |2A. 8
LINEAR INEQUALITIES

 (3x – 1) (x + 1) > 0 ⇒ 4x|2 − 12|x| + 9 ≤ 0


1 3
⇒x> or x < −1 ⇒ (2|x| − 3)2 = 0 ⇒ |x| =
3 2
Solution set is (−∞, 1) ∪ ( , ∞)
1 81. Ans (A)
3
Let x be the shortest side. Then x + 3 and 2x are
76. Ans (A)
1 3 1 3
the remaining lengths
< ⇒ − <0
x+2 x−3 x+2 x−3
Given x + (x + 3) + 2 x  91 and
x − 3 − 3x − 6
⇒ <0 2x  (x + 3) + 5
(x + 2)(x − 3)
−(2x + 9)  4x  88 and x  8  8  x  22
⇒ <0
(x + 2)(x − 3)  Shortest side = 8 but not more than 22cm.
2x + 9 82. Ans (A)
⇒ >0
(x + 2)(x − 3)
Let 2(2x + 3) – 10 < 6 (x – 2)→(1)
 4x + 6 – 10 – 6x + 12 < 0
 –2x + 8 < 0

9
 –2x < – 8  x > 4 i.e., x  (4, )
 Solution set is (− , −2) ∪ (3, ∞) 2x−3 2+4x
2
+6≥ →(2)
4 3
77. Ans (B)
2x−3 2+4x
+6≥
We have, x2 – 5x + 4 < 0 4 3

 (x – 4) (x – 1) < 0  6x + 63  8 – 63

1<x<4  6x – 16x  8 – 63⇒ −10x ≥ −55

Since, x  Z, x = 2, 3 55 55
⇒x≤ i. e. , x ∈ (−∞, ]
10 10
Thus, there are 2 integral solutions.
Solution set is given by
78. Ans (A)
55 55
Let x and x + 2 be two even integers (−∞, ] ∩ (4, ∞) = (4, ]
10 10
such that x < 12, x < 10 and sum 2x + 2 > 14 83. Ans (C)
 x < 10 and x > 6 2
Case (i): when x > 0, < 3 ⇒ 2 < 3x
x
 6 < x < 12x=8 2 2
⇒ < x or x >
Thus, the pair of numbers is (8, 10). 3 3
2
79. Ans (C) Case (ii): when x < 0, < 3 ⇒ 2 > 3x
x
x+1 1 4x + 4 − x 2 − 2 2
⇒ >x⇒x .
2
> ⇒ >0 3 3
x2 + 2 4 4(x 2 + 2)
Which is satisfied when x < 0
x 2 − 4x − 2
⇒ <0 2
4(x 2 + 2) ∴ x ∈ ( , ∞) ∪ (−∞, 0)
3
 x2 – 4x – 2 < 0(∵ 4(x 2 + 2) > 0) 84. Ans (C)
4 ± √16 + 8 10x − 1
⇒x= = 2 ± √6 4x − 3 ≥
2 3
⇒ 2 − √6 < x < 2 + √6 ⇒ 12x − 9 ≥ 10x − 1
 x = 0, 1, 2, 3, 4. Thus 5 solutions.  12x – 10x  + 9 – 1  2x  8
80. Ans (A)  x  4  x  [4, )
12x 12|x| 85. Ans (A)
| |≥ 1⇒ ≥ 1(∵ 4x 2 + 9 > 0)
4x2 +9 4x2 +9
We have, –3x + 17 < – 13
⇒ 4x 2 − 12|x| + 9 ≤ 0
MATHEMATICS Page |2A. 9
LINEAR INEQUALITIES

 –3x < – 13 – 17 ∴ Solution set is (−∞, 3).


 –3x < –30 89. Ans (C)
 3x > 30 4x + 3 < 6x + 7 ⇒ −2x < 4

 x > 10  x  (10, ) ⇒ −x < 2 ⇒ x > −2 ⇒ x ∈ (−2, ∞)

86. Ans (A) 90. Ans (D)

We have, 3x − 7 > 5x − 1 5 − 2x x 5 − 2x x − 30
≤ −5⇒ ≤
3 6 3 6
Transferring the term 5x to L.H.S. and the term −7
x − 30
to R.H.S. ⇒ 5 − 2x ≤ ⇒ 10 − 4x ≤ x − 30
2
Dividing both sides by 2 ⇒ 40 ≤ 5x
3x − 5x > −1 + 7 ⇒ −2x > 6 ⇒ 8 ≤ x ⇒ x ∈ [8, ∞)
2x 6 91. Ans (C)
⇒ < − ⇒ x < −3
2 2
If x cm is the breadth, then: 2(3x + x) ≥ 160
With the help of number line, we can easily look
⇒ x ≥ 20
for the numbers less than −3.
92. Ans (B)
Let Ravi got x marks in third unit test.
∴ Average marks obtained by Ravi
∴ Solution set is (−∞, −3), i.e., all the numbers Sum of marks in all tests
=
lying between −∞ and −3 but −∞ and −3 are not Number of tests
70 + 75 + x 145 + x
included as x < −3. = =
3 3
87. Ans (C)
Now, it is given that he wants to obtain an average
We have, 37 − (3x + 5) ≥ 9x − 8(x − 3)
of at least 60 marks.
(37 − 3x − 5) ≥ 9x − 8x + 24
At least 60 marks means that the marks should be
⇒ 32 − 3x ≥ x + 24
greater than or equal to 60.
Transferring the term 24 to L.H.S. and the term 145 + x
≥ 60 ⇒ 145 + x ≥ 60 × 3
(−3x) to R.H.S. 3
32 − 24 ≥ x + 3x ⇒ 8 ≥ 4x ⇒ 4x ≤ 8 ⇒ 145 + x ≥ 180
Dividing both sides by 4, Now, transferring the term 145 to R. H. S..
4x 8 x ≥ 180 − 145 ⇒ x ≥ 35
⇒ ≤ ⇒x≤2
4 4 i.e., Ravi should get greater than or equal to 35
marks in third unit test to get an average of at
least 60 marks.
∴ Solution set is (−∞, 2]. ∴ Minimum marks Ravi should get = 35.
88. Ans (A) 93. Ans (B)
We have, 3x − 2 < 2x + 1 We have, a < b and c < 0 Dividing both sides of
Transferring the term 2x to L.H.S. and the term (- a < b by c. Since, c is a negative number, sign at
2) to R.H.S. a b
inequality will get reversed. Hence, >
c c

94. Ans (C)


3(2 − x) ≥ 2(1 − x) ⇒ 6 − 3x ≥ 2 − 2x
All the numbers on the left side of 3 will be less −x ≥ −4 ⇒ x ≤ 4 ⇒ x ∈ (−∞, 4]
than it. 95. Ans (A)

MATHEMATICS Page |2A. 10


LINEAR INEQUALITIES
2x−1 15x−10−8+4x 2x−1 19x−18
≥ ⇒ ≥ 100. Ans (C)
3 20 3 20
−8 ≤ 5x − 3 ⇒ −5 ≤ 5x ⇒ −1 ≤ x
⇒ 40x − 20 ≥ 57x − 54 ⇒ −17x ≥ −34
5x − 3 < 7 ⇒ 5x < 10 ⇒ x < 2
⇒x≤2
Hence, common sol is −1 ≤ x < 2
⇒ x ∈ (−∞, 2]
⇒ x ∈ [−1,2) ⇒ a = 1, b = 2 and a + b = 3
96. Ans (C)
101. Ans (B)
We have 3x − 7 > 2(x − 6) ⇒ 3x − 7 > 2x − 12
C F−32
Transferring the term 2x to L.H.S. and the term Given: = and 10 < C < 20.
5 9

(−7) to R.H.S., 5 F − (32)5


⇒C=
3x − 2x > −12 + 7 ⇒ x > −5 9
5 F−160
and 6 − x > 11 − 2x Since, 10 < C < 20 ⇒ 10 < < 20
9

Transferring the term (−2x) to L.H.S. and the term ⇒ 90 < 5 F − 160 < 180
6 to ⇒ 90 + 160 < 5 F < 180 + 160
R.H.S.; −x + 2x > 11 − 6 ⇒ x > 5 250 340
⇒ 250 < 5 F < 340 ⇒ <F<
5 5
Draw the graph of inequations (i) and (ii) on the
⇒ 50 < F < 68
number line,
102. Ans (C)
The given system of inequalities is x + 2 > 11…….
(i)
Hence, solution set of the equations are real 2x ≤ 20…...(ii)
numbers, x lying on greater than 5 excluding 5.i.e., Now, x + 2 > 11 ⇒ x > 11 − 2 ⇒ x > 9 ⇒ x ∈
x>5 (9, ∞)
∴ Solution set is (5, ∞) ∴ Solution of inequality (i) is x > 9……(iii)
97. Ans (D) and 2x ≤ 20 ⇒ x ≤ 10 ⇒ x: ∈ (−∞, 10]
Given inequality is 5x + 1 > −24 ∴ Solution of inequality (ii) is x ≤ 10……(iv)
⇒ 5x > −25 ⇒ x > −5
Also, 5x − 1 < 24 ⇒ 5x < 25 ⇒ x < 5
Hence, −5 < x < 5 ⇒ x ∈ (−5,5)
98. Ans (B) Clearly the common values of x satisfying (iii) and
2x − 7 < 11 ⇒ 2x < 18 ⇒ x < 9 (iv) lie between 9 and 10 .
3x + 4 < −5 ⇒ 3x < −9 ⇒ x < −3 Hence, the solution of the given system is given by
Hence, common solution is x < −3. 9 < x ≤ 10 ⇒ x ∈ (9,10]
So, x ∈ (−∞, −3)
99. Ans (C)
Case I :
2 2 2 103. Ans (C)
When x > 0, < 3 ⇒ 2 < 3x ⇒ < x or x >
x 3 3
Conceptual
Case II:
104. Ans (B)
2 2 2
When x < 0, < 3 ⇒ 2 > 3x ⇒ > x or x < ,
x 3 3 We have 3x − 7 > x + 1
which is satisfied when x < 0. ⇒ 2x > 8 ⇒ x > 4,So x ∈ (4, ∞)
2
∴ x ∈ (−∞, 0) ∪ ( , ∞)
3
105. Ans (D)

MATHEMATICS Page |2A. 11


LINEAR INEQUALITIES

2(x−1) 3(2+x) (ii)B − A = xϵ(−∞, −2] ∪ [5, ∞)


We have ≤
5 7
= R − (−2,5)
⇒ − x − 44 ≤ 0 ⇒ x ≥ −44
(iii)A ∪ B = x: xϵ(∞, 2] ∪ [5, ∞)
⇒ x ∈ (−44, ∞)
(iv)A − B = (−1,2)
106. Ans (A)
5−2x x 112. Ans (C)
We have < −5
3 6
Given equation has no real sol
5 − 2x x
⇒ − +5<0 ∴𝐷<0
3 6
⇒ x ∈ (8, ∞) ⇒ 4(1 + 3𝑚)2 < 4(1 + 𝑚2 )(1 + 8𝑚)

107. Ans (C) ⇒ 8𝑚3 − 8𝑚2 + 2𝑚 > 0

Conceptual , x ∈ (−1, ∞) ⇒ 2m(2𝑚 − 1)2 >0


1
108. Ans (C) ⇒ m > 0 & m≠ [because ½ is not an integer]
2
6x−5
We have <0 ⇒Number of integral values are infinitely many.
4x+1

5 113. Ans(C)
x−
⇒ 6 < 0 ⇒ x ∈ (− 1 , 5) If x < 5 then −x > −5
5 4 6
x+ 114. Ans(C)
4
109. Ans (C) Given that x < y, b < 0
We have
3
<1 x y
x−2 ⇒ > ,b < 0
b b
3
⇒ −1<0 115. Ans(A)
x−2
3 − (x − 2) Given that −3x + 17 < −13
⇒ <0
x−2 ⇒ −3x < −17 − 13 ⇒ −3x < −30
5−x 3x > 30 ⇒ x > 10
⇒ < 0 ⇒ x ∈ (−∞, 2) ∪ (5, ∞)
x−2
⇒ x ∈ (10, ∞)
110. Ans (A)
116. Ans(B)
A= {m ∈ 𝐑: x2-(m+1)x+m+4=0 has real roots }
Given that |x| < 3 ⇒ − 3 < x < 3
D ≥ 0 ⇒ (m + 1)2 − 4(m + 4) ≥ 0
117. Ans(D)
⇒ 𝑚2 + 2𝑚 + 1 − 4𝑚 − 16 ≥ 0
Given that |x| > b, b > 0
⇒ 𝑚2 − 2𝑚 − 15 ≥ 0
⇒ x < −b or x > b
⇒ (m − 5)(m + 3) ≥ 0
⇒ x ∈ (−∞, −b) ∪ (b, ∞)
⇒ m€(−∞, −3] ∪ [5, ∞)
118. Ans(C)
So, A=(−∞, −3] ∪ [5, ∞) & B[-3,5)
Given that |x − 1| > 5
Now, check the options
⇒ (x − 1) < −5 or (x − 1) > 5
A-B=(−∞, −3] ∪ [5, ∞)
⇒ x < −5 + 1 or x > 5 + 1
AB= {-3}
⇒ x < −4 or x > 6
B-A=(-3,-5)
⇒ x ∈ (−∞, −4) ∪ (6, ∞)
A∪B=R
119. Ans(B)
Hence option (A) is the correct answer.
Given that |x + 2| ≤ 9
111. Ans (B)
⇒-9≤ x + 2 ≤ 9
Given,
⇒-7≤ x ≤ 7
A=x: xϵ (-2,2) and B=x: xϵ (-∞, -1]∪[5, ∞)
⇒ x ∈ [−11,7]
(i)A ∩ B = (−2, −1]
120. Ans(A)
MATHEMATICS Page |2A. 12
LINEAR INEQUALITIES
7
The given graph represents ⇒3≤3x≤7⇒1≤x≤
3
x > −5& x < 5
130. Ans A
Combine to get |x| < 5
R(x) − c(x) > 0;
121. Ans(D)
60x + 2000 − 20x − 4000 > 0 & 𝑥 > 50
The gives graph represents all values of x greater
131. Ans (C)
than 5 including 5 on the real number line. So, x ∈
3𝑥=6⇒𝑥=2⇒4𝑥−𝑦=−1
(5, ∞)
⇒8−𝑦=−1⇒9=𝑦
122. Ans(B)
132. Ans (𝐂)
The given graph has all real values of x greater
a<b
9
than and equal to . a b
2
> (x < 0)
9 9 x x
So, x ≥ ⇒ x ∈ [ , ∞)
2 2

123. Ans(A) 133. Ans (𝐁)


The given graph represents all the values of x less 134. Ans (B)
7
than on real number line.
2
7
So, x ∈ (−∞, )
2

124. Ans(B) KCET MOCKTEST SOLUTIONS


The given graph represents all real values of x 1. Ans(B)
less than or equal to −2 x−2>0
So, x ∈ (−∞, −2] ⇒ x > 2 ⇒ x ∈ (2, ∞)
125. Ans(A) 2. Ans(C)
-7<2x+3<7 6 ⩽ −3(2x − 4) ⩽ 12
⇒-10<2x<4 ⇒ 2 ⩽ −2x + 4 ⩽ 4 ⇒ −2 ⩽ −2x ⩽ 0
⇒-5<x<2 ⇒ xϵ(−5,2) ⇒ −1 ⩽ −x ⩽ 0 ⇒ 0 ⩽ x ⩽ 1
126. Ans(A) 3. Ans(D)
x2 +6x−7 x 1
<0 − ⩽0
|x+4|
x+2 x
⇒ critical points for wavy curve are x2 − x − 2 (x + 1)(x − 2)
⇒ ⩽0⇒ ⩽0
x=-7, -4 & 1 x(x + 1) x(x + 2)
⇒ critical points for wavy curve are
x=-1,2,0 &- 2

So, x ϵ (-7, -4)∪(-4,1)


127. Ans(A)
-1≤x-2≤1
So,x ∈ (−2, −1] ∪ (0,2]
⇒1≤x≤3⇒xϵ [1,3]
4. Ans(A)
128. Ans(C)
(x + 2)2 −1
x+5≥10 or x+5≤-10 > 0, x ≠ −2,1,
(x − 1)(2x + 1) 2
⇒x≥5 or x≤-15⇒xϵ(-∞,-15]∪[5,∞) ⇒ (x − 1)(2x + 1) > 0
129. Ans(C) −1
⇒ x ∈ (−∞, ) ∪ (1, ∞)
-2≤3x-5≤2 2

MATHEMATICS Page |2A. 13


LINEAR INEQUALITIES

−1 9. Ans(A)
⇒ x ∈ (−∞, −2) ∪ (−2, ) ∪ (1, ∞)
2 x+5
5. Ans(D) ⩽0
x−2
(x − 4)(x − 3) ⇒ (x + 5)(x − 2) ⩽ 0 and x ≠ 2
>0
2x 2 + 4x + 5 ⇒ x ∈ [−5,2)
(x − 4)(x − 3) > 0
10. Ans(C)
∵ 2x 2 + 4x + 5 > 0, ∀x ∈ R
x+1 1 4(x + 1) − (x 2 + 2)
(Δ = 16 − 40 < 0) − > 0 ⇒ >0
x2 + 2 4 4(x 2 + 2)
x ∈ (−∞, 3) ∪ (4, ∞) 4x + 2 − x 2
⇒ > 0 ⇒ 4x + 2 − x 2 > 0
6. Ans(A) 4(x 2 + 2)
1 3 ⇒ x 2 − 4x − 2 < 0 ⇒ x ∈ (2 − √6, 2 + √6)
− <0
x+2 x−3
⇒ x ∈ (2 − 2.4 , 2 + 2.4)
x − 3 − 3x − 6
⇒ <0 ⇒ x ∈ (−0.4 , 4.4)
(x + 2)(x − 3)
−2x − 9 So, the integral values of x= {0,1,2,3,4}
⇒ <0
(x + 2)(x − 3) 11. Ans(D)
(2x + 9) x−4
⇒ >0 −2≥0
(x + 2)(x − 3) x+1
x − 4 − 2x − 2
⇒ critical points for wavy curve are ≥0
−9
x+1
x= , -2 & 3 −x − 6 x+6
2
≥0⇒ ⩽0
x+1 x+1
⇒ (x + 6)(x + 1) ⩽ 0 and x ≠ −1
⇒ x ∈ [−6, −1] and x ≠ −1 ∴ x ∈ [−6, −1)
12. Ans(A)
9
So x ∈ (− , −2) ∪ (3, ∞)
2 5x − 6 ≥ 8, ( or )5x − 6 ≤ −8
7. Ans(D) x≥
14
(or) x ≤
−2
5 5
7 + 9(x − 2) + (x − 2)(x − 3)
<0 2 14
(x − 2)(x − 3) ⇒ x ∈ (−∞, − ] ∪ [ , ∞)
5 5
(x + 5)(x − 1)
<0 13. Ans(C)
(x − 2)(x − 3)
x − 1 < −5( or )x − 1 > 5
⇒ critical points for wavy curve are
x < −4( or )x > 6
x=-5,1,2 & 3
x ∈ (−∞, −4) ∪ (6, ∞)
14. Ans (D)
2x − 3
−3≥0
3x − 5
xϵ (-5,1)∪ (2,3) 2x − 3 − 9x + 15
≥0
8. Ans(A) 3x − 5
x 2 + 2 ⩽ 3x and 3x ⩽ 2x 2 − 5 12 − 7x
≥0
3x − 5
x 2 − 3x + 2 ⩽ 0 and 2x 2 − 3x − 5 ⩾ 0
⇒ (12 − 7x)(3x − 5) ⩾ 0, x ≠ 5/3
(x − 1)(x − 2) ⩽ 0 and 2x 2 − 5x + 2x − 5 ⩾ 0
5 12
x ∈ [1,2] and (x + 1)(2x − 5) ⩾ 0 ⇒x∈( , ]
3 7
5 15. Ans (D)
x ∈ (−∞, −1] ∪ [ , ∞)
2
Solution set x = ϕ
MATHEMATICS Page |2A. 14
LINEAR INEQUALITIES

x − 2 2x − 3 ⇒ x < −√2 or x > √2


− >0
x + 2 4x − 1
 Solution set is (−∞, −√2) ∪ (√2, ∞)
(x − 2)(4x − 1) − (2x − 3)(x + 2)
>0 19. Ans (D)
(x + 2)(4x − 1)
2(x 2 − 5x + 4) We have,
>0
(x + 2)(4x − 1) |x| > b  x < – b or x > b
(x − 4)(x − 1)  x  (–, –b)  (b, )
>0
(x + 2)(4x − 1)
20. Ans (C)
⇒ critical points for wavy curve are x 1 x 1
≤ ⇒ − ≤0
1 x−3 x x−3 x
x= -2, , 1 & 4
4 x2 −x+3
⇒ ≤0
x(x−3)

x2 – x + 3  0 for all x  R
(∵ a > 0 and b2 – 4ac < 0)
x2 − x + 3
1 ∴ ≤0
x ∈ (−∞, −2) ∪ ( , 1) ∪ (4, ∞) x(x − 3)
4
⇒ x(x − 3) < 0 ⇒ 0 < x < 3
So, x ∉ (1,4)
⇒ x ∈ (0,3)
Solution set is (0, 3).
16. Ans (D)
21. Ans (B)
|4x – 3| < 27 –27 < (4x – 3) < 27
The given inequality
 –24 < 4x < 30
6 ≤ −3(2x − 4) < 12
 – 6 < x < 15/2
6 ≤ −6x + 12 < 12
15
Solution set is (−6, ) Adding ( -12) to each term,
2

17. Ans (A) 6 − 12 ≤ −6x + 12 − 12 < 12 − 12


Consider the real line ⇒ −6 ≤ −6x < 0
Dividing by (−6) to each term,
−6 −6x 0
≥ >
For, x < –1, then we have, –x+1–x –1<4 −6 −6 −6
⇒1≥x>0
 –2x < 4  x > –2
⇒0<x≤1
For, –1 < x < 1, then x – 1 – x – 1 < 4
∴ Solution set is (0,1].
–2 < 4 which is true
22. Ans (B)
For, x > 1, then x – 1 + x + 1 < 4
We have, −8 < 5x − 3 < 7
 2x < 4  x < 2
⇒ − 5 ≤ 5x < 10
Solution set is (–2, 2)
⇒ − 1 ≤ x < 2 ⇒ x ∈ [−1,2)
18. Ans (B)
23. Ans (C)
If x + 2 > 0, then we have,
We have,
x2 + x + 2 + x > 0  x2 + 2x + 2 > 0
MA
which is true for all x > – 2 IQ = × 100
CA
(∵discernment is < 0) MA
⇒ IQ = × 100(∵ CA = 12yr)
If x + 2 < 0, then we have, 12
25
x2 – x – 2 + x > 0  x2 > 2 = MA
3

MATHEMATICS Page |2A. 15


LINEAR INEQUALITIES

Given, 80 ≤ 1Q ≤ 140 The given system of inequalities is x + 2 > 11…….


25 (i)
⇒ 80 ≤ MA ≤ 140
3 2x ≤ 20…...(ii)
⇒ 240 ≤ 25MA ≤ 420
Now, x + 2 > 11 ⇒ x > 11 − 2 ⇒ x > 9 ⇒ x ∈
240 420
⇒ ≤ MA ≤ (9, ∞)
25 25
⇒ 9.6 ≤ MA ≤ 16.8 ∴ Solution of inequality (i) is x > 9……(iii)

24. Ans (C) and 2x ≤ 20 ⇒ x ≤ 10 ⇒ x: ∈ (−∞, 10]

Given, 3 < 3t − 18 ≤ 18 ∴ Solution of inequality (ii) is x ≤ 10……(iv)

Adding 18 to each term,


3 + 18 < 3t − 18 + 18 ≤ 18 + 18
⇒ 21 ≤ 3t ≤ 36
Dividing by 3 to each term, Clearly the common values of x satisfying (iii) and
21 3t 36 (iv) lie between 9 and 10 .
≤ ≤
3 3 3
Hence, the solution of the given system is given by
⇒ 7 ≤ t ≤ 12
9 < x ≤ 10 ⇒ x ∈ (9,10]
Adding 1 to each term,
7 + 1 ≤ t + 1 ≤ 12 + 1
⇒ 8 ≤ t + 1 ≤ 13
25. Ans (C) 28. Ans (C)
−8 ≤ 5x − 3 ⇒ −5 ≤ 5x ⇒ −1 ≤ x Conceptual
5x − 3 < 7 ⇒ 5x < 10 ⇒ x < 2 29. Ans (D)
5−2x x
Hence, common sol is −1 ≤ x < 2 We have ≤ −5
3 6
⇒ x ∈ [−1,2) ⇒ a = 1, b = 2 and a + b = 3 or 2(5 − 2x) ≤ x − 30 or 10 − 4x ≤ x − 30
26. Ans (B) or −5x ≤ −40 or x ≥ 8
C F−32
Given: = and 10 < C < 20. Thus, all real numbers which are greater than or
5 9

5 F − (32)5 equal to 8 are the solutions of the given inequality,


⇒C=
9 i.e., x ∈ [8, ∞).
5 F−160
Since, 10 < C < 20 ⇒ 10 < < 20 30. Ans (C)
9

⇒ 90 < 5 F − 160 < 180 4x + 3 < 6x + 7 ⇒ −2x < 4


⇒ 90 + 160 < 5 F < 180 + 160 ⇒ −x < 2 ⇒ x > −2 ⇒ x ∈ (−2, ∞)
250 340
⇒ 250 < 5 F < 340 ⇒ <F<
5 5
⇒ 50 < F < 68
27. Ans (C)

MATHEMATICS Page |2A. 16


RELATIONS AND FUNCTIONS

HINTS AND SOLUTIONS

1. Ans (A) 7. Ans (A)


A×B I. Given, (4x + 3, y) = (3x + 5, −2)
= {(1,3), (1,4), (2,3), (2,4), Two ordered pairs are equal when their
(3,3), (3,4)} corresponding elements are equal.
and A × C = {(1,4), (1,5), (1,6), (2,4), 4x + 3 = 3x + 5 and y = −2
(2,5), (2,6), (3,4),(3,5), (3,6)} 4x − 3x = 5 − 3
Therefore, (A × B) ∩ (A × C) x=2
= {(1,4), (2,4), (3,4)}. II. Now, A = {−1,3,4}
2. Ans (A) ∴ A × A = {(−1, −1), (−1,3), (−1,4), (3, −1),
n(A × A × B) = n(A) × n(A) × n(B) (3,3), (3,4), (4, −1), (4,3), (4,4)}
= 3 × 3 × 4 = 36 8. Ans (B)
3. Ans (B) Given, n(A) = 4, n(B) = 5 and
A − C = {1,4} and B − C = {4} n(A ∩ B) = 3
∴ (A − C) × (B − C) ∴ n[(A × B) ∩ (B × A)] = 32 = 9
= {1,4} × {4} = {(1,4), (4,4)}. 9. Ans (A)
4. Ans (B) X × Y = {a, b, c} × {} = ϕ
We have B ∪ C = {3,4,5,6} Hence there are no ordered pairs formed in X × Y.
∴ A × (B ∪ C) = 10. Ans (A)
{(1,3), (1,4), (1,5), (1,6), (2,3), (2,4), use distributive law
(2,5), (2,6), (3,3), (3,4), (3,5), (3,6)}. 11. Ans (C)
5. Ans (A) It is obvious that the relation R is ' x is the square
A = Set of first elements of the ordered pairs of of y ′ . In Set-builder form, R = {(x, y): x is the
A × B = {1,2,3} square of y, x ∈ P, y ∈ Q}
B = Set of second element of the ordered pairs of 12. Ans (A)
A × B = {3,5} If n(A) = p and n(B) = q, then total number of
A × B = {(1,3), (1,5), (2,3), relations that can be defined from a set A to a set B
(2,5), (3,3), (3,5)} is 2pq .
So, the remaining elements of A × B are Therefore, the number of relations from A to B
(1,5), (2,3), (3,5) will be 24 = 16.
6. Ans (C) 13. Ans (A)
2 2 Given, A = {x: |x| < 3, x ∈ I}
2a + 3b = 35, a, b ∈ Z
If a = 2, b = 3, then A = {x: −3 < x < 3, x ∈ I}
2 × 22 + 3 × 32 = 35 = {−2, −1,0,1,2}
∴ (a, b) is (2,3). In the same way, other sets are Also, R = {(x, y): y = |x|}
(−2, −3), (4,1), (−4, −1), (2, −3), ∴ R = {(−2,2), (−1,1), (0,0), (1,1), (2,2)}
(−2,3), (−4,1), (4, −1) 14. Ans (C)
So, number of elements in the set is 8 .
MATHEMATICS Page |3A. 1
RELATIONS AND FUNCTIONS

Recall that a|b stands for a divides b. For the ∴ Total number of non-empty relations from A to
elements of the given sets A and B, B = 28p − 1
We find 2|6,2|10, 3|3,3|6,5|10 20. Ans (B)
R={(2,6), (2,10), (3,3), (3,6), (5,10)} 6
Given, R = {(x, y): y = x + ,
x
∴ R−1 is given by
where x, y ∈ N and x < 6}
= {(6,2), (10,2), (3,3), (6,3), (10,5)} 6
For, x = 1, y = 1 + = 7
1
15. Ans (C)
6
The given relation in Roster form is, x = 2, y = 2 + =5
2
R = {(3,5), (4,6), (5,7), (6,8), 6
x = 3, y = 3 + = 5
(7,9), (8,10), (9,11)} 3
∴ Domain of R = {3,4,5,6,7,8,9} 6 11
x = 4, y = 4 + = ∉N
4 2
So, A is true.
6 31
Range of R = {5,6,7,8,9,10,11} x = 5, y = 5 + = ∉N
5 5
So, R is false. ∴ The given relation in Roster form is
16. Ans (A) R = {(1,7), (2,5), (3,5)}
We have, y = x + 5, x < 4, x, y ∈ N Hence, domain of R = {1,2,3}
For, x = 1, y = 6 Range of R = {7,5}
x = 2, y = 7 21. Ans (C)
x = 3, y = 8 Prime numbers less than 10 are 2,3,5,7.
∴ In Roster form R = {(1,6), (2,7), (3,8)} Hence, relation R in Roster form is
Domain of R = {1,2,3} R = {(2,8), (3,27), (5,125), (7,343)}
Range of R = {6,7,8} 22. Ans (B)
17. Ans (A) The difference of two integers is also an integer.
Clearly, the difference between the elements in P ∴ Domain of R = Z
and Q is 2. So, in Set-builder form, Range of R = Z
Relation R = {(x, y): x − y = 2, x ∈ P and y ∈ Q} 23. Ans (C)
In Roster form, R = {(5,3), (6,4), (7,5)} ∵ 3 + 7 = 10,3 + 11 = 14,
Domain of R = {5,6,7} 5 + 7 = 12,5 + 11 = 16.
Range of R = {3,4,5} ∴ There is no element that belongs to R. Hence,
18. Ans (B) R = {}
Let the number of elements in set B be x. 24. Ans (A)
Number of elements in set A = 3 Given, A = {1,2,3,4,5,
Number of relations from A to B = 4096 R: A → A and R = {(x, y): y = 2x − 1}
3x
⇒2 = 4096 ∴ R = {1,1), (2,3), (3.5)}
3x 12
⇒2 =2 Hence, domain of R = {1,2,3}
On comparing, we get 3x = 12 25. Ans (C)
⇒x=4 Given, S = {(a, b): b = |a − 1|, a ∈ Z
So, number of elements in set B = 4 and |a| < 3}z → set of integers.
19. Ans (D) ∴ S = {(−2,3), (−1,2), (0,1), (1,0), (2,1)}
Given n(A) = 8 and n(B) = p ∴ Range of set (S) = {0,1,2,3}
8p
∴ Total number of relations from A to B = 2 (∵ |a| < 3 ⇒ −3 < a < 3)
KCET MATHEMATICS Page |3A. 2
RELATIONS AND FUNCTIONS

26. Ans (B) 1 1


⇒ y ∈ [− , ]
1 2 2
f(x) = 1 1
√9 − x 2 ∴ Range of f = [− , ]
2 2
Clearly, 9 − x 2 > 0 31. Ans (B)
2
⇒x −9 <0 Here f(x) is defined only when x + 3 ≠ 0
⇒ (x + 3)(x − 3) < 0 i.e., when x ≠ −3.
Thus, domain of f(x) is (−3,3). ∴ D(f) = R − {−3}
27. Ans (D) 32. Ans (A)
We have, D(f + g) = D(f) ∩ D(g)
log (x + 5)
f(x) = 2 = (−∞, 0] ∩ [0, ∞) = {0}
x + 4x + 3
33. Ans (C)
log (x + 5)
= Given function is
(x + 3)(x + 1)
f(x) will be defined, if x + 3 ≠ 0, y = √x − 2 + √1 − x
x+1≠0 Since x − 2 ≥ 0 and 1 − x ≥ 0
and x + 5 > 0 i.e, x ≠ −3, ⇒x≥2
x ≠ −1 and x > −5 ⇒x≤1
∴ Domain of function is ∴ from (i) and (ii), x = ϕ
(−5, ∞) − {−3, −1} 34. Ans (D)
28. Ans (C) π π
− ≤x≤
4 4
x2 +1
Given, f(x) = π
⇒ − ≤ x < 0 and 0 ≤ x ≤
π
x2 −3x+3
4 4
At no point of R, x 2 − 3x + 3 = 0 ⇒ [x] = −1 and x] = 0
So, required domain = R 35. Ans (B)
29. Ans (D) For f(x) = g(x)
f(x) = √log10 x2 is real ⇒ 2x 2 − 1 = 1 − 3x
If log10 x 2 ≥ 0 ⇒ 2x 2 + 3x − 2 = 0
⇒ x2 ≥ 1 ⇒ 2x 2 + 4x − x − 2 = 0
⇒ x ≤ −1 and x ≥ 1 ⇒ (2x − 1)(x + 2) = 0
⇒ x ∈ (−∞, −1] ∪ [1, ∞) Thus, domain for which the function f(x) = g(x) is
30. Ans (C) 1
{−2, }.
x 2
We have, f(x) =
1+x2 36. Ans (D)
Let, f(x) = y f(0) = 0 − 4 = −4,
x 1 1 9
∴y= f (− ) = − − 4 = − ,
1 + x2 2 2 2

⇒ x2 y + y = x f(3) = 12(3) + 5 = 41,


⇒ yx 2 − x + y = 0 f(−5) = −5 − 4 = −9 ≠ 9.
x assumes real values, if Δ ≥ 0 37. Ans (A)
2
(−1) − 4(y)(y) ≥ 0 Since f is a linear function,
2
⇒ 1 − 4y ≥ 0 f(x) = mx + c. Also
2
⇒ 4y − 1 ≤ 0 since (1,1), (0, −1) ∈ R,
⇒ (2y + 1)(2y − 1) ≤ 0 f(1) = m + c = 1 and f(0) = c = −1.

KCET MATHEMATICS Page |3A. 3


RELATIONS AND FUNCTIONS

This gives m = 2 45. Ans (C)


∴ f(x) = 2x − 1 Given, f(x) = x 3 − 1
38. Ans (C) ∴ f(3) = (3)3 − 1 = 27 − 1 = 26
f f(x) x3 f(2) = (2)3 − 1 = 8 − 1 = 7
( ) (x) = = 2
g g(x) x + 1 f(3)+f(2) 26+7 33
Now, = = =3
11 11 11
39. Ans (A)
46. Ans (C)
(fg)(x) = f(x)g(x) = (√x − x)x 2
Since, 2x is positive for every x ∈ R.
= x 5/2 − x 3
So, f(x) = 2x is a positive real number for every
40. Ans (C)
x ∈ R.
We have, (f + g)(x)
∴ Range of f = R+
= √x + x, (f − g)(x) = √x − x
Now, f(x) = 1
3
(fg) x = √x(x) = x 2
⇒ 2x = 1 ⇒ 2x = 20 ⇒ x = 0
1
f √x
and ( ) (x) = = x −2 , x ≠ 0 ∴ {x: f(x) = 1} = {0}
g x
We have, f(x + y) = 2x+y
41. Ans (B)
= 2x ⋅ 2y = f(x) ⋅ f(y)
(f + g)(x) = f(x) + g(x)
∴ f(x+y) =f(x)⋅f(y) for all x, y ∈ R
= √x − x + x 2
47. Ans (B)
42. Ans (B)
f(a + 1) − f(a − 1)
Consider R1 = {(x, y): x, y ∈ A and y = x + 1}
= [4(a + 1) − (a + 1)2 ]
= {(1,2)(2,3), (3,4)}
−[4(a − 1) − (a − 1)2 ]
Since, 4 has no image. So, R1 is not a function.
= 8 − 4a = 4(2 − a).
Now, R 2 = {(x, y): x, y ∈ A and x + y = 5}
48. Ans (C)
= {(1,4), (2,3), (3,2), (4,1)}
We have, f(x) = x 4 + x 2
Since, every element of A has its image in A.
Now,
So, R 2 is a function in A.
1 1 1
43. Ans (B) f(x) ⋅ f ( ) = (x 4 + x 2 ) ( + 2)
x x4 x

Given, f(x) = 2x − 5 1 1
1 + x2 + + 1 = 2 + x2 + 2
x2 x
Put x = 0, f(0) = 2(0) − 5 = −5
1 2
x = 7, f(7) = 2(7) − 5 = 9 = (x + )
x
∴ f(0) ≠ f(7) 49. Ans (C)
x = −3, f(−3) = 2(−3) − 5 f(x + y) = f(x) ⋅ f(y) ∀x, y ∈ N,
= −6 − 5 = −11 f(1) = 3
44. Ans (C)
f(2) = f 2 (1) = 32
2
f(x) = 2x + bx + c,
f(3) = f(1)f(2) = 33
f(0) = 3 ⇒ 3 = c
f(4) = 34
and f(2) = 1
f(k) = 3k
⇒ 1 = 8 + 2b + c
∑nk=1 f(k) = 3279
⇒ 2b + 3 = −7
f(1) + f(2) + f(3) + ⋯ … … + f(k) = 3279
⇒ b = −5
3 + 32 + 33 + ⋯ … 3k = 3279
2
∴ f(1) = 2 × 1 + (−5) × 1 + 3
3(3k − 1)
= 2 − 5 + 3 = 0. = 3279
3−1
KCET MATHEMATICS Page |3A. 4
RELATIONS AND FUNCTIONS

3k − 1 = 23(2) = 26 = 64
= 1093
2 55. Ans (B)
3k − 1 = 2186
2n(A)n(B) = 64 = 26 ⇒ n(A) ⋅ n(B) = 6
k
3 = 2187
And 2n(B)⋅n(C) = 512 = 29
k=7
⇒ n(B) ⋅ n(C) = 9
50. Ans (D)
n(A) ⋅ n(B) = 6 = 2 × 3(or)6 × 1
In the given function,
n(B) ⋅ n(C) = 9 = 3 × 3( or )1 × 9
1−x
f(x) = log ( ) n(B) = 3(or)1
1+x
2x Required sum of values of n(B)
Putting in place of x we get,
1+x2
=3+1
2x
2x 1+ 56. Ans (A)
f( ) = log ( 1 + x2 )
1+x 2 2x n(A) = n
1−
1 + x2
2
1 + x 2 + 2x 2n(A×A) = 2n
= log ( )
1 + x 2 − 2x 57. Ans (D)
1−x 2 1+x A = {1,2,3,4,5}
= log ( ) = 2log ( )
1+x 1−x verify the options
= 2f(x) 58. Ans (A)
A = {1,2,3,4,5,6,7,8}
51. Ans (B) x + 2y = 9
A = {1,2,3,4,5,6, … . ,10} if, x = 1, ⇒ y = 4
R = {(2,4), (4,3), (6,2)(8,1)} x = 3, ⇒ y = 3
∴ Domain of R is {2,4,6,8} x = 5, ⇒ y = 2
52. Ans (C) x = 7, ⇒ y = 1
x2 + y2 ⩽ 4 ∴ R = {(1,4), (3,3), (5,2), (7,1)}
2 2
y ⩽4−x R−1 = {(4,1), (3,3), (2,5), (1,7)}
y ⩽ √4 − x 2 domain of R−1 = {4,3,2,1}
4 − x2 ⩾ 0 59. Ans (B)
⇒ −2 ⩽ x ⩽ 2 R 2 is a function
Therefore the domain R
= {−2, −1,0,1,2}
53. Ans (C)
A = {1,2,3}, B = {1,4,6,9}. R ⊆ A × B
R = {(x, y): x > y, x ∈ A, y ∈ B}
R = {(2,1)(3,1)} 60. Ans (A)
Range = {1} f(x)=4x-x 2 =x(4-x)
54. Ans (C) f (a + 1) − f (a − 1) = (a + 1)(4 − a −
A = {a, b, c} ⇒ n(A) = 3 1) − (a − 1)(4 − a + 1)
B = {1,2} ⇒ n(B) = 2 = (a + 1) (3 − a) − (a − 1)(5 − a)
Therefore, Number of relations from A to B= = 3a – a2 +3-a-5a+ a2 +5-a
2n(A)⋅n(B) =8-4a

KCET MATHEMATICS Page |3A. 5


RELATIONS AND FUNCTIONS

67. Ans (C)


61. Ans (B) x, x > 0
|x| = {
−x, x ⩽ 0
2x, x ⩾ 0
f(x) + g(x) = x + |x| = {
0, x < 0
68. Ans (D)
−x 2 , x<0
f(x) = { 0, x=0
x2 , x>0
= x|x|, ∀x ∈ ℝ
69. Ans (D)
62. Ans (C)
R1 = {(1,3), (2,4), (3,5), (4,6), (5,7)}
f(x) = x 2 − 3x + 1
Since 4 and 6 do not belongs to Y
f(2α) = 2f(α)
(2,4), (4,6) ∉ R1
(2α)2 − 6α + 1 = 2[α2 − 3α + 1]
R1 = {(1,3), (3,5), (5,7)} ⊂ A × B
4α2 − 6α + 1 = 2α2 − 6α + 2
Hence R1 is a relation but not a mapping as the
2α2 = 1
elements 2 and 4 do not having any image.
1
α=± R 2 : It is a relation.
√2
R 3 : it is a relation being a subset of A × B but the
63. Ans (B)
elements 1 and 3 do not have a unique image and
n(A) = 7
hence it is not a mapping.
n(B) = 5
R 4 : It is both a mapping and a relation. each
The number of functions from A into B
element in A has a unique image. it is also one -
= n(B)n(A) = 57
one and onto mapping and hence a bijection.
64. Ans (B)
70. Ans (A)
f(−1) = −5, f(3) = 3
1 1
−a + b = −5, 3a + b = 3 f(x)f ( ) = (1 + x 4 ) (1 + 4 )
x x
a−b=5 1
= 2 + x4 + 4
3a + b = 3 x
on solving, a = 2 and b = −3 1
= f(x) + f ( )
x
65. Ans (A)
Alternatively, use T.S.R
f(x) = αx + β
71. Ans (C)
∵ f(1) = 1
f(2) = 65
α + β = 1 → (1)
f(x) = 1 + x n
f(2) = 3
f(2) = 1 + 2n = 65
2α + β = 3 → (2)
2n = 64 = 26
From (1) and (2)
n=6
α = 2, β = −1
f(x) = 1 + x 6
66. Ans (A)
f(−3) = 1 + (−3)6
f(x) = 2x − 1
= 730
g(x) = x 2
72. Ans (C)
∵ (3f − 2g)(1) = 3f(1) − 2g(1)
1 1
= 3(1) − 2 = 1 f(x) ⋅ f ( ) = f(x) + f ( )
x x
KCET MATHEMATICS Page |3A. 6
RELATIONS AND FUNCTIONS

f(3) = −80 x−7


⩾ 0, x ≠ 9
∴ f(x) = 1 − x n x−9
⇒ x ∈ (−∞, 7] ∪ (9, ∞)
f(3) = 1 − 3n
[∵ using T. S. R of linear inequality
−80 = 1 − 3n
80. Ans (A)
−3n = −81 = −34
3x 2 + 2x + 1 ⩾ 0, ∀x ∈ R
n=4
∴ Domain of f = R
f(x) = 1 − x 4
81. Ans (A)
1 1
f(x) − f ( ) = 1 − x 4 − 1 + 4 f(x) is well defined when
x x
1 x 2 − x − 6 ≠ 0, ⇒ (x − 3)(x + 2) ≠ 0
= 4 − x4
x ⇒ x ≠ 3, −2
73. Ans (C)
∴ Domain = R − {3, −2}
f(xy) = f(x) + f(y) ⇒ f(x) = log a x
82. Ans (A)
f(2) = 1
f(x) = g(x)
log a 2 = 1
3x 2 − 1 = 3 + x
2=a
3x 2 − x − 4 = 0
∴ f(x) = log 2 x
(3x − 4)(x + 1) = 0
74. Ans (A)
4
f(x + y) = f(x) ⋅ f(y) x = {−1, }
3
Then f(x) = ax . a ≠ 0 83. Ans (B)
f(0) = 1 Domain = R
75. Ans (B) Range = |x − 5| ≥ 0
f(x) = kx = 2 − (+ ve integer)
f(1) = 2 ⇒ k = 2 = (−∞, 2]
f(x) = 2x 84. Ans (A)
76. Ans (D) D1 = 4 − x ≥ 0 ⇒ 4 ≥ x
(x − 6)(x − 9) > 0 D2 = x 2 − 1 > 0 ⇒ (x + 1)(x − 1) > 0
x ∈ (−∞, 6) ∪ (9, ∞) Thus, x ≤ 4 x, < −1 and x > 1
77. Ans (B) D = (∞, −1) ∪ (1,4]
2
2 − 2x − x ⩾ 0 [∵ by taking intersection of above intervals using
x 2 + 2x − 2 ⩽ 0 number line]
2
x + 2x + 1 ⩽ 3 85. Ans (D)
(x + 1)2 ≤ 3 It is defined for |x| − x > 0
−√3 ⩽ x + 1 ⩽ √3 |x| > x ⇒ x < 0

−1 − √3 ⩽ x ⩽ √3 − 1 Hence x ∈ (−∞, 0)

78. Ans (D) 86. Ans (D)


x 2 − 5x + 6 > 0
f(x) = √(x − 7)(9 − x)
(x − 2)(x − 3) > 0
(x − 7)(9 − x) ≥ 0
x < 2( or )x > 3
x ∈ [7,9]
x ∈ (−∞, 2) ∪ (3, ∞)
79. Ans (A)
87. Ans (A)

KCET MATHEMATICS Page |3A. 7


RELATIONS AND FUNCTIONS

logarithms is well defined for + ve reals ⇒ x ∈ (0,1) and 2x 2 − 3x + 1 ⩽ 0


18x − x 2 − 77 > 0 ⇒ (x − 1)(2x − 1) ⩽ 0
x 2 − 18x + 77 < 0 1
⇒ x ∈ [ , 1]
(x − 7)(x − 11) < 0 2
1
x ∈ (7,11) ∴ x ∈ [ , 1)
2
88. Ans (D) [∵ by taking intersection of above intervals using
f(x) = |x| + x − [x] number line]
|x| + {x} 93. Ans (B)
0 < |x| < 3,0 ⩽ {x} < 1
f(x) = √25 − 9x 2 + √x 2 + x + 1
⇒ 0 < |x| + {x} < 4
⇒ 25 − 9x 2 ⩾ 0 & x 2 + x + 1 > 0, ∀x ∈ R
89. Ans (A) 5 5
⇒ x ∈ [− , ] and x ∈ R
Case (i): 1 ⩽ x < 2 ⇒ [x] = 1 3 3
5 5
1 ⩽ x2 < 4 ⇒ 2 ⩽ x2 + 1 < 5 Domain = [− , ]
3 3
2 x2 + 1 94. Ans (D)
⇒ ⩽ < 5 − − − (1)
1 [x]
x 2 − 4|x| + 3 ⩾ 0
Case (ii) ; 2 ⩽ x < 3 ⇒ [x] = 2
(|x| − 1)(|x| − 3) ⩾ 0
4 ⩽ x 2 < 9 ⇒ 5 ⩽ x 2 + 1 < 10
|x| ∈ (−∞, 1] ∪ [3, ∞)
5 x2 + 1
⇒ ⩽ < 5 − − − (2) |x| ⩽ 1 (or) |x| ⩾ 3
2 [x]
x ∈ [−1,1] (or )x ∈ (−∞, −3] ∪ [3, ∞)
Case (iii) : 3 ⩽ x < 4 ⇒ [x] = 3
x ∈ (−∞, −3] ∪ [−1,1] ∪ [3, ∞)
9 ⩽ x 2 < 16
95. Ans (C)
10 ⩽ x 2 + 1 < 17
f(x) = x logx e
10 x 2 + 1 17
⩽ < − − − −(3) x > 0, x ≠ 1
3 [x] 3
17 ⇒ x ∈ (0,1) ∪ (1, ∞)
from (1), (2)&(3) Range = [2, )
3
96. Ans (C)
90. Ans (B)
x 2 − [x]2 ⩾ 0
1
should not come
0 x 2 ⩾ [x]2 ⇒ x ∈ ℝ+ ∪ ℤ
Thus, 10x − 10−x ≠ 0 97. Ans (C)
102x ≠ 1 ef(x) = e − ex
x≠0 f(x) = log (e − ex )
Domain = R − {0} e − ex > 0
91. Ans (D) e > ex
x 3 + 4x 2 + 3x ≠ 0 x<1
⇒ x(x 2 + 4x + 3) ≠ 0 ⇒ x ∈ (−∞, 1)
⇒ x(x + 3)(x + 1) ≠ 0 98. Ans (C)
⇒ x ≠ 0, −1, −3 f(x) = cot 5x
∴ x ∈ ℝ − {0, −1, −3} cos 5x
=
92. Ans (B) sin 5x
1 sin 5x ≠ 0
f(x) = + √3x − 1 − 2x 2
√x − x2 5x ≠ nπ
2 2 nπ
⇒ x − x > 0 and 3x − 1 − 2x ⩾ 0 x≠ ,n ∈ z
5
KCET MATHEMATICS Page |3A. 8
RELATIONS AND FUNCTIONS

99. Ans (A) |x| < 71


f(x) = 5cos x − 3sin x + 2 −71 < x < 71
x
Range =[c − √a2 + b 2 , c + √a2 + b 2 ] −14.5 < < 14.5
5
= [2 − √34, 2 + √34] x
[ ] = {−15, −14, −13, … ,0,1, … ,13,14}
100. Ans (B) 5
f(x) = x 2 − 4x + 5 107. Ans (B)

= x 2 − 4x + 4 + 1 Range of sin x is [−1,1]

= (x − 2)2 + 1 ≥ 1 Range of [sin x] is {−1,0,1} = A

Range of f = [1, ∞) Range of tanx is R

Or use T.S.R Range of [ tanx ] is Z = B

101. Ans (B) A ∩ B = {−1,0,1}


1 108. Ans (C)
Given f(x)= (3x )2 − 2. 3x + 1
2 π π
− <x<
1 2 1 2 2
= (3x − ) + 1 −
2 4 [x] = {−2, −1,0,1}
2
1 3 cos ([x]) = {cos 2, cos 1,1}
f(x) = (3x − ) +
2 4
109. Ans (A)
3
f(x) ⩾ f(x) = [x] − x = −{x}
4
3 0 ⩽ {x} < 1
Hence Range of f = [ ∞)
4
−1 < −{x} ⩽ 0
102. Ans (A)
A = (−1,0]
|x| −1, −4 ≤ x < 0
f(x) = ={ g(x) = x − [x] = {x}
x 1, 0<x≤4
Range = {−1,1} 0 ⩽ {x} < 1

103. Ans (D) B = [0,1)

f(x) = |x| + |x + 1| ∴ A ∩ B = {0}

f(x) = |x| + |x + 1| ⩾ |x − x − 1| 110. Ans (B)

⩾1 0, x∈ℤ
f(x) = [2x] − 2[x] = {
1, x∉ℤ
Range = [1, ∞)
Range = {0,1}
104. Ans (A)
111. Ans (A)
1
f(x) = 1 − f: [2,3] → ℝ
1 + x2
0 ⩽ f(x) < 1 f(x) = x 3 + 3x − 2 is monotonically increasing

Range = [0,1) function.

105. Ans (B) ⇒fmax and fmin exist at end point of given interval.

1 f(2) = 23 + 6 − 2 = 12
2
x + 2⩾2
x f(3) = 27 + 9 − 2 = 34
x4 + 1 Range = [12,34]
⩾2
x2
112. Ans (A)
x2 1
0⩽ ⩽ 1
1 + x4 2 f(x) = x 2 +
1 + x2
1
Range = [0, ] 1
2
= (x 2 + 1) + −1⩾2−1
106. Ans (C) (1 + x 2 )

KCET MATHEMATICS Page |3A. 9


RELATIONS AND FUNCTIONS

⩾ 1, ∀x ∈ R ⇒ [x] = 2, 3 ⇒ x ∈ [2, 3) or x ∈ [3, 4).


[∵ A. M ⩾ G. M] So, x ∈ [2, 4).
113. Ans (B) 118. Ans(B)
1
max. value = f(−1) = f(3) = 5 Given that: f(x) =
1−2cos x
min. value = f(1) = f(2) = 1
We know that −1 ≤ cos x ≤ 1
range = [1,5]
⇒ 1 ≥ cos x ≥ −1
alternatively, use graph
⇒ − 1 ≤ −cos x ≤ 1
⇒ − 2 ≤ −2cos x ≤ 2
⇒ −2 + 1 ≤ 1 − 2cos x ≤ 2 + 1
⇒ − 1 ≤ 1 − 2cos x ≤ 3
1 1
⇒ −1 ≤ ≤
1−2cos x 3
1
⇒ − 1 ≤ f(x) ≤
3
1
so, the range of f(x) = [−1, ]
114. Ans (B) 3

x 119. Ans (C)


f(x) = ,x ∈ R
1 + |x|
Given that: f(x) = √1 + x 2
If x > 0, |x| = x
⇒ f(xy) = √1 + x 2 y 2 &
x 1
⇒ f(x) = = 1−
1+x x+1 f(x) ⋅ f(y) = √1 + x 2 ⋅ √1 + y 2
If x < 0, |x| = −x = √1 + x 2 + y 2 + x 2 y 2
x 1
⇒ f(x) = = −1 + ∵ √1 + x 2 y 2 ≤ √1 + x 2 + y 2 + x 2 y 2
1−x x+1
115. Ans (B) ⇒ f(xy) ≤ f(x) ⋅ f(y)

f(x) = 3|sin x| − 2|cos x| 120. Ans (B)

0 ⩽ |cos x| ⩽ 1 Let f(x) = √a2 − x 2 ,

−2 ⩽ −2|cos x| ⩽ 0 → (1) f(x) is defined if a2 − x 2 ≥ 0

0 ⩽ |sin x| ⩽ 1 ⇒ x 2 − a2 ≤ 0 ⇒ x 2 ≤ a2

0 ⩽ 3|sin x| ⩽ 3 → (2) ⇒ x ≤ ±a ⇒ −a ≤ x ≤ a

(1) +(2) ∴ Domain of f(x) = [−a, a]

⇒ −2 ⩽ 2|sin x| − 2|cos x| ⩽ 3 121. Ans (B)

Range of f(x) = [−2,3] Given that: f(x) = ax + b

116. Ans (D) ⇒ f(−1) = a(−1) + b

Given that: n( A) = m and n( B) = n ⇒ −5 = −a + b

∴ n( A × B) = n( A) ⋅ n( B) = mn ⇒ a−b=5 ---(1)

So, the total number of non-empty relations and f(3) = 3a + b

from A to B=2mn − 1 ⇒ 3a + b = 3 ---(2)

117. Ans (D) Solve above equations (1) and (2)

We have [x]2 − 5[x] + 6 = 0 We get, a = 2, b = −3

⇒ [x]2 − 3[x] − 2[x] + 6 = 0 122. Ans(A)


1
⇒ [x]([x] − 3) − 2([x] − 3) = 0 Given that: f(x) = √4 − x +
√x2 −1
⇒ ([x] − 3)([x] − 2) = 0 f(x) is defined if

KCET MATHEMATICS Page |3A. 10


RELATIONS AND FUNCTIONS

4 − x ≥ 0 and x 2 − 1 > 0 x
f(x) =
x2 +1
−x ≥ −4 and (x − 1)(x + 1) > 0
2
x ≤ 4 and x < −1 and x > 1 f(2) =
5
∴ Domain of f(x) is (−∞, −1) ∪ (1,4] 2 2/5 10
f(f(2)) = f ( ) = =
[∵ by taking intersection of above intervals using 5 4 29
1+
25
number line]
129. Ans (C)
123. Ans(D)
f(x) = 2x 2
4−x
Given that: f(x) = f(b) − f(a) 2b2 − 2a2
x−4
=
We know that f(x) is defined if b−a b−a
= 2(b + a)
x−4≠0⇒x≠4
∴ required = 2(3.8 + 4) = 2 × 7.8 = 15.6
So, the domain of f(x) is = R − {4}
4−x 130. Ans (A)
Let f(x) = y = = −1
x−4
Domain of f(x) = (−1,1)
⇒ Range of f(x) = −1
3 + 4x − 4x 2 ≥ 0
124. Ans (D)
4x 2 − 4x − 3 ≤ 0
Given that: f(x) = √x − 1
(2x + 1)(2x − 3) ≤ 0
f(x) is defined if x − 1 ≥ 0 ⇒ x ≥ 1 −1 3
⇒x∈[ , ]
∴ Domain of f(x) = [1, ∞) 2 2
−1 3
Let f(x) = y = √x − 1 ⇒ Domain of g(x) = [ , ]
2 2
If x ∈ [1, ∞), then x − 1 ∈ [0, ∞) −1 3
Domain of f(x) + g(x) = (−1,1) ∩ [ , ]
2 2
⇒ √x − 1 ∈ [0, ∞)
−1
=[ , 1]
∴ Range of f(x) = [0, ∞) 2

125. Ans (A) 131. Ans (D)

f(x) is defined if x 2 − x − 6 ≠ 0 f(x) = √9 − x 2


⇒ x 2 − 3x + 2x − 6 ≠ 0 9 − x 2 ⩾ 0 ⇒ x ∈ [−3,3]
⇒ (x − 3)(x + 2) ≠ 0 ⇒ x ≠ −2, x ≠ 3 Range = [0,3]
So, the domain of f(x) = R − {−2,3} 132. Ans (B)
126. Ans (B) f(x) = √x 2 − 7x + 12 = √(x − 4)(x − 3) is well
Given that: f(x) = 2 − |x − 5| defined, if (x − 4)(x − 3) ⩾ 0
Here, f(x) is defined for x ∈ R ⇒ x ∈ (−∞, 3] ∪ [4, ∞)
∴ Domain of f(x) = R 133. Ans (B)
Now, |x − 5| ≥ 0 ⇒ − |x − 5| ≤ 0 We have, f (x) = x2 – 4x + 5
⇒ 2 − |x − 5| ≤ 2  x2 – 4x + 4 + 1  (x – 2)2 + 1
⇒ f(x) ≤ 2 We know that (x – 2)2  0  (x – 2)2 + 1  1
∴ Range of f(x) = (−∞, 2]
 Range: [1, )
127. Ans (D)
134. Ans (D)
cos x ⩾ 0
|x| ≠ ±1x ≠ ±1
π 3π
x ∈ [0, ] ∪ [ , 2π]  Df = R − {−1, 1}
2 2
135. Ans (D)
[i.e., 1st and 4th quadrant]
f(−2) + f(3) + f(4) = −6 + 9 + 8 = 11
128. Ans (B)

KCET MATHEMATICS Page |3A. 11


RELATIONS AND FUNCTIONS

136. Ans (A) Log (x3 – x) is defined for


[𝑥 2 ] − [𝑥] − 6 > 0 ([𝑥] − 3)([𝑥] + 2) > 0 x3 – x > 0  x(x2 – 1) > 0
[x] < −2 or [x] > 3 ⇒ x ∈ (−∞, −2) ∪ [4, ∞) x > 0 and x 2 − 1 > 0
⇒{ OR
137. Ans (A) x < 0 and x 2 − 1 < 0
2b = 27 − 3a  x > 1 Or x  (–1, 0)
27 − 3a Combining the two we have the domain as
b=
2
R = {(1,12), (3,9), (5,6), (7,3)} (–1, 0)  (1, 2)  (2, )
138. Ans (A) 5. Ans (C)
Odd number = {1,3,5,7,9} Clearly 2  N, 4  N satisfy 4x + 3y = 20.
No. of sub sets =25−1=3
 (2, 4)  R
139. Ans (D)
6. Ans (D)
f(−1) = −5
π π
−a + b = −5 We have, = 0.785 < 1. ⇒[ ]=0
4 4
π
3a + b = 3 Now, for x  [ 0, )we have sin [x] = sin 0 = 0
4
a = 2, b = −3 π
Again, x ∈ ( − , 0), we have,
4
140. Ans (C)
[x] = −1 ⇒ sin[x] = − sin 1
3a + 2b = 27
7. Ans (D)
R = {(1,12), (3,9), (5,6), (7,3)}
Number of relations from A to B is 2mn
141. Ans (B) Number of non – empty relations = 2mn – 1
142. Ans (C) 8. Ans (B)
(𝑥 − 2)(𝑥 − 5) > 0 The number of functions from the set A into set B
Now use TSR in nm, where n (A) = m and n (B) = n
Thus, the number of functions from A into A, is
MOCK TEST SOLUTIONS: 1010,
1. Ans (C)
(∵ n(A) = 10 = n(B))
y = ex and y = x never intersects
9. Ans (C)
2. Ans (B)
x 1 x−1
= ⇒y=
Use A  B = {(a, b) : a  A & b  B} x−1 y x
(P  Q)  (P  Q) = x−1
x−1 x
{(a, b) : a  P  Q & b  P  Q} f(y) = f ( )=
x x−1
−1
x
3. Ans (B)
x−1
We have, f(x) = 2x + |x| = =1−x
x−1−x
Now, f(2x) = 4x + |2x| = 4x + 2|x| 10. Ans (C)
f(−x) = −2x + |−x| = −2x + |x| f (2) = 2f ()
 f(2x) + f(−x) − f(x) ⇒ (2α)2 − 6α + 1 = 2(α2 − 3α + 1)
= 4x + 2|x| − 2x + |x| − 2x − |x| 1
⇒ 2α2 = 1 ⇒ α = ±
⇒ f(2x) + f(−x) − f(x) = 2|x| √2
4. Ans (A) 11. Ans (B)

We have, f(x) =
3
+ log10 (x 3 − x). √a2 − x 2 is defined for a2 – x2  0
4−x2
3 Now, a2 – x2  0  a2  x2  x2  a2
is defined for 4 – x2  0 i. e ., x   2
4−x2
KCET MATHEMATICS Page |3A. 12
RELATIONS AND FUNCTIONS

 –a x  a ⇒ x∈ [ – a, a] ⇒ x2 y − x + y = 0
12. Ans (B) For x to be real
We have, f (x) = x2 – 4x + 5 1 − 4y 2 ≥ 0
 x2 – 4x + 4 + 1  (x – 2)2 + 1 ⇒ (1 − 2y)(1 + 2y) ≥ 0
We know that (x – 2)2  0  (x – 2)2 + 1  1 1 1
⇒ ( − y) ( + y) ≥ 0
2 2
 Range: [1, )
1 1
13. Ans (C) ⇒ (y + ) (y − ) ≤ 0
2 2
In the given options only option (C) satisfies the 1 1
⇒ − ≤y≤
condition of a function. 2 2
1 1
Hence, option (C) is a function. ∴ y = f(x) ∈ [− , ]
2 2
14. Ans (A)
18. Ans (A)
For x ∈ (π, 3 π/2), we have
For f(x) to be defined
−1 < sin x < 0
x − 1 > 0 and 2x − 1 > 0 and 2x − 1 ≠ 1
⇒ 0 < 1 + sin x < 1 and 1 < (2 + sin x) < 2 1
⇒ x > 1, x > and x ≠ 1
2
∴ [sin x] = −1, [1 + sin x] = 0 and [2 + sin x] = 1
⇒x>1
⇒ f(x) = [sin x] + [1 + sin x] + [2 + sin x]
Hence, domain is (1, ∞).
= −1 + 0 + 1 = 0
19. Ans (D)
For x = π, we have
We have,
[sin x] = 0, [1 + sin x] = 1 and [2 + sin x] = 2
1 1 1
∴ f(x) = 0 + 1 + 2 = 3 f(x 2 ) = x 2 − 2
= (x − ) (x + )
x x x

For x = , we have 1
2 = (x + ) f(x)
x
[sin x] = −1, [1 + sin x] = 0 and
20. Ans (D)
[2 + sin x] = 1
Given, f(x) = √x
∴ f(x) = −1 + 0 + 1 = 0
Hence, range of f(x) = {0, 3} f(25) √25 5
∴ = = =1
f(16) + f(1) √16 + √1 4+1
15. Ans (C)
21. Ans (B)
f(a) = a
For domain of f(x), 2 − 2x − x 2 ≥ 0
αa2
⇒ =a ⇒ x 2 + 2x − 2 ≤ 0
α+1
1 ⇒ −1 − √3 ≤ x < −1 + √3
⇒ αa2 = a2 + a ⇒ α = 1 + (∵ a ≠ 0)
a 22. Ans (C)
16. Ans (C)
We have,
1
Given, f(x) = 1 1
√4−x2
f(x)f ( ) = f(x) + f ( ) ⇒ f(x) = x n + 1
x x
For domain of f(x),
Now,
⇒ 4 − x2 > 0
f(10) = 1001 ⇒ 10n + 1 = 1001 ⇒ n = 3
⇒ x2 < 4
∴ f(x) = x 3 + 1 ⇒ f(20) = 203 + 1 = 8001
⇒ −2 < x < 2
23. Ans (A)
∴ Domain= (−2, 2)
Given, y = x − 3 ⇒ x − y = 3
17. Ans (C)
x ∴ R = {(11, 8), (13, 10)}
Let y =
1+x2 ⇒ R−1 = {(8, 11), (10, 13)}
KCET MATHEMATICS Page |3A. 13
RELATIONS AND FUNCTIONS

24. Ans (A)


We have,
|x| 1, 0 < x ≤ 4
f(x) = ={
x −1, −4 ≤ x < 0
∴ Range (f) = {−1, 1}
25. Ans (C)
Given function is f(n) = 8−n Pn−4 , 4 ≤ n ≤ 6. It is
defined, if
1. 8 − n > 0 ⇒ n < 8 … (i)
2. n − 4 ≥ 0 ⇒ n ≥ 4 … (ii)
3. n − 4 ≤ 8 − n ⇒ n ≤ 6 … (iii)
From Eqs. (i), (ii) and (iii), we get n = 4, 5, 6
Hence, range of f(n) = { 4 P0 , 3 P1 , 2 P2 } = {1, 3, 2}
26. Ans (B)
∴ f(2.5) = [2.5 − 2] = [0.5] = 0
27. Ans (D)
Using T.S.R.
Range of f is [−√2, √2]
28. Ans (B)
We have,
x−1
f(x) =
x+1
f(x)+1 2x
⇒ = [Applying componendo-dividendo]
f(x)−1 −2

f(x) + 1
⇒x=
1 − f(x)
f(x) + 1
2 x − 1 2 {1 − f(x)} − 1 3 f(x) + 1
∴ f(2 x) = = =
2x+1 f(x) + 1 f(x) + 3
2{ }+1
1 − f(x)
29. Ans (A)
Given a set containing 10 distinct elements and
f: A → A Now, every element of a set A can make
image in 10 ways.
∴ Total number of ways in which each element
makes images = 1010 .
30. Ans (D)
since, f(x + y) = f(x) + f(y) and f(1)=7,
take f(x)=7x
Now, ∑ f(r)=∑ 7r
= 7(1 + 2 + 3+. . . +n)
7n(n + 1)
=
2

KCET MATHEMATICS Page |3A. 14


TRIGONOMETRY

HINTS AND SOLUTIONS

1. Ans (B) = 220 × 3600 = 792000 cm = 7.92 km.


Required radian measure 6. Ans (C)
π 60π π
=
180
× Degree measure Here, l = 37.4 cm and θ = 60∘ = radian =
180 3
π 26π l
= × 520 = Hence, by r = , we have
θ
180 9
37.4 × 3 37.4 × 3 × 7
2. Ans (B) r= = = 35.7 cm
π 22
We know that, 180∘ = π radian
7. Ans (C)
∘ ′ 1
Hence, 40 20 = 40 degree Since, length of arcs is equal.
3
π 121 121π
= × radian = radian. ∴ l1 = l2 = l (say)
180 3 540
121π Let the length of radius of two circles be r1 and r2 ,
Therefore, 40∘ 20′ = radian.
540
respectively.
3. Ans (B) π π
Also, given that θ1 = 60∘ × = radian and
In 60 minutes, the minute hand of a watch 180 3
π 5π
completes one revolution. Therefore, in 40 θ2 = 75∘ × = radian.
180 12
2 I
minutes, the minute hand turns through of a We know that, l = rθ ⇒ r =
3 θ
2 4π I1 I 3l
revolution. Therefore θ = × 360∘ or radian. ∴ r1 = =π=
3 3
θ1 π
Hence, the required distance travelled is given by 3
I2 l 12l
4π and r2 = = =
l = rθ = 1.5 × θ2 5π

3 12

= 2π = 2 × 3.14 = 6.28 cm. r1


3l
15 5
π
So, = 12l = =
r2 12 4
4. Ans (C) 5π

Length of wire = 2π × 3 = 6πcm and r = 48 cm is 8. Ans (B)


the radius of the hoop. Therefore, the angle θ (in Given radius, r = 100 cm and arc length, I =
radian) subtended at the centre of the hoop is 22 cm
given by We know that, l = rθ
Arc 6π π l 22
θ= = = = 22.5∘ θ= = = 0.22rad
Radius 48 8 r 100
180 180×7
5. Ans (A) = 0.22 × degree = 0.22 ×
π 22
Radius of the wheel = 35 cm 126 ∘
6 ∘
= = 12
∴ Circumference of the wheel 10 10
= 2π × 35 cm = 220 cm. 6
= 12∘ + × 60′ [∵ 1∘ = 60′ ]
10
Hence, the linear distance travelled by a point of
= 12∘ + 36′ = 12∘ 36′
the rim in one revolution = 220 cm. Number of
Hence, the degree measure of the required angle is
revolutions made by the wheel in 3 minutes
12∘ 36′ .
= 20 × 3 × 60 = 3600.
9. Ans (A)
∴ The linear distance travelled by a point of the
Given, length of pendulum = 75 cm
rim in 3 minutes
Radius (r) = length of pendulum
MATHEMATICS Page | 4A. 1
TRIGONOMETRY

= 75 cm (sin θ + cos θ)
=
Length of arc (l) = 21 cm sin θ ⋅ cos θ
l 21 7
1 1
Now, θ = = = radian = + = sec θ + cosec θ
r 75 25 cos θ sin θ
13. Ans (D)
In II quadrant, only sin θ and cosec θ are positive.
∴ cos 2 θ = (1 − sin2 θ)
9 16
= (1 − )=
25 25

16 −4
10. Ans (D) ⇒ cos θ = −√ =
25 5
Since, diameter of the circle = 40 cm
−5
∴ Radius of the circle (r) = 20 cm ∴ sec θ =
4
and length of the chord (l) = 20 cm cos θ −4 5 −4
and cot θ = = × =
sin θ 5 3 3
From the shown figure, it is clear that ∠AOB =
(−5) (−4)
π
60∘ = radian ∴ (2sec θ − 3cot θ) = {2 × −3× }
3 4 3
∴ Length of minor arc of the chord (l) = radius of −5 3
=( + 4) =
2 2
circle (r) × angle subtended in radian by the
14. Ans (D)
minor arc (θ)
In III quadrant, only tan θ and cot θ are positive.
π 20π
= 20 × = cm sin2 θ = (1 − cos 2 θ)
3 3
9 16
= (1 − )=
25 25

16 4
⇒ sin θ = ±√ =±
25 5
−4
⇒ sin θ =
5
11. Ans (D) −4 5 4
∴ tan θ = ( × )=
tan 10∘ tan 20∘ tan 30∘ tan 40∘ 5 −3 3
3 −5
tan 50∘ tan 60∘ tan 70∘ tan 80∘ and cot θ = ⇒ cosec θ =
4 4
= tan (90 − 80)∘ tan (90 − 70)∘ −5
and sec θ =
3
tan (90 − 60)∘ tan (90 − 50)∘
−5 3
tan 50∘ tan 60∘ tan 70∘ tan 80∘ (cosec θ + cot θ) ( 4 + 4)
∴ =
(sec θ − tan θ) −5 4
= tan 80∘ × cot 80∘ × tan 70∘ × cot 70∘ ( − )
3 3
× tan 60∘ × cot 60∘ × tan 50∘ −2
( ) −2 3 1
× cot 50∘ = 1 = 4 = × =
−9 4 −9 6
( )
12. Ans (A) 3
(sin θ + cos θ)(tan θ + cot θ) 15. Ans (D)
π π
sin θ cos θ Given, f(x) = sin [x], − < x <
= (sin θ + cos θ) [ + ] 4 4
cos θ sin θ
Clearly, sin 0 = 0
sin2 θ + cos 2 θ
= (sin θ + cos θ) ( ) π 3.14
sin θ ⋅ cos θ and [ ] = [ ]=0
4 4

(∵ sin2 θ + cos 2 θ = 1) π
∴ ∀x ∈ [ 0, ) , sin [x] = 0
4
MATHEMATICS Page | 4A. 2
TRIGONOMETRY
π (∵ cos 2 θ + sin2 θ = 1)
∀x ∈ [− , 0) , [x] = −1
4
21. Ans (D)
∴ sin [x] = sin (−1) = −sin 1
3 π
We have: sin A = , where 0 < A <
So, the range of f(x) is {0, −sin 1}. 5 2

16. Ans (C) ∴ cos A = ±√1 − sin2 A


3 4
cos A = , cos B = 9 4
5 5 ⇒ cos A = +√1 − sin2 A = √1 − =
25 5
∵ ∠A and ∠B lies on the 4 th quadrant.
[∵ cosA is positive in first quadrant ]
9 16
∴ sin A = −√1 − , sin B = −√1 − −12
25 25 It is given that: cos B =
13
4 3 3π
⇒ sin A = − , sin B = − and π < B <
5 5 2

4 3 ∴ sin B = ±√1 − cos 2 B


∴ 2sin A + 4sin B = 2 (− ) + 4 (− )
5 5
8 12 20 ⇒ sin B = −√1 − cos 2 B
=− − =− = −4
5 5 5 [∵ Sin is negative in the third quadrant ]
17. Ans (B)
−12 2 5
sin x sec x tan x ⇒ sin B = −√1 − ( ) =−
× × =9 13 13
cos x cosec x cot x
⇒ tan4 x = 9 Now, sin (A − B) = sin Acos B − cos Asin B
3 −12 4 −5 16
⇒ tan2 x = 3 = × − × =−
5 13 5 13 65
⇒ tan x = ±√3
22. Ans (B)
π π
⇒ x = ∈ (0, ) 1
Given, cot α = ⇒ tan α = 2 and sec β =
−5
3 2 2 3
18. Ans (C) Then, tan β = √sec 2 β − 1
Since, −1 ≤ cos θ ≤ 1
25 16
⇒ −5 ≤ 5cos θ ≤ 5 ⇒ tan β = ±√ − 1 = ±√
9 9
⇒ −5 + 12 ≤ 5cos θ + 12 < 5 + 12
4 −4
⇒ 7 ≤ 5cos θ + 12 < 17 ⇒ tan β = ± But, tan β =
3 3

[∵ tan β is negative in II nd
Hence, minimum value is 7 quadrant ]
19. Ans (A) tan α + tan β
∴ tan(α + β) =
1 − tan α ⋅ tan β
π π 2π
(1 + cos ) (1 + cos ) (1 + cos ) (1 4
6 3 3 2+(−3) 2
= −4 = .
1−(2)( ) 11
7π 3
+ cos )
6 23. Ans (D)
√3 1 1 √3 1
= (1 + ) (1 + ) (1 − ) (1 − ) √3
2 2 2 2 −
sin 10∘ cos 10∘
3 1 1 3 3 [cos 10∘ − √3sin 10∘ ]
= (1 − ) (1 − ) = × = =
4 4 4 4 16 sin 10∘ cos 10∘
20. Ans (B) 1 √3
2 [ cos 10∘ − sin 10∘ ]
Given equations can be rewritten as 2 2
=
a b sin 10∘ cos 10∘
cos θ = , sin θ = 2[sin 30∘ cos 10∘ − cos 30∘ sin 10∘ ]
x−h y−k
=
a2 b2
sin 10∘ cos 10∘
Now, + =1 2[sin (30∘ −10∘ )] 2⋅2sin (30∘ −10∘ )
(x−h)2 (y−k)2
= = = 4.
sin 10∘ cos 10∘ 2sin 10∘ cos 10∘

MATHEMATICS Page | 4A. 3


TRIGONOMETRY

24. Ans (C) ⇒ 2cos 2 − cos x − 1 = 0


1 + cos 2x + cos 4x + cos 6x ⇒ cos x = 1/2 or 1
= (1 + cos 6x) + (cos 2x + cos 4x) ∴ x = 2nπ or x = 2nπ + π/3
= 2cos 2 3x + 2cos 3xcos x 30. Ans (C)
= 2 cos 3x(cos 3x + cos x) 3π 3π
cos ( + x) − cos ( − x)
= 4cos xcos 2xcos 3x 4 4
3π 3π 3π 3π
25. Ans (D) +x+ −x +x− +x
= −2sin 4 4 sin 4 4
1 2 2
sin x + cos x = 3π
5 = −2sin sin x
1 4
⇒ sin2 x + cos 2 x + 2 sin x cos x = π
25 = −2sin (π − ) sin x
4
−24 −7
sin 2x = ⇒ cos 2x = π 1
25 25 = −2 sin sin x = −2 × sin x
4 √2
24
⇒ tan 2x =
7 = −√2sin x
26. Ans (A) Alternatively, put x= π/4 and verify the options
sin (x − y) = 1/2 31. Ans (D)
π 5π 1 1
⇒ x − y = or Since, sin x ⋅ cos x = sin 2x ≤
6 6 2 2
cos (x + y) = 1/2 (∵ |sin 2x| ≤ 1)
π 5π 32. Ans (A)
⇒x+y= ,
3 3
We have,
∴ After solving we get x = 45∘ , y = 15∘
sin (40∘ + θ) ⋅ cos (10∘ + θ) − cos (40∘ + θ)
27. Ans (A)
⋅ sin (10∘ + θ)
Roots will be of opposite sign if product of roots is
= sin {(40∘ + θ) − (10∘ + θ)}
negative
1
1 = sin 30∘ =
⇒ sin θ < 2
2
33. Ans (A)
28. Ans (B)
sin2 6x − sin2 4x
q
tan α + tan β = , = sin (6x + 4x)sin (6x − 4x)
p
r = sin 10xsin 2x
tan α . tan β = , p ≠ 0
p [∵ sin2 A − sin2 B]=sin (A + B)sin(A-B)
q
p q 34. Ans (B)
tan (α + β) = r = p−r
1−( ) cos 2 45∘ − sin2 15∘
p
= cos (45∘ + 15∘ )cos (45∘ − 15∘ )
29. Ans (A)
∵ cos 2 A − sin2 B
Given equation is [ ]
= cos (A + B)cos (A − B)
sin x + sin 5x = sin 2x + sin 4x
1 √3 √3
⇒ 2sin (3x)cos (2x) = 2sin (3x)cos x = cos 60∘ cos 30∘ = × =
2 2 4
⇒ sin (3x)[cos (2x) − cos x] = 0 35. Ans (A)
Either sin (3x) = 0 ⇒ 3x = nπ We know that,
⇒ x = nπ/3 x+y x−y
cos x + cos y = 2cos ⋅ cos
Or cos (2x) − cos x = 0 2 2

MATHEMATICS Page | 4A. 4


TRIGONOMETRY

x+y x−y 2cos 3xcos x + cos 3x


sin x − sin y = 2cos ⋅ sin =
2 2 2sin 3xcos x + sin 3x
cos 7x+cos 5x
Thus, we have cos 3x(2cos x + 1)
sin 7x−sin 5x = = cot 3x
sin 3x(2cos x + 1)
7x + 5x 7x − 5x
2 cos cos
= 2 2
7x + 5x 7x − 5x
2 cos sin 41. (B)
2 2
cos x 1
Use θ = |60x − 11y|, where x=4 and y=20
= = cot x 2
sin x
36. Ans (D) 42. (B)
1
sin x − sin 3x 2cos 2x ⋅ sin (−x) Use θ = |60x − 11y|, where x=2 and y=15
2
2 2
=
sin x − cos x −cos 2x
43. (B)
= 2sin x
Perimeter of sector: l+2r=r θ+2r
37. Ans (C)
Length of arc of semicircle =πr.
π
cos θ sin ( − θ) By given data, r θ +2r= πr
= 2
1 + sin θ 1 + cos (π − θ) ⇒ θ= π − 2
2
π θ π θ
2sin ( − ) cos ( − )
44. (B)
= 4 2 4 2
π θ l = rθ, where l=44m, θ = 72°
2
2cos ( − )
4 2 l 44
∴ r = = 2π =35m
π θ θ
5
sin ( − )
= 4 2 = tan (π − θ)
π θ 4 2
45. (B)
cos ( − )
4 2 1:2:3=30°:60°: 90°
38. Ans (D) 46. (C)
∘ ∘
We have, sin θ = sin 15 + sin 45 By given data,
15 + 45 ∘ 15 − 45 ∘ l=Circumference=2π=2π(20)=40πcm
sin θ = 2sin ( ) cos ( )
2 2
l 40π 2π
= 2sin 30∘ cos 15∘ θ= = =
r 100 5

1 47. (C)
= 2 × cos 15∘ = cos 15∘
2 Area of circle = πr2=9 π ⇒r=3cm
∘ ∘
⇒ sin θ = sin (90 − 15) ⇒ θ = 75 π
l = rθ=3. =π
3
39. Ans (D)
So Req. perimeter of sector=l+2r=π + 6
sin 55∘ − sin 35∘
sin 10∘ 48. (C)
2cos 45∘ ⋅ sin 10∘ For 1min=120 revolutions
= = √2
sin 10∘ ⇒2 sec=4 revolutions
[∵ sin C − sin D
Now, for 1 revolution angle turned is
C+D C−D
= 2cos sin ] 4× 2π=8π revolutions.
2 2

40. Ans (B) 49. (D)


cos 4x + cos 3x + cos 2x l=24π cm, θ = 72° =

5
sin 4x + sin 3x + sin 2x
l 24π
(cos 4x + cos 2x) + cos 3x r= = × 5 = 60 cm
θ 2π
=
(sin 4x + sin 2x) + sin 3x
50. (B)
4x+2x 4x−2x
2cos cos 2 +cos 3x
= 2
4x+2x 4x−2x (By transformation 1min==180 revolution
2sin cos +sin 3x
2 2 180
⇒ 1 sec= = 3 revolution.
formulae) 60

MATHEMATICS Page | 4A. 5


TRIGONOMETRY

For, 1 revolution angle turned is =2π 57. (B)


⇒ For,3 revolutions angle turned is π
A+B=
2
=3× 2π = 6π
⇒ cos 2 A + cos 2 B = 1
51. (C)
R. V = cos 2 5 + cos 2 10 + ⋯ . +cos 2 85 + cos 2 90
60′ = 360∘
1′ = 60 1
=8
θ = 250 × 60 = 1500 2
π 5 22 55 58. (D)
θ = 150 × = × =
180 6 7 21
ℓ = rθ 1
= 4 + 2√3
55 1 − sin x
= 21 × = 55 1
21 1 − sin x =
4 + 2√3
52. (A)
π
4 − 2√3
A−C= =
9 4
A − C = 20∘ √3
A + B + C = 180∘ 1 − sin x = 1 −
2
A + 90∘ + C = 180∘ (∵ B = 90∘ ) √3
A + C = 90∘ sin x =
2
A + C + A − C = 900 + 200 π π
2A = 110∘ x = ( or )π −
110∘
3 3
A= = 55∘
2 59. (B)
C = 35∘
Given,(sin α + cosec α)2 + (sec α + cos α)2
53. (D)
= k + tan2 α + cot 2 α
Interior angles of regular pentagon = 1080
π ⇒ sin2 α + cosec 2 α + 2cosecαsinα + sec 2 α
= 108 ×
180
3π + cos 2 α + 2secα. cosα = k + tan2 α + cot 2 α
= .
5

54. (D)
⇒ 1 + cosec 2 α + 2(1) + sec 2 α
A⋅M⩾ G⋅M
1 +2(1) = k + tan2 α + cot 2 α
cos θ = x + ⩾ 2
x ⇒ 5 + 1 + cot 2 α + 1 + tan2 α
but maximum value of cos θ is 1
= k + tan2 α + cot 2 α
⇒k=7
60. (D)
55. (C)
sin θ1 + sin θ2 + sin θ3 = 3
2P6 − 3P4 + 1 ⇒ sin θ1 = sin θ2 = sin θ3 = 1
= 2(cos 6 θ + sin6 θ) − 3(cos 4 θ + sin4 θ) + 1 π
⇒ θ1 = θ2 = θ3 =
2
= 2[1 − 3sin2 αcos 2 α] − 3[1 − 2 cos 2 α sin2 α] + 1 ∴ cos θ1 + cos θ2 + cos θ3 = 0
= 2−3+1 =0 61. (A)
56. (B) sin θ + cosec θ = 2
3 ⇒ sin θ = 1 = cos ecθ
sec θ + tan θ = ∴ sinn θ + cos ec n θ = 1 + 1 = 2
2
2 62. (A)
sec θ − tan θ =
3 AM ≥ GM
3 2 5 Tan2 α + cot 2 α ≥ 2
2tan θ = − =
2 3 6 63. (C)
5
tan θ =
12

MATHEMATICS Page | 4A. 6


TRIGONOMETRY

Q ∈ I quadrant −4
tan θ =
sec θ + tan θ = p 3
1 ⇒ θ ∈ IIQ(or)IVQ
sec θ − tan θ =
p 4 −4
1 ⇒ sin θ = (or)
2sec θ = p + 5 5
p
p2 +1 72. (A)
sec θ =
2p
We can write,
64. (B)
Sin190=sin(180+10)= -sin10
acos θ + bsin θ = p → (1)
asin θ − bcos θ = q → (2) Sin200=sin(180+20)= -sin20
(1)2 + (2)2 ⇒ a2 + b2 = p2 + q2 ...... .......... .............
65. (A) ...... .......... .............

sin θ = (√2 − 1)cos θ Sin350=sin (180+170) = -sin170


(√2 + 1)sin θ = cos θ Finally, Sin 360=0
√2sin θ = cos θ − sin θ So, sin 10∘ + sin 20∘ + ⋯ + sin 340∘ + sin 350∘ +
66. (D) sin 360∘ = 0
0 ⋅tan 20 …….tan 890 )
Req. Value = elog10 (tan 1
1
= elog10 = e0 = 1 73. (A)
π
A + B = ⇒ tan A ⋅ tan B = 1
2 sin2 x(1 − cos 2 x) + cos 2 x
f(x) =
67. (A) sin2 x + cos 2 x(1 − sin2 x)
π sin x − sin2 xcos 2 x + cos 2 x
2

A+B= =
2 sin2 x + cos 2 x − sin2 xcos 2 x
⇒ cot A ⋅ cot B = 1 1 − sin2 xcos 2 x
=1
Req. Value = 1 1 − sin2 xcos 2 x
f(x) = 1
68. (D) f(2023) = 1
sin x = cos 2 x
74. (A)
now, cos 2 x + cos 4 x
= cos 2 x + sin2 x = 1 cos θ = 1
sec θ = 1
69. (B) cos 999 θ − sec 999 θ = 1 − 1 = 0.
π
A+B= ⇒ sin2 A + sin2 B = 1 75. (A)
2
sin A 1 + cot A
∴ K = sin2 50 + sin2 100 + ⋯ + sin2 80∘ [ ]
sin A 1 − cot A
+ sin2 850 3
1+
1 1 = 4 = 7.
= 8+ =8 3
2 2 1−
4
70. (A) 76. (C)
cot α1 ⋅ cot α2 … … ⋅ cot αn = 1 sin (6(180∘ ) + 60∘ )
π
⇒ α1 = α2 … … αn = √3
4 = sin 60∘ =
∴ max of 2
cos α1 ⋅ cos α2 … … ⋅ cos αn 77. (C)
1 1 1
= ⋅ …⋯⋅ (n times ) cos 4 x + cos 2 x = 1
√2 √2 √2 (cos 4 x + cos 2 x)2 = 1
1 cos 8 x + 2cos 6 x + cos 4 x = 1
= n/2
2
78. (D)
71. (B)
Cos 1> cos2
cos x is decreasing in (0, π)

MATHEMATICS Page | 4A. 7


TRIGONOMETRY

A is false, but R is true. π


0<α<
2
π
0<β<
2
−π π
⇒ 0 < α + β < π, <α−β<
2 2
4 5
cos (α + β) = , sin (α − β) =
5 13
⇒ α + β ∈ IQ, α − β ∈ IQ
cos 2α = cos [(α + β) + (α − β)]
79. (D) = cos (α + β)cos (α − β) − sin (α + β)sin (α − β)
4 12 3 5
WKT,1c = 570 , 2c = 1140 , 3c = 1710 = × − ×
5 13 5 13
Hence z < x < y 48 − 15 33
= =
65 65
80. (A)
83. (C)
A, B ∈ IQ
cos θ+sin θ
cos (A + B) = cos Acos B − sin A ⋅ sin B WKT = tan (450 + θ)
cos θ−sin θ
12 3 5 4
× − × (when θ = 90 )
13 5 13 5 = tan (540 )
36 − 20 16
= = 84. (B)
65 65
36 − 20 16 cos 2 33∘ − cos 2 57∘ sin 90∘ ⋅ sin 24∘
= = =
65 65 ∘
sin 21 − sin 69 ∘ −2cos 45∘ ⋅ sin 24∘
−1
=
√2
85. (B)
Tan A
Tan (A − B) =
1 + Tan A
81. (A)
3√3
4√3 −
50 + 20 = 700 = 13
Tan 50∘ + tan 20∘ 4√3 × 3√3
1+
= Tan 70∘ 13
1 − Tan 500 ⋅ tan 200 √3(52 3)

Tan 50∘ + Tan 20∘ = Tan 70∘ − Tan 50∘ = = √3
13 + 36
2Tan 50∘ = Tan 70∘ − Tan 20∘
Tan 70∘ − Tan 20∘
=2 π
Tan 50∘ A − B = 60∘ =
3

82. (B)

86. (A)
cos αcos β 2
=
sin αsin β 1
Using Componendo and Dividendo.
cos (α − β) 2 + 1
= =3
cos (α + β) 2 − 1
87. (A)

MATHEMATICS Page | 4A. 8


TRIGONOMETRY

1 1 2tan β/2
sin Acos B = tan α = , tan β =
4 3 1 − tan2 β/2
3sin Acos B = sin Bcos A 1 4
3 = =
sin Bcos A = 1 3
4 1−
4
1 3 tan α + tan β
sin (A + B) = + = 1 tan (α + β) =
4 4 1 − tan α ⋅ tan β
π
A+B= 1 4
2 + 5×3
=3 3= =3
88. (B) 4 5
1−
9
√3sin θ + cos θ 93. (B)
√3 1 π
= 2 [ sin θ + cos θ] = 2sin (θ + ) > 0 sin 120 ⋅ sin 480 ⋅ sin 540
2 2 6
sin 120 sin (600 − 120 )sin (60∘ + 12)sin 540
π =
0<θ+ <π sin 720
6
−π 5π 1 sin 360 cos 360 1
<θ< . = =
6 6 4 sin 720 8

89. (B) 94. (B)


m 1 64
sin (π)
Tan α = , Tan β = Req, Value = 65
m−1 2m − 1 π
m − (m − 1) 26 sin
65
= π
m + (m − 1) sin (π − )
= 65 = 1
Tan α − 1 π
= 64. sin ( ) 64
Tan α + 1 65
π
β= α− 95. (A)
4
π π 2π 4π
α−β= . cos ⋅ cos ⋅ cos
4 7 7 7
90. (C) 8π
sin
= 7 = −1
π 5π 7π π 8
sin ⋅ sin ⋅ sin 8sin
7
18 18 18
sin 10∘ ⋅ sin 50∘ ⋅ sin 70∘ 96. (A)
sin 10∘ ⋅ sin (60∘ − 10∘ ) ⋅ sin (60∘ + 10∘ ) 1
1 1 tan α =
= sin (3(10∘ )) = sin 30∘ 7
4 4 1
1 sin β =
= √10
8
1
91. (B) ⇒ tan β =
3
π
b 0<α<β<
tan θ = 2
a 2tan β
a(1 − tan2 θ) b2tan θ tan 2β =
acos 2θ + bsin 2θ = + 1 − tan 2
2
1 + tan θ 1 + tan2 θ 1

3=6
b2 b =
1 8
a [1 −
] 2b 1−
a2 a 9
= +
b2 b 2
7−1 π
1+ 2 1+( ) = = tan ( − α)
a a 7+1 4
a(a2 − b2 ) 2b2 a π
= + 2 2β = −α
2
a +b 2 a + b2 4

a(a2 + b2 ) 97. (D)


= =a θ+ϕ θ−ϕ
a2 + b 2
2 sin ( ) ⋅ cos ( )
92. (D) 2 2
θ+ϕ θ−ϕ
= 2√3sin ( ) ⋅ sin ( )
2 2
MATHEMATICS Page | 4A. 9
TRIGONOMETRY

θ+ϕ sin 470 − sin 25∘ + sin 61∘ − sin 110


⇒ sin ( )=0
2 = 2cos 36 ⋅ sin 11 + 2cos 36 ⋅ sin 25
= 2cos 36(sin 25∘ + sin 11∘ )
⇒ θ+ϕ =0
= 4cos 360 ⋅ sin 180 ⋅ cos 70
ϕ = −θ √5 + 1 √5 − 1
=4× × × cos 70
∴ sin 3θ + sin 3ϕ = 0 4 4
4
98. (D) = × cos 70 = cos 70
4
103. (B)
√2 + √2 + √2 + 2cos 8θ
sin A + sin 3A + sin 3A + sin 5A
= 2sin 2A ⋅ cos A + 2sin 4A ⋅ cos A
= √2 + √2 + √2(2cos 2 4θ) = 2cos A(2sin 3A ⋅ cos A)
= 4cos 2 Asin 3A.
= √2 + √2(2cos 2 2θ) 104. (B)

= √2(2cos 2 θ) = 2cos θ A+B+C= π


sin2 A − sin2 B + sin2 C
99. (C) = sin (A + B)sin (A − B) + sin2 C
π 2π = sin C(sin (A − B) + sin (A + B))
sin 32 sin (3π + ) = 2sin Csin Acos B
10 = 10
π π
32sin 32sin 105. (B)
10 10
2π π π x+y x−y 3
− sin −2 sin cos 2sin ⋅ cos
= 10 10 10 2 2 4
π = π x+y x−y = 2
32 sin 32 sin 2cos ⋅ sin
10 10 2 2 5
−1 π x+y
= cos Tan 15
16 10 2
x−y = 8
Tan ( )
100. (A) 2
sin 2x n 106. (A)
=
sin 2y 1 −2sin 2α ⋅ sin α2 ⋅ sin 5α ⋅ cos 3α
using C and D =1
−2cos 3α ⋅ sin 2α ⋅ 2sin 5α ⋅ sin α
sin 2x + sin 2y n + 1
= 107. (B)
sin 2x − sin 2y n − 1
2sin (x + y) ⋅ cos (x − y) n + 1 A−B A − B 2024
= cot 2024 ( ) + (−cot )
2cos (x + y)sin (x − y) n−1 2 2
tan (x + y) n + 1 A−B
= = 2 ⋅ cot 2024 ( )
tan (x − y) n − 1 2

101. (B)
108. (A)

sin 70∘ + cos 40∘ cos 5x + cos x = 0


2cos 3x ⋅ cos 2x = 0
cos 70∘ + sin 402
⇒ cos 3x = 0( or )cos 2x = 0
sin 70∘ + sin 50∘
=
sin 20∘ + sin 40∘
2sin 60∘ cos 10∘
= = √3 π π
2sin 30∘ cos 10∘ ⇒ 3x = ( or )2x =
2 2
102. (D) π π
⇒ x = ( or )x =
6 4
109. (D)

MATHEMATICS Page | 4A. 10


TRIGONOMETRY

tan (θ + 15∘ ) 116. (D)


=3
tan (θ − 15∘ ) sin2 θ + 3cos θ = 3
usin gC and D 1 − cos 2 θ + 3cos θ − 3 = 0
tan (θ + 15∘ ) + tan (θ − 15∘ ) cos 2 θ − 3cos θ + 2 = 0
tan (θ + 15∘ ) − tan (θ − 15∘ ) cos θ = 1
4 θ=0
= =2
2 117. (A)
sin (2θ)
=2 2cos 2 θ − 1 = cos 2 θ
sin (30∘ )
cos 2 θ = 1
sin 2θ = 1
π θ = nπ
2θ =
2 118. (A)
π
θ= 2sin2 x + 6sin x − sin x − 3 = 0
4
2sin x(sin x + 3) − 1(sin x + 3) = 0
110. (C) (2sin x − 1)(sin x + 3) = 0
sin2 x √3sin x 1
+ =0 sin x =
cos x cos x 2
tan x[sin x + √3] = 0 Number of solution x = 4
tan x = 0(∵ sin x ≠ −√3) 119. (C)
x = nπ, n ∈ Z 3sin2 x − 7sin x + 2 = 0
111. (D) 3sin2 x − 6sin x − sin x + 2 = 0
(3sin x − 1)(sin x − 2) = 0
sin x + cos x = 2 1
Sq. on both sides ⇒ sin x =
3
1 + sin 2x = 4
sin 2x = 3 120. (A)
sol does not exist (sinx − 2)(2sinx − 1) > 0
112. (A) 1
(sinx − 2) (sinx − ) > 0
2
1 − sin θ + sin θ + 1 = 0 2
sin2 θ − sin θ − 2 = 0 Since always (sinx − 2) < 0
sin2 θ − 2sin θ + sin θ − 2 = 0 1
(sin θ + 1)(sin θ − 2) = 0 We have sinx − < 0
2
⇒ sin θ = −1 1
5π 7π ⇒ sinx <
θ∈( , ) 2
4 4
1
113. (A) ⇒ sin x ϵ (0, )
2
π 5π
sin 7θ + sin θ = sin 4θ ⇒x ϵ(0, ) ∪ ( , 2π)
6 6
2sin 4θ ⋅ cos 3θ = sin 4θ
1 We can get the answer directly without performing
⇒ sin 4θ = 0( or )cos 3θ =
2 above steps (graphical method).
π
⇒ 4θ = 0, π( or )3θ =
3
π π
⇒ θ = (or) θ =
4 9
114. (A)
tan2 x = −2sin2 x
sin2 x[sec 2 x + 2] = 0
⇒ sin2 x = 0
⇒ x = nπ, n ∈ Z
115. (D)
1 √3 2
sin x + = +
sin x 2 √3
√3
⇒ sin x =
2
⇒ x = 60∘
MATHEMATICS Page | 4A. 11
TRIGONOMETRY
π
∴ θ+ϕ =
4
121. (B) 127. (C)
[sin 5θ + sin θ] − sin 3θ = 0 1
sin θ = − is correct. ∵ −1 ≤ sin θ ≤ 1
2sin 3θ ⋅ cos 2θ − sin 3θ = 0 5
sin 3θ[2cos 2θ − 1] = 0 So (A) is correct.
2cos 2θ − 1 = 0 or sin 3θ = 0
1 cos θ = 1 is correct. ∵ cos 0∘ = 1
cos 2θ = or 3θ = nπ
2 So (B) is correct.
π nπ
2θ = or θ = , for, n = 1 1
sec θ = ⇒ cos θ = 2 is not correct.
3 3 2
π
or θ = ∵ −1 ≤ cos θ ≤ 1
3
π π
θ = ∈ (0, ) So, (C) is not correct.
6 2
tan θ=20 is also correct as -∞ ≤tan θ ≤ ∞
122. (A)
So, (D) is also correct.
2sin (ex ) ⩾ 2
sin (ex ) ⩾ 1
π
sin (ex ) = 1 ⇒ ex = 128. (B)
2
π Given that: tan 1∘ tan 2∘ tan 3∘ … . tan 89∘
x = log e ( )
2
= tan 1∘ tan 2∘ tan 3∘ … ⋅ tan 45∘ ⋅ tan (90 − 44∘ )
5x + 5−x
= 1 at x = 0 ⋅ tan (90 − 43∘ )
2
so solution doesnot exist = tan 1∘ cot 1∘ ⋅ tan 2∘ ⋅ cot 2∘ ⋅ tan 3∘
123. (C)
⋅ cot 3∘ … tan 89∘ ⋅ tan (99∘ − 1∘ )
cosecθ ⋅ sec θ = 1
= tan 1∘ cot 1∘ ⋅ tan 2∘ ⋅ cot 2∘ ⋅ tan 3∘ ⋅
sin θ ⋅ cos θ = 1 ⇒ sin 2θ = 2 > 1
cot 3∘ … tan 89∘ ⋅ cot 89∘ = 1.1.1.1 … 1.1 = 1
has no solution
129. (C)
124. (C) 1−tan2 15∘
Given that:
If sin θ + cosec θ = 2, then 1+tan2 15∘

sinn θ + cosec n θ = 2 Let θ = 15∘ ∴ 2θ = 30∘


1 − tan2 θ
125. (D) cos 2θ =
1 + tan2 θ
Given that: f(x) = cos 2 x + sec 2 x 1 − tan2 15∘
cos 30∘ =
We know that AM ≥ GM 1 + tan2 15∘
cos2 x+sec2 x √3 1 − tan2 15∘
⇒ ≥ √cos 2 x ⋅ sec 2 x ⇒ =
2
2 1 + tan2 15∘
cos x + sec 2 x
2
⇒ ≥1 130. (B)
2
Given expression is
⇒ cos 2 x + sec 2 x ≥ 2
cos 1∘ ⋅ cos 2∘ ⋅ cos 3∘ … cos 179∘
⇒ f(x) ≥ 2
⇒ cos 1∘ ⋅ cos 2∘ ⋅ cos 3∘ … cos 90∘
126. (D)
⋅ cos 91∘ … cos 179∘
We know that
=0 [∵ cos 90∘ = 0]
tan θ + tan ϕ
tan(θ + ϕ) = 131. (C)
1 − tan θ tan ϕ
1 1 5 tan θ = 3, θ lies in third quadrant
+
= 2 3 = 6 =1
∴ sin θ =
−3
where θ lies in third quadrant
1 1 6 √10
1− ×
2 3 6
π
⇒ tan (θ + ϕ) = tan
4

MATHEMATICS Page | 4A. 12


TRIGONOMETRY

Minimum value of y − 8 = −√(3)2 + (4)2


⇒ y−8 = −√9 + 16 = −5

⇒ y= 8−5 =3
132. (A) So, the minimum value of the given expression is 3 .
tan 75∘ − cot 75∘ 136. (A)
∘ ∘)
= tan 75 − cot (90 − 15 The given expression is
sin 75∘ sin 15∘
= tan 75∘ − tan 15∘ = − tan 3A − tan 2A − tan A
cos 75∘ cos 15∘
sin 75 cos 15 − cos 75 sin 15∘
∘ ∘ ∘ tan 3 A = tan (2 A + A)
=
cos 75∘ cos 15∘ tan 2 A + tan A
sin (75∘ − 15∘ ) ⇒ tan 3 A =
= 1 − tan 2 Atan A
1
× 2cos 75∘ cos 15∘ ⇒ tan 3 A(1 − tan 2 Atan A)
2
2sin 60∘ = tan 2 A + tan A
=
cos (75∘ + 15∘ ) + cos (75∘ − 15∘ ) ⇒ tan 3 A − tan 3 Atan 2 Atan A
√3 = tan 2 A + tan A
2× √3
= 2 = = 2√3
cos 90∘ + cos 60∘ 0 + 1 ⇒ tan 3 A − tan 2 A − tan A
2
= tan 3 Atan 2 Atan A
133. (B)
137. (D)
We know that if θ increases then the value of sin θ
sin (45∘ + θ) = sin 45∘ cos θ + cos 45∘ sin θ
also increases
1 1
π = cos θ + sin θ
So sin 1∘ < sin 1 [∵ 1 radian = sin 1]
180 √2 √2
134. (D) cos (45∘ − θ) = cos 45∘ cos θ + sin 45∘ sin θ

Given that tan α =


m
and tan β =
1 1 1
m+1 2m+1 = cos θ + sin θ
√2 √2
tan α + tan β
tan (α + β) = sin (45∘ + θ) − cos (45∘ − θ)
1 − tan αtan β
1 1 1 1
m 1 = cos θ + sin θ − cos θ − sin θ
+ √2 √2 √2 √2
= m + 1 2m +1
m 1 138. (C)
1− ×
m + 1 2m + 1 π π
2 cot ( + θ) ⋅ cot ( − θ) =
2m + m + m + 1 4 4
(m + 1)(2m + 1) π π
= cot cot θ − 1 cot cot θ + 1
(m + 1)(2m + 1) − m 4 × 4
(m + 1)(2m + 1) π π
cot θ + cot cot θ − cot
4 4
2m2 + 2m + 1 1 ⋅ cot θ − 1 1 ⋅ cot θ + 1
= = ×
2m2 + 2m + m + 1 − m cot θ + 1 cot θ − 1
2m2 + 2m + 1 cot θ − 1 cot θ + 1
= =1 = × =1
2m2 + 2m + 1 cot θ + 1 cot θ − 1
⇒ tan (α + β) 139. (B)
π π Given that:
= tan ∴α+β=
4 4
cos 2θcos 2ϕ + sin2 (θ − ϕ) − sin2 (θ + ϕ)
135. (D)
cos 2θcos 2ϕ + sin2 (θ − ϕ) − sin2 (θ + ϕ)
The given expression is 3cos x + 4sin x + 8
= cos 2θcos 2ϕ + sin (θ − ϕ + θ + ϕ) ⋅ sin (θ
Let y = 3cos x + 4sin x + 8
− ϕ − θ − ϕ)
⇒ y − 8 = 3cos x + 4sin x
[∵ sin2 A − sin2 B]
MATHEMATICS Page | 4A. 13
TRIGONOMETRY

= sin (A + B) ⋅ sin (A − B) = −sin 18∘ ⋅ cos 36∘


= cos 2θcos 2ϕ + sin 2θ ⋅ sin (−2ϕ) √5 − 1 √5 + 1
= −( )( )
= cos 2θcos 2ϕ − sin 2θsin 2ϕ 4 4
= cos (2θ + 2ϕ) = cos 2(θ + ϕ) √5 − 1 √5 + 1
[∵ sin 18∘ = , cos 36∘ = ]
[∵ sin (−θ) = −sin θ] 4 4

140. (C) 5−1 1


= −( )=−
16 4
The given expression is cos 12∘ + cos 84∘ +
143. (B)
cos 156∘ + cos 132∘ (cos 132∘ + cos 12∘ ) +
Given, (sin 50∘ − sin 70∘ ) + sin 10∘
(cos 156∘ + cos 84∘ )
50∘ +70∘ 50∘ −70∘
132∘ +12∘ 132∘ −12∘ = 2cos ⋅ sin + sin 10∘
= (2 cos ⋅ cos )+ 2 2
2 2 = 2cos 60 ⋅ (−sin 10 + sin 10∘
∘ ∘)
156∘ +84∘ 156∘ −84∘ 1
(2cos ⋅ cos ) = −2 × sin 10∘ + sin 10∘
2 2 2
∘ ∘ ∘ ∘ = −sin 10∘ + sin 10∘
= 2cos 72 ⋅ cos 60 + 2cos 120 ⋅ cos 36
=0
1 1
= 2 cos 72∘ × + 2 × (− ) cos 36∘ 144. (C)
2 2
Given that: sin θ + cos θ = 1
= cos 72∘ − cos 36∘
⇒ (sin θ + cos θ)2 = (1)2
= cos(90∘ − 18∘ ) − cos 36∘
⇒ sin2 θ + cos 2 θ + 2sin θcos θ = 1
= sin 18∘ − cos 36∘
⇒ 1 + sin 2θ = 1 ⇒ sin 2θ = 1 − 1 = 0
√5 − 1 √5 + 1
= − 145. (B)
4 4
π
[∵ sin 18∘ =
√5−1
, cos 36∘ =
√5+1
] Given that: α + β =
4
4 4
tan α + tan β
√5 − 1 − √5 − 1 −1 ⇒ =1
= = 1 − tan αtan β
4 2
⇒ tan α + tan β = 1 − tan αtan β
141. (C)
1 1 ⇒ tan α + tan β + tan αtan β = 1
Given that: tan A = and tan B =
2 3
⇒ 1 + tan α + tan β + tan αtan β = 1 + 1
2 tan A
tan 2 A = ⇒ 1(1 + tan α) + tan β(1 + tan α) = 2
1 − tan2 A
1 ⇒ (1 + tan α)(1 + tan β) = 2

= 2 = 1 = 1 =4
1 2 1−1 3 3
1−( ) 4 4 146. (C)
2
4 1 4
So, tan 2 A = and tan B = Given that: sin θ = − , θ lies in third quadrant
3 3 5
tan 2 A + tan B
tan (2 A + B) =
1 − tan 2 A ⋅ tan B 4 2
cos θ = √1 − sin2 θ = √1 − (− )
5
4 1 5
+
= 3 3 = 9 − 4 =5×9=3 16 9 +3
4 3 9 3 5 = √1 − =√ =
1− × 25 25 −5
3 3
142. (C) 3
∴ cos θ = − , θ lies in third quadrant
5
π 13π π 3π
sin ⋅ sin = sin ⋅ sin (π + ) θ
10 10 10 10 cos θ = 2cos 2 − 1
2
π 3π
= sin ⋅ (−sin ) = −sin 18∘ ⋅ sin 54∘ 3π π θ 3π
10 10 [∵ π < θ < ,∴ < < ]
2 2 2 4
= −sin 18∘ ⋅ sin (90∘ − 36∘ )

MATHEMATICS Page | 4A. 14


TRIGONOMETRY

−3 θ
⇒ = 2cos 2 − 1
5 2
θ 3 2
⇒ 2cos 2 = 1 − =
2 5 5
θ 2 1 θ 1
⇒ cos 2 = = ⇒ cos = ±
2 5×2 5 2 √5
θ 1 π θ 3π
⇒ cos =− [∵ < < ]
2 √5 2 2 4
147. (C) 4 −3
And sin A = and cot A =
Given equation is tan x + sec x = 2cos x 5 4

sin x 1 ∴ 2 cot A − 5 cos A + sin A


⇒ + = 2cos x
cos x cos x −3 −3 4
= 2( ) − 5( ) +
⇒ 1 + sin x = 2cos x 2 4 5 5
⇒ 2cos 2 x − sin x − 1 = 0 −3 4 −15 + 30 + 8 23
= +3+ = =
2 5 10 10
⇒ 2(1 − sin2 x) − sin x − 1 = 0
−3 −3 4
⇒ 2 − 2sin2 x − sin x − 1 = 0 = 2( ) − 5( ) +
4 5 5
⇒ − 2sin2 x − sin x + 1 = 0 150. (A)
2
⇒ 2sin x + sin x − 1 = 0 Given expression is cos 2 48∘ − sin2 12∘
Since, the equation is a quadratic equation in cos 2 48∘ − sin2 12∘ =
sin x. So it will have 2 solutions. cos (48∘ + 12∘ ) ⋅ cos (48∘ − 12∘ )
148. (A) ∵ cos 2 A − sin2 B = cos (A + B) ⋅ cos (A − B)
The given expression is 1 √5 + 1 √5 + 1
= cos 60∘ ⋅ cos 36∘ = × =
π π 2π 5π 2 4 8
sin + sin + sin + sin
18 9 9 18 151. (B)
5π π 2π π 1 1
= (sin + sin ) + (sin + sin ) Given that: tan α = and tan β =
18 18 9 9 7 3

5π π 5π π 2π π 1 2 1
+ − + 1 − tan2 α 1 − (7) 1−
= 2sin ( 18 18 ) ⋅ cos (18 18) + 2sin (9 9) cos 2α = = = 49
2 2 2 1 + tan2 α 1 2 1+ 1
1+( ) 49
7
2π π 48 24

⋅ cos ( 9 9) = =
50 25
2
2tan β
Now tan 2β =
π π π π 1−tan2 β
= 2sin ⋅ cos + 2sin ⋅ cos
6 9 6 18 1 2

1 π 1 π π π = 3= 3 =2×9=3
= 2 × cos + 2 × cos = cos + cos 1 8 3 8 4
2 9 2 18 9 18 1−
9 9
π π π π 7π 8π 3
= sin ( − ) + sin ( − ) = sin + sin ∴ tan 2β =
2 9 2 18 18 18 4
7π 4π 2tan 2β
= sin + sin sin 4β =
18 9 1 + tan2 2β
149. (B)
Given that: 3tan A + 4 = 0, A lies in second 3 3
2× 3 16 24
= 4 = 2 = × =
quadrant 3 2 1+ 9 2 25 25
1+( ) 16
−4 −3 4
∴ tan A = & cos A =
3 5 24
cos 2α = sin 4β =
[A lies in second quadrant] 25

MATHEMATICS Page | 4A. 15


TRIGONOMETRY

152. (B) tan(1°) + tan(89°)


a
Given that: tan θ = = tan(1°) + tan(90° − 1°)
b
= tan(1°) + cot(1°) =
bcos 2θ + asin 2θ
1
1 − tan2 θ 2tan θ tan(1°) +
tan1°
= b[ 2
]+ a[ ]
1 + tan θ 1 + tan2 θ 1+tan2 1° sec2 1° 1 cos1°
= = = ×
tan1° tan1° cos2 1° sin1°
a2 2a
1−
2 2 2
= b[ b ]+ a[ b 2] = =
2sin1°cos1° sin(2°)
a2 a
1+ 2 1+ 2
b b 157. (B)
2 1
2a Given equation is, sin x + sin y =
b2 − a2 2
= b[ 2 ]+[ 2 b 2
b + a2 b +a ⇒ 2sin
x+y
cos
x −y
=
1
]
b2 2 2 2
x+y x −y 1
b3 − a2 b 2a2 b b3 − a2 b + 2a2 b ⇒ 2sin
2
cos
2
=
4
...(i)
= 2 + =
b + a2 b 2 + a2 b 2 + a2 and cos x + cos y = 1
b3 + a2 b b(b2 + a2 ) x+y x −y
= 2 = =b ⇒ 2cos cos =
b + a2 b 2 + a2 2 2
x+y x −y 1
⇒ cos cos = ...(ii)
2 2 2
153. (D) x +y 1
Dividing (i) by (ii), we get tan ( )=
2 2
1 x2 +1
Given that: cos θ = x + ⇒ cos θ = x +y
x x 2tan( 2 )
2 2
Now, tan(x + y) = x +y
⇒ x + 1 = xcos θ ⇒ x − xcos θ + 1 = 0 1−tan2 ( )
2

For real value of x, b2 − 4ac ≥ 0 1


2× 4
= 2 =
⇒ (−cos θ)2 − 4 × 1 × 1 ≥ 0 1 3
1−
4
⇒ cos 2 θ − 4 ≥ 0 ⇒ cos 2 θ ≥ 4
158. (B)
⇒ cos θ ≥ ±2 [−1 ≤ cos θ ≤ 1]
A 1−cos (A)
So, the value of θ is not possible. WKT, tan ( ) =
2 sin (A)
π
154. (None of the options is correct) Put A= in the above formula,
4
sinθ = sin ∝ ⇒ sinθ − sin ∝ = 0 π
(4 ) 1−cos 4
π
1−
1
√2
θ+∝ θ−∝ tan ( ) =( π )= 1
2 sin
∴ 2cos ( ) sin =0 4 √2
2 2
θ+∝ θ−∝ √2−1
∴ = 0 or sin is a multiple of π. = =√2 − 1
2 2 1

𝐍𝐨𝐭𝐞: option (B) would have been the correct 159. (A)
answer if ‘and’ is replaced by ‘or’. sin1° + sin2°+. . . . . + sin359°

155. (A) = sin1° + sin2°+. . . . . + sin180°. . . ..

Since x lies in quadrant. sin(360° − 2°) + sin(360° − 1° )

So, cosx and cos


x
are negative = sin1° + sin2°+. . . . . + sin180°. . . . . − sin2°
2
3 4 − sin1° = 0
Now, tan x = ⇒ cox −
4 5
160. (C)
x
Also, cosx = cos 2 − 1
2 1 + sinθ + sin2 θ+. . . . . upto ∞ = 2√3 + 4
x 1 1 4
⇒ cos 2 = {cosx + 1} = {− + 1} = 1 Since, R.H.S. is an infinite geometric progression
2 2 2 5
x 1 with common ration sinθ.
⇒ cos = − .
2 √10 1
∴ = 2√3 + 4
1 − sinθ
156. (A)
⇒ 2√3 + 4 − 2√3 sinθ − 4sinθ = 1
MATHEMATICS Page | 4A. 16
TRIGONOMETRY

⇒ 2√3 + 3 = sinθ(2√3 + 4) 167. (B)

⇒ sinθ =
2√3 +3
×
2√3 − 4
=
√3
⇒θ=
π tan θ ⋅ cot θ = 1
2√3 +4 2√3 −4 2 3

161. (B) 168. (C)


π 180∘
cos 2 45° − sin2 15° = = 5∘ 37′ 30′′
32 32
= cos(45° + 15°)cos(45° − 15°)
1 √3 √3
169. (D)
= cos60° cos30° = . = 5π π
2 2 4
sin ⋅ sin
12 12
162. (B)
1 2 1 π 1 1 1
Let tan ( cos −1 )=θ = sin = + =
2 √5
2 6 2 2 4
2
⇒ cos −1 = 2tan−1 θ
√5
170. (B)
2 1−θ2
⇒ cos −1 = cos −1
( ) θ
√5 1+θ2 1 + cos θ = 2cos 2 ( )
2 1−θ2
2
⇒ =( ) ⇒ 2 + 2θ2 = √5 − √5θ2
√5 1+θ2

⇒ θ2 (√5 + 2) = √5 – 2 √2 + √2 + √2 + 2cos 8θ = 2cos θ


√5 – 2
⇒ θ2 = ⇒ θ2 = (√5 – 2)2
√5 + 2

⇒ θ = √5 – 2
171. (D)
By graph
163. (B)
Consider √3 cosec 20° − sec20° 172. (B)
=
√3

1
=
√3cos20°− sin20° log ⋅ tan 1∘ tan 2∘ … . . tan 89∘
sin20° cos20° sin20°cos20°
√3 1
log 1 = 0
2( 2 cos20°− 2sin20°)
= ⇒ e0 = 1
sin20°cos20°
2(sin60°cos20°−cos20° sin20°) 173. (D)
=
sin20°cos20°
174. (C)
2×2sin(60°−20°) 4sin40°
= = =4
2sin20°cos20° sin40° 175. (A)
164. (D) Note that
2 2
We have, sin 51° + sin 39° sin[𝜋 2 ]𝑥 = sin 9𝑥 and sin [−𝜋 2 ]𝑥 = − sin 10𝑥
2 2
= sin 51° + sin (90° − 51°) 176. (B)
2 2
= sin 51° + cos 51° = 1 177. (C)
165. (D)
We have, tanA + cotA = 2
MOCK TEST SOLUTIONS:
Squaring both sides,
1. (C)
then we get tan2 A + cot 2 A + 2 = 4
1
We have,
(∵ tanA = ) ⇒ tan2 A + cot 2 A = 2
cot A cos(θ + ϕ) = m cos(θ − ϕ)
Again squaring both sides we get, 1 cos(θ − ϕ)
⇒ =
tan4 A + cot 4 A + 2 tan2 A. cot 2 A = 4 m cos(θ + ϕ)
⇒ tan4 A + cot 4 A = 2 1 + m 2 cos θ cos ϕ
⇒ =
166. (A) 1 − m 2 sin θ sin ϕ
1−m 1−m
cos (3 × 360∘ + 120∘ ) + tan (4 × 360∘ + 45∘ ) = ⇒ tan θ tan ϕ = ⇒ tan θ = cot ϕ
1/2 1+m 1+m
2. (D)
MATHEMATICS Page | 4A. 17
TRIGONOMETRY
1 1 sin(80 − 10) . cos70
Given that, sin A = and sin B = =
√10 √5
cos80cos10sin70
We know that, cos80cos10 + sin80sin10
=
sin(A + B) = sin A cos B + sin B cos A cos80cos10
= 1 + tan80. tan10
1 1 1 1
= √1 − + √1 − = 1 + tan(90 − 10) tan10
√10 5 √5 10
= 1 + cot10tan10
1
4 1 9 = 1+1= 2
= √ + √
√10 5 √5 10
6. (B)
1 5 1 π 2π 32π
= (2 + 3) = √ = cos cos … cos
√50 √50 √2 65 65 65

π π 2π 25 π
⇒ sin(A + B) = sin = cos . cos … cos
4 65 65 65
π 26 π 65π
⇒ A+B= sin sin
= 65 65
4 π = π
26 sin 64 sin
3. (B) 65 65
1 π
sin (π − ) 1
1 + | cos x| + cos 2 x + | cos 3 x|+. . . ∞ = = 65
1 − | cos x| π = 64
64 sin
65
1
∴ = 43 7. (B)
81−| cos x|
1
3 Given, cos x + sin x =
⇒ = 26 ⇒ 1 = 2 − 2| cos x| 2
21−| cos x|
1
1 ⇒ 1 + sin 2x =
⇒ | cos x| = 4
2
2 tan x −3
1 ⇒ =
⇒ cos x = ± 1 + tan x 4
2
⇒ 8 tan x = −3 − 3 tan2 x
π π 2π 2π
⇒ x = ,− , ,− ⇒ 3 tan2 x + 8 tan x + 3 = 0
3 3 3 3
∴ Number of solutions =4 −8 ± √64 − 36 −8 ± 2√7
⇒ tan x = =
4. (A) 6 6

∵ sin 2x + cos 4x = 2 4 ± √7
⇒ = −( )
3
It is possible only when
8. (B)
sin 2x = 1 and cos 4x = 1
π We have,
⇒ 2x = 2nπ + and 2x = 2mπ 12 3
2 cos(α + β) = and sin(α − β) =
π 13 5
∴ x = nπ + and x = mπ, n ∈ I 5 4
4 ⇒ sin(α + β) = and cos(α − β) =
13 5
Then, solution,
Now,
π
= (nπ + , n ∈ I) ∩ (mπ, m ∈ I) = ϕ sin 2α = sin{(α + β) + (α − β)}
4
5. (C) ⇒ sin 2α = sin(α + β) cos(α − β)
sin80 sin10 + cos(α + β) sin(α − β)

= cos80 cos10 5 4 12 3 56
sin70 ⇒ sin 2α = × + × =
cos70 13 5 13 5 65
sin80cos10 − sin10cos80 9. (D)
= cos80cos10 We have, sin10 2x = 1 + cos10 x
sin80
cos80
MATHEMATICS Page | 4A. 18
TRIGONOMETRY

Minimum value of RHS = 1 and maximum values of 3π


⇒ sin θ = −1 = sin [∵ sin θ ≥ 1]
LHS = 1. Therefore, solution is possible only when 2
3π 5π 7π
sin10 2x = 1 and cos10 x=0. But this is not possible. ∴ θ= ∈( , )
2 4 4
Therefore, it has no solution. 15. (D)
10. (B) We have,
We know that, cos 1° + cos 2° + cos 3° + ⋯ + cos 180°
− √a 2 + b2 ≤ a cos θ + b sin θ ≤ √a 2 + b2 90°

= ∑{cos θ + cos(180°) − θ} + cos 180°


∴ −√3 + 1 ≤ √3 sin x + cos x ≤ √3 + 1
θ=1
⇒ −2 ≤ √3 sin x + cos x ≤ 2 90°

But, √3 sin x + cos x = 4 = ∑(cos θ − cos θ) − 1 = −1


θ=1
Hence, given equation has no solution
16. (D)
11. (D)
1 √3
1 2 [ sin 80° − cos 80°]
We have, x + = 2 cos θ 2 2
x
3 sin 80° cos 80°
1
⇒ (x + ) = (2 cos θ)3 4[sin(80° − 60°)] 4 sin 20°
x = =
2 sin 80° cos 80° sin 160°
1 1 1
⇒ x 3 + 3 + 3x. (x + ) = 8 cos 3 θ 4 sin 20°
x x x = =4
sin(180° − 20°)
1
⇒ x 3 + 3 + 3.2 cos θ = 8 cos 3 θ 17. (A)
x
1 Since, sin x + cosec x = 2
⇒ x 3 + 3 = 2(4 cos 3 θ − 3 cos θ)
x 1
⇒ sin x + =2
= 2 cos 3θ sin x
12. (B) ⇒ sin2 x − 2 sin x + 1 = 0
sin (x + y) a + b ⇒ (sin x − 1)2 = 0 ⇒ sin x = 1
=
sin (x − y) a − b 1
Now, sinn x + cosec n x = sinn x +
sinn x
sin (x + y) + sin (x − y)
⇒ = 1+1= 2
sin (x + y) − sin (x − y)
(a + b) + (a − b) 18. (C)
=
(a + b) − (a − b) sin2 5° + sin2 10° + sin2 15°+. . . + sin2 90°
2 sin x cos y 2a tan x a = sin2 5° + sin2 10° +
⇒ = ⇒ =
2 cos x sin y 2b tan y b … + sin2 45°+. . . + sin2 80° + sin2 85° + sin2 90°
13. (B) 1
= sin2 5° + sin2 10°+. . . + + … + cos 2 10° +
2
We have, cot θ − tan θ = 2 2 2
cos 5° + sin 90°
⇒ cos 2 θ − sin2 θ = 2 sin θ cos θ
= (sin2 5° + cos 2 5°) + (sin2 10° + cos 2 10°) +
⇒ cos 2θ = sin 2θ
⋯+
π π
⇒ tan 2θ = tan ⇒ 2θ = nπ + (sin2 40° + cos 2 40°) + sin2 45° + sin2 90°
4 4
nπ π 1
⇒ θ= + =1+1+1+1+1+1+1+1+ +1
2 8 2
14. (D) 1
=9
2 2
Given, cos θ + sin θ + 1 = 0 2
19. (B)
⇒ sin2 θ − sin θ − 2 = 0
sec θ + tan θ = k …(i)
⇒ (sin θ + 1)(sin θ − 2) = 0

MATHEMATICS Page | 4A. 19


TRIGONOMETRY

sec 2 θ − tan2 θ ⇒ sin θ − cos θ = −√2


⇒ =k
sec θ − tan θ 1 1
1 ⇒ sin θ − cos θ = −1
⇒ sec θ − tan θ = …(ii) √2 √2
k

On adding Eqs. (i) and (ii), we get 1 1


⇒ cos θ − sin θ = 1
√2 √2
1 k2 + 1
2 sec θ = k + = π π
k k ⇒ cos cos θ − sin sin θ = 1
4 4
2k π
⇒ cos θ = 2 ⇒ cos (θ + ) = cos 2nπ
k +1 4
20. (A) π
⇒ θ + = 2nπ
We have, 4
2
π
1 + sin x + sin2 x + ⋯ + to ∞ = (√3 + 1) ⇒ θ = 2nπ −
4
1 2 24. (A)
⇒ = (√3 + 1)
1 − sin x We have,
1
⇒ 1 − sin x = 2 sin(π + θ) sin(π − θ) cosec 2 θ
(√3 − 1)
= − sin θ sin θ cosec 2 θ = −1
2
(√3 − 1) 25. (B)
⇒ 1 − sin x =
4 4
Given that, sin θ = − and θ lies in the IIIrd
4 − 2√3 5
⇒ sin x = 1 − ( ) quadrant
4

√3 π 2π 16 3
⇒ sin x = ⇒ x = or, ⇒ cos θ − √1 − =−
2 3 3 25 5
21. (A)
3
θ 1+cos θ 1− 1
We have, Now, cos = ±√ = ±√ 5
= ±√
2 2 2 5
2 2 2 2
1 + 8 sin x cos x = 1 + 2 θ 1
But we take cos = −
2 2 )2 2 )2 2 √5
(2 + sin x cos x = 1 + 2(sin 2x
θ
= 1 + 2 sin2 2x 2 = 1 + (1 − cos 4x 2 ) Since, if θ lies in IIIrd quadrant then
2
2 2 will be in IInd quadrant
= 2 − cos 4x Now, −1 ≤ cos 4x ≤ 1
θ 1
⇒ 1 ≤ 2 − cos 4x 2 ≤ 3 Hence, cos = −
2 √5
⇒ 1 ≤ 1 + 8 sin2 x 2 cos x 2 ≤ 3 26. (D)
Hence, the required maximum value is 3
We have,
22. (D)
sin x + sin2 x = 1 ⇒ sin x = 1 − sin2 x
Since sin θ, cos θ are the roots of
⇒ sin x = cos 2 x
ax 2 − bx + c = 0
∴ cos 8 x + 2 cos 6 x + cos 4 x
b c
∴ sin θ + cos θ = and sin θ cos θ = = sin4 x + 2 sin3 x + sin2 x
a a

b2 = (sin x + sin2 x)2 = 1


⇒ sin2 θ + cos 2 θ + 2 sin θ cos θ =
a2 27. (D)
c c b2
and sin θ cos θ = ⇒ 1 + 2 ( ) = Since the sum of a positive number and its
a a a2

⇒ b2 − a2 = 2ac reciprocal is always greater than or equal to 2.

23. (C) Therefore, y ≥ 2. But, y = 2 only when θ = 0.

We have, √2 sec θ + tan θ = 1 Hence, y > 2


28. (D)
√2 sin θ
⇒ + =1 The quadratic equation is x 2 − x cos θ + 1 = 0
cos θ cos θ
MATHEMATICS Page | 4A. 20
TRIGONOMETRY

Since, x is real, therefore discriminant ≥ 0


⇒ B 2 − 4 AC ≥ 0
⇒ cos 2 θ ≥ 4(1)(1) ⇒ cos 2 θ ≥ 4
Which is impossible because cos 2 θ is not greater
than 1
29. (D)
θ+ϕ θ−ϕ
2 sin ( ) ⋅ cos ( )
2 2
θ+ϕ θ−ϕ
= 2√3sin ( ) ⋅ sin ( )
2 2
θ+ϕ
⇒ sin ( )=0
2
⇒ θ+ϕ =0
ϕ = −θ
∴ sin 3θ + sin 3ϕ = 0
30. (D)

√2 + √2 + √2 + 2cos 8θ

= √2 + √2 + √2(2cos 2 4θ)

= √2 + √2(2cos 2 2θ)

= √2(2cos 2 θ) = 2cos θ

MATHEMATICS Page | 4A. 21


COMPLEX NUMBERS

HINTS AND SOLUTIONS


1. Ans (B) 7. Ans (C)
√−3 = i√3, z1 ⋅ z2 = z1

√−6 = i√6 ⇒ z2 = 1 + 0i[∵ z1 (1 + 0i) = z1 ]


Then, z2 is called multiplicative identity of complex
So, √(−3)√(−6)
number z1 . Hence, Statement I is true.
= i2 3√2 = −3√2
For every non-zero complex number (z), we have
2. Ans (D)
complex number called the multiplicative inverse
(3 + i√5)(3 − i√5) 1
of z such that z ⋅ = 1 + 0i
(√3 + √2i) − (√3 − i√2) z

32 − (i√5)2 8. Ans (C)


= (1+i)2
√3 + √2i − √3 + i√2 Given that, = x + iy
2−i
[∵ (z1 − z2 )(z1 + z2 ) − z12 − z22 ]
1 + i2 + 2i
9+5 4
14i ⇒ = x + iy
= = (∵ i4 = 1) 2−i
2√2i 2√2i 1 − 1 + 2i
⇒ = x + iy
7 −7√2 7√2i 2−i
= i3 = i=0− 2i 2+i
√2 2 2
⇒ × = x + iy
2−i 2+i
3. Ans (B)
2i(2 + i)
We have, (1 + i)2n = (1 − i)2n ⇒ = x + iy
(2)2 + (i)2
1 + i 2n 4i − 2
⇒( ) =1 ⇒ = x + iy
1−i 5
⇒ (i)2n = 1 which is possible if n = 2 2 4
⇒ x + iy = − + i
(∵ i4 = 1) 5 5
2 4 2
4. Ans (D) Now, x + y = − + =
5 5 5

(−√3 + √−2)(2√3 − i) 9. Ans (C)


3
= (−√3 + √2i)(2√3 − i) 1 1 3 1 2 1
( + 3i) = ( ) + 3 ( ) (3i) + 3 ( ) (3i)2
3 3 3 3
= −6 + √3i + 2√6i − √2i2
+ (3i)3
= (−6 + √2) + √3(1 + 2√2)i
1
5. Ans (B) = + i + 9(−1) + 27(−i)
27
Since x 2 + y 2 = (x + iy)(x − iy) 1
= − 9 − 26i
x2 + y2 27
∴ (p + iq) = ( )i 1 − 243 −242
x + iy = − 26i = − 26i
27 27
= (x − iy)i = y + ix
10. Ans (A)
∴ p = y, q = x
We have,
6. Ans (A)
sin⁡ 60∘ + icos⁡ 60∘ cos⁡ 15∘ + isin⁡ 15∘
For every complex number there exists the ×
cos⁡ 15∘ − isin⁡ 15∘ cos⁡ 15∘ + isin⁡ 15∘
complex number 0 + i0 (denoted as 0 ) called = sin⁡(60∘ − 15∘ ) + icos⁡(60∘ − 15∘ )
additive identity or zero complex number such 1 1
= sin⁡ 45∘ + icos⁡ 45∘ = +i
that, for complex number z, z + 0 = z √2 √2
MATHEMATICS Page |5A. 1
COMPLEX NUMBERS

11. Ans (B) We have, x = 1 + 2i


(x + iy)1/3 = a + ib ⇒ x − 1 = 2i
⇒ x + iy = (a + ib)3 ⇒ x 2 − 2x + 5 = 0
⇒ x + iy Now, x 3 + 7x 2 − x + 16
= a3 − ib3 + i3a2 b − 3ab2 = x(x 2 − 2x + 5) + 9(x 2 − 2x + 5) + (12x − 29)
= a3 − 3ab2 + i(3a2 b − b3 ) = x(0) + 9(0) + 12x − 29
⇒ x = a3 − 3ab2 and y = 3a2 b − b3 = 12(1 + 2i) − 29
x y
So, − = a2 − 3b2 − 3a2 + b2 = −17 + 24i
a b

= −2a2 − 2b2 = −2(a2 + b2 ) 18. Ans (A)


z1 1
12. Ans (B) = (2 + 3i) ( )
z2 1 + 2i
3 3
5i (− i) = 5 × − i2 1 2
5 5 = (2 + 3i) ( − i)
5 5
= −3(−1) = 3 = 3 + 0i
2 6 4 3
13. Ans (C) = ( + ) + i (− + )
5 5 5 5
ik = i4n ⋅ i7 = (i4 )n (i2 )3 i 8 1
= − i
= 1(−1)(i) = −i 5 5
19. Ans (C)
14. Ans (B)
(1 + 𝑖)𝑛 (1 + 𝑖)𝑛
We have, =
(1 − 𝑖)𝑛−2 (1 − 𝑖)𝑛 (1 − 𝑖)−2
(z1 + z2 )2 = (z1 + z2 )(z1 + z2 )
1+i n
= (z1 + z2 )z1 + (z1 + z2 )z2 (distributive law) =( ) (1 − i)2
1−i
= z12 + z2 z1 + z1 z2 + z22 1+𝑖 1+𝑖 𝑛
=( × ) (1 + 𝑖 2 − 2𝑖)
(distributive law) 1−𝑖 1+𝑖
𝑛
= z12 + z1 z2 + z1 z2 + z22 1 + 2𝑖 + 𝑖 2
=( ) (1 + 𝑖 2 − 2𝑖)
1 − 𝑖2
(commutative law, z1 z2 = z2 z1 )
= z12 + 2z1 z2 + z22 1 + 2𝑖 − 1 𝑛
=( ) (1 − 1 − 2𝑖)
1 − (−1)
15. Ans (B)
[∵ i2 = −1]
1 + i 1000 1 − i 2000
( ) +( ) 2i n
1−i 1+i = ( ) (−2i)
2000 2
1 + i 1 + i 1000 1 + i2 − 2i
=( × ) +( ) = in (−2i) = −2in+1
1−i 1+i 1 − i2
20. Ans (C)
= i1000 + (−i)2000
Additive inverse of 𝑧 is −𝑧.
= i4×250 + [(−i)2 ]4×250
∴ Additive inverse of (1 − i) is
=1+1=2
−(1 − i) = −1 + i
16. Ans (A)
21. Ans (B)
Given x = 2 + 5i
Let z = x + iy
⇒ x − 2 = 5i
x+iy−6i |x+i(y−6)|
∴| | = 1 or =1
⇒ x 2 − 4x + 29 = 0 x+iy+6i |x+i(y+6)|

Now, x 3 − 5x 2 + 33x − 49 ⇒ √x 2 + (y − 6)2 = √x 2 + (y + 6)2


= x(x 2 − 4x + 29) − 1(x 2 − 4x + 29) − 20 ⇒ x 2 + y 2 + 36 − 12y = x 2 + y 2 + 36 + 12y
= −20 ⇒ 24y = 0 ⇒ y = 0
17. Ans (B) ∴ ⁡z = x i.e., z lies on the real axis.
MATHEMATICS Page |5A. 2
COMPLEX NUMBERS

22. Ans (A) Case (i) : z‾ = 0,


1
Let z = 2 − 3i, then, z −1 = ∴ z = 0 = 0 + i0
2−3i
2 + 3i 2 + 3i Case (ii): z = −7
= =
(2 − 3i)(2 + 3i) 22 − (3i)2 ∴ z = −7 + 0i
2 + 3i 2 3 Hence there is only two solutions.
= = + i
13 13 13 27. Ans (C)
23. Ans (D)
We have,
(1 + i√3)(2 + 2i)
| | (√3 + i)3 (3i + 4)2
√3 − i z=
(8 + 6i)2
|1 + i√3||2 + 2i| |√3 + i|3 |3i + 4|2
= ⁡⇒ |z| =
|√3 − i| |8 + 6i|2
3
2 × 2√2 2
= = 2√2 (√(√3)2 + (1)2 ) (√(3)2 + (4)2 )
2 ⁡= 2
24. Ans (D) (√82 + 62 )
(1+i)2 (2)3 (5)2
Given complex number is ⁡= =2
1−i (10)2
2
(1 + i + 2i) 1 + i
= × 28. Ans (C)
1−i 1+i
|cos⁡ θ + isin⁡ θ| = |cos⁡ θ − isin⁡ θ| = 1
2i(1 + i)
= |1 − i√3| 2 1
1 − i2 ∴ |z| = = =
2i + 2i 2
2i − 2 2|1 − i| 2√2 √2
= =
1+1 2 29. Ans (C)
=i−1 We have, |z − i| = |z + 1| = 1
∴ Required conjugate is −1 − i Let, z = x + iy
25. Ans (B) ∴ |z − i| = 1
I. Let z = 2 − 3i ⇒ |x + iy − i| = 1
Then, z‾ = 2 + 3i and |z|2 = 22 + (−3)2 = 13 ⇒ |x + (y − 1)i| = 1
Therefore, the multiplicative inverse of 2 − 3i is ⇒ x 2 + (y − 1)2 = 1
z‾ 2 + 3i 2 3 Also, |z + 1| = 1
z −1 = = = + i
|z|2 13 13 13
⇒ |x + iy + 1| = 1
II. We have,
⇒ |(x + 1) + iy| = 1
5 + √2i 5 + √2i 1 + √2i
= × ⇒ (x + 1)2 + y 2 = 1
1 − √2i 1 − √2i 1 + √2i
From Eqs. (i) and (ii), we get
5 + 5√2i + √2i − 2
= x = y = 0 and x = −1, y = 1
1 − (√2i)2
∴ z = 0, −1 + i
3 + 6√2i 3(1 + 2√2i)
= = = 1 + 2√2i 30. Ans (C)
1+2 3
1 1 Let z1 = 3 + pi
III. i−35 = = (i2)17
i35 i 3
and z2 = + pi
1 i i 4
= × = =i Given that |z1 | = 2|z2 |
−i i −i2
26. Ans (B) ⇒ √9 + p2
Given |z|2 + 7z‾ = 0
9
⇒ zz‾ + 7z‾ = 0 2√ + P2
16
⇒ z‾(z + 7) = 0
MATHEMATICS Page |5A. 3
COMPLEX NUMBERS

On squaring both sides, we get 5 − 5i + 12i − 4 1 + 7i


= =
9 5(3 + i) 5(3 + i)
9 + p2 = 4 ( + p2 )
16 (1 + 7i)(3 − i)
=
9 5(9 + 1)
⇒ 4P 2 − P 2 = 9 −
4 10 + 20i 1 + 2i
= =
27 9 3 50 5
⇒ 3P 2 = ⇒ P2 = ⇒ P = ±
4 4 2
1 2 2 2
31. Ans (A) ∴ |z| = √( ) + ( )
5 5
Let z = −3 + ix 2 y
1 √5
Now, z‾ = −3 − ix 2 y = √1 + 4 =
5 5
According to the question,
35. Ans (D)
−3 − ix 2 y = x 2 + y + 4i
4
On equating real and imaginary parts both sides, z=
1−i
we get 4 1+i
⇒z= ×
−3 = x 2 y and −x 2 + y = 4 1−i 1+i
4(1 + i)
4 = 2 = 2(1 + i)
∴ −3 = x 2 − 1 − i2
x2
2(1 − i) × (1 + i)
⇒ −3x 2 = x 4 − 4 ∴ z‾ =
1+i
⇒ x 4 + 3x 2 − 4 = 0 4
=
⇒ x 4 + 4x 2 − x 2 − 4 = 0 1+i
⇒ x 2 (x 2 + 4) − 1(x 2 + 4) = 0 36. Ans (B)
⇒ (x 2 − 1)(x 2 + 4) = 0 |1 + i| × |2 + i|
|3 + i|
⇒ x 2 = 1 or x 2 + 4 = 0
⇒ x = ±1[∵ x 2 ≠ −4] √2 × √5
= =1
2 √10
⇒ y = −4[∵ −x y = 4]
37. Ans (B)
∴ (x, y) = (1, −4), (−1, −4)
3+4i
Let z =
32. Ans (A) 4−5i
1
7−i 7 − i 3 + 4i we have to calculate z −1 i.e.,
z= = × z
3 − 4i 3 − 4i 3 + 4i
1 4 − 5i 3 − 4i
21 + 4 + i(28 − 3) ∴ ⁡z −1 = = ×
= =1+i z 3 + 4i 3 − 4i
25
12 − 15i − 16i + 20i2 12 − 31i − 20
∴ |z| = |1 + i| = √2 = =
9 − 16i2 9 − 16(−1)
∴ |z|14 = (√2)14 −8 − 31i −8 31
= = − i
7 9 + 16 25 25
= [(√2)2 ]
−8 −31
= 27 ∴ ⁡z −1 = ( , )
25 25
33. Ans (D) 38. Ans (B)
z‾
Reciprocal of z = i584 (i8 + i6 + i4 + i2 + 1)
|z|2 −1
i574 (i8 + i6 + i4 + i2 + 1)
3−√7i
Therefore, reciprocal of 3 + √7i =
16 i584
= −1
3 √7i i574
= −
16 16 = i10 − 1 = −1 − 1 = −2
34. Ans (C) 39. Ans (B)
1−i 4i
Let z = +
3+i 5
MATHEMATICS Page |5A. 4
COMPLEX NUMBERS

We have,
(3−2i)(2+3i) i4n+1 − i4n−1 i − i−1 2i
(1+2i)(2−i) = = =i
2 2 2
6 + 9i − 4i + 6
= 48. Ans (B)
2 − i + 4i + 2
in=i4m+3=i4mi3=1. (−i) = −i
12 + 5i 4 − 3i
= × 49. Ans (C)
4 + 3i 4 − 3i
48 − 36i + 20i + 15 63 − 16i 1+i 1−i
= = = i, = −i
16 + 9 25 1−i 1+i
63 16 1+i 3 1−i 3
= − i ( ) −( ) = i3 − (−i)3 = 2i3 = −2i
25 25 1−i 1+i
63 16
∴ required conjugate = + i 50. Ans (B)
25 25

40. Ans (B) 1 + i and 1 – i are conjugate to each other Also z +

If (1 − i)n = 2n …..(i) z̄ = 2 Re(z)

We know that, if two complex numbers are equal, (1 + i)2n + (1 – i)2n = a purely real number

their modulus must also be equal. 51. Ans (D)

Therefore, from Eq. (i), we have 1 + i2 + i4 + i6 +⁡…⁡+⁡i2n

|(1 − i)|n = |2n | = 1 – 1 + 1 – 1⁡….+⁡(–1)n

⇒ |1 − i|n − |2|n (2n > 0) This can't be evaluated unless n is specified


n 52. Ans (A)
⇒ [√12 + (−1)2 ] = 2n
1 + a 1 + cos θ + i sin θ θ
= = i cot
⇒ (√2)n = 2n 1 − a 1 − cos θ − i sin θ 2
n n 53. Ans (D)
⇒ 2 2 = 2n ⇒ = n ⇒ n = 0
2 1
x + = 2 cos α
41. Ans (C) x

p+qω+rω2 p+qω+rω2  x=cos+ i sin ,x n = cos⁡n + ⁡i⁡sin⁡n


ω2 ( 4) + ω ( 3)
rω2 +pω3 +qω qω+rω2 +pω 1 1
 = cos α − i sin α  = cos⁡n − ⁡i⁡sin⁡n
2 x xn
= ω + ω = −1
1
42. Ans (B) x n + = 2 cos n α
xn

For n = 2, (1 + i)2 = −2i 54. Ans (C)


43. Ans (D) If n = 100, ∑100 k
k=1 i = 0

(1 + i)6 = ((1 + i)2 )3 = (2i)3 = −8i Now ∑99 k 96 k


k=1 i = ∑k=1 i + i
97
+ i98 + i99
(1 − i)3 = (1 − i)(1 − i)2 = −2 − 2i = 0 + i + i2 + i3 = i – 1 – i = –1
 required sum=−2 − 10i Now 1 + ∑99 k
k=1 i = 1 − 1 = 0

44. Ans (C)  n = 99


Sum of n terms of G. P 55. Ans (D)
a(rn −1) i(i1000 −1) i(1−1)
Sn= = = =0 √a + ib = x + iy
r−1 i−1 i−1

45. Ans (C)  a + ib = (x + iy)2

i4 + i9 + i26 1+i−1  a + ib = (x – y2) + (2xy) i


=
8
2−i +i −i10 15 2−1−1+i  a – ib = (x – y2) – (2xy) i
= 1 = 1 + i0  a – ib = (x – iy)2
46. Ans (D)
√a − ib = ⁡x– iy
1 + i 4n
( ) = (i)4n n = 1 56. Ans (A)
1−i z̅
47. Ans (C) Use z −1 =
|z|2

MATHEMATICS Page |5A. 5


COMPLEX NUMBERS

57. Ans (A) 67. Ans (B)


9y2 – 4 – 10xi = 8y2 + 20i xyz= (a + b)( a + b2)( a2 + b)=a3+b3
 9y2 – 4 = 8y2 and –10x = 20 68. Ans (A)
 x = – 2, y = 2 1 1 3 9 27
( + + + +. . . . . ) ω + ω2
2 2 8 32 128
58. Ans (A)
1 1/2
π π 6 π 6 = ( ) ω + ω2
6 2 1−3/4
1 + i√3 cos + i sin
3 3 ei 3
( ) =[ π π] = ( −iπ )
1
= . 2ω + ω2
1 − i√3 cos − i sin e 3 2
3 3
=  + 2 = –1
= ei4π
69. Ans (C)
= cos 4 π + i sin 4 π = 1
6 Given (1 – ) (1 – 2) (1 – 4) (1 – 8)
1−i√3
Also ( ) =1
1+i√3 =(1 – ) (1 – 2) (1 – ) (1 – 2)
59. Ans (C) =[(1 – ) (1 – 2)]2=9
1+i√3
Rationalise and⁡verify⁡the⁡option⁡ 70. Ans (A)
1−i√3

60. Ans (B) 1. .2=3=1

z+z̅ = 2 Re z = purely real ⇒ Im⁡z = 0 71. Ans (A)


z−1
61. Ans (A) Since, is purely imaginary
z+1
Given (1 + 2 + +)3n – ( 1 +  + 2+2)3n z−1 ̅̅̅̅̅̅̅̅̅̅
z−1
∴ ⁡⁡⁡ = −( )
=3n-(2)3n=1-1=0 z+1 z+1
z − 1 z̅ − 1
62. Ans (D) ⇒ ⁡⁡⁡⁡ =
z + 1 z̅ + 1
(x – 2)3 + 27 = 0
[Apply⁡componendo⁡dividendo]
(x – 2)3 = – 27=(-3)3
2z 2
⇒ ⁡⁡⁡⁡ = ⁡⁡⁡⁡ ⇒ ⁡⁡⁡⁡zz̅ = 1
x – 2 = –3, –3, –32 −2 −2z̅
 x = –1, 2 – 3, 2 –32 ⇒ |z|2 = 1⁡⁡⁡⁡ ⇒ ⁡⁡⁡⁡ |z| = 1
63. Ans (D) 72. Ans (D)

Use the result n + 2n = –1 We have,

64. Ans (B) 2n (1 + i)2n


+
(1 − i)2n 2n
(1 + )7 = A + B
2n {(1 + i)2 }n
(–2)7 = A + B = 2 n
+
{(1 − i) } 2n
–2 = A + B
2n (1 + 2i + i2 )n
= +
1 +  = A + B 2
(1 − 2i + i ) n 2n
 A = 1, B = 1 2n (2i)n 1 n
= + = (− ) + in
65. Ans (D) (−2i)n 2n i

(x – 1)3 + 8 = 0 = in + in = 2in

(x – 1)3 = −8 73. Ans (D)


We have,
x − 1 = −2, −2, −22
z + z −1 = 1 ⇒ z 2 − z + 1 = 0
x = 1 – 2, 1 − 2, 1 − 22
⇒ z = −ω⁡or − ω2
x = −1, 1 − 2, 1 − 22
For z = −ω, we have
66. Ans (B)
z100 + z −100 = (−ω)100 + (−ω)−100
1, , 2 are in G. P

MATHEMATICS Page |5A. 6


COMPLEX NUMBERS

1 ∴ Radius= |
k(α−β)
|=|
5(i+i)
|=
5
=ω+ = ω + ω2 = −1 1−k2 1−25 12
ω
Or, For z = −ω2 , we have 78. Ans (B)
33
z100 + z −100 = (−ω2 )100 + (−ω2 )−100 Given, (√5 + √3i) = 249 z
1 Let √5 = r cos θ,⁡⁡⁡⁡√3 = r sin θ
= ω200 +
ω200
∴ ⁡⁡⁡⁡⁡ r 2 = 5 + 3⁡⁡⁡⁡ ⇒ ⁡⁡⁡⁡r = 2√2
1
⇒ z100 + z −100 = ω2 + 2 ∴ ⁡⁡⁡ (r cos θ + ir sin θ)33 = 249 z
ω
= ω2 + ω = −1 ⇒ ⁡⁡⁡⁡ |r 33 (cos 33θ + i sin 33θ)| = |249 z|
33
74. Ans (C) ⇒ (2√2) | cos 33θ + i sin 33θ| = 249 |z |
3+2i sin θ 99
Let z =
1−2i sin θ ⇒ 2 2 (1) = 249 |z| ⇒ |z| = √2
3 + 2i sin θ (1 + 2i sin θ) 79. Ans (D)
⇒z= ×
1 − 2i sin θ 1 + 2i sin θ
We have,
3 − 4 sin2 θ + 8i sin θ
= (1 − i)n = 2n
1 + 4 sin2 θ
For purely imaginary of z, put Re(z) = 0 ⇒ |1 − i|n = |2|n
n n n
3 − 4 sin2 θ ⇒ (√2) = 2n ⁡ ⇒ 22 = 2n ⇒ 22 = 1
ie,⁡⁡⁡⁡⁡⁡⁡⁡ =0
1 + 4 sin2 θ
⇒n=0
√3
⇒ sin θ = ± ⁡ So, there is no non-zero integral solution of the
2
π π given equation
⇒ ⁡θ = nπ + (−1)n (+ ) = nπ ±
3 3 80. Ans (B)
75. Ans (B) z = 0 is the only complex number which satisfies
We know that, sum of any four consecutive powers the given relations
of i is zero 81. Ans (B)
13
The coordinates of the points representing 1 +
∴ ⁡⁡⁡⁡ ∑(in + in+1 )
n=1 i, i − 1 and 2i are (1,1), (−1,1) and (0,2)
= (i + i2 +. . . . +i13 ) + (i2 + i3 +. . . +i14 ) respectively
= i13 + i14 ∴ Required⁡area = 1⁡sq. unit.
=i−1 82. Ans (D)
76. Ans (C) −1+√3i
We know, ω =
2
We have, 1000
1 √3i
√x + iy = ±⁡(a + i⁡b) ∴ ⁡⁡⁡⁡⁡ (− + ) = (ω)1000 = ω⁡⁡⁡⁡⁡⁡
2 2
⇒ x + iy = a2 − b2 + 2i⁡ab [∵ ω3 = 1]
2 2
⇒ x = a − b , y = 2⁡ab 83. Ans (D)
∴ √−x − iy = √−(a2 − b 2 ) − 2i⁡ab The vertices of the triangle are

⇒ √−x − iy = √b 2 − a2 − 2i⁡ab 1 √3 1 √3
A(0,1), B (− , ) ⁡and⁡C (− , − )
2 2 2 2
= √(b − ia)2 = ±(b − ia)
0 1 1
77. Ans (B) 1 √3
z−α
1 |− 1|
On comparing the given circle with | | = k, we ∴ Area⁡of⁡∆ABC = 2 2
z−β 2| |
1 √3
get − − 1
2 2
α = i,⁡⁡⁡β = −i, k=5 ⇒ Area⁡of⁡∆⁡ABC
MATHEMATICS Page |5A. 7
COMPLEX NUMBERS

1 1 1 √3 √3 ∴ x 2 − 3x + [x] = 0
= [− (− + ) + 1 ( + )]
2 2 2 4 4 ⇒ x 2 − 3x + 2 = 0 ⇒ x = 1, 2 ⇒ x = 2
√3 Hence, the given equation has two solutions only
⇒ Area⁡of⁡∆ABC = sq. units
4 88. Ans (C)
84. Ans (A) 100 100
−1 + √−3 −1 − √−3
√3 1 ( ) +( )
+i 2 2
= 2 2
1 √3 = ω100 + ω200 = ω + ω2 = −1
−i
2 2
89. Ans (B)
√3 + i
= We have,
1 − i√3
π π
√3 + i 1 + i√3 1 + i = √2 (cos + i sin )
= × 4 4
1 − i√3 1 + i√3 π π
and, 1 − i = √2 (cos + i sin )
4i 4 4
= =i
4 ∴ (1 + i)8 + (1 − i)8
85. Ans (A) π π 8 π π 8
= 24 (cos + i sin ) + 24 (cos − i sin )
(2 − ω)(2 − ω 2 )(2
−ω 10 )(2
−ω ) 11 4 4 4 4
4 (cos 4
= (2 − ω)(2 − ω2 )(2 − ω)(2 − ω2 ) =2 2⁡π + i sin 2⁡π) + 2 (cos 2⁡π − i sin 2⁡π)

= [(2 − ω)(2 − ω2 )]2 = 24 (2 cos 2⁡π) = 25

= [4 − 2(ω+ω2 ) + 1]2 90. Ans (B)


1
= (4 + 2 + 1)2 = 49 Given, x + iy = (
1+2i 2
)
3+4i
86. Ans (A)
1 + 2i
⇒ (x + iy)2 =
3 + 4i
We have, x = √1 + √1 + √1 + ⋯ ∞
Taking modulus from both sides we get
⇒ x = √1 + x 1 + 2i
2 2
|x + iy|2 = | |
⇒ x = 1 + x⁡ ⇒ x − x − 1 = 0 3 + 4i
1 ± √1 + 4 1 ± √5 1+4
⇒x= = ⇒ x2 + y2 = √
2 2 9 + 16
1+√5
As x > 0, we take only x = . 5 1
2
⇒ (x 2 + y 2 )2 = =
87. Ans (C) 25 5
91. Ans (C)
We have the following cases:
Given, |z1 | = |z2 | = ⋯ |zn | = 1
CASE I: When x ∈ [0, 1)
⇒ ⁡⁡⁡ |z1 |2 = |z1 |2 = ⋯ |zn |2 = 1
In this case, we have [x] = 0
⇒ ⁡⁡⁡ z1 z̅1 = z1 z̅2 = ⋯ = zn z̅n = 1
∴ x 2 − 3x + [x] = 0
1 1 1
⇒ x 2 − 3x = 0 ⇒ x = 0,3 ⇒ x = 0 ⇒ ⁡⁡⁡ z̅1 = ,⁡⁡⁡z̅2 = , … . z̅n = …(i)
z1 z2 zn

CASE II: When x ∈ [1, 2) Now, |z1 + z2 +. . . +zn |


In this case, we have [x] = 1 = |z̅̅̅̅̅̅̅̅̅̅̅̅̅̅̅̅̅̅̅̅
1 + z2 +. . . +zn | = |z̅1 + z̅2 + ⋯ + z̅n |

∴ x 3 − 3x + [x] = 0 ⇒ x 2 − 3x + 1 = 0 =|
1
+
1
+. . . +
1
| [using Eq. (i)]
z1 z2 zn
3 ± √5
⇒x= 92. Ans (C)
2
(1+i)2
Clearly, these values of x do not belong to Given, = x + iy
2−i

[1, 2]. So, the equation has no solution in [1, 2) 2i 2+i


⇒ ⁡⁡⁡⁡ × = x + iy
CASE III: When x ∈ [2, 3) 2−i 2+i

MATHEMATICS Page |5A. 8


COMPLEX NUMBERS

4i − 2 z1 z̅1 1
⇒ ⁡⁡⁡⁡ = x + iy For collinear points |z2 z̅2 1| = 0
5
2 4 z3 z̅3 1
⇒ ⁡⁡x + iy = − + i 1 + 2i 1 − 2i 1
5 5
∴ |2 + 3i 2 − 3i 1|
2 4 2
∴ x+y =− + = 3 + 4i 3 − 4i 1
5 5 5
4i 1 − 2i 1
93. Ans (A) = |6i 2 − 3i 1| [C1 → C1 − C2 ]
1 𝓏 8i 3 − 4i 1
𝓏3 + 𝓏2 − + 1 = 0
i i −2i −1 + i 0
= |−2i −1 + i 0| = 0
⇒ ⁡⁡ 𝓏 3 − i𝓏 2 + i𝓏 + 1 = 0
8i 3 − 4i 1
⇒ ⁡⁡ 𝓏 2 (𝓏 − i) + i(𝓏 − i) = 0 [R1 → R1 − R 2 ,⁡⁡⁡R 2 → R 2 − R 3 ]
2
⇒ ⁡⁡ (𝓏 − i)(𝓏 + i) ⁡⁡ ⇒ ⁡ |𝓏| = 1 Alternatively, use distance formula.
94. Ans (A) 101. Ans (A)
The number z=-4+5i is in second quadrant. verify⁡the⁡options⁡by⁡squaring
After rotation in counter clock wise direction at
Hence, a = √2i = ±⁡(1 + i)
angle 180°, it comes to the 4th quadrant
102. Ans (C)
∴ option A is correct
We have,
95. Ans (A) z − 12 5 z−4
| | = ⁡and⁡ | |=1
Mid point of vertices=origin z − 8⁡i 3 z−8
Only option A satisfies Let z = x + iy. Then,
96. Ans (D) z − 12 5
| |=
verify⁡the⁡options⁡by⁡squaring z − 8⁡i 3
⇒ 3|z − 12| = 5|z − 8⁡i|
97. Ans (D)
⇒ 3|(x − 12) + i⁡y| = 5|x + (y − 8)⁡i|
Given, z1/3 = p + iq
⇒ 9(x − 12)2 + 9⁡y 2 = 25⁡x 2 + 25(y − 8)2
⇒ (x − iy) = (p + iq)3 [put z = x − iy]
…(i)
⇒ (x − iy) = p3 — iq3 + 3p2 qi − 3pq2
z−4
⇒ (x − iy) = (p3 − 3pq2 ) + i(3p2 q − q3 ) and, | |=1
z−8
⇒ x = (p3 − 3pq2 ) and −y = 3p2 q − q3 ⇒ |z − 4| = |z − 8|
x 2 2 y 2 2
⇒ ⁡ = (p − 3q ) and = (q − 3p ) ⇒ |x − 4 + i⁡y| = |x − 8 + i⁡y|
p q
x y ⇒ (x − 4)2 + y 2 = (x − 8)2 + y 2
∴ ⁡⁡ + = −2p2 − 2q2
p q ⇒x=6
x y
+ Putting x = 6 in (i), we get
p q
⇒ 2 = −2
(p + q2 ) y 2 − 25⁡y − 136 = 0 ⇒ y = 17, 8
98. Ans (D) Hence, z = 6 + 17⁡i or z = 6 + 8⁡i
4 103. Ans (C)
z̅ =
1+i
Let z = x + iy, then z̅ = x − iy
99. Ans (D)
∴ ⁡⁡⁡⁡z + z̅ = 2x and z − z̅ = 2iy
1
Let z =
i−1 Given, (3 + i)(z + z̅) − (2 + i)(z − z̅) + 14i = 0
̅̅̅̅̅̅̅
1 1 1
⇒ ⁡⁡⁡ (3 + i)2x − (2 + i)2iy + 14i = 0
Then, z̅ = ( ) = =−
i−1 −i−1 i+1

100. Ans (D) ⇒ ⁡⁡⁡6x + 2ix − 4yi + 2y + 14i = 0 + oi


On comparing real and imaginary part, we get
6x + 2y = 0
MATHEMATICS Page |5A. 9
COMPLEX NUMBERS

And 2x − 4y + 14 = 0 i(1 − i200 )


= ⁡ (since⁡sum⁡in⁡G.P.)
On solving, we get x = −1, y = 3 1−i
2 i(1 − 1)
∴ zz̅ = |z|2 = (√(−1)2 + (3)2 ) = 10 = =0
1−i
104. Ans (B) 108. Ans (D)

Given, z = −z̅ Given that; x − 3 = i


̅̅̅̅̅̅̅̅̅̅
⇒ x + iy = −(x + iy) [Put z = x + iy] ⇒ (x − 3)2 = i2 ⇒ x 2 − 6x + 10 = 0

⇒ x + iy = −(x − iy) So, x 3 − 3x 2 − 8x + 15

⇒x=0 = x(x 2 − 6x + 10) + 3(x 2 − 6x + 10) − 15

Hence, z is a purely imaginary. Now, = 0 + 0 − 15 = −15.

105. Ans (B) 109. Ans (D)


3 (1 + i)2n = ((1 + i)2 )n = (1 + i2 + 2i)n
Given, = a + ib
2+cos θ+i sin θ
= (1 − 1 + 2i)n = 2n in (1 − i)2n = ((1 − i)2 )n
3[(2 + cos θ) − i⁡sin⁡θ]
⇒ = a + ib = (1 + i2 − 2i)n = (1 − 1 − 2i)n = (−2)n in
(2 + cos θ)2 + sin2 θ
3[2 + cos θ − i sin θ] 2n (1 + i)2n
⇒ = a + ib n
+
5 + 4 cos θ (1 − i)2 2n
3(2 + cos θ) 2n 2n in 1
⇒ ⁡a = ,⁡⁡ ⁡= n n
+ n
= + in
5 + 4 cos θ (−2) i 2 (−1)n in
⁡⁡
3 sin θ
And b = −
5+4 cos θ
1 + (−1)n i2n 1 + (−1)n (i2 )n
= =
(−1)n in (−1)n in
∴ (a − 2)2 + b2
2 1 + (−1)n (−1)n 1 + (−1)2n
6 + 3 cos θ 9 sin2 θ = =
=( − 2) + (−1)n in (−1)n in
5 + 4 cos θ (5 + 4 cos θ)2
1+1 2
(−4 − 5 cos θ)2 + 9 sin2 θ = = .
(−1)n in (−1)n in
=
(5 + 4 cos θ)2 110. Ans (D)
16 + 25 cos 2 θ + 40 cos θ + 9 sin2 θ 1 − 2i 4 − i
= +
(5 + 4 cos θ)2 2 + i 3 + 2i
16 + 16 cos 2 θ + 40 cos θ + 9 (1 − 2i)(3 + 2i) + (4 − i)(2 + i)
= =
(5 + 4 cos θ)2 (2 + i)(3 + 2i)
(5 + 4 cos θ)2 50 − 120i 10 24
= =1 = = − i
(5 + 4 cos θ)2 65 13 13
106. Ans (B) 111. Ans (D)
(1−i√3)(2+2i) 1 3 3 + 4i
Let z = ( + )( )
(√3−i) 1 − 2i 1 + i 2 − 4i
(2 − 2√3) + 2i(1 + √3) (√3 − i) 1 + 2i 3 − 3i 6 − 16 + 12i + 8i
= × =[ 2 + ][ ]
(√3 − i) (√3 − i) 1 + 22 12 + 12 22 + 42
2 + 4i + 15 − 15i −1 + 2i
=
2√3−6+2i−2√3i+2√3i+6i−2−2√3⁡ =( )( )
3+1 10 2
(17−11i)(−1+2i) 5+45i 1 9
−8 + 8i = = = + i.
= = −2 + 2i 20 20 4 4
4
112. Ans (C)
∴ Magnitude of z = √4 + 4 = 2√2
Given equation
Or use properties of modulus and arguments
(x 4 + 2xi) − (3x 2 + yi) = (3 − 5i) + (1 + 2yi)
107. Ans (C)
200
⇒ (x 4 − 3x 2 ) + i(2x − 3y) = 4 − 5i
∑ in = i + i2 + i3 + ⋯ . . +i200 Equating real and imaginary parts, we get
n=1
MATHEMATICS Page |5A. 10
COMPLEX NUMBERS

x 4 − 3x 2 = 4 ….. (i) and 2x − 3y = −5 Alternatively, take z=1, solve and verify the
1
Form (i) and (ii), we get x = ±2 and y = 3, . options
3
1 118. Ans (A)
Trick: Put x = 2, y = 3 and then x = −2, y = ,
3 z−5i
| | = 1 ⇒ z would lie on the right bisector of the
we see that they both satisfy the given equation. z+5i

113. Ans (C) line segment connecting the points 5i and −5i.

Here z + z‾ = (x + iy) + (x − iy) = 2x (Real) and Thus z would lie on the x-axis

zz‾ = (x + iy)(x − iy) = x 2 + y 2 (Real) Alternatively, take z=x+iy and solve

114. Ans (B) 119. Ans (D)

2 − 3i (2 − 3i)(4 + i) Let x = 3 + 2i
=
4−i (4 − i)(4 + i) ⇒ (x − 3)2 = 4i2 = −4
8 + 3 − 12i + 2i 11 − 10i ⇒ x 2 − 6x + 13 = 0
= =
16 + 1 17
Thus, x 4 − 8x 3 + 4x 2 + 4x + 39
11+10i
⇒ Conjugate = . = (x 2 − 6x + 13)(x 2 − 2x − 21) − 96x + 312
17

115. Ans (C) ∴ f(3 + 2i) = 0(x 2 − 2x − 21) − 96(3 + 2i) + 312
1 1 3 − √7i = 24 − 192i = a + ib
= ⋅
3 + √7i 3 + √7i 3 − √7i ∴ a = 24 and b = −192
3 − √7i ∴ =
a 24
=− .
1
= b −192 8
9+7
3−√7i 3 √7
120. Ans (A)
= = − i.
16 16 16 Taking modulus on both sides x 2 + y 2
116. Ans (C)
|u + iv| u2 + v 2
(1 + i)(2 + i) 1 + 3i 3 − i = = =1
|u − iv| u2 + v 2
z= = ×
(3 + i) 3+i 3−i
121. Ans (D)
3 + 4i
= (1 + i)2 −2 + 6i 1 3
5 = =− + i
3−i 10 5 5
⇒ |z| = 1
122. Ans (C)
|z1 ||z2 | √2⋅√5
Trick : |z| = |z3 |
= =1 Verify⁡for⁡n=1,2,…
√10

117. Ans (B) 123. Ans (B)


Let z = x + iy, ⁡z + iz |z|2 = |z ⋅ z| = |z| ⋅ |z| = |z|2
= (x − y) + i(x + y) and 124. Ans (C)
iz = −y + ix Put z = x + iy
x+iy−8i
If A denotes the area of the triangle formed by So, Re⁡ ( )=0
x+iy+6
z, z + iz and iz, then
[x + (y − 8)i][x + 6 − iy]
Re⁡ ( )=0
1 x y 1 (x + 6)2 + y 2
A = |x − y x+y 1|
2 −y x 1 ⇒ x(x + 6) + y(y − 8) = 0

Applying transformation x 2 + y 2 + 6x − 8y = 0

(R 2 → R 2 − R1 − R 3 ) 125. Ans (B)

We get Given equation x 4 − 1 = 0


x y 1 ⇒ (x 2 − 1)(x 2 + 1) = 0 ⇒ x 2 = 1 and x 2 = −1⁡ ⇒
1
A= | 0 0 −1| x = ±1, ±i
2
−y x 0
1 1
126. Ans (D)
= (x 2 + y 2 ) = |z|2 .
2 2
MATHEMATICS Page |5A. 11
COMPLEX NUMBERS

On putting z = x + iy z‾ x − iy x − iy x − iy
= = ×
we get x 2 − y 2 + 2ixy + x − iy = 0 z x + iy x + iy x − iy
x 2 + i2 y 2 − 2xyi
⇒ x 2 − y 2 + x = 0, y(2x − 1) = 0 =
1
x 2 − i2 y 2
⇒ x = or y = 0
2 x 2 − y 2 − 2xyi
=
When x = , y = ±
1 √3 x2 + y2
2 2
x2 − y2 2xy
When y = 0, x = 0 or -1 = − 2 i
2
x +y 2 x + y2
⇒ System has four solutions. z‾
When z lies in third quadrant then will also be lie
z

127. Ans (C) in third

z = sin x + icos2x x2 − y2 −2xy


∴⁡ < 0⁡ & 2 <0
x2 + y2 x + y2
z‾ = sin x − icos⁡2x
x 2 − y 2 < 0⁡&⁡−2xy < 0 as x 2 + y 2 > 0
But we are given that
⇒ x 2 < y 2 & xy > 0
z‾ = cos x − isin⁡2x
So, x < y < 0.
∴ sin x − icos⁡2x = cos x − isin⁡2x⁡
130. Ans (A)
Comparing the real and imaginary parts, we get
Given (z + 3)(z‾ + 3) = (z + 3)(z̅̅̅̅̅̅̅
+ 3)
sin x = cos x & cos 2x = sin 2x
= |z + 3|2
tan x = 1& tan 2x = 1
π π 131. Ans (B)
tan x= tan⁡ & tan2x= tan⁡
4 4 1+i x
π π Given: ( ) =1
1−i
∴ x = nπ + , n ∈ I and 2x = nπ +
4 4 ⇒ (i)x =(i)4n ⇒ x⁡ = 4n, n ∈ N
∴ x = 2x ⇒ 2x − x = 0 ⇒ x = 0 132. Ans (A)
128. Ans (C)
Take modulus on both sides
1−isin⁡ α
Given ⁡z = 133. Ans (A)
1+2isin⁡ α

(1 − isin⁡ α)(1 − 2isin⁡ α) Property of modulus


=
(1 + 2isin⁡ α)(1 − 2isin⁡ α) ∴ |z1 z2 | = |z1 ||z2 |
1 − 2isin⁡ α − isin⁡ α + 2i2 sin2 ⁡ α 134. Ans (B)
=
(1)2 − (2isin⁡ α)2
Given that: z = 2 − i lies in 4th quadrant.
1 − 3isin⁡ α − 2sin2 ⁡ α π
= After rotation at an angle in clockwise direction, z
1 − 4i2 sin2 ⁡ α 2

(1 − 2sin2 ⁡ α) − 3isin⁡ α shift into 3rd quadrant.


=
1 + 4sin2 ⁡ α So, option B is correct
1 − 2sin2 ⁡ α 3sin⁡ α 135. Ans (D)
= − .i
1 + 4sin ⁡ α 1 + 4sin2 ⁡ α
2
x + yi is a non-real complex number if y ≠ 0.
Since Z is purely real
136. Ans (D)
−3sin⁡ α
= = 0 ⇒ sin⁡ α = 0
1+4sin2 ⁡ α Given that: a + ib = c + id
So, α = nπ, n ∈ N. ⇒ |a + ib| = |c + id|
129. Ans (B)
⇒ √a2 + b 2 = √c 2 + d2
Given that: ⁡z = x + iy
Squaring both sides, we get
If z lies in third quadrant
a2 + b2 = c 2 + d2
So, x < 0 and y < 0.
137. Ans (B)
z‾ = x − iy

MATHEMATICS Page |5A. 12


COMPLEX NUMBERS

i+z 141. Ans (D)


Given that: ⁡ | |=1
i−z
1
Let z = x + yi |β| = 1 ⇒ β‾ =
β
i + x + yi x + (y + 1)i
∴ ⁡| |=1⇒| |=1 β−α 1−αβ‾
i − x − yi −x − (y − 1)i | | = |β| | |=1
1−α‾β 1−α‾β

⇒ ⁡|x + (y + 1)i| = | − x − (y − 1)i| 142. Ans(D)


⇒ ⁡√x 2 + (y + 1)2 = √x 2 + (y − 1)2 Given (1 − ω + ω2 )⁡(1 + ω − ω2 )
⇒ ⁡x 2 + (y + 1)2 = x 2 + (y − 1)2 =−2ω × −2ω2 = 4 [∵ use⁡1 + ω + ω2 = 0
⇒ (y + 1)2 = (y − 1)2 143. Ans(C)
y 2 + 2y + 1 = y 2 − 2y + 1 (√3 + 1)3 (√9 + 16)2
|Z| =
y = 0 ⇒ x-axis. (√64 + 36)2

138. Ans (B) 8 × 25


= =2
100
use properties of modulus |z n | = |z|n
144. Ans(B)
So,⁡|z|2 = |z 2 |
in [1 + i + i2 + i3 ]
139. Ans (C)
= in [1 + i − 1 − i] = 0
If z is a real number, then
145. Ans (B)
1+icos⁡ θ
Z= 1
1−2icos⁡ θ x=
(1−cos⁡ θ)+isin⁡ θ
1+icos⁡ θ 1+2icos⁡ θ
= × 1
1−2icos⁡ θ 1+2icos⁡ θ
=
1 + 2icos⁡ θ + icos⁡ θ + 2i cos ⁡ θ 2 2 θ θ θ
= 2sin2 ⁡ + i2sin⁡ cos⁡
2 2 2
1 − 4i2 cos 2 ⁡ θ
θ θ
sin2 −icos 2
1+3icos⁡ θ−2cos2 ⁡ θ =
= θ
1+4cos2 ⁡ θ 2 sin
2
1−2cos2 ⁡ θ 3cos⁡ θ 1
= + i So, Real part of z =
1+4cos2 ⁡ θ 1+4cos2 ⁡ θ 2

Since Z is real 146. Ans (C)


3cos⁡ θ m
=0 (1 + i)2
1 + 4cos 2 ⁡ θ [ ] =1
2
⇒ 3cos⁡ θ=0 2i m
π ⇒[ ] =1
2
So,θ = (2n + 1) , n ∈ N
2 m
⇒i =1⇒m=4
140. Ans (A)
147. Ans (B)
Given that: z = 1 + 2i
1−i 96
( ) = a + ib
⇒ |z| = √(1)2 + (2)2 = √5 1+i

7−z −2i 48
Now f(z) = ⇒( ) = a + ib
1−z2 2i
7 − (1 + 2i) 7 − 1 − 2i 1 = a + ib ⇒ a = 1, b = 0
= =
1 − (1 + 2i)2 1 − 1 − 4i2 − 4i So, (a, b) = (1,0)
6 − 2i 3−i 3 − i 2 + 2i
= = = × 148. (C)
4 − 4i 2 − 2i 2 − 2i 2 + 2i
1 We have, z = x + iy
=1+ i
2 Also, |z + 1| = |z − 1|
1 2 ⇒ |(x + 1) + (iy)| = |(x − 1) + iy|
So⁡,|f(z)| = √(1)2 + ( )
2
⇒ (x + 1)2 + y 2 = (x − 1)2 + y 2
1 √5 |z| ⇒ x 2 + 1 + 2x = x 2 + 1 − 2x
= √1 + = =
4 2 2 ⇒ 4x = 0⁡ ⇒ x = 0
MATHEMATICS Page |5A. 13
COMPLEX NUMBERS
n
i. e. , y-axis
= ∑(r + 1)[r 2 + (ω + ω2 )r + ω3 ]
149. Ans (D)
r=1
1+i x n
( ) = 1 ⇒ ix = 1
1−i = ∑(r + 1)(r 2 − r + 1)
r=1
150. Ans (C) n 2
n(n + 1)
3x = 6 ⇒ x = 2⁡and⁡4x − y = −1 = ∑(r 3 + 1) = [ ] +n
2
⇒ 8 − y = −1⁡ ⇒ 9 = y r=1

151. Ans (B) Alternatively, verify options for n = 1


Apply modulus to every factor of numerator and 4. Ans (D)
denominator, then simplify
Square the options and identify the correct answer.
152. Ans (C)
5. Ans (B)
153. Ans (A)
Given, (1 + i)2n = (1 − i)2n
⇒ ⁡⁡ 2n in = 2n (−1)n in ⁡⁡⁡ ⇒ ⁡⁡⁡1 = (−1)n
MOCK TEST SOLUTIONS: ∴ The smallest value of n is 2
1. Ans (C) 6. Ans (D)
x2 +34x−71 2n (1+i)2n
Let y = We have, (1−i)2n +
x2 +2x−7 2n
⇒ x 2 (y − 1) + x(2y − 34) + 71 − 7y = 0 n
Since, x is complex number 2 {(1 + i)2 }n
= +
∴ ⁡⁡⁡D < 0 {(1 − i)2 }n 2n
⇒ (2y − 34)2 − 4(y − 1)(71 − 7y) < 0 2n (1 + 2i + i2 )n
=(y − 17)2 − (71y − 7y 2 − 71 + 7y) < 0 = +
(1 − 2i + i2 )n 2n
⇒ 8y 2 − 112y + 360 < 0
⇒ y 2 − 14y + 45 < 0 2n (2i)n 1 n
= + = (− ) + in
⇒ ⁡ (y − 9)(y − 5) < 0 (−2i)n 2n i
⇒ ⁡⁡5 < y < 9 = in + in = 2in
∴ ⁡⁡a = 5, b = 9
7. Ans (D)
2. Ans (A)
We have,
x2 + y2 + z2
225 + (3ω + 8ω2 )2 + (3ω2 + 8ω)2
= (a + b)2 + ω2 (a + bω)2 + (aω2 + bω)2
= 225 + 9ω2 + 64ω4 + 48ω3 + 9ω4 + 64ω2 +
= a2 + b2 + 2ab + a2 ω2 + b2 ω4 + 2abω3 + a2 ω4
48ω3
+ b2 ω2 + 2abω3
= 225 + 9ω2 + 64ω + 48 + 9ω + 64ω2 + 48
= a2 (1 + ω + ω2 ) + b2 (1 + ω + ω2 )
= 225 + 73(ω2 + ω) + 96
+ 6ab⁡⁡⁡[∵ ω4 = ω]
= 225 − 73 + 96 = 248
= 6ab⁡⁡⁡⁡⁡⁡[∵ 1 + ω + ω2 = 0]
8. Ans (C)
3. Ans (C)
1 i√3
1 1 1 1 We know, − + =ω
2 (1 + ) (1 + 2 ) + 3 (2 + ) (2 + 2 ) 2 2
ω ω ω ω
∴ ⁡⁡⁡⁡4 + 5(ω)334 + 3(ω)365
1 1
+ ⋯ … + (n + 1) (n + ) (n + 2 ) = 4 + 5(ω3 )111 . ω1 + 3(ω3 )121 . ω2
ω ω
= 2(1 + ω)(1 + ω2 ) + 3(2 + ω)(2 + ω2 ) + ⋯ = 4 + 5ω + 3ω2

+(n + 1)(n + ω)(n + ω2 ) = 3(1 + ω + ω2 ) + 1 + 2ω


n = 1 + (−1 + i√3) = i√3
= ∑(r + 1)(r + ω)(r + ω2 )
9. Ans (D)
r=1

MATHEMATICS Page |5A. 14


COMPLEX NUMBERS

1+ω ω2 −ω And 2x − 4y + 14 = 0
∆= | 1 + ω2 ω −ω2 |
On solving, we get x = −1, y = 3
ω2 + ω ω −ω2
2
−ω2 ω2 −ω ∴ ⁡⁡⁡⁡zz̅ = |z|2 = (√(−1)2 + (3)2 ) = 10
= | −ω ω −ω2 |
−1 ω −ω2 14. Ans (A)
ω2 ω2 ω ω2 ω 1 We have,
⇒ ∆= | ω 2 | = ω2 |
ω ω ω 1 ω|
(1 + i)2n − (1 − i)2n
1 ω ω2 1 1 ω
(1 + ω4 − ω2 )(1 − ω4 + ω2 )
⇒ ∆= ω2 {ω2 (ω − ω) − ω(ω2 − ω) + (ω − 1)}
{(1 + i)2 }n − {(1 − i)2 }n
⇒ ∆= ω2 {0 − ω3 + ω2 + ω − 1} = −3⁡ω2 =
(1 + ω4 − ω2 )(1 − ω4 + ω2 )
10. Ans (A)
(2i)n − (−2i)n
1+i x
(1 + i)(1 + i)
x =
( ) =[ ] (1 + ω − ω2 )(1 − ω + ω2 )
1−i (1 − i)(1 + i)
(2i)n − (−2i)n
x x =
(1 + i) 1 − 1 + 2i (−2⁡ω2 )(−2ω)
=[ 2
] =[ ]
1−i 2 = 2n−2 {in − (−i)n }
1+i x
⇒( ) = (i)x = 1 [given] 0⁡⁡⁡⁡⁡⁡⁡⁡⁡⁡, if⁡n⁡is⁡even
1−i ={
2n−1 ⁡in ,⁡⁡⁡if⁡n⁡is⁡odd
∴ x = 4n
15. Ans (B)
11. (A) 3
(n+1)2 x + iy =
Given, an = i 2 + cos θ + i sin θ
2
Here, a1 = i2 = 1, a 2 = i3 = i,
2 3(2 + cos θ − i sin θ)
=⁡
2 2
(2 + cos θ)2 + sin2 θ
a 3 = i4 = 1, a 4 = i5 = i,
6 + 3 cos θ − 3i sin θ
2 =
a 5 = i6 = 1, … 4 + cos 2 θ + 4 cos θ + sin2 θ
Here, we see that for all odd values of n, we get the 6 + 3 cos θ −3 sin θ
=[ ] + i[ ]
5 + 4 cos θ 5 + 4 cos θ
value of a n is 1
On equating the real and imaginary parts on both
∴ a1 + a3 + a 5 + ⋯ + a 25 =
sides, we get
3(2 + cos θ) −3 sin θ
x= ,y =
(5 + 4 cos θ) 5 + 4 cos θ
12. Ans (D) ∴ ⁡⁡ x 2 + y 2
9
We have, =(5+4 [4 + cos 2 θ + 4 cos θ + sin2 θ]⁡
cos θ)2
n n
1 − i√3 −1 − i√3 9 6 + 3 cos θ
( ) +( ) =⁡ = 4[ ]−3
2 2 5 + 4 cos θ 5 + 4 cos θ
= ωn + (ω2 )n = ω6⁡k + ω12k = 4x − 3

= (ω3 )2k + (ω3 )4⁡k = 2 16. Ans (B)

13. Ans (C) We have,

Let z = x + iy, then z̅ = x − iy |z1 | = |z2 | = ⋯ = |zn | = 1

∴ ⁡⁡⁡⁡z + z̅ = 2x and z − z̅ = 2iy ⇒ z1 z̅1 = z2 z̅2 = ⋯ = zn z̅n = 1

Given, (3 + i)(z + z̅) − (2 + i)(z − z̅) + 14i = 0 1 1 1


⇒ z̅1 = , z̅2 = , … , z̅n =
z1 z2 zn
⇒ ⁡⁡⁡ (3 + i)2x − (2 + i)2iy + 14i = 0
Now,
⇒ ⁡⁡⁡6x + 2ix − 4yi + 2y + 14i = 0 + oi
|z1 + z2 + ⋯ + zn | = |z̅̅̅̅̅̅̅̅̅̅̅̅̅̅̅̅̅̅̅̅̅̅
1 + z 2 + ⋯ + zn |
On comparing real and imaginary part, we get
⇒ |z1 + z2 + ⋯ + zn | = |z̅1 + z̅2 + ⋯ + z̅n |
6x + 2y = 0
MATHEMATICS Page |5A. 15
COMPLEX NUMBERS

1 1 1 −1 + i√3
⇒ |z1 + z2 + ⋯ + zn | = | + + ⋯+ | = 28 ( ) ⇒ |z| = 28 (1) = 28
z1 z2 zn 2
17. Ans (C) ∴ Modulus= 28
1−i 4i
Let z = + 22. Ans (C)
3+i 5

5 − 5i + 12i − 4 1 + 7i
= = If n = 100, ∑100 k
k=1 i = 0
5(3 + i) 5(3 + i)
(1 + 7i)(3 − i) 10 + 20i 1 + 2i Now ∑99 k 96 k
k=1 i = ∑k=1 i + i
97
+ i98 + i99
= = =
5(9 + 1) 50 5 = 0 + i + i2 + i3 = i – 1 – i = –1
99
2 2
1 2 1 √5 1 + ∑ ik = 1 − 1 = 0
∴ |z| = √( ) + ( ) = √1 + 4 =
5 5 5 5 k=1

18. Ans (C)  n = 99


23. Ans (B)
Given vertices 𝓏, 𝓏 + i𝓏 and i𝓏 form vertices of
We know that, sum of any four consecutive powers
right-angled triangle with base 𝓏 and height i𝓏
of i is zero
Then the area of the triangle is 13
1
△= |𝓏|2 = 200 (given) ∴ ⁡⁡⁡⁡ ∑(in + in+1 )
2
n=1
⇒ ⁡ |𝓏|2 = 400
= (i + i2 +. . . . +i13 ) + (i2 + i3 +. . . +i14 )
⇒ ⁡ |𝓏| = 20
= i13 + i14
∴ ⁡ |3𝓏| = 3|𝓏| = 60
=i−1
19. Ans (C)
24. Ans (C)
5
Let 𝓏1 = 1 + i, 𝓏2 = −2 + 3i and 𝓏3 = 0 + i
3 We have,
x1 y1 1 1 1 1
√x + iy = ±⁡(a + i⁡b)
x
Then, | 2 y2 1| = |−2 3 1|
5
x3 y3 1 0 1 ⇒ x + iy = a2 − b2 + 2i⁡ab
3

5 −10 ⇒ x = a2 − b2 , y = 2⁡ab
= 1 (3 − ) + 1(2) + 1 ( )
3 3 ∴ √−x − iy = √−(a2 − b 2 ) − 2i⁡ab
4 10
= +2− ⇒ √−x − iy = √b 2 − a2 − 2i⁡ab
3 3
4 + 6 − 10 = √(b − ia)2 = ±(b − ia)
=⁡ =0
3
25. Ans (B)
Hence, area of triangle is zero, therefore points are
z−α
On comparing the given circle with | | = k, we
collinear. z−β

Alternatively, use distance formula to verify get α = i,⁡⁡⁡β = −i, k = 5


k(α−β) 5(i+i) 5
options. ∴ Radius= | |=| |=
1−k2 1−25 12
20. Ans (C)
π
verify⁡for⁡θ = 26. Ans (B)
2
21. Ans (B) Given that: ⁡z = x + iy
8 If z lies in third quadrant
Let z = (1 + i√3)
So, x < 0 and y < 0.
1 + i√3
= (−2)8 ( ) = (−2)8 (ω2 )8 ⁡⁡[∵ ω3 = 1 z‾ = x − iy
−2
z‾ x − iy x − iy x − iy
= 28 ω16 = 28 ω = = ×
z x + iy x + iy x − iy

MATHEMATICS Page |5A. 16


COMPLEX NUMBERS

x 2 + i2 y 2 − 2xyi ⇒ |x + iy − i| = 1
=
x 2 − i2 y 2 ⇒ |x + (y − 1)i| = 1
2 2
x − y − 2xyi ⇒ x 2 + (y − 1)2 = 1
=
x2 + y2
Also, |z + 1| = 1
x2 − y2 2xy
= 2 2
− 2 i ⇒ |x + iy + 1| = 1
x +y x + y2
z‾
⇒ |(x + 1) + iy| = 1
When z lies in third quadrant then will also be lie
z ⇒ (x + 1)2 + y 2 = 1
in third
From Eqs. (i) and (ii), we get
x2 − y2 −2xy
∴⁡ 2 < 0⁡ & 2 <0 x = y = 0 and x = −1, y = 1
x +y 2 x + y2
∴ z = 0, −1 + i
x 2 − y 2 < 0⁡&⁡−2xy < 0 as x 2 + y 2 > 0
30. Ans (C)
⇒ x 2 < y 2 & xy > 0
Let z1 = 3 + pi
So, x < y < 0. 3
and z2 = + pi
27. Ans (A) 4

Given (z + 3)(z‾ + 3) = (z + 3)(z̅̅̅̅̅̅̅


+ 3) Given that |z1 | = 2|z2 |

= |z + 3|2 ⇒ √9 + p2
28. Ans (C) 9
2√ + P2
|cos⁡ θ + isin⁡ θ| = |cos⁡ θ − isin⁡ θ| = 1 16
|1 − i√3| 2 1 On squaring both sides, we get
∴ |z| = = =
2|1 − i| 2√2 √2 9
9 + p2 = 4 ( + p2 )
29. Ans (A) 16
We have, |z − i| = |z + 1| = 1 9
⇒ 4P 2 − P 2 = 9 −
Let, z = x + iy 4
27 9 3
∴ |z − i| = 1 ⇒ 3P 2 = ⇒ P2 = ⇒ P = ±
4 4 2

MATHEMATICS Page |5A. 17


STRAIGHT LINE

HINTS AND SOLUTIONS

1. Ans ( C) ⇒ x1 = −9, y1 = 9
1 and
x2 +3
= 9,
y2 +3
=6
× 1(k − 1) = ±1 2 2
2
⇒ x2 = 15, y2 = 9
k − 1 = ±2 ⇒ k = 3 & k = −1

−9+15+3 9+9+3
Now, centroid = ( , ) = (3,7)
3 3

2. Ans (A) 4. Ans (A)


Since, D is the mid point of BC. So, coordinate of Given that,
x2 +x3 y2 +y3
BC are ( , ) AP = PQ = QR = RB
2 2
and AQ = RP = QB
Given, G(7,5) is the centroid of △ ABC
2 + x2 + x3 Since, AQ = QB, that means Q is
∴7=
3 the mid-point of
3+y2 +y3
and 5 = 2 + 6 −7 + 5
3 AB = [ , ] = (4, −1)
2 2
⇒ x2 + x3 = 21 − 2
Midpoint of PR = (4, −1) [∵ PQ = QR]
and y2 + y3 = 15 − 3
5. Ans (A)
⇒ x2 + x3 = 19
Since, the vertices of the triangle are
and y2 + y3 = 12
A ≡ (2,3), B ≡ (8,10) and C ≡ (5,5).
x2 + x3 19
⇒ = ∴ The centroid of
2 2
y2 +y3 2 + 8 + 5 3 + 10 + 5
and =6 ΔABC = ( , ) = (5,6)
2
3 3
19
∴ Coordinate of D are ( , 6) 6. Ans (D)
2
The slope of a line whose inclination is 90∘ , is not
defined.
7. Ans (C)
∵ ABCD is a parallelogram.
∴ AB ∥ CD ⇒ Slope of AB = Slope of CD
and BC ∥ AD ⇒ Slope of BC = Slope of AD. Hence,
Statement I is false.
3. Ans (A)
Given, A = (3,3), E = (−3,6) and
F = (9,6)
Let B = (x1 , y1 ) and C = (x2 , y2 )
x1 +3 y1 +3
Then, = −3, =6
2 2

MATHEMATICS Page | 6A. 1


STRAIGHT LINE

Now, mid-point of AC
−2 + 3 −1 + 3 1 2 1
=( , ) = ( , ) = ( , 1)
2 2 2 2 2
and mid-point of BD
4−3 0+2 1
=( , ) = ( , 1)
2 2 2
⇒ ∠PAX = 180∘ − 60∘ = 120∘
⇒ Mid-point of AC = Mid-point of BD
∴ Slope of line AQ = m = tan 120∘
Hence, Statement II is true.
= tan (180∘ − 60∘ )
8. Ans (C)
Let slope of one line is m, then slope of another line = −tan 60∘ = −√3

is 2m. Given, the tangent of the angle between them 11. Ans (B)
1 Since, points are A(x, −1), B(2,1) and C(4,5) are
is tan θ = .
3
collinear.
m1 − m2 1
∴| |= Here, x1 = x, y1 = −1, x2 = 2, y2 = 1, x3 = 4 and
1 + m1 m2 3
1 m − 2m y3 = 5
⇒ =| |
3 1 + m × 2m Slope of AB = Slope of BC
1 −m y2 − y1 y3 − y2
⇒ =| | ⇒ =
3 1 + 2m2 x2 − x1 x3 − x2
⇒ (1 + 2m2 ) = 3m ⇒
1+1
=
5−1
(∵ slope of line =
y2 −y1
)
2−x 4−2 x2 −x1
2
⇒ 2m − 3m + 1 = 0
2 4
Factorize it by splitting the middle term ⇒ = ⇒2−x=1
2−x 2
⇒ 2m2 − 2m − m + 1 = 0 ⇒x=2−1=1
⇒ 2m(m − 1) − 1(m − 1) = 0 12. Ans (B)
⇒ (2m − 1)(m − 1) = 0 Let the incident ray strike X-axis at the point A
⇒ 2m − 1 = 0 whose coordinates be (x, 0). From the figure, the
1
or m − 1 = 0 ⇒ m = , m = 1 slope of the reflected ray is given by
2
3
9. Ans (D) tan θ = …. (i)
5−x
The inclination of the line (θ) lies within 0∘ to 180∘ Again, the slope of the incident ray is given by
∘ ∘
i.e., 0 ≤ θ ≤ 180 . −2
tan (π − θ) =
10. Ans (C) x−1
−2
Given, ∠YPQ = 30∘ or −tan θ =
x−1

To find slope of line AQ. Solving Eqs. (i) and (ii), we get
Here, ∠YPQ = ∠OPA
(vertically opposite angles)
∵ ∠OPA + ∠POA + ∠PAO = 180∘
(∵ sum of all angles of a triangle is 180∘ )
⇒ 30∘ + 90∘ + ∠PAO = 180∘
⇒ ∠PAO = 180∘ − 120∘ = 60∘ 3 2
=
5−x x−1
13
⇒x =
5

Therefore, the required coordinates of the point A


13
are ( , 0)
5

MATHEMATICS Page | 6A. 2


STRAIGHT LINE

13. Ans (D) Since, the points A(1,6), B(3, −4) and C(x, y) are
Let A(x1 , y1 ) = (p + 1,1), B(x2 , y2 ) = (2p + 1,3) collinear.
and C(x3 , y3 ) = (2p + 2,2p) 1 6 1
∴ |3 −4 1| = 0
Points A, B and C are collincar if, x y 1
Slope of AB = Slope of BC ⇒1(-4-y)-6(3-x) +1(3y+4x) =0
3−1 2p−3
i.e., = ⇒ 10x + 2y − 22 = 0
(2p+1)−(p+1) (2p+2)−(2p+1)

2 2p − 3 ⇒ 5x + y − 11 = 0
⇒ =
p 1 17. Ans (C)
2
⇒ 2p − 3p − 2 = 0 Slope of the line 3x + y = 3 is -3.
⇒ (2p + 1)(p − 2) = 0 Slope of any line perpendicular to the line 3x + y =
−1 −1 1
⇒p= ,2 3 is =
2 −3 3
1
14. Ans (A) Now, equation of line with slope and passing
3
We know that the acute angle θ between two lines through (2,2) is y − 2 = (x − 2)
1
3
with slopes m1 and m2 is given by
⇒ x − 3y + 4 = 0
m2 − m1
tan θ = | | x y
1 + m1 m2 ⇒ + =1
−4 4
1 3
Let m1 = ,
2 4
Hence, its y-intercept is .
m2 = m and 3

θ= .
π 18. Ans (B)
4
Any line parallel to 2x − 3y = 4 is
Now, putting these values in (i), we get
2x − 3y + λ = 0
1
π m− λ λ
tan = | 2|
4 1 A ≡ [− , 0] , B ≡ [0, ]
1+ m 2 3
2
1 λ λ
2m−1 area of △ AOB = |( ) ( )|
⇒1=| |, 2 2 3
2+m
2
2m−1 2m−1 = 12λ = 144
which gives = 1 or = −1
2+m 2+m
⇒ λ = 12, −12
1
Therefore m = 3 or m = − line is 2x-3y+12=0 and 2x-3y-12=0
3
1
Hence, slope of the other line is 3 or − .
3

15. Ans (A)


Slope of line passing through (k − 1, k + 2) and
k+1−k−2
(k, k + 1) is m1 = = −1 (independent of k )
k−k+1

Slope of line passing through (k, k + 1) and (k +


1, k) is 19. Ans (B)
k−k−1 S is mid-point of QR
m2 = = −1 (independent of k )
k+1−k
7+6 3−1 13
So S = ( , ) = ( , 1)
∵ m1 = m2 2 2 2
2−1 2
∴ Points (k − 1, k + 2), (k, k + 1) and (k + 1, k) are Slope of PS = 13 =−
2− 9
2
collinear, for any value of k.
16. Ans (A)

MATHEMATICS Page | 6A. 3


STRAIGHT LINE

22. Ans (D)


Equation of line
3x
2 Given line 3x + 4y = 12 can be rewritten as +
12
⇒ y − (−1) = − (x − 1)
9 4y x y
=1⇒ + =1
12 4 3
⇒ 9y + 9 = −2x + 2
⇒ x-intercept = 4 and y-intercept = 3 Let the
⇒ 2x + 9y + 7 = 0
required line be
20. Ans (C)
x y
L: + = 1 where
x−y=4 a b

To find equation of R a = x-intercept and b = y − intercept

slope of L = 0 is 1 According to the question

⇒ slope of QR = −1 a = 4 × 2 = 8 and b = 3/2


x 2y
Let QR is y = mx + c ∴ Required line is + =1
8 3
y = −x + c ⇒ 3x + 16y = 24
−3 24
⇒y= x+
16 16
−3
Hence, required slope = .
16

23. Ans (C)


Since the required line is perpendicular to x-axis.
∴ Its slope is not defined.
So, equation of required line is x + 2 = 0
24. Ans (D)
x+y−c=0 x y
+ = 1 where a + b = −1 and
a b
distance of QR from (2,1) is 2√3
4 3
|2 + 1 − c| + =1
a b
2√3 =
√2 ⇒ a = 2, b = −3 or a = −2, b = 1.
⇒ 2√6 = |3 − c| x y x y
Hence − = 1 and + = 1.
2 3 −2 1
⇒ c − 3 = ±2√6c = 3 ± 2√6 25. Ans (D)
Line can be x +y = 3 ± 2√6 It is given that (1,1) is the midpoint of line AB.
⇒ x + y = 3 − 2√6 a+0 0+b
⇒1= ,1 =
21. Ans (D) 2 2
x y ⇒ a = 2 and b = 2
Let the line be + = 1
a b
Equation of line AB is
a b
(−3,4) = ( , ) x y
2 2 + =1
2 2
a = −6, b = 8
⇒x+y=2
equation of line is 4x − 3y + 24 = 0

MATHEMATICS Page | 6A. 4


STRAIGHT LINE

Here, p = 5 and α = 30∘


So, equation of line in normal form is
xcos α + ysin α = p
∴ xcos 30∘ + ysin 30∘ = 5
√3 1
⇒ x+ y=5
2 2
26. Ans (A)
⇒ √3x + y = 10
Any line ⊥ to 2x − y + 3 = 0 has slope
1
m= −
2
λ−3 λ−3
Slope of line through (4,3) and (2, λ) is =
2−4 −2

λ−3 1
=− ⇒λ=4
−2 2
27. Ans (D)
co-ordinates of point D are A-B+C= (4,7)
⇒ line AD is 5x − 3y + 1 = 0
30. Ans (B)
28. Ans (C)
1−0
Equation of line in intercept form is The slope of the line AB is = 1 or tan 45∘ . After
3−2
x y rotation of the line through 15∘ , the slope of the line
+ =1
a b
AC in new position is tan 60∘ = √3.
Given, a + b = 9
Therefore, the equation of the new line AC is
and line (i) passes through the point (2,2), it will
y − 0 = √3(x − 2) or y − √3x + 2√3 = 0
satisfy the line (i)
i.e., put x = 2, y = 2 in Eq. (i)
2 2
⇒ + =1
a b
Solve Eqs. (ii) and (iii), to find the values of a and b.
From Eq. (ii), b = 9 − a put in Eq. (iii), we get
2 2
+ =1
a 9−a
⇒ 2(9 − a) + 2a = a(9 − a) 31. Ans (B)

⇒ 18 − 2a + 2a = 9a − a2 Given, equation of lines are

⇒ a2 − 9a + 18 = 0 x − 7y + 5 = 0

Factorize it by splitting the middle term, and 3x + y = 0

⇒ a2 − 6a − 3a + 18 = 0 On solving (i) and (ii), we get,

⇒ a(a − 6) − 3(a − 6) = 0 −5 15
x= ,y =
22 22
⇒ (a − 6)(a − 3) = 0 5 15
Hence, the intersection point is (− , ).
⇒ a = 6 or 3 ⇒ b = 3 or 6 22 22

Hence, Eq. (i) becomes ∴ Equation of required line is


x y x y 15 1 5
+ = 1 or + = 1 y− = (x + )
6 3 3 6 22 0 22
⇒ 3x + 6y = 18 or 6x + 3y = 18 5
⇒0=x+
⇒ x + 2y = 6 or 2x + y = 6 (divide by 3) 22
29. Ans (B) (∵ Line is parallel to y-axis

MATHEMATICS Page | 6A. 5


STRAIGHT LINE

1 a2 c2
∴ m = tan 90∘ = ) and y = − x− …. (ii)
b2 b2
0
a1
⇒ 22x + 5 = 0 Slopes of the lines (i) and (ii) are m1 = − and
b1
a
32. Ans (A) m2 = − 2 , respectively.
b2
−a
Slope of the line ax + by = c is , Now,
b
−a′ I. Lines are parallel, if m1 = m2 , which gives
Also, slope of the line a′ x + b′ y = c ′ is .
b′
a1 a2 a1 a2
The lines are perpendicular if − =− or =
b1 b2 b1 b2

−a −a′ II. Lines are perpendicular, if m1 , m2 = −1, which


( )( ) = −1 or aa′ + bb′ = 0
b b′
gives
33. Ans (C)
a1 a 2
Let Q(h, k) be the image of the point P(1,2) in the ⋅ = −1
b1 b2
line x − 3y + 4 = 0 or a1 a 2 + b1 b2 = 0
Therefore, the line (i) is the perpendicular bisector 36. Ans (B)
of line segment PQ. x 3 − yx 2 + x − y = 0
⇒ x 2 (x − y) + (x − y) = 0
(x 2 + 1)(x − y) = 0
So only possibility is x = y as x 2 + 1 ≠ 0
So, it represents a straight line.
37. Ans (C)
Point of intersection of the two sides is A ≡ (1,2)
∴ Slope of diagonal AB = 2
1
⇒ Slope of another diagonal BD = −
2
y+2 1
−1 ⇒ Eq. of BD is =−
Hence, slope of line PQ = x+1 2
Slope of line x−3y+4=0
⇒ x + 2y + 5 = 0
k−2 −1
so that = 1 or 3h + k = 5 … (ii) 1 −8
h−1 ∴ Co-ordinates of D = ( , )
3
3 3
h+1 k+2
and the mid-point of PQ, i.e., point ( , ) will 38. Ans (D)
2 2

satisfy the equation (i) so that Consider the given equation of lines,
h+1 k+2 x − (t + α) = 0
− 3( ) + 4 = 0 or h − 3k = −3
2 2
y + 16 = 0
Solving (ii) and (iii),
6 7
and −αx + y = 0
we get h = and k = .
5 5 Since, these lines are concurrent, therefore the
Hence, the image of the point (1,2) in the line (i) is system of equations is consistent.
6 7
( , ). 1 0 −(t + α)
5 5
Now, | 0 1 16 | = 0
Alternatively, use T.S.R −α 1 0
34. Ans (B) ⇒ 1(0 − 16) − (t + α)(0 + α) = 0
3×1+3−9 3
Ratio = − = (Internally) ⇒ −16 − α(t + α) = 0
3×2+7−9 4

35. Ans (B) ⇒ α(t + α) + 16 = 0

Given lines can be written as ⇒ α2 + tα + 16 = 0


a1 c1 Clearly, α should be real.
y=− x− … (i)
b1 b1
∴ t 2 − 4 × 16 ≥ 0
MATHEMATICS Page | 6A. 6
STRAIGHT LINE

⇒ t 2 − 64 ≥ 0 The distance of this point from line (iii) is


Hence, least positive value of t is 8. −20 15
|5 × ( ) − 2 × | | − 130|
13 13 =
39. Ans (C)
√52 + (−2)2 13√29
Sides of the triangle are x − 3y = 0,
130 10√29
= = units
4x + 3y = 5 and 13√29 29

3x + y = 0 42. Ans (C)

By seeing the equation of the sides, we can easily Here, A = 3, B = −4, C1 = 7, and C2 = 5. Therefore,

find out that x − 3y = 0 and 3x + y = 0 are the required distance is

perpendicular to each other. |7 − 5| 2


d= =
√32 + (−4)2 5
So, they will cut each other at orthocenter.
Solving x − 3y = 0 and 3x + y = 0, 43. Ans (C)

we find x = 0,y = 0. Given equation of line is

When we put x = 0, ⇒ 12(x + 6) = 5(y − 2)

y = 0 in 3x − 4y = 0, ⇒ 12x + 72 = 5y − 10

we find 0 = 0 (satisfied). ⇒ 12x − 5y + 82 = 0

So, line 3x − 4y = 0 passes through orthocentre. Hence, distance from the point (−1,1) is

40. Ans (A) 12 × (−1) − 5 × 1 + 82


d=| |
√(12)2 + (−5)2
Let (h, k) be the point of reflection of the given point
−12 − 5 + 82
(4, −13) about the line 5x + y + 6 = 0. The mid- =| |
√144 + 25
point of the line segment joining points (h, k) and
82 − 17 65
h+4 k−13 =| |= =5
(4, −13) is given by ( , ) √169 13
2 2

This point lies on the given line, so we have 44. Ans (A)
h+4 k − 13 △ ABC is an equilateral triangle. AD is altitude.
5( )+ +6=0
2 2 Equation of BC is 2x − y = 1
or 5h + k + 19 = 0 So perpendicular distance,
Again, the slope of the line joining points (h, k) and 2(−1) − 2 − 1
k+13
AD = | |
(4,13) is given by . This line is perpendicular to √4 + 1
h−4
k+13 = | − √5| = √5
the given line and hence (−5) ( ) = −1
h−4

This gives 5k + 65 = h − 4
or h − 5k − 69 = 0 … ( ii )
On solving (i) and (ii), we get h = −1 and k = −14.
Thus, the point (−1, −14) is the reflection of the
given point.
41. Ans (D)
AD
The given lines are In △ ABD, = sin 60∘
AB
x − 3y + 5 = 0 √5 √3
⇒ =
3x + 4y = 0 AB 2
5x − 2y = 0 2√5 20
∴ AB = =√ units
√3 3
20 15
Solving (i) and (ii), we get (− , ) as their
13 13 45. Ans (A)
intersection point.
MATHEMATICS Page | 6A. 7
STRAIGHT LINE

Given line is 4x − y = 0. 5
4−
d= 2 = 3 = 3√2 > 1
In order to find the distance of the line from the 4
√1 + 12 2√2
2

point P(4,1) along another line, we have to find the Hence, no point lies in it
point of intersection of both the lines. 48. Ans (D)
For this purpose, we will first find the equation of k
+1
∘ According to the question, |2 | = 2
the second line. Slope of second line is tan 135 = 13

−1. Equation of the line with slope -1 and through k


⇒ + 1 = ±26 ⇒ k = 50 or -54 .
2
the point P(4,1) is
49. Ans (D)
Given, 3,4 are intercepts of a line.
x y
∴ Equation of line is + = 1
3 4

⇒ 4x + 3y − 12 = 0
Now, distance from origin is
4×0+3×0−12 12
| |= unit
√32 +4 2 5

50. Ans (D)


Let the slope be m of the straight line which passes
through
(−2,2), then equation (y − 2) = m(x + 2)
mx − y + (2m + 2) = 0
∵ Perpendicular distance of line (i) from point
y − 1 = −1(x − 4) or x + y − 5 = 0
(given) (3, −1) = 6
Solving (i) and (ii), we get x = 1 and y = 4 so that
|m(3) − (−1) + (12m + 2)|
point of intersection of the two lines is Q(1,4). ⇒ =6
√m2 + (−1)2
Now, distance of line (i) from the point P(4,1) along
|3m + 1 + 2m + 2| = 6√m2 + 1
the line (ii)
Squaring on both sides,
= distance between the points P(4,1) and Q(1,4)
⇒ (5m + 3)2 = 36(m2 + 1)
= √(1 − 4)2 + (4 − 1)2 = 3√2 units.
⇒ 25m2 + 9 + 30m = 36m2 + 36
46. Ans (D)
⇒ 11m2 − 30m + 27 = 0..(ii)
The given lines are
Now, Δ = B 2 − 4AC
5x − 12y + 65 = 0 … (i)
= 900 − 4(11)(27) < 0
and 5x − 12y − 39 = 0 … (ii)
∵ Discriminate of Eq. (ii) is negative. i.e., slope of the
65+39
Required distance = | |
√25+144 given line is imaginary. So, no line drawn through
|c1 − c2 | the point (2,2).
(∵ d = )
√a2 + b2 51. Ans (D)
104
=| | = 8 units A = (2, −3), B = (10, y)
13

47. Ans (D) AB = 10


Given lines are ⇒ (10 − 2)2 + (y + 3)2 = 100
x + y = 4 and 2x + 2y = 5 ⇒ (y + 3)2 = 36

or x + y =
5 ⇒ y + 3 = ±6
2
⇒ y = 3( or ) − 9
The distance between two parallel lines,

MATHEMATICS Page | 6A. 8


STRAIGHT LINE

∴ y1 + y2 = 3 − 9 = −6
3 2 √10
52. Ans (A) AB = √(1 − 0)2 + (3 − ) =
8 3
√a2 (cos θ − cos ϕ)2 + a2 (sin θ − sin ϕ)2 = 2a
BC = √(82 − 1)2 + (30 − 3)2 = 27√10
2 (cos 2 2 2
⇒a θ + cos ϕ − 2cos θcos ϕ + sin θ
+ sin2 ϕ − 2sin θsin ϕ) = 4a2 8 2 82√10
CA = √(82 − 0)2 + (30 − ) =
3 3
⇒ 1 + 1 − 2cos θcos ϕ − 2sin θsin ϕ = 4
Clearly AB + BC = CA
⇒ 2 = −2(sin θsin ϕ + cos θcos ϕ)
∴ A, B, C are collinear
⇒ −1 = cos (θ − ϕ)
59. Ans (C)
⇒ θ − ϕ = 2nπ ± π
1 x1 − x 3 x2 − x4
⇒ θ = 2nπ ± π + ϕ Area of Quadrilateral = |y − y y2 − y4 |=18
2 1 3
53. Ans (D) 60. Ans (D)
A = (0, b) on y − axis x1 +x2 +x3 y1 +y2 +y3 19
Centroid G = ( , ) = (5, )
3 3 3
B = (4,5)
61. Ans (B)
AB = C
O = (0,0), A = (a, 0), B = (0, b)
√16 + (b − 5)2 = c
O, A, B are form a right angle triangle
16 + (b − 5)2 = c 2
Right angle at O and circumcentre is midpoint of AB
(b − 5)2 = c − 16 < 0 not possible a b
Thus, circumcentre S = ( , ) and orthocentre is
54. Ans (D) 2 2

0=(0, 0)
A = (x1 , y1 ) = (−7,1), B = (x2 , y2 ) = (3,6)
P = (x, y) = (−1,4) a2 b 2 1 2
∴ OS = √ + = √a + b 2
P divides ̅̅̅̅
AB in the ratio x1 − x: x − x2 4 4 2
= −7 + 1: −1 − 3 62. Ans (C)
= −6: −4 = 3: 2 Given D = (−3, −5) and E = (3,3)
55. Ans (B) ∴ BC = 2DE
Let A(2, −3), P(10, y) = 2√36 + 64 = 20
Given AP = 10

√(10 − 2)2 + (y + 3)2 = 10


64 + (y + 3)2 = 100
(y + 3)2 = 36
y + 3 = ±6
y = 3 or − 9 63. Ans (D)
56. Ans (A) Let A(−2,2), B(1,0), C(x, 0), D(1, y)
1 mid point of AC = mid point of BC
Midpoint of PQ = midpoint of AB = ( , 3)
2
−2 + x 2 + 0 1+1 0+4
57. Ans (B) ⇒( , )=( , )
2 2 2 2
10 − 4 14 + 3 17 ⇒ −2 + x = 2, 2 = y
P=( , ) = (2, )
3 3 3
⇒ x = 4, y = 2
64. Ans (D)
A = E + F − D [∵ AEDF is a parallelogram
58. Ans (D) A = (4,3) + (6,4) − (1,2)

MATHEMATICS Page | 6A. 9


STRAIGHT LINE

A = (9,5)

68. Ans (A)


Given, A = (1, −2), C = (−5,6)
65. Ans (B) note that B, D are the missing vertices of right angled
2a = √9 + 16 triangle
a = 5/2 Thus, B, D =
length of median = 3a (x1 + x2 ) ± (y1 − y2 ) y1 + y2 ∓ (x1 − x2 )
[ , ]
2 2
1−2−5−6 6−5−2−1
B=( , ) = (−6, −1)
2 2
−5 + 6 + 1 + 2 −2 + 1 + 6 + 5
D=( , ) = (2,5)
2 2
5 5
= 3( ) =
2 2

66. Ans (B)


Area of △ ABC = 9
1
⇒ |6 × k| = 9 ⇒ 6|k| = 18
2
⇒ |k| = 3 ⇒ k = ±3

69. Ans (D)


A = (2,1), B = (3, −2)
C = (x1 , x1 + 3)
Area of triangle=5
2 3 x1 2
⇒| | = 10
1 −2 x1 + 3 1
⇒ |−4 − 3 + 3x1 + 9 + 2x1 + x1 − 2x1 − 6| = 10
67. Ans (B) ⇒ |4x1 − 4| = 10
x+4+4 y+2+5 ⇒ 4x1 = ±10 + 4
(2,3) = ( , )
3 3
x+8 y+7 ⇒ 7 −3
(2,3) = ( , ) x1 = ( or )
3 3 2 2
(x, y) = (−2,2) 7 13 −3 3
∴ real points are ( , ) , ( , )
2 2 2 2
1 4 2 1
∴ required area Δ = |4 5 1| 70. Ans (D)
2
−2 2 1
Number of points = 39 + 38 + 37 + ⋯ + 1
1 0 −3 0
= |4 5 1| 39 × 40
2 = = 780
−6 −3 0 2
1
= |18| = 9
2

MATHEMATICS Page | 6A. 10


STRAIGHT LINE

⇒ xa + xb + ya − yb = xa − xb + ya + yb
⇒ 2xb = 2yb ⇒ x = y ⇒ x − y = 0

71. Ans (C)


A = (3, −4), B = (−4,3), C = (x, y) 74. Ans (C)
centroid (a, b) Let, P(x, y) and A = (0,5)
3 − 4 + x −4 + 3 + y Given PA = 2|x|
(a, b) = ( , )
3 3 ⇒ PA2 = 4x 2
−1 + x −1 + y
=( , ) ⇒ x 2 + (y − 5)2 = 4x 2
3 3
⇒ y 2 − 10y + 25 = 3x 2
3a + 1 = x, y = 3b + 1
x−1 y−1 ⇒ 3x 2 − y 2 + 10y − 25 = 0
a= ,b =
3 3 75. Ans (C)
(a,b) satisfies 3x − 2y = 4 y y−b
× = −1
3a − 2b = 4 x x−a
x−1 y−1 ⇒ y(y − b) + x(x − a) = 0
3( ) −2( )=4
3 3 ⇒ x 2 + y 2 − ax − by = 0
3(x − 1) − 2(y − 1) = 12
3x − 2y − 3 + 2 = 12
3x − 2y = 13
72. Ans (A)
locus of P is circle
76. Ans (B)
(i)Image (4,1) w.r.t y = x is (1,4)
(ii) Translation 2 units along +ve x - axis
(2,0) + (1,4) = (3,4)
77. Ans (A)
(i) (−2,3)

73. Ans (B) (ii) Translation 3 units along the negative y-axis =

PA = PB (−2,0)

⇒ PA2 = PB 2
⇒ [x − (a + b)]2 + [y − (a − b)]2
= [x − (a − b)]2 + [y − (a + b)]2
⇒ x 2 + (a + b)2 − 2x(a + b) + y 2 + (a − b)2 − 2y(a
− b)
= x 2 + (a − b)2 − 2x(a − b) + y 2 + (a + b)2 + 2y(a
+ b)
⇒ −2x(a + b) − 2y(a − b)
= −2x(a − b) + 2y(a + b) 78. Ans (B)

MATHEMATICS Page | 6A. 11


STRAIGHT LINE

(x1, y1) = (0, 0) 4 −4


Slope of ̅̅̅̅
PQ m2 = =
(x1+h, y1+k) = (1, 2) ⇒ (h, k) = (1,2), −7 7
⇒ ̅̅̅̅
AB parallel to ̅̅̅̅
PQ
Now, (x2, y2) = (4, 3)
∴ Angle between ̅̅̅̅
AB and ̅̅̅̅
PQ is '0'
(x2+h, y2+k) = (5, 5)
84. Ans (B)
79. Ans (A)
5−4
Area of triangle does not change in any system tan 135∘ =
x−3
Thus, required area= area of △ PQR = 43 1
⇒ −1 = ⇒ −x + 3 = 1
80. Ans (C) x−3
Let O=(0, 0), P=(2√2, √2) and Q=(x, y) ∴x=2

Now angle between OP and OQ is 45° 85. Ans (A)

Verify the options P = (2, −3), A = (1,2), B = (−1,5)


5−2 3 −3
81. Ans (D) Slope of AB = = =
−1−1 −2 2
mid point = (1,2) required line is y + 3 =
−3
(x − 2)
2
2−6
Slope of line through (−3,6) & (1,2) is = ⇒ 2y + 6 = −3x + 6
1+3

m = −1 ⇒ tan θ = −1 ⇒ 3x + 2y = 0
π 3π
⇒θ=π− ⇒θ=
4 4
82. Ans (B)
A = (0, b), B = (0,0), C = (a, 0)
a a b
D = ( , 0) , E = ( , )
2 2 2 86. Ans (D)
̅̅̅̅ ̅̅̅̅
AD ⊥ BE P=(2,3), A=(-5,6) and B = (−6,5)
̅̅̅̅ × slope of BE
Slope of AD ̅̅̅̅ = −1 1
Slope of ̅̅̅̅
AB = = 1
1
b b/2
⇒ × = −1 ̅̅̅̅ = −1
Slope of the line perpendicular to AB
(−a/2) a/2
−2b2 Required Equation, y − 3 = −1(x − 2)
⇒ = −1
a2 ⇒x+y=5
2 2
⇒ a = 2b 87. Ans (C)
⇒ a = ±√2b mid point of ̅̅̅̅
AB = P(2,2)
−1 −1
Slope of ̅̅̅̅
CD = = = −3
̅̅̅̅ 1/3
Slope of AB
∴ eqn. of ̅̅̅̅
CD y − 2 = −3(x − 2)
⇒ y − 2 = −3x + 6
⇒ 3x + y = 8 ⇒ 3x + y − 8 = 0

83. Ans (C)


A = (2, −3), B = (−5,1)
P = (7, −1), Q = (0,3)
88. Ans (A)
4 −4
̅̅̅̅ m1 =
Slope of AB = AB = √(x2 − x1 )2 + (y2 − y1 )2
−7 7
MATHEMATICS Page | 6A. 12
STRAIGHT LINE

̅̅̅̅ Parallel to x-axis ⇒ y1 = y2


∵ AB
∴ AB = |x2 − x1 |
89. Ans (A)
m = tan 15∘ = 2 − √3
equation of line y + 4 = (2 − √3)(x − 0)
⇒ y + 4 = (2 − √3)x
⇒ (2 − √3)x − y − 4 = 0 93. Ans (A)
−1 − 3 3 + 5
D=( , ) = (−2,4)
2 2
−9 − 4 −13
Slope of CD = =
7+2 9
Equation of CD is 13x + 9y = −26 + 36
⇒ 13x + 9y − 10 = 0

90. Ans (D)


a b
( , ) = (1,2)
2 2
⇒ a = 2; b = 4
x y 94. Ans (D)
Therefore, equation of the line is + = 1
2 4
y-intercept = −2
⇒ 2x + y = 4
1
91. Ans (A) slope m = tan 30∘ =
√3
Given b = 2a
equation of the line is y = mx + c
x
The equation of the line in the intercept form is + 1
a
y ⇒y= x−2
=1 √3
b
x y ⇒ √3y = x − 2√3
⇒ + =1
a 2a 95. Ans (A)
⇒ 2x + y = 2a
Eqn. ̅̅̅̅
AC ⇒ y = x
passing through (1,2) ̅̅̅̅ ⇒ y + x = 1
& eqn. of BD
⇒ 2 + 2 = 2a ⇒ a = 2
∴ Required line is 2x + y = 4
92. Ans (A)
a b
∴ (3,2) = ( , )
2 2
a = 6, b = 4
x y
∴ Eqn. of line + = 1 ⇒ 2x + 3y = 12
6 4
96. Ans (C)
A = (3, −4), B = (5,2)
̅̅̅̅ is P = (4, −1)
midpoint of AB
a 2
=
b 1

MATHEMATICS Page | 6A. 13


STRAIGHT LINE

⇒ a = 2b ⇒ x1 + y1 = 2
equation of the line in the intercept form is
x y
+ =1
a b
x y
⇒ + =1
2b b
⇒ x + 2y = 2b
4 − 2 = 2b
b=1
100. Ans (B)
required line x + 2y = 2 4
Required line = y − 2 = 11 (x − )
97. Ans (D) 3
a + b = −1 11
⇒y−2= (3x − 4)
3
The equation of the straight lines in the intercept
⇒ 3y − 6 = 33x − 44
form is
x y ⇒ 33x − 3y − 38 = 0
+ =1
a b
4 3
+ =1
a b
⇒ 4b + 3a = ab 101. Ans (A)
4(−1 − a) + 3a = a(−1 − a) P = (3,2)
−4 − 4a + 3a = a = a2 ̅̅̅̅ is y − y1 = m(x − x1 )
Equation of BD
a2 = 4 ⇒ y − 2 = 2(x − 3)
a = ±2 ⇒ 2x − y − 4 = 0
case (i): a = 2, b = −3
x y
− =1
2 3
case (ii): a = −2, b = 1
x y
+ =1
−2 1
98. Ans (C) 102. Ans (B)
A = (a, 0), B = (0, b) x-y=a
Let P = (x1 , y1 ) is midpoint of ̅̅̅̅
AB (3, -1)⇒ a = 4
a b ∴ Equation of the line line x − y = 4
⇒ (x1 , y1 ) = ( , )
2 2
103. Ans (A)
⇒ a = 2x, b = 2y
1−0
∴ a + b = 10 Slope of ̅̅̅̅
AB = =1
3−2
⇒ x1 + y1 = 5 ⇒ tan θ = 1 ⇒ θ = 45∘
Locus of P is x + y = 5 ∴ Slope of ̅̅̅̅
AC = tan 60∘ = √3
99. Ans (C) and eqn. of ̅̅̅̅
AC is y − 0 = √3(x − 2)
̅̅̅̅. The locus of P is
Let P(x1 , y1 ) is the midpoint of AB
y = √3x − 2√3
x y
+ =2
x1 y1 ⇒ √3x − y = 2√3
⇒ a = 2x1 , b = 2y1
⇒a+b =4
⇒ 2x1 + 2y1 = 4
MATHEMATICS Page | 6A. 14
STRAIGHT LINE
x
The equation of the line having intercepts a, b is +
a
y
=1
b
x y
⇒ + =1
8 9
⇒ 9x + 8y = 72
104. Ans (C) 108. Ans (C)
−π A = (3,3), B = (7,6)
P = 10, α =
4
Equation of ̅̅̅̅
AB is 3x − 4y = −3
eqn. of line xcos α + ysin α = P x y
x y ⇒ + =1
− = 10 −1 (3/4)
√2 √2
9 5
⇒ x − y = 10√2 ∴ AB = √1 + =
16 4
105. Ans (B)
A line meets x-axis at P and y-axis at Q
Let OP = a, OQ = b
⇒ P = (a, 0), Q = (0, b)
A divide ̅̅̅̅
PQ in the ratio 1: 2
2a b
⇒ A = (4,3) = ( , )
3 3
12 109. Ans (C)
⇒a= , b = 9, ⇒ a = 6, b = 9
2 −x − y = √2
x y
The equation of PQ is + =1 −1 −1
a b
⇒ ( )x + ( )y = 1
x y √2 √2
+ = 1 ⇒ 3x + 2y = 18
6 9 5π 5π
⇒ xcos + ysin =1
4 4
110. Ans (D)
2x − 3y = −6
x y
⇒ + =1
−3 2
x y
from given line + =1
−8 −8
( ) ( )
a b
106. Ans (A)
−8 −8
−√3x + y = 8 = 3, = −2
a b
divide √3 + 1 = 2 −8
⇒a= ,b = 4
3
−√3x 1
+ y=4 111. Ans (C)
2 2
cos (150∘ )x + sin 150∘ y = 4 (cos θ + sin θ)x + (sin 2θ)y = 1

α = 150∘ , P = 4 −(sin θ + cos θ) π


slope m = at θ =
sin 2θ 6
107. Ans (B)
1 √3
3x + 4y = 12 −[ + ]
2 2
x y K=m=
+ =1 √3
4 3 2
let a = 8, b = 9 (√3 + 1)
=−
√3
MATHEMATICS Page | 6A. 15
STRAIGHT LINE

1 ⇒ k 2 = 10 × 36
= − [1 + ] = −1.574
√3 ⇒ k = ±6√10
⇒ [K] = −2
⇒required number of lines = 2
112. Ans (A)
√3x + y = 1 → (1)
Slope of (1) is m1 = −√3
Let slope of the required line m2 = m
m1 −m2
Thus, tan θ = | |
1+m1 m2

−√3 − m m + √3
√3 = | | =| |
1 − √3m 1 − m√3 115. Ans (B)

⇒ m + √3 = ±√3(1 − √3m) = ±(√3 − 3m) x + y + 2 = 0 → (1)


A = (−2, −1)
⇒ m + √3 = √3 − 3m or m + √3 = −√3 + 3m
| − 2 − 1 + 2| 1
⇒ 4m = 0 or 2 m = 2√3 AP = =
√2 √2
⇒ m = 0 or m = √3 AP
From the figure, sin 60∘ =
a
case (i) : m = 0, P = (3, −2)
√3 1
Equation of the line ⇒ =
2 √2a
y+2 =0
2
case (ii) : m = √3, P = (3, −2) ⇒a=√
3
Equation of the line, y + 2 = √3(x − 3)
y + 2 = √3x − 3√3
⇒ √3x − y − 2 − 3√3 = 0
113. Ans (A)
2b = a + c
⇒ a − 2b + c = 0 → (1)
116. Ans (B)
ax + by + c = 0 → (2)
7x − y + 8 = 0 → (1)
(1) and (2) represents same line
A = (−4,5)
x y 1
⇒ = = The equation of ̅̅̅̅
AC is perpendicular to (1) passing
1 −2 1
So,(1, −2) through A(−4,5)

114. Ans (B) x + 7y = −4 + 35

2x + 6y − 7 = 0 → (1) ⇒ x + 7y = 31
The equation of the line parallel to (1) is
2x + 6y = k
k
x-intercept =
2
k
y-intercept =
6

AB = 10 117. Ans (B)


3x + 4y + 6 = 0
k2 k2
⇒√ + = 10 The equation of the line perpendicular to (1) passing
4 36
through (0,0) is 4x − 3y = 0
⇒ 40k 2 = 100 × 4 × 36

MATHEMATICS Page | 6A. 16


STRAIGHT LINE

118. Ans (D)


1 2
Slope of required line is =−
(−9/2) 9
2
Equation is y + 1 = − (x − 1)
9

⇒ 9y + 9 = −2x + 2 or 2x + 9y + 7 = 0

121. Ans (B)


2x − 3y + 7 = 0 → (1)
The equation of the line perpendicular to (1) is
3x + 2y + c = 0 → (2)
Area of the triangle formed by (2) with co-ordinate
axes = 3
c2
119. Ans (B) ⇒ =3
2|ab|
x + 2|y| = 1 → (1) and x = 0 → (2)
c2
from (1) x ± 2y = 1 ⇒ =3
12
⇒ x + 2y = 1 → (3) ⇒ c = ±6
⇒ x − 2y = 1 → (4) Required line is 3x + 2y = ±6
Point of intersection of (3) and (4) is A = (1,0) 122. Ans (A)
points of intersections (1) and (2) −4
O = (0,0), P = (3, −4) ⇒ slopeof OP =
1 −1 3
B = (0, ) , C = (0, )
2 2 The equation of the line perpendicular to ̅̅̅̅
OP and
1 1 1 1
Area of triangle ABC = | + |= passing through P is
2 2 2 2
3
120. Ans (A) y + 4 = (x − 3)
4
A = (1,3), C = (7,5) ⇒ 3x − 4y = 25
2 1
Slope of ̅̅̅̅
AC m1 = =
6 3

̅̅̅̅ m2 = m
Let slope of AB
∠BAC = 450
m1 − m2
⇒ tan 45∘ = | |
1 + m1 m2
1
−m 123. Ans (D)
⇒ 1 = |3 m|
1+ Equation of AB is x. cos α + y. sin α=p
3
x cosα y sinα
⇒ 3 + m = ±(1 − 3m) ⇒ + =1
p p
⇒ 3 + m = 1 − 3m (or) 3 + m = −1 + 3 m
x y sinα
⇒ 4 m = −2 (or) 2 m = 4 ⇒ p + p =1
( ) ( )
cosα sinα
−1
⇒m= (or)m = 2 p p
2 So co-ordinates A and B are ( , 0) and (0, );
cos α sin α
The required equation of the line passing through ' A So, coordinates of midpoint of AB are
' p p
( , ) = (x1 , y1 )
−1 2cos α 2sin α
y−3 = (x − 1)(or)y − 3 = 2(x − 1) p p
2 Let x1 = , y1 =
2cos α 2sin α
⇒ x + 2y − 7 = 0(or)2x − y + 1 = 0

MATHEMATICS Page | 6A. 17


STRAIGHT LINE
p p
⇒ cos α = and sin α = 127. Ans (A)
2x1 2y1
m1 − m2
p2 1 1 tan θ = | |
′ 2 2
we ve cos α + sin α = 1 ⇒ ( 2 + 2 ) = 1 1 + m1 m2
4 x1 y1
m+1
1 1 4 ⇒ tan 60∘ = | |
Locus of (x1 , y1 ) is + = 1−m
x2 y2 p2
m+1
⇒ √3 = | |
1−m
⇒ √3 ∙ (1 − m) = (m + 1) or
√3(1 − m) = −m − 1
⇒ √3 − √3m = m + 1 or √3 − √3m = −m − 1
⇒ m(√3 + 1) = √3 − 1 or √3 + 1 = (√3 − 1)m

124. Ans (B) √3 − 1 √3 + 1


⇒m= or m =
a 2 √3 + 1 √3 − 1
=1
24 ⇒ m = 2 − √3 or m = 2 + √3
a = 2√6(∵ a > 0) ∴ required lines: y-3 = (2 ± √3)(x − 2)
125. Ans (A)
Equation of line passing through A = (2,3) with
slope m is (y − 3) = m(x − 2)
⇒ mx − y − 2m + 3 = 0 → (1)
c2
Area = 12 ⇒ = 12
2|ab|
(3 − 2m)2
⇒ = 12 128. Ans (B)
2|m|
case1: 9 + 4m2 − 12m = −24m Let (a, b) be coordinates of B

⇒ 4m2 + 12m + 9 = 0 From diagram, it is clear that midpoint of BQ is equal

⇒ (2m + 3)2 = 0 to midpoint of RP

−3 a−1 b−2 3+2 3+1


⇒m= ( , )=( , )
2 2 2 2 2
case2: 4m2 − 12m + 9 = 24m
⇒ a = 6 and b = 6
⇒ 4m2 − 36m + 9 = 0
No real values of ' m ' ⇒ coordinates of point B are (6,6)

1−6
126. Ans (A) ∴ equation of BC is y − 6 = (x − 6)
2−6

A = (5,6), B = (1, −4)


⇒ 4y − 24 = 5x − 30
m = midpoint = (3,1)
10 5 ⇒ 5x − 4y = 6
Slope of ̅̅̅̅
AB = =
4 2

Equation of perpendicular of ̅̅̅̅


AB is
−2
y−1 = (x − 3)
5
⇒ 5y − 5 = −2x + 6
⇒ 2x + 5y − 11 = 0
121
∴ Required area =
20

MATHEMATICS Page | 6A. 18


STRAIGHT LINE

129. Ans (C) The equation of the diagonal not passing through
Required distance A, B is
y − 2 = −3(x − 2)
0 + 0 + 25 25
OP = | |=| |=5 ⇒ 3x + y − 8 = 0 → (1)
√32 + 42 5
The perpendicular distance from (1,2) to (1)
By Pythagoras theorem AP = PB = 12 |3 + 2 − 8| 3
= =
1 √10 √10
So, Area of △ OAB = × 24 × 5 = 60
2
133. Ans (B)
x y
Given line equation is + = 1, passing through
5 b

(13, 32)
13 32
⇒ + =1
5 b
we get b = −20
x y
thus, line L is − = 1 ⇒ 4x − y = 20 ... (1)
5 20
130. Ans (C) ⇒ Slope = 4.
3x + 4y + 7 = 0 x y
Now, the line K: + = 1
c 3
⇒ 6x + 8y + 14 = 0 → (1) −3
Its slope is
and 6x + 8y + k = 0 → (2) c

−3 −3
The distance between (1) and (2) = 4 ∴ =4⇒c=
c 4
|14 − k|
⇒ =4 Thus K: 4x − y = −3
√36 + 64
The lines are
⇒ |14 − k| = 40
4x − y = 20
⇒ 14 − k = ±40
4x − y = −3
⇒ k = 54 (or ) − 26
|−20−3| 23
131. Ans (A) Required distance = =
√16+1 √17

Let P=(2, 3) 134. Ans (A)


And L: 2x − 3y + 9 = 0 → (1) 2x − 3y − 4 = 0 → (1)
P = (1,1)
x + y = 1 → (2)
The equation of the parallel to (1) passing through P
is
x + y = 2 → (3)
sol (1) and (2), we get Q(2,0)
Equation of AP: y − 3 = 1(x − 2)
PQ = √1 + 1 = √2
⇒ x − y + 1 = 0… (2)
135. Ans (D)
Solve (1) and (2), we get A=(6, 7)
6x + 8y − 6 = 0
Thus, AP=√32 = 4√2
6x + 8y − 1 = 0
132. Ans (C)
Now, 2r = distance between the lines
A = (−1,1), B = (5,3) 5 5 1
2 1 = = =
slope of AB = = √36 + 64 10 2
6 3
̅̅̅̅ is P = (2,2) 1
midpoint of AB ⇒r=
4
MATHEMATICS Page | 6A. 19
STRAIGHT LINE

Circumference = 2πr |ax1 + by1 + c| 28


= , y1 < 0
π 22 11 √a2 + b2 √29
= = =
2 2×7 7 |6 − 5k + 2| 28
⇒ =
√29 √29
⇒ 8 − 5k = ±28
⇒ 5k = 8 − 28 = −20
⇒ k = −4
140. Ans (D)
136. Ans (C) L ≡ 3x − 5y + a = 0
L1: 5x + 12y + 1 = 0 → (1)
P = (1,2)
L2: 5x + 12y + 7 = 0 → (2)
6
⇒ distance between L1 and L2 is Q = (3,4)
13

Now L3 is parallel to L1 and L2 such that L11 = a − 7


Distance between L1 and L3 = Distance between L2
3 L12 = a − 11
and L3=
13

Verify the options now P,Q lies on same side of L = 0

137. Ans (B) L11 L22 > 0


|5 − 2| |2 + 5|
d1 : d2 = : = 3: 7
√25 √25 ⇒ (a − 7)(a − 11) > 0
⇒ a ∈ (−∞, 7) ∪ (11, ∞)

141. Ans (A)


L ≡ x + 2y − k = 0
A = (2,3), B = (−1,2)
L11 = 8 − k, L22 = 3 − k
L = 0 divides A,B in the ratio − L11 : L22 = 3: 4
−(8 − k) 3
⇒ =
3−k 4
138. Ans (C) ⇒ −32 + 4k = 9 − 3k
−14 ⇒ 7k = 41
a= ,b = 7
4 41
⇒k=
7
2x y
⇒ line AB: + = 1 ⇒ −2x + y = 7 142. Ans (A)
−7 7
L≡x−y=0
|−2+1−7| 8
perpendicular distance d = = A = (2,3), B = (8,6)
√5 √5

⃗⃗⃗⃗⃗ in the ratio − L11 : L22 = 1: 2


L = 0 divides AB
(use T.S.R)
143. Ans (A)
L11 = 5 > 0
L22 = cos θ + sin θ − 1 > 0
⇒ cos θ + sin θ > 1
π
139. Ans (D) ⇒ θ ∈ (0, )
2
MATHEMATICS Page | 6A. 20
STRAIGHT LINE

144. Ans (B) 1 −6 a


|2 3 4| = 0
−L11 λ
= 1 4 1
L22 1
−(−1 + 1 − 4) λ ⇒ 1(3 − 16) + 6(2 − 4) + a(8 − 3) = 0
⇒ =
(5 + 7 − 4) 1
⇒ −13 − 12 + 5a = 0 ⇒ 25 = 5a ⇒ a = 5
1 λ
⇒ =
2 1
1
⇒λ=
2 148. Ans (C)
Given lines
145. Ans (A) x y
+ = 1 → (1)
a b
−1 15
Equation of PQ: y + 1 = 3
+
7 7 x y
7 25 (x − ) + = 1 → (2)
3 −
3 7
3 b a
−β−19 8 points intersection of (1) and (2)
⇒ (3y + 1) = (3x − 7)
13 ab ab
13 P=( , )
a+b a+b
⇒ 39y + 13 = −27x + 133 The equation of ̅̅̅̅
OP is x = y
⇒ 57x + 39y − 120 = 0 149. Ans (B)
⇒ 19x + 13y − 40 = 0 Q = (5,3)
R = (5, −3)
150. Ans (A)
P = (−2,6)
Q = (4,2)
Midpoint of PQ is m = (1,4)
4 −2
slope of ̅̅̅̅
PQ = =
−6 3

The equation of L = 0, is the perpendicular bisector


146. Ans (C) of ̅̅̅̅
PQ
x + y − 2 = 0 → (1) 3
y − 4 = (x − 1)
A + 2D 2
= G(0,0) ⇒ A = −2D ⇒ 2y − 8 = 3x − 3
3
̅̅̅̅
AD ⊥ ̅̅̅̅
BC ⇒ 3x − 2y + 5 = 0
̅̅̅̅ is x − y = 0 → (2)
Equation of AD 151. Ans (A)
from (1) & (2) D = (1,1) 2x − y + 3 = 0 → (1)
∴ A = (−2, −2) P = (1,2)
Midpoint of AB
= foot of the perpendicular from P on (1) = Q
Using T.S.R on foot of perpendicular,
−1 13
we get, Q = ( , )
5 5

147. Ans (D)

MATHEMATICS Page | 6A. 21


STRAIGHT LINE

(√3, 0) and (0, −1)


x y
∴ + =1
√3 −1
x − √3y = √3
⇒ √3y = x − √3

152. Ans (C)


h − 2 k − 3 −(2 + 3 − 3)
= =
1 1 1+1
⇒ h = 1; k = 2 ⇒ (h, k) = (1,2)
153. Ans (B) 156. Ans (C)

Image of A(2, 3) w.r.t Y-axis is A = (−2,3) m1 = −m2
B = (−1, −1), C = (−4, λ) are collinear 3 2
⇒ = −( )
Slope A’B=Slope of BC 5−a 1−a
⇒ 3 − 3a = −10 + 2a
⇒ λ = 11
⇒ 13 = 50
13
⇒a=
5

154. Ans (B) 157. Ans (B)


B is the image of A xtan 150 − y = 0
̅̅̅̅
L = 0 is perpendicular bisector of AB xsin 150 − ycos 15∘ = 0 → (1)
M = midpoint of ̅̅̅̅
AB = (4,6) P = (cos 750 , sin 75∘ )
Equation of L = 0 is y − 6 = −1x − 4 = (sin 15∘ , cos 15∘ ) = (x1 , y1 )
⇒ x + y = 10 Let Q = (h, k) is reflected of P w.r.to (1)
h − sin 150 k − cos 150
⇒ =
sin 150 −cos 150
−2(sin2 150 − cos 2 150 )
=
1
= 2cos 30∘ = √3
⇒ h = √3 sin 150 + sin 150
and k = −√3cos 150 + cos 150
155. Ans (D) √3−1 √3+1
⇒ h = (√3 + 1) ( ) & K = (1 − √3)
2√2 2√2
x y
+ =1 1 −2
√3 1 ⇒h= ,K =
√2 2√2
Reflected ray passes through

MATHEMATICS Page | 6A. 22


STRAIGHT LINE

1 −1 Given that: tan θ =


3
⇒Q=( , ) 5
√2 √2 3
⇒ Slope of the line m =
5

158. Ans (A) So, the equation of the line is

2x + 3y + 4 = 0 → (1) y − y1 = m(x − x1 )

A = (1,2) = (x1 , y1 ) 3
⇒ y+3= (x − 0)
5
B = (α, β) is the image of a with respective (1)
⇒ 5y + 15 = 3x
α − 1 β − 2 −2(ax1 + by1 + c)
= = ⇒ 5y − 3x + 15 = 0
2 3 a2 + b 2
α − 1 β − 2 −2(12) −24 162. Ans (A)
= = =
2 3 13 13 Intercept form of a line is
48 −72 x y
α= + 1, β = +2 + =1
13 13 a b
−120 + 39 −81 x y
α+β= = + =1 [∵ a = b
a a
13 13
⇒ x+y= a
⇒ y = −x + a
∴ Slope is −1
163. Ans (B)
Equation of line ' l ' is given by
y − y1 = m(x − x1 ).
Since ′l′ passing through the point P(3,2).
159. Ans (B) ∴ y − 2 = m(x − 3)
P = (4, −1) ⇒ y = mx + 2 − 3m
y = x → (1) Since it is given that lines y = x and ' l ' are
PQ = 2 × ( The perpendicular from P to perpendicular to each other,
x − y = 0) ∴ m × 1 = −1[∵ m1 × m2 = −1]
|4 + 1| Put m = −1 in eqn. (i), we get
= 2( ) = 5√2
√2 y = −x + 2 − 3(−1)
y = −x + 5
x+y=5

160. Ans (B)


a b −c
Using concurrent condition, |b c −a | = 0 164. Ans (B)
c a −b Equation of any line perpendicular to the given line
⇒a+b+c=0 x + y + 1 = 0 is x − y + k = 0.
161. Ans (A) If eq. (i) passes through the point (1,2),
Since the lines cut off intercepts −3 on y-axis, the then 1 − 2 + k = 0 ⇒ k = 1
line is passing through the point (0, −3). Putting the value of k in eq. (i),
MATHEMATICS Page | 6A. 23
STRAIGHT LINE

we have x − y + 1 = 0 ⇒ y − x − 1 = 0 Let m2 be the slope of the required line


165. Ans (C) m1 − m2
∴ tan θ = | |
1 + m1 m2
First equation of line having intercepts on the axes
x y −√3 − m2
a, −b is − = 1 ⇒ bx − ay = ab ⇒ tan 60∘ = | |
a b 1 + (−√3)m2
Second equation of line having intercepts on the −√3 − m2
x y ⇒ √3 = ± ( )
axes b, −a is − = 1 ⇒ ax − by = ab 1 − √3m2
b a

Slope of eq. (i) m1 =


b −√3 − m2
a ⇒ √3 = [taking (+) sign]
a
1 − √3m2
Slope of eq. (ii) m2 =
b
⇒ 2m2 = 2√3 ⇒ m2 = √3
m1 − m2
tan θ = | | −√3−m2
1 + m1 m2 & √3 = − ( ) [taking (−) sign]
1−√3m2
b a
− b2 − a2 √3 + m2
= a b = √3 = ⇒ 4m2 = 0 ⇒ m2 = 0
ab 2ab 1 − √3m2
1+
ba
∴ Equation of line passing through (3, −2) with
166. Ans (D)
slope √3 is
Equation of line passing through the points (2, −3)
y+2= √3(x − 3) ⇒ √3x − y − 2 − 3√3 = 0
and (4, −5) is
−5 + 3 and the equation of line passing through (3, −2)
y+3 = (x − 2)
4−2 with slope 0 is
−2 y + 2 = 0(x − 3) ⇒ y + 2 = 0
⇒ y+3= (x − 2)
2
169. Ans (A)
⇒ y + 3 = −(x − 2)
√3
Distance of the line from origin is
⇒ y + 3 = −x + 2 2

⇒ x + y = −1 Equation of any line passing through (1,0) is


x y y − 0 = m(x − 1) ⇒ mx − y − m = 0
⇒ + = 1 (Intercept form)
−1 −1
√3 m×0−0−m
∴ a = −1, b = −1 ∴ =| |
2 √1 + m2
167. Ans (A)
√3 −m
Given equations are: =| |
2 √1 + m2
2x − 3y + 5 = 0, 3x + 4y = 0 Squaring both sides, we get
20 15
On solving, Point of intersection is (− , ). 3 m2
17 17 =
4 1 + m2
Now perpendicular distance from the point
⇒ 4m2 = 3 + 3m2
20 15
(− , ) to the given line 5x − 2y = 0 is
17 17 ⇒ m2 = 3 ∴ m = ±√3
20 15 −100 30
5 (− ) − 2( )
| 17 17 | = | 17 − 17| = 130 ±√3x − y ∓ √3 = 0
√25 + 4 √29 17√29 √3x − y − √3 = 0 And -√3x − y + √3 = 0
⇒ √3x + y − √3 = 0
168. Ans (A) 170. Ans (B)
Equation of line is given by Given equations are

√3x + y + 1 = 0 y = mx + c1 and y = mx + c2

⇒ y = −√3x − 1 Since slopes of eq. (i) and eq. (ii) are same, they are
parallel lines.
∴ Slope of this line, m1 = −√3
MATHEMATICS Page | 6A. 24
STRAIGHT LINE

|c1 −c2 | Different form of equation of straight lines are Slope


∴ Distance between the two lines = .
√1+m2
intercept form,
171. Ans (B)
y = mx + c, Parameter = 2
Use T.S.R on foot of perpendicular x y
Intercept form, + = 1, Parameter = 2
172. Ans (A) a b

slope-point form, y − y1 = m(x − x1 ), Parameter = 2


Normal form, xcos α + ysin α = P, Parameter = 2
176. Ans(B)
(i) Reflection of (4, 1) w.r.t y=x is (1, 4)
(ii) The point after translation is
(1 + 2, 4) = (3,4).
Let the given line meets the axes at A(a, 0) and 177. Ans(C)
B(0, b). Verify the options by using distance between point
Given that C(3,2) is the mid-point of AB and line
a+0 0+b 178. Ans (D)
3= ⇒ a= 6&2= ⇒b=4
2 2
Any line perpendicular to 3x + y = 3 is
Intercept form of the line AB
x y x − 3y = λ. If it passes through the point (2,2) then
+ =1
a b
2 − 3(2) = λ ⇒ λ = −4
x y
So, + = 1 ⇒ 2x + 3y = 12
6 4 ∴ Required equation is x − 3y = −4
173. Ans (C) ⇒ − 3y = −x − 4
Given equation is y = 3x − 1 1 4
⇒ y = x + [∵ y = mx + c]
Slope of the line passing through the given point 3 3
4
(1,2) and parallel to the given line = 3 So, the y-intercept is .
3

So, the equation of the required line is 179. Ans (B)


y − 2 = 3(x − 1) Clearly, eq. (i), (ii) and (iii) are parallel to each other
174. Ans (A) as the coefficients of x and y are same.
Given equation x = 0, y = 0, x = 1 and y = 1 form a
square of side 1 unit

Distance between parallel lines ( i ) and (iii) we get


5−2 3
| |=
√(3)2 +(4)2 5

From figure, we get that OABC is square having ∵ Distance between two
[ parallel lines = | c1 − c2 |]
corners O(0,0), A(1,0), B(1,1) and C(0,1) Equation of
√a2 + b 2
diagonal AC: Distance between parallel lines (ii) and (iii) we get
1−0
y-0= (x − 1) ⇒ y + x = 1 −5 − 2 7
0−1
| |=
1−0 2
√(3) + (4)2 5
Equation of diagonal OB is y= (x − 0) ⇒y=x
1−0
3 7
∴ Ratio between the distances = : = 3: 7
175. Ans(B) 5 5

180. Ans (C)


MATHEMATICS Page | 6A. 25
STRAIGHT LINE

Let ABC be an equilateral triangle with vertex ⇒ y − intercept = 4/3


(x1 , y1 ). 183. Ans(A)
AD ⊥ BC and let (a, b) be the coordinates of D. a1 a 2 + b1 b2 = 0
Given that the centroid G lies at the origin i.e., (0,0) −2
2 + 3k = 0 ⇒ k =
Since, the centroid of a triangle, divides the median 3
184. Ans (A)
in the ratio 1: 2
1×x1 +2×a Verify the options
So, 0 =
1+2
185. Ans (A)
⇒ x1 + 2a = 0
1 K 0 1
and 0 =
1×y1 +2×b
⇒ y1 + 2b = 0 |4 0 1| = 4
1+2 2
0 2 1
Equation of BC is given by x + y − 2 = 0 ⇒ |K(−2) + 1(8)| = 8
D (a, b) lies on the line x + y − 2 = 0 ⇒ −2K + 8 = ±8
So a + b − 2 = 0 ⇒ −2K = −16( or ) − 2K = 0
Slope of BC is = −1 ⇒ K = 8 (or) K = 0
y1 −0 y1
and the slope of AG = = 186. Ans (A)
x1 −0 x1

Since, they are perpendicular to each other Given 3x-4y+2=0 and P =(-2,3)
y1 The equation of the line parallel to above line and
∴ − 1 × = −1 ⇒ y1 = x1
x1 passing through P is 3x-4y=-6-12
From eq. (i) and (ii) we get x1 + 2a=0 ⇒3x-4y+18=0
⇒ 2a = −x1 187. Ans (A)
y1 + 2b = 0 ⇒ 2b = −y1 x + y = a ⇒ y = −x + a
From eq. (iv) we get,a + b − 2 = 0 Thus, m=-1⇒ θ = 135°
a+a−2=0
a = 1 and b = 1 [∵ a = b]
∴ x1 = −2 × 1 = −2
y1 = −2 × 1 = −2
181. Ans (D)
A = (5,0), B = (0,3) 188. Ans (D)
x y m1 m2 = −1
Equation of ̅̅̅̅̅
AB is + = 1
5 3 17 − β 2
3x + 5y = 15 → (1) × = −1 ⇒ β = 5
−8 3
The ⊥er distance from (4,4) to (1) 189. Ans (D)
|12 + 20 − 15| 17 Since the given lines are perpendicular.
= =
√9 + 25 √34 ∴ Product of their slopes = −1
17 −l −l′
=√ ⇒ × = −1 ⇒ ll′ = −mm′
2 m m′
⇒ ll′ + mm′ = 0
182. Ans (C)
190. Ans (B)
Given P = (2,2)
Put θ = 45° and solve using point slope form
3x + y − 3 = 0 → (1)
191. Ans (B)
The eq. of the line perpendicular to (1) passing
x − 3y = −4
through P is x − 3y = −4
⇒ x = 0 ⇒ y = 4/3
MATHEMATICS Page | 6A. 26
STRAIGHT LINE

192. Ans (C) Since, the coordinates of three verities A, B and C


193. Ans (C) 5 4 2 7
are ( , − ) , (0, 0) and (− , ) respectively. Also, the
3 3 3 3
1 1
mid point of AC is ( , ) Therefore, the equation of
2 2

MOCK TEST SOLUTIONS 1 1


line passing through ( , ) and (0, 0) is x − y = 0,
2 2

1. Ans (B) which is the required equation of another diagonal


Using T.S.R ∴ a = 1, b = −1 and c = 0
2 6. Ans (C)
c
Area of ∆ =
2|ab| The equation of a line passing through the
2. Ans (B) intersection of x − 3 y + 1 = 0 and
The equation of any line passing through (1, 1) and 2 x + 5 y − 9 = 0 is
(−5, 5) is (x − 3 y + 1) + λ(2 x + 5 y − 9) = 0
5−1 ⇒ x(2 λ + 1) + y(5λ − 3) + 1 − 9λ = 0
y−1 = (x − 1)
−5 − 1
This is at a distance of √5 units from the origin
⇒ −6(y − 1) = 4(x − 1)
1−9λ 7
Since, the point (13, λ) lies on this line. ∴| | = √5 ⇒ λ =
√(2 λ + 1)2 + (5 λ − 3)2 8
∴ −6(λ − 1) = 4(13 − 1) ⇒ λ = −7
Hence, the required line is 2 x + y = 5
3. Ans (C)
7. Ans (B)
Points (3, 4) and (7, 13) are on the same side of
Let the coordinates of the other two vertices be
straight line y = x. Take image of A about y =
B(3, y1 ) and D(3, y2 ). Since the diagonals of a
x ie, A′′ ≡ (4, 3)
rectangle bisect each other

Now, P is a intersection point of line y = x and A′′ B


10
y1 + y2 2 + 5
Equation of line A′′ B is y − 3 = (x − 4) ∴ = ⇒ y1 + y2 = 7 … (i)
3 2 2
⇒ 3y − 9 = 10x − 40 Also, AC = BD
⇒ 10x − 3y = 31 ⇒ √(5 − 1)2 + (5 − 2)2 = y2 − y1
31 31 ⇒ y2 − y1 = 5 …(ii)
⇒ ( , ) satisfy the line A′′ B such that
7 7
From (i) and (ii), we get y1 = 1, y2 = 6
PA + PB is minimum
Hence, the coordinates of the other two vertices are
31 31
∴ Coordinates of P are ( , ) (3,1) and (3,6)
7 7
4. Ans (A) 8. Ans (B)

Note that, for given two lines = m1 × m2 = −1 The coordinates of the point dividing the line joining

Hence, θ = 90° (−1,1) and (5,7) in the ratio λ: 1 are

5. Ans (B) 5λ−1 7λ+1


( , )
λ+1 λ+1

MATHEMATICS Page | 6A. 27


STRAIGHT LINE

This point lies on the line x + y = 4


∴ 5λ−1+7λ+1 =4λ+4 ⇒8λ =4
1
⇒λ=
2
9. Ans (B)
If given lines are concurrent, then 1 1
0( ) + 0( ) −1
2 −3 k ∴ OP = || a b ||
|3 −4 −13| = 0 1 1
√ 2+ 2
8 −11 −33 a b
⇒ −22 + 15 − k = 0 ⇒ k = −7 1
⇒p=
10. Ans (C) 1 1
√ +
a2 b 2
For the lines y = x and y = −x, x − axis and y −
1
axis are the angle bisectors. ⇒ p2 = [squaring both sides]
1 1
+
a2 b 2
11. Ans (C)
1 1 1
Line making equal intercepts therefore, its equation ⇒ 2= 2+ 2
p a b
is
16. Ans (D)
x ± y = a …(i)
Required equation can be 4x − 3y − K = 0
Since, it passes through (2, 4)
4 × −1 − 3 × −4 − K
∴ a = −2, 6 ∴| |=1
√42 + (−3)2
Hence, equation of the required lines are −4 + 12 − K
⇒ = ±1
x±y=a 5
⇒ x + y = −2 ⇒ 8 − K = ±5
or x + y = 6 ⇒ K = 3 or K = 13
⇒ x+y−6=0 ∴ Equation of lines are 4x − 3y − 3 = 0 and 4x −
12. Ans (B) 3y − 13 = 0
Let the image or (reflection) of the origin with
reference to the line
4x + 3y − 25 = 0 is (h, k) 17. Ans (B)
h − 0 k − 0 −2(0 + 0 − 25) 50 It is evident from the figure that P moves on the line
∴ = = = =2
4 3 16 − 9 25 x = 1. Clearly, y-coordinate of P varies between 0
h
∴ =2 ⇒ h=8 and 1
4
∴ 0 ≤ α ≤ ⇒ α ∈ [0, 1]
k
and = 2 ⇒ k = 6
3
∴ The required point is (8, 6)
13. Ans (A)
|b − a| |b − a|
Required distance = =
√12 + 12 √2
14. Ans (D)
Use T.S.R on image of a point w.r.t line.
15. Ans (C)
x y 18. Ans (D)
Here the equation of AB is + = 1
a b Since, S is mid point of QR
From the figure, OP ⊥ AB, ∴ Coordinate of S are
MATHEMATICS Page | 6A. 28
STRAIGHT LINE

6 + 7 −1 + 3 13 ∴ Equation of line is
( , ) = ( , 1)
2 2 2 x − 3y = −4 …(i)
2−1 2 −4 4
∴ Slope of PS = =− Hence, y-intercept = =
13 9 −3 3
2−
2
22. Ans (C)
Given equation of lines are
5x + 3y − 7 = 0 …(i)
and 15x + 9y + 14 = 0
14
⇒ 5x + 3y + = 0 …(ii)
3

c1 − c2 35
d=| |=
√a2 + b 2 3√34
The required equation which is 23. Ans (B)

2 Lines 3 x + 4 y + 2 = 0 and 3 x + 4 y + 5 = 0 are on


passing through (1, −1) and slope − , is
9 the same side of the origin. The distance d1 between
2 these lines is given by
y + 1 = − (x − 1)
9
2−5 3
⇒ 9y + 9 = −2x + 2 d1 = | |=
√32 + 42 5
⇒ 2x + 9y + 7 = 0
Lines 3 x + 4 y + 2 = 0 and 3 x + 4 y − 5 = 0 are on
19. Ans (C)
the opposite sides of the origin.
Let p be the length of the perpendicular from the
The distance d2 between these lines is given by
vertex (2, −1) to the base 2+5 7
d2 = | |=
x + y = 2, then √32 + 42 5
2−1−2 Thus, 3 x + 4 y = 0 divides the distance between
p=| |
√12 + 12 3x + 4y + 5 = 0 and 3 x + 4 y − 5 = 0 in the ratio
1
= d1 ∶ d2 i.e., 3 ∶ 7
√2
24. Ans (B)
If a be the length of the side of triangle then,
∵ Slope of given line y = x is 1
p = a sin 60°
∴ Slope of required line which is perpendicular to
1 a√3
⇒ = given line is −1
√2 2
Thus, the equation of required line passing through
2
⇒ a=√ (3, 2) and slope −1, is
3
y − 2 = −1(x − 3)
20. Ans (A)
⇒ x+y=5
Clearly, L = 0 is the perpendicular bisector of the
25. Ans (A)
segment joining (−2,6) and (4,2). The equation of
We have,
which is 1−0 π
Slope of AB = = 1 ⇒ ∠BAX =
3 3−2 4
y − 4 = (x − 1) ⇒ 3x − 2y + 5 = 0 But, ∠BAC = 15°. Therefore, ∠CAX = 60°
2
∴ L = 3x − 2y + 5
21. Ans (D)
∵ Line perpendicular to 3x + y = 3 is x − 3y = λ
Also, it passes through (2, 2)
∴ 2 − 6 = λ ⇒ λ = −4
MATHEMATICS Page | 6A. 29
STRAIGHT LINE

28. Ans (A)


Reflection (h, k) of the point (4, −13) about 5x + y +
6 = 0 is given by
h − 4 k + 13 −2(20 − 13 + 6)
= =
5 1 25 + 1
h − 4 k + 13
⇒ = = −1 ⇒ h = −1, k = −14
5 1
So, the equation of AC is  reflection = (−1, −14).
y − 0 = tan 60°(x − 2) 29. Ans (B)
1 1
⇒ y = √3 x − 2√3 ⇒ √3 x − y = 2√3 Intercepts on co – ordinate axes are and .
a b
26. Ans (C)  Its equations is
x
+
x
= 1 ⇒ ax + by = 1
1 1
a b
Use foot of perpendicular from given point onto
30. Ans (C)
given line formula
27. Ans (B) 1 2a − a 2a − a
Area of triangle = 18 ⇒ | | = 18
a
Slope of ax by + c = 0 is − = m1 (say). Slope of (a
2 a−a a − 2a
b a a
⇒| | = 36
+ b)x − (a − b) y is
a+b
= m2 0 −a
a−b
 a2 = 36  a =  6
a a+b
m1 − m2 − − Given triangle is right angled at (a, a)
tan θ = | |=| b a−b|
1 + m1 m2 a a+b
1− [
b a−b
]  Circumcentre = midpoint of (2a, a), (a, 2a) =
3a 3a
−a2 + ab − ab − b2 ( , )
2 2
b(a − b)
=| | = 1 ⇒ θ = 45°
ab − b 2 − a2 − ab  (9, 9)
b(a − b)

MATHEMATICS Page | 6A. 30


CONIC SECTIONS

HINTS AND SOLUTIONS


1. Ans (D) Let A(2,1) and B(α, β) be the ends of the

AB = √22 + 1 = √5 diameter.

according to question Since C is mid-point of AB,


α+2 3 β+1 5
√32 + 52 − p < 3 and √32 + 52 − p < 5, ∴ = and =−
2 2 2 2

⇒ −29 < −P < −25 ⇒ α = 1 and β = −6.


⇒ 29 > p > 25 Hence, the other end of the diameter is (1, −6).

2. Ans (B)
5. Ans (A)
We have the equation of circle
The point of intersection of
x 2 + y 2 + 2gx + 2fy + c = 0
3x + y − 14 = 0 and 2x + 5y − 18 = 0 are
But it passes through (0,0)
x = 4, y = 2, i.e., the point (4,2)
and (2,1), then c = 0.
Therefore, the radius is = √9 + 16 = 5 and
5 + 4g + 2f = 0
hence the equation of the circle is given by
As it touches y-axis
(x − 1)2 + (y + 2)2 = 25
or x 2 + y 2 − 2x + 4y − 20 = 0
6. Ans (C)
Let (h, k) be the centre of the circle. Then k =
h − 1.
Therefore, the equation of the circle is given by
thus 2√f 2 − c = 0[∵ c = 0] (x − h)2 + [y − (h − 1)]2 = 9
⇒ f = 0, Given that the circle passes through the point

∴g=−
5 (7,3) and hence we get
4
(7 − h)2 + (3 − (h − 1))2 = 9
3. Ans (B)
or (7 − h)2 + (4 − h)2 = 9
Centre (2,2), r = 2 so touches both axes
or h2 − 11h + 28 = 0
4. Ans (B)
which gives (h − 7)(h − 4) = 0
Given equation is
⇒ h = 4 or h = 7
x 2 + y 2 − 3x + 5y − 4 = 0.
Therefore, the required equations of the circles
It can be easily seen that (i)
are
represent a circle with centre
3 5
x 2 + y 2 − 8x − 6y + 16 = 0
C ( , − ).
2 2 (Substituting h = 4,7 in (i) )
or x 2 + y 2 − 14x − 12y + 76 = 0
MATHEMATICS Page |7A. 1
CONIC SECTIONS

7. Ans (D) 12. Ans (A)


r = √4 + 9 − 4 = 3 ⇒ 2r = 6 The given equation is (x 2 + 8x) + (y 2 + 10y) =
8. Ans (C) 8
We write the given equation in the standard Now, completing the squares within the
form as parenthesis, we get
(x 2 − 2x) + (y 2 + 4y) = 8 (x 2 + 8x + 16) + (y 2 + 10y + 25)
Now, completing the squares, we get = 8 + 16 + 25
(x 2 − 2x + 1) + (y 2 + 4y + 4) = 8 + 1 + 4 i.e. (x + 4)2 + (y + 5)2 = 49
(x − 1)2 + (y + 2)2 = 13 i.e. {x − (−4)}2 + {y − (−5)}2 = 72
Comparing it with the standard form of the Therefore, the given circle has center at (−4, −5)
equation of the circle, we see that the center of and radius 7.

the circle is (1, −2) and radius is √13. 13. Ans (A)

9. Ans (B) Since, the circle is in the first quadrant touching

x 2 + y 2 − 6x + 8y + (25 − a2 ) = 0 each co-ordinate axis at a distance of one unit


from origin.
Radius = 4√9 + 16 + (25 − a2 )
∴ It passes through the points (1,0) and (0,1).
⇒ a = ±4
Also, its center will be (1,1) and radius is one
unit. Hence, the equation of required circle is
(x − 1)2 + (y − 1)2 = 12

10. Ans (C)


The given equations of diameters are
3x − 4y − 7 = 0. 14. Ans (A)
and 2x − 3y − 5 = 0 x 2 − 8x + 12 = 0 or (x − 6)(x − 2) = 0
On solving Eqs. (i) and (ii), we get y 2 − 14y + 45 = 0 or (y − 5)(y − 9) = 0
x = 1 and y = −1 Thus, the sides of squares are
∴ Centre of circle is (1, −1). x = 2, x = 6, y = 5, y = 9
Let r be the radius of circle, then Then the center of the circle inscribed in the
πr 2 = 49π ⇒ r = 7 units square will be (
2+6 5+9
, ) ≡ (4,7)
2 2
∴ Equation of required circle is
15. Ans (A)
2 2
(x − 1) + (y + 1) = 49
The point of intersection of
⇒ x 2 + y 2 − 2x + 2y + 1 + 1 = 49
3x + y − 14 = 0 and
⇒ x 2 + y 2 − 2x + 2y − 47 = 0
2x + 5y − 18 = 0 are x = 4, y = 2,
11. Ans (A)
i.e. the point (4,2).
(3,4)&(2,5) are ends of diameter of circle
Therefore, the radius = √9 + 16 = 5 Hence the
So, Equation
equation of the circle is given by (x − 1)2 + (y +
(x − 3)(x − 2) + (y − 4)(y − 5) = 0
2)2 = 25
x 2 + y 2 − 5x − 9y + 26 = 0
or x 2 + y 2 − 2x + 4y − 20 = 0

MATHEMATICS Page |7A. 2


CONIC SECTIONS

16. Ans (B) 19. Ans (D)


Let the vertex be at the lowest point and the axes Since directrix of parabola is y = 2
are vertical. Let the coordinate axis be chosen as ⇒y−2=0
shown in figure. So, vertex of parabola is (0,0) and focus S is
The equation of the parabola takes the form (0, −2).
2
x = 4ay. Equation of parabola will be x 2 = −4ay

Since it passes through (6,


3
), we have ⇒ x 2 = −4 × 2y = −8y.
100
3
(6)2 = 4a ( ),
100
36×100
i.e. a =
12

Let AB is the deflection of the beam which is


1
m.
100

20. Ans (D)


Given, equation of parabola is y 2 = 12x
which is of the form y 2 = 4ax
i.e., focus lies on the positive direction of X-axis.
Here, 4a = 12 ⇒ a = 3
2
Coordinates of B are (x, ). Focus = (a, 0) = (3,0)
100
2 Axis = X-axis
Therefore x 2 = 4 × 300 × = 24
100
Directrix, x = −d ⇒ x = −3
i.e. x = 2√6 metres
Length of latus rectum = 4a = 4 × 3 = 12
17. Ans (C)
21. Ans (D)
∴a=2
8y
We have + y2 = 1
focus is (0, −2) 3

Clearly for A, B both, y = −2 ⇒ (3y − 1)(y + 3) = 0


1
∴ x 2 = −8(−2) = 16 ∴ y = ( Reject y = −3)
3

22. Ans (C)


Since the distance from the focus to the vertex is
5 cm. We have, a = 5. If the origin is taken at the
vertex and the axis of the mirror lies along the
positive x-axis, the equation of the parabolic
∴ x = ±4
section is y 2 = 4(5)x = 20x
∴ A is (−4, −2), B is (4, −2)
We have, x = 45, so y 2 = 900
18. Ans (C)
Therefore, y = ±30
From the figure, OPQ represents the triangle
Hence, AB = 2y = 2 × 30 = 60 cm
whose area is to be determined. The area of the
1
triangle = PQ × OF
2
1
= (12 × 3) = 18
2

MATHEMATICS Page |7A. 3


CONIC SECTIONS

For ellipse r − 2 > 0 and 5 − r > 0


⇒2<r<5
28. Ans (D)
I. Since, foci (±5,0) lie on X-axis, as
y-coordinate is zero. Hence, equation of ellipse
x2 y2
will be of the form + = 1..
a2 b2

Given that, length of major axis 2a = 26


⇒ a = 13 and c = 5
∵ c 2 = a2 − b2
23. Ans (D) ⇒ (5)2 = (13)2 − b2 ⇒ 25 = 169 − b2
Since the focus (2,0) lies on the x-axis, the x-axis ⇒ b2 = 169 − 25 = 144
itself is the axis of the parabola. Hence the Put the values of a2 = 169 and b2 = 144 in Eq.
equation of the parabola is of the form y 2 = 4ax (i), we get
2
or y = −4ax. x2 y2
+ =1
Since the directrix is x = −2 and the focus is 169 144
(2,0), the parabola is to be of the form y 2 = 4ax II. Since, foci (0, ±6) lie on Y-axis, as

with a = 2. Hence the required equation is y 2 = x-coordinate is zero. Hence, equation of ellipse
x2 y2
4(2)x = 8x. will be of the form + =1
b2 a2
24. Ans (A) Given that, length of minor axis 2b = 16 and c =
Given, vertex = (0,0) 6 ⇒ b = 8 and c = 6
Point = (2,3) and axis = X-axis ∵ c 2 = a2 − b2 ⇒ (6)2 = a2 − (8)2
Since, point (2,3) lies in first quadrant and axis is ⇒ a2 = 36 + 64 = 100
X-axis. Hence, equation of parabola will be of the Put the values of a2 = 100 and b2 = 64 in Eq. (i),
form y 2 = 4ax, which passes through (2,3) i.e., x2 y2
we get + =1
64 100
Put x = 2, y = 3 in y 2 = 4ax
29. Ans (A)
∴ (3)2 = 4a × (2)
Since, foci (±3,0) lie on X-axis, as y-coordinate is
9
⇒a= zero.
8
Hence, required equation of parabola is Hence, equation of ellipse will be of the form
9 9 x2 y2
y2 = 4 ( ) x ⇒ y2 = x + =1
a2 b2
8 2
Given that, foci (±c, 0) = (±3,0)
25. Ans (B)
c = 3 and a = 4
(2)2 − 4(3) < 0
∵ c 2 = a2 − b2 ⇒ (3)2 = (4)2 − b2
26. Ans (C)
⇒ 9 = 16 − b2 ⇒ b2 = 16 − 9 ⇒ b2 = 7
9x 2 + 4y 2 = 1
Put the values of a2 = 16 and b2 = 7 in Eq. (i),
x y2
⇒ + =1 x2 y2
1/9 1/4 we get + =1
16 7
2a2 4
⇒ Length of latus rectum = = 30. Ans (B)
b 9
2b2
27. Ans (B) According to given condition, = 2ae
a
x2 y2
+ =1 b2
r−2 5−r ⇒ b2 = a2 e or e =
a2

MATHEMATICS Page |7A. 4


CONIC SECTIONS

b2 Hence, equation of ellipse will be of the form


Also, e = √1 − or e2 = 1 − e
a2 x2 y2
+ =1
b2 a2
or e2 + e − 1 = 0
−1±√5 where, a = √5 and b = 1
Therefore, e =
2 x2 y2
Hence, Eq. (i) becomes + =1
√5−1 12 (√5)2
As, e < 1, e =
2 x2 y2
or + =1
31. Ans (D) 1 5

given that be = 2 and a = 2 5x 2 + y 2 = 5

(here a < b ) 36. Ans (C)


k
∵ a2 = b2 (1 − e2 ) Put y = in x 2 + y 2 = k 2 ,
x
∴ b2 = 8 k2
we get x 2 + = k2
x2
x2 y2
∴ equation of ellipse + =1
4 8 ⇒ x4 − k2 x2 + k2 = 0
32. Ans (B) k 2 ± √k 4 − 4k 2
x2 y2
∴ x2 =
Ellipse is + = 1. 2
8 6
For no intersection
Here a2 = 8
k 2 (k 2 − 4) < 0(k ≠ 0)
⇒ a = 2√2
∴ k ∈ (−2,2) − {0}
sum of focal distances = 2a = 4√2
Hence integral values of k are {−1,1}
33. Ans (C)
37. Ans (C)
2 1 1
e12 = 1 − = ⇒ e1 = x2 y2
3 3 √3 Let equation of hyperbola be − =1
a2 b2

1 √3
As, e1 e2 = ⇒ e2 = b2
2 2
∴ e = √1 +
3 b2 a2
Now, = 1 − ⇒b=2
4 16
5 2 b2 5
34. Ans (A) ⇒ ( ) = (1 + 2 ) [∵ e = ]
4 a 4
We have d = F1 F2 = 2ae
b2 9 b 3
d2 = 4a2 e2 = 4(a2 − b2 ) ⇒ = ⇒ =
a2 16 a 4
= 4(4 − 1) = 12 2b2 3
We have, =9⇒ 2× b =9
a 4
⇒ d = 2√3
⇒ b = 6 and a = 8
35. Ans (A)
x2 y2
∴ Equation of hyperbola is − =1
I. Since, ends of major axis (±3,0) are along X- 64 36

axis and minor axis (0, ±2) are along Y-axis. 38. Ans (D)
x2 y2
Hence, equation of ellipse will be of the torm Given that the hyperbola − − 1 is passing
a2 b2
x2 y2
+ = 1, where, a = 3 through the points (3,0) and (3√2, 2), so we get
a2 b2

and b = 2 a2 = 9 and b2 = 4.
x2 y2 Again, we know that b2 = a2 (e2 − 1). This gives
Hence, Eq. (i) becomes + =1
32 22
13
x2 y2
4 = 9(e2 − 1) or e2 =
9
or + =1
9 4
√13
or e =
II. Since, ends of major axis (0, ±√5) are along Y- 3

axis and minor axis (±1,0) are along X-axis. 39. Ans (C)
3 9
ae = 3, e = , b2 = 4 ( − 1) , b2 = 5
2 4
MATHEMATICS Page |7A. 5
CONIC SECTIONS

x2 y2 x2 y2
− =1 − + =1
4 5 4 3
The eccentricity of this hyperbola is given by

a2 4 7
e = √1 + 2
= √1 + = √
b 3 3

45. Ans (C)


For a hyperbola
b2 81 225
e2 = 1 +2
=1+ =
a 144 144
40. Ans (C)
15 5
1 ∴e= =
2b = (2ae) ⇒ 4b2 = a2 e2 12 4
2 144
Also, a2 =
⇒ 4b2 = a2 + b2 ⇒ 3b2 = a2 25
12 5
b2 1 4 2 Hence, the foci are (± ae, 0) ≡ (± , )
Hence e2 = 1 + = 1+ = ⇒ e= 5 4
a2 3 3 √3

41. Ans (A) ≡ (±3,0)

Since foci are (0, ±12), it follows that Now, for an ellipse, ae = 3 or a2 e2 = 9

c = 12. Now, b2 = a2 (1 − e2 )

2b2 ⇒ b2 = a2 − a2 e2 = 16 − 9 = 7
Length of the latus rectum = = 36
a
46. Ans(A)
2
⇒ b = 18a.
x 2 + y 2 − 2x + 3y + k = 0
2 2 2
Therefore, c = a + b gives 5
We have r=√g 2 + f 2 − c =
2 2 2
144 = a + 18a i.e., a + 18a − 144 = 0
9 25 13 25
⇒ a = −24,6. ⇒1+ −k= ⇒ −k=
4 4 4 4
Since, a > 0 we take a = 6 and so ⇒ k = −3
b2 = 108. 47. Ans(C)
Therefore, the equation of the required
hyperbola is
y2 x2
− = 1 ⇒ 3y 2 − x 2 = 108
36 108
42. Ans (D)
144 9 7
e12 = 1 − × = B = 2C − A = (4,2) − (3,4) = (1, −2)
16 144 16
44 9 25 48. Ans(A)
e22 = 1 + × =
16 144 16 Circle with ̅̅̅̅
AB as diameter
⇒ e12 + e22 =2 x 2 + y 2 − 4x + 3y = 0
43. Ans (D)
2b = 5 and 2ae = 13
25 169
b2 = a2 (e2 − 1) ⇒ = − a2
4 4
13
⇒a=6⇒e=
12
44. Ans (A)
The given equation can be written as 49. Ans(A)
A = (4,0), B = (0,3)
MATHEMATICS Page |7A. 6
CONIC SECTIONS

̅̅̅̅ as diameter is
Required Circle with AB
x 2 + y 2 − 4x − 3y = 0

From the figure, equation of circle is


50. Ans(A) x 2 + y 2 + 2rx − 2ry + r 2 = 0
s = x 2 + y 2 − 4x − 6y − 12 = 0 ⇒ 𝑓𝑜𝑟 𝑟 = 1, x 2 + y 2 + 2x − 2y + 1 = 0
⇒ C = (2,3) and r = √4 + 9 + 12 = 5 54. Ans(B)
Put L: x = 7 in equation of circle
We get y 2 − 6y + 9 = 0
⇒y=3
∴ Point of contact = (7,3)

center = (3,2) and radius = 3


∴ Equation of circle is
(x − 3)2 + (y − 2)2 = 32
⇒ x 2 + y 2 − 6x − 4y + 4 = 0
51. Ans(A) 55. Ans(C)
Given c1 = (3,4), r1 = 4
c2 = (0,0), r2 =?
By the given data c1 c2 = r1 + r2
⇒ 5 = 4 + r2 ⇒ r2 = 1
52. Ans(C)

C = (±5,3), r = 5
∴ Equation of circle is
(x ± 5)2 + (y − 3)2 = 25
⇒ x 2 + y 2 ± 10x − 6y = 0
Given C1 = (2,2) , r1 =2 56. Ans(D)
and C2 (6,5) , r2 =? πr 2 = 4π ⇒ r = 2
By the given data C1 C2 =r1 + r2 we have (x − r)2 + (y − r)2 = r 2
⇒ √(16 + 9)=2+r2 ⇒2+r2 =5⇒ r2 =3 ⇒ (x − 2)2 + (y − 2)2 = 4
∴ Centre C2 =(6,5) & r2 =3 57. Ans(C)
2 2
⇒ x + y − 12x − 10y + 52 = 0 3x − 2y − 8 = 0 → (1)
53. Ans(A) 2x − y − 5 = 0 → (2)
2 −1
after solving, center C = ( , ) = (2, −1)
1 1

⇒r=1

MATHEMATICS Page |7A. 7


CONIC SECTIONS

∴ Equation of circle is The equation of the circle passing through O, A, B


(x − 2)2 + (y + 1)2 = 12 is
⇒ x 2 + y 2 − 4x + 2y + 4 = 0 x2 + y2 − x − y = 0
When (t, t)⇒ t 2 + t 2 − t − t = 0
⇒ 2t 2 − 2t = 0 ⇒ t = 1
62. Ans(D)

58. Ans(B)
Equation of circle passing through the points
(0,0), (4,0) and (0,5) is x 2 + y 2 − 2ay = 0
5 2 25 C = (a, 0)
(x − 2)2 +(y − ) =4+
2 4
x + 2y = 4 → (1)
x 2 +y 2 -4x-5y=0
required line is ⊥er to (1) passing through C
2x − y = 2a
63. Ans(D)
S ≡ x 2 + y 2 − 2x − 4y + 4 = 0
P = (1,2)
S(1,2) = 1 + 4 − 2 − 8 + 4 = −1 < 0

59. Ans(B) P lies inside of S = 0


The equation of the circle concentric with S = 0 The no. of tangents from P to S is 0
is x 2 + y 2 + 2gx + 2fy + K = 0 64. Ans(D)

C (-2, -3)⇒ x 2 + y 2 + 4x − 2y + k = 4 + 9 − 8 + s1 ≡ x 2 + y 2 − 6x − 8y + c = 0
6+k = 0 c1 = (3,4), r1 = √9 + 16 − c = √25 − c
⇒ 11 + k = 0 ⇒ k = −11 Also given s2 ≡ x 2 + y 2 = 9
∴ option B is correct c2 = (0,0), r2 = 3
60. Ans(B) Two circles touch externally,
x 2 + y 2 = K/5 so, c1 c2 = r1 + r2

K ⇒ 5 = √25 − c + 3 ⇒ 4 = 25 − c ⇒ c = 21
C = (0,0), r = √
5 65. Ans(D)

x + 2y + 3 = 0 → (1) Given S1 ≡ x 2 + y 2 − 2x − 4y − 20 = 0

(1)touches the circle S = 0 C1 = (1,2), r1 = √1 + 4 + 20 = 5

r = distance between centre and tangent Also given S2 ≡ (x + 3)2 + (y + 1)2 = p2


C2 = (−3, −1), r2 = P
K |3|
√ = ⇒K=9 C1 C2 = √16 + 9 = 5
5 √5
For one tangent, C1 C2 = |r1 − r2 |
61. Ans(B)
⇒ 5 = |5 − P| ⇒ 5 = −5 + P ⇒ P = 10
O = (0,0), A = (1,0), B = (0,1)
66. Ans(C)
S1 ≡ x 2 + y 2 + 2x + 8y − 23 = 0
MATHEMATICS Page |7A. 8
CONIC SECTIONS

C1 = (−1, −4) ⇒ x 2 + y 2 − 16x + 18y + 85 = 0


r1 = √1 + 16 + 23 = √40 = 2√10
S2 ≡ x 2 + y 2 − 4x − 10y + 19 = 0
C2 = (2,5), r2 = √4 + 25 − 19 = √10
C1 C2 = √9 + 81 = 3√10
C1 C2 = r1 + r2 (touch externally)
∴ no. of common tangents = 3
67. Ans(B)
S1 ≡ x 2 + y 2 = 4 70. Ans(C)

C1 = (0,0), r1 = 2 radius = 2a and centre=(0, 0)

S2 ≡ x 2 + y 2 − 24x − 10y + a2 = 0 x 2 + y 2 = 4a2

C2 = (12,5), r2 = √144 + 25 − a2

= √169 − a2
For exactly two tangents
|r1 − r2 | < c1 c2 < r1 + r2

⇒ |2 − √109 − a2 | < 13 < 2 + √169 − a2


71. Ans(B)
⇒ 11 < √169 − a2 y 2 = 16x ⇒ a = 4
⇒ |2| < 169 − a2 ⇒ a2 < 48 ∴ Focus S = (a, 0) = (4,0)
⇒ −4√3 < a < 4√3 72. Ans(D)
⇒ −6.928 < a < 6.928 x 2 + 12y = 0
⇒ a = −6, −5, −4, −3, −2, −1,0, 1, 2, 3, 4, 5, 6 x 2 = −12y
68. Ans(B) a=3
73. Ans(D)
y 2 = −8x ⇒ a = 2
Equation of directrix is x = a
⇒x =2

Given circle is x 2 + y 2 − 4x + 6y − 7 = 0
C = (2, −3), r = √4 + 9 + 7 = √20
Image of centre C(2, -3) w.r.t X- axis is
C ′ = (2,3), r ′ = √20 74. Ans(C)
2 2
∴ 𝑟𝑒𝑞𝑢𝑖𝑟𝑒𝑑 𝑐𝑖𝑟𝑐𝑙𝑒: (x − 2) + (y − 3) = 20 −9
2 2 x2 = y
⇒ x + y − 4x − 6y − 7 = 0 2
69. Ans(B) 4a = −9/2

x 2 + y 2 + 16x + 18y + 85 = 0 So a=-9/8

C = (−8, −9) and r = √64 + 81 − 85 = √60 And equation of directrix is y=-a⇒ y = 9/8

Image of centre C(-8, -9) w.r.t Y- axis is 75. Ans(A)

C ′ = (8, −9) and r ′ = √60


∴ 𝑟𝑒𝑞𝑢𝑖𝑟𝑒𝑑 𝑐𝑖𝑟𝑐𝑙𝑒: (x − 8)2 + (y + 9)2 = 60
MATHEMATICS Page |7A. 9
CONIC SECTIONS

S: y 2 = 4ax 81. Ans(D)


Put P = (2, −6) to equation of parabola, we get Let P(x, y) be a point the parabola
36 = 8a ⇒ 4a = 18 Slope of OP: m=𝑡𝑎𝑛30° =
1
√3
∴ L. L. R = 18 1
Equation of OP: 𝑦 = x
76. Ans(A) √3

Given Focus F = (3,3) Putting this equation in parabola, we get

directrix L ≡ 3x − 4y = 2 x2
= 4ax ⇒ x = 12a ⇒ y = 4√3𝑎
3
2a = The ⊥er dis tance from F to L = 0
∴ PQ: 2y = 8√3𝑎
|9 − 12 − 2|
⇒ 2a = =1
5
∴ The length of latus rectum = 4a = 2
77. Ans(B)
option B satisfies both tangent and parabola
78. Ans(D)
Given y 2 = 8x ⇒ a = 2
Let P = (x1 , y1 ) be a point on the parabola 82. Ans(B)

⇒ y12 = 8x1 Vertex A = (0,0)

Focal distance = |x1 + a| directrix L ≡ x + 5 = 0

⇒ |x1 + a| = 8 L. L. R = 4a

⇒ x1 + 2 = ±8 = 4( The ⊥er distance from A to L = 0)

⇒ x1 = 6, −10 = 4|5| = 20

79. Ans(A) 83. Ans(B)

S ≡ y 2 = 4x y 2 = −4ax

P = (3,2) = (x1 , y1 ) Focus (-a,0)

S(3,2) = 4 − 12 < 0 84. Ans(B)

P lies inside of S = 0 Given P(k, −1) is

∴ no. of common tangents = 0 exterior to the parabola S ≡ y 2 − |x| = 0

80. Ans(B) S(k,−1) > 0

y 2 = 4ax ⇒ 1 − |k| > 0


2a π ⇒ |k| − 1 < 0
Tan θ/2 = >1⇒θ>
a 2 ⇒ −1 < k < 1
2Tan θ/2 4 85. Ans(A)
Tan θ = =
1 − Tan2 θ/2 −3
S = (0, −3)
θ = π − Tan−1 (4/3)
L≡y−3=0
From definition of the parabola
MATHEMATICS Page |7A. 10
CONIC SECTIONS

SP = PM 91. Ans(C)
⇒ x 2 + (y + 3)2 = (y − 3)2 Focus S = (3,0) = (a, 0)
⇒ x 2 + 12y = 0 directrix x = −3 ⇒ a = 3
∴ Equation of the parabola y 2 = 4ax
⇒ y 2 = 12x
92. Ans(B)
S ≡ x 2 = 4y ⇒ a = 1
Equation of latus rectum y = −a
86. Ans(B)
L≡y+1=0
Given Focus S(−4,0) and directrix x = 4
P = (2a, a) lies inside S = 0
⇒ a = −4
S(2a,a) < 0
We have y 2 = −4ax
4a2 − 4a < 0 ⇒ a(a − 1) < 0
∴ y 2 = −16x
⇒ a ∈ (0,1)
87. Ans(B)
P, O lies on same side of L = 0
a + 1 > 0 ⇒ a > −1
⇒0<a<1
93. Ans(C)
S ≡ y 2 = 12x

A = (−a, 0) P(x1 , y1 ) be a point on S = 0

x − a = 0 → (1) ⇒ y12 = 12x1


y1
Let P(x1 , y1 ) be a point on locus. From figure, Tan 30∘ = ⇒ x1 = √3y1
x1
⇒ PA = The ⊥er distance from P to (1) ⇒ y12 = 12 × √3y1

⇒ √(x1 + a)2 + y12 = |x1 − a| ⇒ y1 = 12√3 ⇒ x1 = 36 = AR

⇒ (x1 + a)2 + y12 = (x1 − a)2


⇒ y12 = −4ax1
Locus of P is y 2 = −4ax
88. Ans(B)
vertex A(0,0) directrix x + 5 = 0 ⇒ x = −5
94. Ans(A)
⇒a=5
L𝐿𝑅 = 4a = 4 × 5 = 20
89. Ans(C)
A = (0,0), axis is y-axis
⇒ Equation of parabola x 2 = 4ay, passing
through (−3,1)
⇒ 9 = 4a L = (1,3), L′ = (1, −1)
2
∴ Equation of parabola x = 9y 1 ′ 1
AS = LL = (4) = 1 = a
90. Ans(C) 4 4
1
Use distance formula Area of ALL' = (a)(4a)
2
Distance of (5, 2) from (4,4) = √5 which is least
= 2a2 = 2
when compared to other points.
MATHEMATICS Page |7A. 11
CONIC SECTIONS

95. Ans(C)
8−6 1
S ≡ y 2 = 24x e=√ =
8 2
⇒a=6
foci = (0, ±be)
P = (6,12) = (a, 2a)
2√2
Q = (6, −12) = (a, −2a) = (0, ± ) = (0, ±√2)
2
are ends of latus rectum of " S=0
102. Ans(A)
⇒ Angle b/w the normal =π/2
x2 y2
96. Ans(C) + =1
cot 2 α cos 2 α
Focus S(−1,0) 1
LLR =
2
m = −√3
2cos 2 α 1
y − 0 = −√3(x + 1) =
cot α 2
⇒y + √3(x + 1) = 0 1
⇒2cos 2 αtanα =
2
97. Ans(C) 1 1
⇒2sinαcosα = ⇒sin 2α =
2 2
x + y = 0 ⇒ y = −x
π π
⇒2α = ⇒α=
x 2 + x 2 − 4x = 0 6 12

⇒2x(x − 2) = 0
(0,0)(2, −2) 103. Ans(B)

Equation parabola,y 2 = 4ax Passes (2, −2) 2b2


Length of latus rectum = 10 =
1 a
⇒ 4 = 8a ⇒ a = 1
2 e2 =
2
⇒4a = 2
a2 − b2 b2
y 2 = 2x e2 = = 1 −
a2 a2
98. Ans(C) 1 5
= 1−
x 2 = 4ay 4 a
(6, −3) lies on curve 5 3 20
= ⇒a=
a 4 3
⇒ 36 = −12a ⇒ a = −3
40
∴ x 2 = −12y 2a =
3
99. Ans(B) 104. Ans(B)
√5
Given 2b = 8, e =
3
a2 −b2
We have e2 =
a2
5 16
⇒ = 1−
9 a2

length of major axis: 2a=12⇒ a = 6


Maximum number of parabola = 2
105. Ans(B)
100. Ans(C)
By translation length are not changed
i.e, ⇒4a=4
101. Ans(C)
x2 y2
+ = 1, a < b
6 8

MATHEMATICS Page |7A. 12


CONIC SECTIONS

b a√1−e2 10 − a > 0,4 − a > 0


From figure tan 30∘ = =
ae ae
⇒a < 10, a < 4 so, a < 4
1 √1 − e2
= 112. Ans(B)
√3 e
Equation of the ellipse of the form
⇒ e2 = 3(1 − e2 )
lx 2 + my 2 = 1
√3
⇒ 4e2 = 3 ⇒ e = 9l + m = 1
2
106. Ans(B) 4l + 4m = 1

x2 y2 3 5
+ =1 (l, m) = ( , )
36 9 32 32
SP + S ′ P = 2a(a > b) = 12 ∴ 3x 2 + 5y 2 = 32

107. Ans(C) 113. Ans(B)

2b = a 4x 2 + 16y 2 = 64
x2 y2
a2 −b2 + =1
We have e = √ 16 4
a2
SP + S ′ P = 2a(a > b) = 8
4b 2 − b 2 √3
=√ = 114. Ans(B)
4b 2 2
F = (4,0) = (ae, 0)
108. Ans(D)
4
e=
5
ae = 4 ⇒ a = 5
we have b2 = a2 (1 − e2 )
= 25 − 16 = 9
x2 y2
∴ + =1
25 9
ae 115. Ans(D)
Tan 30∘ =
b case (i) a < b
1 a̸
⇒ = 2a = 6 and 2be = 8
√3 a̸√1−e2
⇒a = 3 and be = 4
⇒1 − e2 = 3e2
1 We have a2 = b2 (1 − e2 )
⇒4e2 = 1 ⇒ e =
2
⇒9 = b2 − 16 ⇒ b2 = 25
109. Ans(B)
x2 y2
∴ + =1
2 a2 − b 2 2 9 25
e= ⇒√ =
3 a2 3 case (ii)a > b

b 2 4
2b = 6 ⇒ b = 3
⇒1 − ( ) =
a 9 ⇒2ae = 8 ⇒ ae = 4
5 b 2 a 3
⇒ =( ) ⇒ = We have b2 = a2 (1 − e2 )
9 a b √5

110. Ans(C) ⇒9 = a2 − 16 ⇒ a2 = 25
x2 y2
x2 y2 ⇒ + =1
+ =1 25 9
25 16
116. Ans(B)
⇒a = 5, b = 4
PS1 + PS2 = 2a = 2(5) = 10
111. Ans(A)

MATHEMATICS Page |7A. 13


CONIC SECTIONS

x2 y2
+ =1
25 16

Here a = 5, b = 4
PF1 + PF2 = 2a = 2(5) = 10
121. Ans(A)
Focus = (c, 0) = (4,0) ⇒ c = 4
4
eccentricity =
5
c 4 4 4
ae = ⇒ = ⇒a=5
From the figure Tan 450 = a 5 a 5
b
Now, c = a − b22 2
b = ae
x2 y2
∴Equation of ellipse is + =1
we have a√1 − e2 = ae 25 9

⇒1 − e2 = e2 ⇒ 2e2 = 1 122. Ans(C)


1 x
⇒e = = cos t + sin t
√2 2
117. Ans(B) y
= cos t − sin t
5
x 2 y 2
( ) + ( ) = 2(cos 2 t + sin2 t)
2 5
x2 y2
+ = 2(1) = 2
4 25
x2 y2
⇒ + =1
8 50

ae The curve is an ellipse


From △ OBS, cosθ =
a
π 1
123. Ans(C)
⇒e = cos ⇒e =
4 √2 2b2
=b
118. Ans(B) a
C = (0,0) a = 2b
1
e= b 2 √3
2 e = √1 − =
a2 2
a
x=4=
e 124. Ans(D)
a
⇒ =4⇒a=2
e
x2 y2
E≡ + −1
We have b2 = a2 (1 − e2 ) 9 4
C ≡ x2 + y2 − 9
1
= 4 (1 − ) = 3 1 4 1
4 EP ≡ + − 1 = > 0 P lies outside E
x2 y2 9 4 9
∴Equation of ellipse is + =1
4 3 4 1 4 3
EQ ≡ + − 1 = − < 0 Q lies inside E
⇒3x 2 + 4y 2 = 12 9 4 9 4
119. Ans(C) CP = 1 + 4 − 9 < 0 P lies inside of C

Given 2ae = 6, 2b = 8 CQ = 4 + 1 − 9 < 0 Q lies inside of C

⇒ ae = 3, b = 4 ∴ lies outside of E inside of C


We have b2 = a2 − (ae)2 125. Ans(B)

⇒a2 = 25 A = (+5,0), A′ = (−5,0)

⇒e = =
c 3 ⇒a = 5
a 5
S = (4,0), S ′ = (−4,0)
120. Ans(C)
c = ae = 4
MATHEMATICS Page |7A. 14
CONIC SECTIONS

we have b2 = a2 (1 − e2 ) = 25 − 16 = 9 We have b2 = a2 e2 − a2
∴Equation of ellipse ⇒ 16 = 52 − a2
x2 y2 ⇒ a2 = 52 − 16
+ =1
a2 b 2 ⇒ a2 = 36
x2 y2
⇒ + =1 ⇒a=6
25 9

126. Ans(A) ae = 2√13


x
= sin θ + cos θ √13
a ⇒e=
3
y
= sin θ − cos θ 132. Ans(C)
b

eliminate θ from the above equation by squaring


x2 y2
on both the side and adding, we get + =
a2 b2

2 is an ellipse.
127. Ans(C)

18 − 4 14 7 7
e1 = √ =√ =√ =√ b2
18 18 9 3
tan45° =
a(a + ae)
9 + 4 √13 𝑤𝑒 ℎ𝑎𝑣𝑒 b2 = a2 (1 + e)
e12 = √ =
9 3
⇒ a2 (e2 − 1) = a2 (1 + e)
2e12 + e22 = 3 ⇒ e2 − 1 = 1 + e
128. Ans(C) ⇒ e2 = e + 2
x2 y2
For an ellipse + =1 ⇒ e2 − e − 2 = 0
16 b2
⇒ (e − 2)(e + 1) = 0
Foci S (±√16 − b 2 , 0)
⇒e=2
x2 y2
For a hyperbola 144 − 81 = 1, foci S(±3,0) 133. Ans(C)
25 25
Given 2ae = 18 ⇒ ae = 9
√16 − b 2 = 3 ⇒ 16 − b2 = 9
2a a
⇒ b2 = 16 − 9 and =8⇒ =4
e e
⇒ b2 = 7 a
thus, ae × = 36
129. Ans(D) e
⇒ a2 = 36
SE = (±√12, 0), SH = (±√a2 + 3, 0)
c 9 3
By given data 12 = a2 + 3 e= = ⇒e=
a 6 2
⇒ a2 = 9 ⇒ a = 3 b2 = a2 e2 − a2 = 81 − 36
130. Ans(D) ⇒ b2 = 45
x2 y2 x2 y2
Given hyperbola − =1 ∴ Equation of hyperbola is − =1
9 4 a2 b 2
x2 y2
9 + 4 √13 ⇒ − =1
e=√ = 36 45
9 3
134. Ans(C)
131. Ans(A)
2b = 8 ⇒ b = 4
2𝑐 = 2ae = 4√13 P = (3, −2)
⇒ ae = 2√13 x2 y2
Equation of Hyperbola − =1
a2 b2
⇒ 2b = 8 ⇒ b = 4
MATHEMATICS Page |7A. 15
CONIC SECTIONS

9 4 27
− =1 2 2 × 27
a2 16 = 2 = = 2 × 3√3
36 3√3 3√3
a2 = 2
5
∴ LLR = 6√3
5x 2 y 2
− =1 140. Ans(A)
36 16
135. Ans(B) Since e = √2 ⇒ a = b
2 2
x y ⇒ 2ae = 24
2
− 2=1
a b ⇒ ae = 12
Passes through (3,0)
⇒ a√2 = 6 × √2√2
x2 y2
⇒ − 2=1 ⇒ a = 6√2
9 b
It also passes through (3√2, 2) ⇒ b = 6√2
9×2 4 x 2 − y 2 = a2
⇒ − 2=1
9 b ⇒ x 2 − y 2 = 72
4
⇒ 1 = 2 ⇒ b2 = 4 141. Ans(C)
b
2b2
√a2 +b2 √13 L𝐿𝑅 = 8 =
We have e = = a
a 3

136. Ans(C) 2b2


= 8 ⇒ b2 = 4a
a
a = b = √7 ⇒ c = √2
2b = ae
∴ Distance between two foci = 2ae
⇒ 4b2 = a2 e2
= 2√7√2 = 2√14
4b2 = a2 + b2
137. Ans(C)
3b2 = a2
2 2
x y
+ =1 3a2 (e2 − 1) = a2
12 − k 8 − k
1 4 2
For Hyperbola, 12 − k > 0,8 − k < 0 e2 = 1 + ⇒ e2 = ⇒ e =
3 3 √3
8 < K < 12
142. Ans(D)
138. Ans(B)
√5 a2 +b2 5
Given e= ⇒ =
a=b=2 2 a2 4
2
Distance between directrices 36 + K 5
⇒ =
36 4
2a 2a
= = = a√2 = 2√2 K 2 = 45 − 36 = 9
e √2
139. Ans(B) Check S11 = 0
143. Ans(D)
Given 2ae = 9, e = √3
x2 y2
2√3a = 9 Hyperbola − =1
64 36

9 SS1 = 2ae
ae =
2
64 + 36
3√3 =2×8×√
a= 64
2
b2 = a2 (e2 − 1) = 2 × 10 = 20
27 27 144. Ans(B)
b2 = (2) =
4 2
25 − 16
2b2 e1 = √
LL1 = 25
a
MATHEMATICS Page |7A. 16
CONIC SECTIONS

b>a 2
⇒ 3e2 = 4 ⇒ e =
3 √3
e=
5 149. Ans(D)
e1 e2 = 1 x2 y2
+ =1
5 9 5
e2 ( Hyperbola ) =
3 1 2
e=√ =
Foci (0, ±3) 9−5 3
9
x2 y2
− =1 x2 y2
a2 b 2 − =1
9 45/4
⇒b=3
x2 y2 1 81 9 3
− =1 e′ = √ √9 = √ = =
a2 9 45 36 6 2
9+
16 4
a2 = b2 2 3
9 ee′ = ⋅ =1
3 2
⇒ a2 = 16
150. Ans(A)
x2 y2
− =1 4
16 9 ae = 2 ⇒ a =
3
145. Ans(A)
b2 = a2 (e2 − 1)
a = 3 b ⇒ a2 = 9b2
16 5 20
a2 = 9a2 (e2 − 1) = ( )=
9 4 9
√10 9x 2 9y 2
9e2 = 10 ⇒ e = Equation of Hyperbola − =1
3 16 20
146. Ans(C) x2 y2 4
⇒ − =
2a = 7, P = (5, −2) 4 5 9
x2 y2 151. Ans(C)
− =1
a2 b 2 Given that the center of the circle is (1,2)
2
4 2 y Radius of the circle
x − 2=1
49 b
= √(4 − 1)2 + (6 − 2)2 = √9 + 16 = 5
4 4 196
× 25 − 2 = 1 ⇒ b2 = So, the area of the circle = πr 2 = π × (5)2 = 25π
49 b 51
4x 2 51y 2 152. Ans(C)
Equation of Hyperbola − =1
49 196
147. Ans(C)
x2 y2
− =1
4−r 4+r
a2 = 4 − r > 0 ⇒ 4 > r ⇒ r < 4 → (1)
b2 = 4 + r > 0 ⇒ r > −4 → (2)
from (1) and (2) −4 < r < 4
148. Ans(C) Let the required circle touch the axes at (a, 0)
By the given data 2 b = ae and (0, a)
2
2b ∴ Centre is (a, a) and r = a
=8
a So, the equation of the circle is
⇒ b2 = 4a
(x − a)2 + (y − a)2 = a2
2 2 2
4b = a e
If it passes through a point P(3,6), then
⇒ 4a2 (e2 − 1) = a2 e2
MATHEMATICS Page |7A. 17
CONIC SECTIONS

(3 − a)2 + (6 − a)2 = a2 Let ABC be an equilateral triangle in which


⇒ 9 + a2 − 6a + 36 + a2 − 12a = a2 median AD = 3a.
⇒ a2 − 18a + 45 = 0 Centre of the circle is same as the centroid of the
⇒ (a − 3)(a − 15) = 0 triangle i.e., (0,0) ,AG: GD = 2: 1
2 2
⇒ a = 3 and a = 15 So, AG = AD = × 3a = 2a
3 3
(Which is not possible), so, a = 3
∴ The equation of the circle is
So, the required equation of the circle is (x −
(x − 0)2 + (y − 0)2 = (2a)2
2 2
3) + (y − 3) = 9 ⇒ x 2 + y 2 = 4a2
2 2
x + y − 6x − 6y + 9 = 0 155. Ans(A)
153. Ans(A) According to the definition of parabola
y−3
√(x − 0)2 + (y + 3)2 = | |
√(0)2 + (1)2

⇒ √x 2 + y 2 + 9 + 6y = |y − 3|
Squaring both sides, we have x 2 + y 2 + 9 + 6y =
y 2 + 9 − 6y
⇒ x 2 = −12y
Let the equation of the circle be 156. Ans(B)
(x − h)2 + (y − k)2 = r 2 Given parabola is y 2 = 4ax
Let the center be (0, a) If the parabola is passing through (3,2)
1
∴ Radius r = a Then, (2)2 = 4a × 3 ⇒ 4 = 12a ⇒ a =
3
So, the equation of the circle is Now length of the latus rectum = 4a
2 2 2
(x − 0) + (y − a) = a 1 4
=4 × =
2 2 3 3
⇒ x + y − 2ay = 0
157. Ans(A)
Now CP = r

⇒ √(2 − 0)2 + (3 − a)2 = a

⇒ √13 + a2 − 6a = a
⇒ 13 + a2 − 6a = a2
13
a= , Putting the value of a in eq. (i) we get
6

13
x2 + y2 − 2 ( ) y = 0
6
⇒ 3x 2 + 3y 2 − 13y = 0
154. Ans(C)

Given that vertex = (−3,0) ∴ a = −3


and directrix is x + 5 = 0
According to the definition of the parabola, we
get AF = AD i.e., A is the mid-point of DF
x1 −5
∴ −3 = ⇒ x1 = −6 + 5 = −1
2
0+y1
&0= ⇒ y1 = 0
2

∴Focus F = (−1,0)
MATHEMATICS Page |7A. 18
CONIC SECTIONS

Now √(x + 1)2 + (y − 0)2 = |


x+5
| Given Length of the latus rectum of the
√12 +02
hyperbola
Squaring both sides, we get,
2b2
Distance between the foci = 2ae = =8
(x + 1)2 + y 2 = (x + 5)2 a

(x + 1)2 + y 2 = (x + 5)2 ⇒ b2 = 4a

y 2 = 10x − 2x + 24 ⇒ y 2 = 8x + 24 Transverse axis=2a and Conjugate axis=2b

y 2 = 8(x + 3) 1 ae
∴ (2ae) = 2b ⇒ ae = 2b ⇒ b =
2 2
158. Ans(A)
a2 e2 a2 e2
⇒ b2 = ⇒ 4a = [from eq. (i)]
Given that focus of the ellipse is (1, −1) and the 4 4

equation of the directrix is x − y − 3 = 0 and e = 16


⇒ 16 = ae2 ∴ a =
1
e2
. Let P(x, y) by any point on the parabola
2 Now b2 = a2 (e2 − 1)
PF
∴e=
Distance of the point P from the directrix
⇒ 4a = a2 (e2 − 1)
4 4
√(x − 1)2 + (y + 1)2 1 ⇒ = e2 − 1 ⇒ = e2 − 1
= = a 16/e2
x−y−3 2
| | e2 e2
√(1)2 + (−1)2 ⇒ = e2 − 1 ⇒ e2 − = 1
4 4
x−y−3
⇒ 2√x 2 + 1 − 2x + y 2 + 1 + 2y = | | 3e2 4
√2 ⇒ = 1 ⇒ e2 =
4 3
Squaring both sides, we have 2
∴ e=
⇒ 4(x 2 + y 2 − 2x + 2y + 2) √3
x 2 + y 2 + 9 − 2xy + 6y − 6x 162. Ans(A)
=
2 We know that the distance between the foci 2c=
⇒ 8x 2 + 8y 2 − 16x + 16y + 16 = x 2 + y 2 −
2ae
2xy + 6y − 6x + 9
Given that 2ae=16⇒ae=8
⇒ 7x 2 + 7y 2 + 2xy − 10x + 10y + 7 = 0
We have, b2 =a2 (e2 − 1)
⇒ b2 = 32(2 − 1)=32
159. Ans(D)
So, the equation of the hyperbola is
Equation of the ellipse is 3x 2 + y 2 = 12 x2 y2
− =1
x y 2 2 a2 b2
⇒ + =1 x2 y2
4 12
⇒ − =1
Here a2 = 4 ⇒ a = 2 & b2 = 12 32 32
⇒ x 2 − y 2 = 32
⇒ b = 2√3
2a2 2×4 4
163. Ans(A)
Length of the latus rectum = = =
b 2√3 √3 Given that e = 3/2 and foci (±ae, 0)=(±2,0)
160. Ans(B) ∴ ae = 2
x2 y2
Given equation is + = 1 (a < b) 3 4
a2 b2 ⇒a× =2⇒a=
2 3
a2 a2 Now we know that b2 = a2 (e2 − 1)
∴ Eccentricity e = √1 − ⇒ e2
= 1 −
b2 b2 16 9 16 5 20
b2 = ( − 1) ⇒ b2 = × =
a2 9 4 9 4 9
⇒ = (1 − e2 ) ⇒ a2 = b2 (1 − e2 ) So, the equation of the hyperbola is
b2
161. Ans(C) x2 y2
− 20 =1
(4/3)2
9

MATHEMATICS Page |7A. 19


CONIC SECTIONS

9x 2 9y 2 x2 y2 1 Given y 2 = x
− =1⇒ − =
16 20 16 20 9 Differentiating both sides w.r.t. x
x2 y2 4 dy dy 1
⇒ − = 2y =1⇒ =
4 5 9 dx dx 2y
164. Ans(A) π
Also, tangent makes an angle of with x axis
4
c = (−a, −b), r = √a2 − b 2
dy π
The equation of the circle is ⇒ = tan = 1
dx 4
(x + a)2 + (y + b)2 = a2 − b2 1 1
∴ =1⇒y=
⇒ x 2 + y 2 + 2ax + 2by + a2 + b2 2y 2

= a2 − b2 Put value of y in equation y 2 = x


1
165. Ans(B) We get x =
4
1 1
Point ( , )
4 2

170. Ans(A)
2ae = 16, ae = 8, e = √2
b2 = a2 (e2 − 1) = 32
x2 y2
Equation of Hyperbola − =1
32 32

⇒ x 2 − y 2 = 32 (or) ae = 8, e = √2
a = 4√2, (∵ e = √2 ⇒The hyperbola is
S = (0, a) = (0,3)
rectangular hyperbola)
OS = 3, PQ = L. L. R = 4a = 12
So, a = b, ⇒ b = 4√2
1 1
Area of △ OPQ = (OS)(PQ) = × a × 4a 171. Ans(C)
2 2
1 x2 y2
= × 3 × 12 = 18 + =1
2 25 9
166. Ans(B)
a2 − b 2
5 e=√
Given e =
4
a2

Put S = (ae, 0) to equation of focal chord a>b


2x + 3y − 6 = 0
25 − 9 4
W get 2ae = 6 =√ =
25 5
12
⇒ ae = 3 ⇒ a = 172. Ans(A)
5
167. Ans(D) The given parabola passes through (2, 1).

Length of transverse axes=2a  (2)2 = 4. a. (1) ⇒ a = 1

24 So, length of latus rectum =4a = 4(1) = 4


=
5 173. Ans (C)
2 2
(x − 2) + (y − 1) = 9 2ae = 16
⇒ c = (2,1), r = 3 ae = 8
168. Ans(A) e = √2 ⇒ a = 4√2; b = 4√2
a2 − b 2 36 − 16 √5 x 2 − y 2 = 32
e=√ 2
=√ =
a 36 3
174. Ans (A)
169. Ans(B)
175. Ans (D)
MATHEMATICS Page |7A. 20
CONIC SECTIONS

MOCK TEST SOLUTIONS: x2 + y2 – 2x + 3y + k = 0


3
1. Ans (D) here g = – 1, f = , c = k
2
Equation is,
5 9 5
radius = ⇒ √1 + − k =
16x2 + 16y2 – 8x + 32y – 257 = 0 2 4 2

1 257 ⇒ 13 − 4k = 25 ⇒ k = −3
⇒ x 2 + y 2 − x + 2y − =0
2 6 9. Ans (A)
1 257
g=− , f = 1, C=− Clearly, C = (1, 1) and radius = 1 unit
4 16
 The equation is x2 + y2 – 2x – 2y +1 = 0
1 257 274
radius = √ + 1 + =√ 10. Ans (B)
16 16 16
y2 = 16 x  4a = 16  a = 4
2. Ans (D)
focus = (a, 0) = (4, 0)
For the equation of a circle
11. Ans (D)
Coeff. x2 = co-eff. y2  p = 7
y2 + 8x = 0  y2 = – 8x  4a = 8  a = 2
3. Ans (B)
directrix is x = a i.e., x = 2
Clearly circle passes through (4, 0) and
12. Ans (A)
(0, 5). Also, it passes through (0, 0). Thus, the
y2 = 4ax passes through (2, –6)
equation of the circle is x2 + y2 – 4x – 5y = 0
9
4. Ans (B) ⇒ 36 = 8a ⇒ a =
2
Equation of the circle concentric with L.L.R = 4a = 18
x2 + y2 + 4x – 2y + 3 = 0 is 13. Ans (C)
x2 + y2 + 4x – 2y + c = 0 Clearly the axis of the parabola is x – axis.
since this circle passes through (–2, –3) Now the point of intersection of the axis and the
 4 + 9 – 8 + 6 + c = 0  11 + c = 0 directrix is Z = (0, 0), V = (4, 0).
 c = – 11 Let S = (a, 0). The vertex is the midpoint of ZS.
 The circle is, x2 + y2 + 4x – 2y – 11 = 0 a
∴ (4,0) = ( , 0) ⇒ a = 8 ∴ S = (8,0)
2
5. Ans (D)
14. Ans (B)
Area = 4  r2 = 4  r = 2 The vertex V = (0, 0) directrix is x = –5.
 centre (2, 2) and r = 2 a = distance of V from the directrix  a = 5
Equation is, (x – 2)2 + (y – 2)2 = 22 Length of the latus rectum = 4a = 20
6. Ans (B) 15. Ans (D)
(3, 4) C: 9 + 25 + 12 − 32 − 29 < 0 y2 + 2y + x = 0  y2 + 2y + 1 = – x+ 1
 point lies inside circle  (y + 1)2 = – (x – 1)
7. Ans (D)
 V = (1, –1) which lies in fourth quadrant
Equation of the circle is
16. Ans (B)
3 5 8
x2 + y2 − x + y − = 0 Focus = (– 4, 0) directrix : x = 4
7 7 7
3 5 Focus is to the left of directrix
g = − ,f =
14 14 Also vertex = (0, 0), a = 4
∴ C = ( ,− )
3 5
 equation is y2 = – 16 x
14 14

8. Ans (A) 17. Ans (A)

Equation of the circle is (y – 2)2 =16 (x – 1)  4a = 16  a = 4

MATHEMATICS Page |7A. 21


CONIC SECTIONS

V = (x1, y1) = (1, 2) 144


⇒ = a2 ⇒ a2 = 36
18. Ans (C) 4
 a = 6  2a = 12
5x2 + 9y2 = 45
25. Ans (C)
x2 y2
⇒ + =1 9x2 + 25y2 = 225
9 5
a2 = 9, b2 = 5 ( a > b) x2 y2
⇒ + =1
a2 − b2 4 2 25 9
e2 = 2
⇒ e2 = ⇒ e =
a 9 3 a2 − b 2 25 − 9 4
Distance between foci = 2ae = 4 e=√ =√ =
a2 25 5
19. Ans (A)
26. Ans (B)
By data, 2a = 3 (2b)  a = 3b x2 y2
4x2 + 16y2 = 64  + =1
a2 −b2 9b2 −b2 8 2√2 16 4
Now, e2 = = = ⇒e=
a2 9b2 9 3 By definition: SP + S'P = 2a = 8 (∵ a = 4)
20. Ans (C) 27. Ans (D)
2b2 1 2b2
By data, = (2b) (∵ L. L. R = ) y2 x2
a 2 a We have, 9y 2 − 4x 2 = 36 ⇒ − =1
4 9
 a = 2b. (conjugate hyperbola)
2 a2 −b2 4b2 −b2 √3
Now e = = ⇒e= Foci are (0, ±√a2 + b 2 )
a2 4b2 2

21. Ans (D) = (0, √13)


By data 2b = 6 and 2ae = 8  b = 3, and ae = 4 28. Ans (D)
Now, b2 = a2 (1 – e2)  9 = a2 – 16 we have, 4x 2 − 9y 2 = 36 ⇒
x2

y2
=1
9 4
 a2 = 25
x2 y2
a2 + b 2 13 √13
 equation is + =1 e=√ 2
=√ =
25 9 a 9 3
22. Ans (D) 29. Ans (D)
9x2 + 5y2 = 45
By data e = 2 and ae=4 a = 2
x2 y2
⇒ + =1 We have b2 = a2 (e2 – 1)  b2 = 4 (3) = 12
5 9
x2 y2
a2 = 5, b2 = 9, ( b > a)  equation is − =1
4 12

distance between foci = 2√b 2 − a2 30. Ans (C)

= 2√4 = 4 By data, 2b = 8 and b = 4


x2 y2
23. Ans (C) Equation is of the form − =1
a2 16
Foci are (0, ±√b 2 − a2 ) This passes through (3, –2)
= (0, ±√2) 9 4 9 20 5
⇒ − =1⇒ 2= =
a2 16 a 16 4
24. Ans (B)
36
√5 ⇒ a2 =
By data 2b = 8, e = 5
3
5x2 y2
2 2 (1 2)
5 2  equation is − =1
b =a −e ⇒ 16 = a (1 − ) 36 16
9

MATHEMATICS Page |7A. 22


INTRODUCTION TO THREE-DIMENSIONAL GEOMETRY

HINTS AND SOLUTIONS

1. Ans (A) (c) The equation of the plane z = 0 represents the


The coordinates of the origin 0 are (0,0,0). The XY-plane and z = 3 represents the plane parallel to
coordinates of any point on the X-axis will be as xy-plane at a distance 3 units above XY-plane.
(x, 0,0) and the coordinates of any point in the YZ-
plane will be as (0, y, z)
2. Ans (A)
Since L is the foot of perpendicular from P on the x-
axis, so its y and z-coordinates are zero. So, the
coordinates of L is (7,0,0). Similarly, the So, only option

coordinates of M and N are (0,9,0) and (0,0,4), (c) is correct.

respectively. 5. Ans (D)

3. Ans (D) The point (−3,1,2) lies in second octant and the

Since M is the foot of perpendicular from A on the point (−3,1, −2) lies in sixth octant.

yz-plane, so its x-coordinate is zero. Hence, 6. Ans (B)

coordinates of M are (0,7,8) Since, L is the foot of perpendicular segment from P

4. Ans (C) on the XY-plane, Z-coordinate is zero in the XY-

(a) The equation of the plane x = 0 represents the plane.

YZ-plane and equation of the plane x = 1 Hence, coordinates of L are (3,4,0). Similarly, we

represents the plane parallel to YZ-plane at a can find the coordinates of M(0,4,5) and N(3,0,5)

distance 1 unit above YZ-plane. A plane


parallel to YZ-plane at a distance 1 unit above
YZ plane is shown below.

7. Ans (A)
Since L is the foot of perpendicular from point
(3,5,6) on the x-axis so, its y and z-coordinates are
zero. Hence, the coordinates of L are (3,0,0).
(b) The equation of the plane y = 0 represents the 8. Ans (B)
XZ-plane and the equation of the plane y = 3 I. Through the point P in the space, we draw three
represents the plane parallel to X∠-plane at a planes parallel to the coordinate planes, meeting
distance 3 units above XZ-plane. the X-axis, Y-axis and Z-axis in the points A, B and
C, respectively. We observe that, OA = x, OB = y
and OC = z. Thus, if P(x, y, z) is any point in the
space, then x, y and z are perpendicular distances
from YZ, ZX and XY-planes, respectively. II. Given
x, y and z, we locate the three points A, B and C on

MATHEMATICS Page | 8A. 1


INTRODUCTION TO THREE-DIMENSIONAL GEOMETRY

the three coordinate axes. Through the points A, B (a) Let A(0,7, −10), B(1,6, −6) and C(4,9, −6) are
and C, we draw planes parallel to the YZ-plane, ZX- the vertices of a triangle. Then, Side AB = Distance
plane and XY-plane, respectively. The point of between points A and B
intersection of these three planes, namely, ADPF, = √(0 − 1)2 + (7 − 6)2 + (−10 + 6)2
BDPE and CEPF is obviously the point P, = √1 + 1 + 16 = √18 = 3√2
corresponding to the ordered triplet (x, yz). and side BC = Distance between points B and C =
√(1 − 4)2 + (6 − 9)2 + (−6 + 6)2
= √9 + 9 + 0 = √18 = 3√2
Clearly, AB = BC. Hence, triangle is an isosceles
triangle.
(b) Let A(0,7,10), B(−1,6,6) and C(−4,9,6) are the
vertices of a triangle. Then,
Side AB = √(0 + 1)2 + (7 − 6)2 + (10 − 6)2
9. Ans (A)
= √1 + 1 + 16 = √18 = 3√2
On yz-plane, x = 0
Side BC = √(−1 + 4)2 + (6 − 9)2 + (6 − 6)2
10. Ans (A)
= √9 + 9 + 0 = √18 = 3√2
According to the question.
Distance of any point from XY-plane is and side CA = √(−4 − 0)2 + (9 − 7)2 + (6 − 10)2

|z| = |8| = 8 = √16 + 4 + 16 = √36 = 6


Now, AB 2 + BC 2 = (3√2)2 + (3√2)2
= 62 = CA2
∴△ ABC is right angled triangle at B.
(c) Let A(−1,2,1), B(1, −2,5), C(4, −7,8) and
D(2, −3,4) are the vertices of a quadrilateral ABCD.
−1+4 2−7 1+8
11. Ans (A) Then, mid-point of AC = ( , , )
2 2 2

Locus of the point y = 0, z = 0 is X-axis, since on X- 3 −5 9


=( , , )
axis both y = 0 and z = 0. 2 2 2
1+2 −2−3 5+4
12. Ans (D) Mid-point of BD = ( , , )
2 2 2

Reflection of any point (x, y, z) in XY-plane is 3 −5 9


=( , , )
(x, y, −z) 2 2 2
Mid-points of both the diagonals are same (i.e.,
∴ Reflection of (α, β, γ) in XY-plane is (α, β, −γ)
they bisect each other.)
13. Ans (B)
Hence, ABCD is a parallelogram.
Locus of the point for which x = 0, z = 0 is y-axis,
16. Ans (C)
since on y-axis both x = 0 and z = 0.
Let the given points are A and B. Let P(x, y, z) be
14. Ans (B)
any point equidistant from A and B.
Since, L is the foot of perpendicular from P on the
∴ PA = PB
XY-plane, Z-coordinate is zero in the XY-plane.
i.e., Distance between P and A = Distance between
Hence, coordinates of L are (6,7,0).
P and B
15. Ans (D)

MATHEMATICS Page | 8A. 2


INTRODUCTION TO THREE-DIMENSIONAL GEOMETRY

⇒ √(x − 1)2 + (y − 2)2 + (z − 3)2 20. Ans (B)


Let L be the foot of perpendicular from point P on
= √(x − 3)2 + (y − 2)2 + (z + 1)2
the Y-axis. Therefore, its x and z-coordinates are
⇒ (x − 1)2 + (y − 2)2 + (z − 3)2
zero, i.e., (0,4,0).
= (x − 3)2 + (y − 2)2 + (z + 1)2
Therefore, distance between the points (0,4,0) and
⇒ 4x − 8z = 0 ⇒ x − 2z = 0
17. Ans (D) (3,4,5) is √9 + 25 i.e., √34.

The point on the X-axis is of form P(x, 0,0). Since, 21. Ans (B)

the points A and B are equidistant from P, PA2 = Three points are collinear if the sum of any two

PB 2, distances is equal to the third distance.

i.e., (x − 3)2 + (0 − 2)2 + (0 − 2)2 PQ = √(−2 − 2)2 + (−2 − 4)2 + (−2 − 6)2
= (x − 5)2 + (0 − 5)2 + (0 − 4)2 = √16 + 36 + 64 = √116 = 2√29
49
⇒ 4x = 25 + 25 + 16 − 17 i.e., x =
4
QR = √(6 + 2)2 + (10 + 2)2 + (14 + 2)2
49
Thus, the point P on the X - axis is ( , 0,0) which is = √64 + 144 + 256 = √464 = 4√29
4

equidistant from A and B. PR = √(6 − 2)2 + (10 − 4)2 + (14 − 6)2

18. Ans (B) = √16 + 36 + 64 = √116 = 2√29


Length of the diagonal AB Since, QR = PQ + PR. Therefore, given points are

= √(1 − 2)2 + (−2 + 3)2 + (3 − 5)2 collinear.


22. Ans (A)
= √1 + 1 + 4 = √6
We know that points are said to be collinear, if they
Now. AC 2 + BC 2 = AB 2
lie on a line.
⇒ x 2 + x 2 = (√6)2 ⇒ 2x 2 = 6 ⇒ x = √3
Now, PQ = √(1 + 2)2 + (2 − 3)2 + (3 − 5)2
= √9 + 1 + 4 = √14

QR = √(7 − 1)2 + (0 − 2)2 + (−1 − 3)2

= √36 + 4 + 16 = √56 = 2√14


and PR = √(7 + 2)2 + (0 − 3)2 + (−1 − 5)2

= √81 + 9 + 36 = √126 = 3√14


19. Ans (B)
Thus, PQ + QR = PR. Hence, P, Q and R are
Since x-coordinate of every point in YZ-plane is
collinear.
zero.
23. Ans (A)
Let P(0, y, z) be a point on the YZ-plane such that
Let P be the point on Y-axis. Therefore, it is of the
PA = PB = PC. Now,
form P(0, y, 0). The point (1,2,3) is at a distance
PA = PB ⇒ (0 − 2)2 + (y − 0)2 + (z − 3)2
= (0 − 0)2 + (y − 3)2 + (z − 2)2, √10 from

i.e. z − 3y = 0 and PB = PC (0, y, 0). Therefore,

⇒ y 2 + 9 − 6y + z 2 + 4 − 4z √(1 − 0)2 + (2 − y)2 + (3 − 0)2 − √10


= y 2 + z 2 + 1 − 2z, i.e., 3y + z = 6 ⇒ y 2 − 4y + 4 = 0
Simplifying the two equations, we get ⇒ (y − 2)2 = 0 ⇒ y = 2
y = 1 and z = 3. Here, the coordinate of the point P Hence, the required point is (0,2,0).
are (0,1,3). 24. Ans (C)
MATHEMATICS Page | 8A. 3
INTRODUCTION TO THREE-DIMENSIONAL GEOMETRY

Let A(5, −1,1), B(7, −4,7), C(1, −6,10) and


D(−1, −3,4) be the four points of a quadrialateral.
Here,
AB = √4 + 9 + 36 = 7
BC = √36 + 4 + 9 = 7
CD = √4 + 9 + 36 = 7
DA = √36 + 4 + 9 = 7 28. Ans (B)
AC = √16 + 25 + 181 = √122 Let the point be P(x, y, z). Then, it is given PA +
BD = √64 + 1 + 9 = √74 PB = 10
Note that AB = BC = CD = DA and AC ≠ BD. ⇒ √(x − 4)2 + (y − 0)2 + (z − 0)2
Therefore, ABCD is a rhombus. +√(x + 4)2 + (y − 0)2 + (z − 0)2 = 10
25. Ans (A)
⇒ √(x − 4)2 + y 2 + z 2 = 10 − √(x + 4)2 + y 2 + z 2
Let P(x, y, z) be the required point.
Squaring on both sides, we get
Then, OP = PA = PB = PC.
(x − 4)2 + y 2 + z 2 = 100 + (x + 4)2 + y 2 + z 2
Now, OP = PA ⇒ OP 2 = PA2 −20√(x + 4)2 + y 2 + z 2
2 2 2 2 2 2
⇒ x + y + z = (x − 1) + (y − 0) + (z − 0) ⇒ x 2 + 16 − 8x = 100 + x 2 + 16 + 8x
1 m −20√(x + 4)2 + y 2 + z 2
⇒ x = , Similarly, OP = PB ⇒ y = and OP =
2 2
n
⇒ −8x − 8x − 100 = −20√(x + 4)2 + y 2 + z 2
PC ⇒ z =
2 ⇒ −16x − 100 = −20√(x + 4)2 + y 2 + z 2
Hence, the coordinates of the required points are ⇒ 4x + 25 = 5√(x + 4)2 + y 2 + z 2
1 m n
( , , ). (dividing both sides by -4 )
2 2 2

26. Ans (D) Again, squaring on both sides, we get

Here, the points are A(1,2,3), B(−1, −1, −1) and (4x + 25)2 = 25[(x + 4)2 + y 2 + z 2 ]

C(3,5,7). ⇒ 16x 2 + 625 + 200x = 25[(x + 4)2 + y 2 + z 2 ]


⇒ 16x 2 + 625 + 200x = 25[x 2 + 16 + 8x + y 2 +
|AB| = √22 + 32 + 42 = √29,
z2 ]
|BC| = √42 + 62 + 82 = √116 = 2√29
⇒ 16x 2 + 625 + 200x = 25x 2 + 400 + 200x +
and |CA| = √22 + 32 + 42 = √29.
25y 2 + 25z 2
So, |BC| = |AB| + |CA|
⇒ 9x 2 + 25y 2 + 25z 2 − 225 = 0
⇒ The points A, B, C are collinear.
which is the required equation.
27. Ans (D)
29. Ans (A)
L is the foot of perpendicular drawn from the point
Let P(0,7,10), Q(−1,6,6) and R(−4,9,6) be the
P(3,4,5) to the XY-plane. Therefore, the coordinate
given three points.
of the point L is (3,4,0). The distance between the
Here, PQ = √1 + 1 + 16 = 3√2
point (3,4,5) and (3,4,0) is 5 . Similarly, we can find
QR = √9 + 9 + 0 = 3√2
the lengths of the foot of perpendiculars on YZ and
PR = √16 + 4 + 16 = 6
ZX-plane which are 3 and 4 units, respectively.
Now, PQ2 + QR2 = (3√2)2 + (3√2)2
= 18 + 18 = 36 = (PR)2
Therefore, △ PQR is a right-angled triangle at Q.

MATHEMATICS Page | 8A. 4


INTRODUCTION TO THREE-DIMENSIONAL GEOMETRY

Also, OQ = QR. Therefore, ABCD is not a rhombus and naturally, it


Hence, △ PQR is a right-angled isosceles triangle. cannot be a square.
30. Ans (D) 33. Ans (C)
Any point in xy-plane is of the form (x, y, 0). Let Let the coordinates of point P be (x, y, z).
P(x, y, 0) is equidistant from A(5,0,6), B(0, −3,2) Here
and C(4,5,0). PA2 = (x − 3)2 + (y − 4)2 + (z − 5)2
∴ PA = PB ⇒ PA2 = PB 2 PB 2 = (x + 1)2 + (y − 3)2 + (z + 7)2
⇒ (x − 5)2 + (y − 0)2 + (0 − 6)2 By the given condition PA2 + PB 2 = 2k 2
= (x − 0)2 + (y + 3)2 + (0 − 2)2 ⇒ (x − 3)2 + (y − 4)2 + (z − 5)2 +
⇒ x 2 + 25 − 10x + y 2 + 36 (x + 1)2 + (y − 3)2 + (z + 7)2 = 2k 2
= x 2 + y 2 + 9 + 6y + 4 ⇒ 5x + 3y = 24 i.e.,2x 2 + 2y 2 + 2z 2 − 4x − 14y + 4z
Also, PB = PC ⇒ PB 2 = PC 2 = 2k 2 − 109.
⇒ (x − 0)2 + (y + 3)2 + (0 − 2)2 34. Ans (A)
2 2 2
= (x − 4) + (y − 5) + (0 − 0) Let P(x, y, z) be any point such that PA = PB
⇒ x 2 + y 2 + 9 + 6y + 4 Now √(x − 3)2 + (y − 4)2 + (z + 5)2
2 2
= x + 16 − 8x + y + 25 − 10y = √(x + 2)2 + (y − 1)2 + (z − 4)2
⇒ 2x + 4y = 7
⇒ (x − 3)2 + (y − 4)2 + (z + 5)2
Solving (i) and (ii), we get
= (x + 2)2 + (y − 1)2 + (z − 4)2
75 −13
x= ,y = or 10x + 6y − 18z − 29 = 0.
14 14
75 −13 35. Ans (B)
∴ Coordinates of P = ( , , 0).
14 14
The point which is equidistant from
Alternatively, verify the options a b c
(0,0,0), (a, 0,0), (0, b, 0)(0,0, c) is ( , , )
2 2 2
31. Ans (B)
D = (a, 0,0), E = (0, b, 0), F = (0,0, c),
Let P(3,4, −1) and Q(−1,2,3) be the end points of
O = (0,0,0)
the diameter of a sphere.
a b c
∴ Length of diameter = PQ Required point is ( , , )
2 2 2

= √(−1 − 3)2 + (2 − 4)2 + (3 + 1)2 36. Ans (C)

= √16 + 4 + 16 = √36 = 6 units AB = √1 + 1 + 1 = √3


6
∴ Radius = = 3 units BC = √9 + 9 + 9 = √27
2

32. Ans (B) CD = √1 + 1 + 1 = √3

Here, the mid-point of AC is DA = √9 + 9 + 9 = √27


4 + 2 −2 − 5 1 + 10 7 11 AC = √4 + 4 + 4 = √12
( , , ) = (3, − , )
2 2 2 2 2
BD = √16 + 16 + 16 = √48
7−1 −4−3 7+4
and that of BD is ( , , ) 37. Ans (B)
2 2 2
7 11
= (3, − , ). Let diagonal of square = d
2 2
d √6
So, the diagonals AC and BD bisect each other. ⇒ The length of the side = = = √3
√2 √2
ABCD is a parallelogram. 38. Ans (C)
As |AB| = √32 + 22 + 62 = 7 and By distance formula, AB = √3 ,BC = √3 &
|AD| = √52 + 12 + 32 = √35 ≠ |AB|
MATHEMATICS Page | 8A. 5
INTRODUCTION TO THREE-DIMENSIONAL GEOMETRY

CA = √3 45. Ans (C)


39. Ans (C) L = (2,4,0), M = (0,4,5)
A = (1,0,0), B = (0,1,0), c = (0,0,1) LM = √4 + 25 = √29
The perimeter of the △ ABC = AB + BC + CA 46. Ans (B)
= √2 + √2 + √2 = 3√2 AP = PQ = QB
40. Ans (C) ⇒ midpoint of PQ = midpoint of ̅̅̅̅
AB
z co-ordinate is + vein 4 octants = (2,3.5,5)
41. Ans (A) 47. Ans (C)
P = (1,2,3) A = (a, 0,0), B = (0, b, 0), C = (0,0, C)

The dis tan ce from p to x − axis = √y 2 + x 2 a b c


P=( , , )
2 2 2
The distance from p to y-axis = √x 2 + z 2
OP = AP = BP = CP
The distance from p to z-axis = √x 2 + y2 48. Ans (D)
The distance of the point A = (1,2,3), B = (3, −1,5), C = (4,0, −3)
p to coordinate axes are √13, √10, √5 AB = √4 + 9 + 4 = √17
42. Ans (C) BC = √1 + 1 + 64 = √66
A = (3,2, −4), B = (5,4, −6), C = (9,8, −10)
AC = √9 + 4 + 36 = √49
AB = √4 + 4 + 4 = 2√3
AB 2 + AC 2 = BC 2 ,△ ABC is right angle Δle , right
BC = √16 + 16 + 16 = 4√3 angle at ′ A′
CA = √36 + 36 + 36 = 6√3 Circumcenter = mid point of BC
AB + BC = CA 7 −1 2
=( , , )
The points A, B, C are collinear 2 2 2

43. Ans (C) 49. Ans (C)

A = (2,9,12), B = (1,8,8), A = (2, −1,1), B = (1, −3, −5), C = (3, −4, −4)

C = (−2,11,8), D = (−1,12,12) a = BC = √4 + 1 + 1 = √6

AB = √1 + 1 + 16 = 3√2, b = CA = √1 + 9 + 25 = √35

BC = √9 + 9 + 0 = 3√2 c = AB = √1 + 4 + 36 = √41

CD = √1 + 1 + 16 = 3√2, BC 2 + CA2 = AB 2 ,△ ABC is right angle Δle , right


angle at ′ C ′
AD = √9 + 9 + 0 = 3√2
hyp AB √41
AC = √16 + 4 + 16 = 6, circum radius = = =
2 2 2

BD = √4 + 16 + 16 = 6 50. Ans (D)


Given points form a square AB = √(a − 1)2 + 32 + 0 = 5
44. Ans (C) ⇒ a − 1 = ±4 ⇒ a = −3,5
A = (2, −1, −4), B = (−4,3,0) CD =
Let P = (x, 0,0) √(a − 1)2 + (a + 5)2 + (a − 1)2 =6
PA = PB ⇒ a2 − 2a − 15 = 0 ⇒ a = −3,5
2 2
(x − 2) + 1 + 16 = (x + 4) + 9 But common solution of (i) and (ii) is -3.
−4x + 4 + 17 = 8x + 16 + 9 51. Ans (A)
−1
12x = −4 ⇒ x = Given point is P(3,4,5)
3
MATHEMATICS Page | 8A. 6
INTRODUCTION TO THREE-DIMENSIONAL GEOMETRY

∴ Distance of P from yz-plane = 62. Ans (D)


√(0 − 3)2 + (4 − 4)2 + (5 − 5)2 We know that on xy-plane, z = 0.

= √9 = 3 units So, the coordinate of the point L are (3,4,0).

52. Ans (B) 63. Ans (A)

On y-axis, x = 0 and z = 0 and given point P(3,4,5) We know that x-axis, y = 0 and z = 0.

∴ The point A is (0,4,0) So, the required coordinates are (3,0,0).


64. Ans (B)
∴ PA=√(0 − 3)2 + (4 − 4)2 + (0 − 5)2
P = (a, b, c)
= √9 + 0 + 25 = √34
The distance from P to x − axis
53. Ans (A)
= √b 2 + c 2
Given points A(3,4,5) and the given O(0,0,0)
65. Ans (A)
∴ OA = √(3 − 0)2 + (4 − 0)2 + (5 − 0)2
P = (6,7,8)
= √9 + 16 + 25 = √50
The perpendicular distance from P to XY - plane
54. Ans (B)
= |Z| = 8
Let the given points be A(a, 0,1) and B(0,1,2)
66. Ans (D)
∴ AB = √(a − 0)2 + (0 − 1)2 + (1 − 2)2 Let P = (α, β, γ)
√27 = √a 2 +1+1 Let M is the foot of perpendicular from P on xy −
Squaring both sides, we get plane
2 2
27 = a + 2 ⇒ a = 25 ∴ a = ±5 ⇒ M = (α, β, 0)
55. Ans (A) Let Q is the image of P w.r.t xy − plane
We know that on the xy and xz-planes, the line of Q = 2M − P
intersection is x-axis. = (2α, 2β, 0) − (α, β, γ)
56. Ans (C) = (α, β, −γ)
On y-axis, x = 0 and z = 0 67. Ans(B)
57. Ans (B) Conceptual
The point (−2, −3, −4) lies in seventh octant. 68. Ans (A)
58. Ans (A) Conceptual
Any plane parallel to yz-plane is perpendicular to 69. Ans (A)
x-axis.
59. Ans (A) MOCK TEST SOLUTIONS:
We know that one equation of x-axis,y=0,z=0 1. Ans (D)
Hence, the locus of the point is equation of x-axis. Perpendicular distance of the point (6, 5, 8) from
60. Ans (B) y-axis = √62 + 82 = 10 units
On the yz-plane, x = 0 2. Ans (D)
Hence, the locus of the point is yz-plane. Let the point on x-axis is A(x, 0, 0)
61. Ans (A) Given, B = (1, 2, 3, )and C = (3, 5, −2)
Given points are A(5,8,10) and B(3,6,8) Since, |AB| = |AC|
∴ AB = √(5 − 3)2 + (8 − 6)2 + (10 − 8)2 ⟹ √(x − 1)2 + (0 − 2)2 + (0 − 3)2
= √4 + 4 + 4 = √12 = 2√3 = √(x − 3)2 + (0 − 5)2 + (0 + 2)2

MATHEMATICS Page | 8A. 7


INTRODUCTION TO THREE-DIMENSIONAL GEOMETRY

⟹ x 2 + 1 − 2x + 4 + 9 = x 2 + 9 − 6x + 25 + 4 2m − 3 3
⟹ =0 ⟹m=
⟹ x = 6 ∴ Required point is (6, 0, 0) m+1 2
3 + 2 × 5 13
Now, x= =
3+2 5
3. Ans (C) 3 × (−2) + 2 × (−2)
and z = = −2
Let A(5, −4, 2), B(4, −3, 1), C(7, −6, 4) and 5
D(8, −7, 5) 13
∴ Required points is ( , 0, −2)
5
Then, AB = √(4 − 5)2 + (−3 + 4)2 + (1 − 2)2
6. Ans (C)
= √1 + 1 + 1 = √3
The distance between given points
BC = √(7 − 4)2 + (−6 + 3)2 + (4 − 1)2
= √(2 − 1)2 + (2 − 4)2 + (3 − 5)2
= √9 + 9 + 9 = 3√3
= √1 + 4 + 4 = 3
CD = √(8 − 7)2 + (−7 + 6)2 + (5 − 4)2 7. Ans (A)
= √1 + 1 + 1 = √3 Let the coordinate of a point Q on x-axis be (a, 0, 0)
AD = √(8 − 5)2 + (−7 + 4)2 + (5 − 2)2 ∴ Distance, PQ = √(a − a)2 + (b − 0)2 + (c − 0)2
= √9 + 9 + 9 = 3√3 = √b 2 + c 2
Position vector of 8. Ans (B)
⃗⃗⃗⃗⃗ = (4 − 5)î + (−3 + 4)ĵ + (1 − 2)k̂
AB Given that, A(5, −1, 1), B(7, −4, 7), C(1, −6, 10) and
= −î + ĵ − k̂ D(−1, −3, 4)
And position vector of Now, AB = √(7 − 5)2 + (−4 + 1)2 + (7 − 1)2
BC = (7 − 4)î + (−6 + 3)ĵ + (4 − 1)k̂
⃗⃗⃗⃗⃗ = √4 + 9 + 36 = 7
= 3î − 3ĵ + 3k̂ BC = √(1 − 7)2 + (−6 + 4)2 + (10 − 7)2
⃗⃗⃗⃗⃗ = (−î + ĵ − k̂). (3î − 3ĵ + 3k̂)
⃗⃗⃗⃗⃗ ∙ BC
Now, AB = √36 + 4 + 9 = 7
= −3 − 3 − 3 ≠ 0
CD = √(−1 − 1)2 + (−3 + 6)2 + (4 − 10)2
∴ ABCD is parallelogram. = √4 + 9 + 36 = 7
4. Ans (C)
DA = √(5 + 1)2 + (−1 + 3)2 + (1 − 4)2
Let point is (α, β, γ)
= √36 + 4 + 9 = 7
∴ (α − α)2 + β2 + γ2 = α2 + (β − b)2 + γ2
∴ AB = BC = CD = DA = 7,
= α2 + β2 + (γ − c)2
⃗⃗⃗⃗⃗ ∙ BC
Also, AB ⃗⃗⃗⃗⃗ ≠ 0 (These are not perpendicular)
= α2 + β2 + γ2
a b c
∴ ABCD is not square. It is rhombus
We get, α = , β = and γ =
2 2 2 9. Ans (D)
a b c
∴ Required point is ( , , ) Let the point in xy-plane be P(x1 , y1 , 0). Let the
2 2 2

5. Ans (A) given points are A(2, 0, 3), B(0, 3, 2)


Let the point P(x, y, z) divides the line joining the And C(0, 0, 1)
points A and B in the ratio m: 1. According to the given condition,
AP 2 = BP 2 = CP 2
∴ (x1 − 2)2 + y12 + 9 = x12 + (y1 − 3)2 + 4
= x12 + y12 + 1
Since, point P is on XOZ-plane
From Ist and IInd terms,
∴ y coordinate = 0

MATHEMATICS Page | 8A. 8


INTRODUCTION TO THREE-DIMENSIONAL GEOMETRY

x12 + 4 − 4x1 + y12 + 9 = x12 + y12 − 6y1 + 9 + 4 ⇒ 8PB = 16 + PB 2 − PA2


⟹ 4x1 − 6y1 = 0 … . . (i) = 16 + (x + 2)2 + (y − β)2 + (k − A)2 − (x − 2)2
From IInd and IIIrd terms, − (y − β)2 − (z − A)2
x12 + y12 + 9 − 6y1 + 4 = x12 + y12 + 1 = 16 + 8x
⟹ 6y1 = 12 ⟹ y1 = 2 PB = 2 + x ⇒ PB 2 = (2 + x)2
On putting the value of y1 in Eq.(i), we get x1 = 3 ⇒ (y − 3)2 + (z − 4)2 = 0
Hence, required point is (3, 2, 0). 16. Ans (B)
10. Ans (D) x y
+ = 1, z = 0
a b
A = (2,1,5), B = (3,2,3), C = (4,0,4)
locus is a straight line in xy plane
AB = √1 + 1 + 4 = √6
17. Ans (C)
BC = √1 + 4 + 1 = √6
Let P = (x, y, z)
CA = √4 + 1 + 1 = √6
d1 = The distance from P to x-axis= √y 2 + x 2
△ ABC is equilateral △le centroid G = (3,1,4)
d2 = The distance from P to y-axis= √x 2 + z 2
11. Ans (B)
d3 = The distance from P to z-axis= √x 2 + y 2
A = (1, −3,4), B = (1,3,4)
Given d12 + d22 + d23 = 8
Let P = (x, y, z)
2(x 2 + y 2 + z 2 ) = 8 ⇒ x 2 + y 2 + z 2 = 4
PA = PB
18. Ans (A)
(x − 1)2 + (y + 3)2 + (z − 4)2
A = (2,0, −3), B = (0,4,1)
= (x − 1)2 + (y − 3)2 + (z − 4)2
Let P = (x, y, z)
⇒ (y + 3)2 − (y − 3)2 = 0
Given PA2 + PB 2 = AB 2
⇒ 4y(3) = 0 ⇒ y = 0
(x − 2)2 + y 2 + (z + 3)2 + x 2 + (y − 4)2 + (z − 1)2
12. Ans (B)
= 4 + 16 + 16
Let P = (x, y, z)
2 2 2
2x + 2y + 2z − 4x − 8y + 4z + 30 = 36
The distance from P to xy plane =The distance
x 2 + y 2 + z 2 − 2x − 4y + 2z − 3 = 0
from p to yz plane
19. Ans (B)
⇒ |z| = |x| ⇒ z 2 = x 2
A = (3,2,5), B = (2,1,1)
13. Ans (C)
C = (−1,4,1), C = (0,5,5)
Let P = (x, y, z)
AB = √1 + 1 + 16 = √18
The distance from P to zx plane=3
⇒ |y| = 3 BC = √9 + 9 + 0 = √18

14. Ans (B) CD = √1 + 1 + 16 = √18


Let P = (x, y, z) AD = √9 + 9 = √18
The distance from P to xy plane =5 AC = √16 + 4 + 16 = 6
2
|z| = 5 ⇒ z = 25 BD = √4 + 16 + 16 = 6
15. Ans (B) A, B, C, D form a square
A = (2,3,4), B = (−2,3,4) 20. Ans (D)
Let P = (x, y, z) A = (2,4, −1), B = (3,6, −1), C = (4,5,1)
PA + PB = 4 ⇒ PA = 4 − PB ⇒ PA2 Let fourth vertex = D
2
= 16 + PB − 8PB D = A + C − B = (3,3,1)

MATHEMATICS Page | 8A. 9


INTRODUCTION TO THREE-DIMENSIONAL GEOMETRY

21. Ans (D) Distance from origin = √1 + 4 + 9


From given = √14 and from y-axis = √1 + 9
AB = √12 + 12 + 52 = √27 = √10.
BC = √02 + 22 + 42 = √20 28. Ans (D)
It is obvious.
CD = √12 + 12 + 52 = √27
29. Ans (B)
and DA = √0 + 22 + 42 = √20
Thus AB = CD and BC = DA Distance from x-axis = √y 2 + z 2

AC = √12 + 32 + 12 = √11 = √(b 2 + c 2 ).


30. Ans (B)
and DB = √12 + 12 + 92 = √83
So, AC ≠ DB ∵ AB = BC = CD = DA = 7. Also ⃗⃗⃗⃗⃗
AB ⋅ ⃗⃗⃗⃗⃗
BC ≠ 0

Hence, A,B,C and D forms a parallelogram ∴ ABCD is not square. It is rhombus.

22. Ans (A)


P = (3,5,4), Q = (5,7,5), R = (6,5,7)
PQ = √4 + 4 + 1 = 3, QR = √1 + 4 + 4 = 3
RP = √9 + 0 + 9 = 3√2
PQ2 + QR2 = RP 2
orthocenter = Q
23. Ans (D)
A = (2,1,5), B = (3,2,3), C = (4,0,4)
AB = √1 + 1 + 4 = √6, BC = √1 + 4 + 1 = √6
CA = √4 + 1 + 1 = √6
Given Δle is equilateral Δle
Distance between circumcenter & orthocenter = 0
24. Ans (A)
H = (−3,5,2), S = (6,2,5)
3G = 2S + H
3G = (12,4,10) + (−3,5,2)
= (9,9,12)
G = (3,3,4)
25. Ans (A)
Third vertex = (3,1,2)
26. Ans (B)
Here a = √4 + 4 + 1 = √9,
b = √1 + 4 + 4 = √9 and
c = √1 + 16 + 1 = √18.
Obviously, it is a right angled and isosceles
triangle.
27. Ans (C)

MATHEMATICS Page | 8A. 10


PROBABILITY

HINTS AND SOLUTIONS

1. Ans (B) The sample space for this experiment is


The sample space is S = {HH, HT, TH, TT} S = {RR, RB, BR, BB}, where R denotes the red ball
2. Ans (C) and B denotes the black ball.
I. There are 13 letters in the word 8. Ans (B)
'ASSASSINATION'. I. Let us denote blue balls by B1 , B2 , B3 and the
II. There are 10 letters in the word 'NAGATATION'. white balls by W1 , W2 , W3 , W4 . Then, a sample space
3. Ans (A) of the experiment is
Let H and T represent a head and tail. S = {HB1 , HB2 , HB3 , HW1 , HW2 , HW3 , HW4 ,
Let red ball represented by R1 , R 2 and black ball T1, T2, T3, T4, T5, T6}
represented by B1 , B2 and B3 . Here, HBi means head on the coin and ball Bi is
drawn, HWi means head on the coin and ball Wi is
drawn. Similarly, Ti means tail on the coin and the
number i on the die.
II. In the experiment, head may come up on the 1st
toss, or the 2nd toss, or the 3 rd toss and so on till
head is obtained. Hence, the desired sample space
is
4. Ans (B)
S = {H, TH, TTH, TTTH, TTTTH, … }
The sample space associated with the given
9. Ans (C)
random experiment will have 36 sample points,
We know that, r objects can be select from n
and also, when outcomes is doublet, then coin is
objects in n Cr ways
tossed. So, sample space will have following sample
∵ Four cards can be drawn from a pack of 52 cards
points also, 52
in C4 ways.
(11, H), (11, T), (22, H), (22, T),(33, H), (33, T),
10. Ans (A)
(44, H)(44, T),(55, H), (55, T),(66, H)(66, T)
Let R, W and B denote the red, white and blue die
Hence, total number of sample points = 42
respectively. When a die will be selected, then
5. Ans (C)
there will be possibilities for colour R, B or W. But
The sample space S is
there will be 6 possibilities for 6 numbers (1 to 6)
S = {HH, TH, (HT, 1), (HT, 2), (HT, 3),
with each colour. So, the sample space is
(HT, 4), (HT, 5), (HT, 6), (TT, 1), (TT, 2),
S = {R1, R2, R3, R4, R5, R6, W1, W2W3,
(TT, 3), (TT, 4), (TT, 5), (TT, 6)}
W4, W5, W6,B1, B2, B3, B4, B5, B6}
Hence, total number of outcomes = 14
11. Ans (D)
6. Ans (A)
The sample space of the experiment is
The sample space is given by S = {(x, y): x is the
S= {HHH,HHT,HTH,THH,HTT,THT,TTH,TTT}
number on the yellow die and y is the number on
And A = {TTT}
the green die }. The number of elements of this
B = {HTT, THT, TTH}
sample space is 6 × 6 =36
C = {HHT, HTH, THH, HHH}
7. Ans (D)
MATHEMATICS Page | 9A. 1
PROBABILITY

Now, A ∪ B ∪ C = S We know that, the set E is a subset of the sample


Therefore, A, B and C are exhaustive space S. Similarly, we find the following
events. Also, A ∩ B = ϕ, A ∩ C = ϕ correspondence between events and subsets of S.
and B ∩ C = ϕ 14. Ans (A)
Therefore, the events are pair-wise For every event A, there corresponds another
disjoint, i.e., they are mutually exclusive. event A′ called the complementary event to A. It is
Hence, A, B and C form a set of mutually also called the event 'not A '
exclusive and exhaustive events. 15. Ans (C)
12. Ans (A) If an event has more than one sample point, then it
The sample space associated with the given is called a compound event.
random experiment is given by 16. Ans (B)
S = {(1,1), (1,2), (1,3), (1,4), (1,5), (1,6) The sample space associated with the given
(2,1), (2,2), (2,3), (2,4), (2,5), (2,6) random experiment is given by
(3,1), (3,2), (3,3), (3,4), (3,5), (3,6) S = {(1,1), (1,2), (1,3), (1,4), (1,5), (1,6)
(4,1), (4,2), (4,3), (4,4), (4,5), (4,6) (2,1), (2,2), (2,3), (2,4), (2,5), (2,6)
(5,1), (5,2), (5,3), (5,4), (5,5), (5,6) (3,1), (3,2), (3,3), (3,4), (3,5), (3,6)
(6,1), (6,2), (6,3), (6,4), (6,5), (6,6)} (4,1), (4,2), (4,3), (4,4), (4,5), (4,6)
Now, (5,1), (5,2), (5,3), (5,4), (5,5), (5,6)
A = Getting an even number or the first die (6,1), (6,2), (6,3), (6,4), (6,5), (6,6)}
A = {(2,1), (2,2), (2,3), (2,4), (2,5), (2,6) Event B given above
(4,1), (4,2), (4,3), (4,4), (4,5), (4,6) and C = Getting atmost 5 as sum of the numbers on
(6,1), (6,2), (6,3), (6,4), (6,5), (6,6)} the two dice
B = Getting an odd number on first die C = {(1,1), (1,2), (1,3), (1,4), (2,1)
B = {(1,1), (1,2), (1,3), (1,4), (1,5), (1,6), (2,2), (2,3), (3,1), (3,2), (4,1)}
(3,1), (3,2), (3,3), (3,4), (3,5), (3,6) B or C = B ∪ C =
(5,1), (5,2), (5,3), (5,4), (5,5), (5,6)} {(1,1), (1,2), (1,3), (1,4), (1,5), (1,6), (2,1),
Thus, A and B = A ∩ B = ϕ (2,2), (2,3), (3,1), (3,2), (3,3),
13. Ans (B) (3,4), (3,5), (3,6), (4,1), (5,1), (5,2),
We have studied about random experiment and (5,3), (5,4), (5,5), (5,6)}
sample space associated with an experiment. The So n(B ∪ C) = 22
sample space serves as a universal set for all 17. Ans (B)
questions concerned with the experiment. Consider the experiment of throwing a die.
Consider the experiment of tossing a coin two We have, S = {1,2,3,4,5,6}.
times. An associated sample space is S = Let us define the following events.
{HH, HT, TH, TT}. A : 'a number less than 4 appears'.
Now, suppose that we are interested in those B : 'a number greater than 2 but less than
outcomes which correspond to the occurrence of 5 appears' and
exactly one head. We find that HT and TH are the C : 'a number greater than 4 appears'
only elements of S corresponding to the occurrence Then, A = {1,2,3}, B = {3,4} and C = {5,6}
of this happening (event). These two elements We observe that,
form the set E = {HT, TH}. A ∪ B ∪ C = {1,2,3} ∪ {3,4} ∪ {5,6} = S
MATHEMATICS Page | 9A. 2
PROBABILITY

Such events A, B and C are called exhaustive events. = P(A) + P(A‾)P(A‾)P(A) + P(A‾)P(A‾)P(A‾)P(A‾)P(A)
In general, if E1 , E2 , … , En are n events of a sample +⋯
space S and if 1 5 5 1 5 5 5 5 1
= + ( )( )( ) + ( )( )( )( )( )+ ⋯
E1 ∪ E2 ∪ E3 ∪ … ∪ En = ⋃ni=1 Ei =S 6 6 6 6 6 6 6 6 6
1 1 5 2 1 5 4
Then E1 , E2 , … , En are called exhaustive events. In = + ( ) + ( ) +⋯
6 6 6 6 6
other words, events E1 , E2 , … , En are said to be
1/6 6
exhaustive, if atleast one of them necessarily = 2 =
5 11
1−( )
occurs whenever the experiment is performed. 6
Further, if Ei ∩ Ej = ϕ for i ≠ j, i.e., events, Ei and Ej 22. Ans (C)
Total number of outcomes = 8
are pairwise disjoint and ⋃ni=1 Ei = S, then
Favourable cases are HTT, THT, TTH.
E1 , E2 , … … , En are called mutually exclusive and
∴ Number of favourable outcomes = 3
exhaustive events.
3
18. Ans (D) ∴ Required probability =
8

Let E = The die shows 4 = {4} 23. Ans (B)


Let F = The die shows even number Let us consider an experiment of 'tossing a coin
= {2,4,6} ∴ E ∩ F = {4} ≠ ϕ "twice". The sample space of this experiment is S =
{HH, HT, TH, TT}
Let, the following probability be assigned to the
outcomes
1 1
P(HH) = , P(HT) = , P(TH)
Hence, E and F are not mutually exclusive. 4 7
2 9
19. Ans (D) = , P(TT) =
7 28
Let the set be S
Clearly, this assignment satisfies the conditions of
Then, S =
axiomatic approach. Now, let us find the
{1,2,3,4,5,6,7,8,9,10,11,12,13,14,15,16,17,18,
probability of the event E :
19,20,21,22,23,24,25}
'Both the tosses yield the same result'.
Now, let the event E = Getting a prime number
Here, E = {HH, TT}
when each of the given number is equally likely to
Now, P(F) = ∑P(mi ), for all nni ∈ F
be selected. E = {2,3,5,7,11,13,17,19,23}.
1 9 4
20. Ans (A) = P(HH) + P(TT) = + =
4 28 7
Let E be the event that a black face card is chosen. For the event F : 'exactly two heads', we have F =
1
The outcomes in E are Jack, Queen, King of spade {HH} and P(F) = P(HH) =
4
and club. 24. Ans (A)
21. Ans (B) A ∪ B = A ∪ (B − A), where A and B − A are
Here, total number of outcomes = 6 mutually exclusive, and
1
Let P(A) = probability of getting 1 = B = (A ∩ B) ∪ (B − A), where A ∩ B and B − A are
6

1 5 mutually exclusive.
∴ P(A‾) = 1 − P(A) = 1 − =
6 6 Using axiom (iii) of probability, we get
Since, first time 1 occurs at the odd throw.
P(A ∪ B) = P(A) + P(B − A)
So, the required probability
and P(B) = P(A ∩ B) + P(B − A).

MATHEMATICS Page | 9A. 3


PROBABILITY

Solve Eq. (iii) from Eq. (ii), (i) Since, P(D) = −0.20, this is not possible as 0 ≤
If A and B are disjoint sets, i.e., they are mutually P(A) ≤ 1 for any event A.
exclusive events, then A ∩ B = ϕ (ii) P(S) = P(A ∪ B ∪ C ∪ D)
9 45 27 46 127
Therefore, P(A ∩ B) = P(ϕ) = 0 = + + + = ≠ 1.
120 120 120 120 120
Thus, for mutually exclusive events A and B, we
This violates the condition that P(S) = 1
have
29. Ans (A)
n(S) = 9 C3
n(E) = 3 C1 × 4 C1 × 2 C1
3×4×2 24×3!
Probability = 9C = × 6!
3 9!
24×6 2
= = .
9×8×7 7

30. Ans (C)


Given, P(A ∪ B) = 0.6
P(A ∪ B) = P(A) + P(B) ∴ P(A) + P(B) = P(A ∪ B) + P(A ∩ B) = 0.9
which is axiom (iii) of probability. and P(A ∩ B) = 0.3
25. Ans (A) P(A′ ) + P(B ′ ) = [1 − P(A)] + [1 − P(B)]
Given, P(A ∪ B) = P(A ∩ B) = 2 − [P(A) + P(B)]
This is Possible only when P(A) = P(B). = 2 − 0.9 = 1.1
26. Ans (D) 31. Ans (D)
Probability when first two cards are queens out of Let M and B denote the students of Mathematics
4C
1
3C
1 and Biology.
52 cards = 52 C × 51 C
1 1
Given, P(M) = 40%, P(B) = 30%
Probability when third card is king out of 50 cards
P(M ∩ B) = 10%
4C
1
= 50 C ( ∵ 10% of the class study both subjects i.e., they
1

Hence, the required probability are common in both subjects)


4
C1 × 3 C1 × 4 C1 ∴ P(M or B) = P(M ∪ B)
= 52 C × 51 C × 50 C
1 1 1 = P(M) + P(B) − P(M ∩ B)
4 3 4 40 30 10 60
= × × = + − = = 60% = 0.6
52 51 50 100 100 100 100
27. Ans (D) 32. Ans (C)
Total numbers = 90 If all the outcomes 1,2, … ,10 are considered to be
Numbers divisible by 6 equally likely, then the probability of each outcome
= 6,12,18,24,30,36,42,48,54,60,66,72, 1
is
10
78,84,90
Now, P(A) = P(2) + P(4) + P(6) + P(8)
Number divisible by 8 =
1 1 1 1 4 2
8,16,24,32,40,48,56,64,72,80,88 = + + + = =
10 10 10 10 10 5
Total number of numbers divisible Also, event 'not A′ = A′ = {1,3,5,7,9,10}
by 6 or 8 = 15 + 11 − 3 = 23 Now, P(A′ ) = P(1) + P(3) + P(5) + P(7) + P(9) +
23 6 3
∴ Required probability = P(10) = =
90 10 5

28. Ans (D) ′) 3


Thus, P(A = = 1 − = 1 − P(A)
2
5 5

MATHEMATICS Page | 9A. 4


PROBABILITY

Also, we know that A′ and A are mutually exclusive By addition theorem of probability,
and exhaustive events i.e., P(A ∪ B) = P(A) + P(B) − P(A ∩ B)
A ∩ A′ = ϕ and A ∪ A′ = S ⇒ P(A ∪ B) = P(A) + P(B)
′)
or P(A ∪ A = P(S) ⇒ P(A ∪ B) = 0.25 + 0.4
Now, P(A) + P(A′ ) = 1, by using axioms (ii) and [∵ P(A) = 0.25, P(B) = 0.4( given )]
(iii) P(A ∪ B) = 0.65..(i)
′)
Or P(A = P(not A) = 1 − P(A) Now, we have P(Ac ∩ B c ) = P(A ∪ B)c
33. Ans (D) ⇒ P(Ac ∩ B c ) = 1 − P(A ∪ B)
Let E and F denote the events that Anil and Ashima = 1 − 0.65 = 0.35
will qualify the examination, respectively. Given 35. Ans (A)
that, Given, P(A ∩ B c ) = 0.25 and
P(E) = 0.05, P(F) = 0.10 and P(Ac ∩ B) = 0.5
P(E ∩ F) = 0.02. Then, ∴ P{(A ∪ B)c } = 1 − P(A ∪ B)
I. The event 'both Anil and Ashima will not qualify = 1 − {P(A ∩ B c ) + P(Ac ∩ B) + P(A ∩ B)}
the examination' may be expressed as E ′ ∩ F ′ . = 1 − (0.25 + 0.5 + 0)

Since, E is 'not E ', i.e., Anil will not qualify the [∵ A and B are mutually exclusive events
examination and F ' is 'not F ', P(A ∩ B) = 0] = 1 − 0.75 = 0.25
i.e., Ashima will not qualify the examination 36. Ans (C)
′ ′ ′ 1
Also E ∩ F = (E ∪ F) We have P(A ∪ B) =
2
Now P(E ∪ F) = P(E) + P(F) − P(E ∩ F) 1
⇒ P(A ∪ (B − A)) =
or P(E ∪ F) = 0.05 + 0.10 − 0.02 = 0.13 2
Therefore, P(E ′ ∩ F ′ ) = P(E ∪ F)′ 1
⇒ P(A) + P(B − A) =
2
= 1 − P(E ∪ F)
(Since A and B − A are mutually exclusive)
= 1 − 0.13 = 0.87
1
II. P (atleast one of them won’t qualify) ⇒ 1 − P(A‾) + P(B − A) =
2
= 1 − P (both of them will qualify) 2 1
⇒ 1− + P(B − A) =
= 1 − 0.02 = 0.98 3 2
1
III. The event only one of them will qualify the ⇒ P(B − A) =
6
examination is same as the event either (Anil will 1
⇒ P(A‾ ∩ B) = ( Since A‾ ∩ B = B − A)
6
qualify and Ashima will not qualify) or (Anil will
37. Ans (B)
not qualify and Ashima will qualify)
1
i.e., E ∩ F ′ or E ′ ∩ F, where E ∩ F ′ and E ′ ∩ F are Given, P(A) = P(B)
2

mutually exclusive. ⇒ P(B) = 2P(A) and A ∪ B = S


Therefore, P (only one of them will qualify) ∴ P(A ∪ B) = P(S) = 1
= P(E ∩ F ′ or E ′ ∩ F) ⇒ P(A) + P(B) = 1
= P(E ∩ F ′ ) + P(E ′ ∩ F) [∵ P(A ∩ B) = 0] ⇒ P(A) + 2P(A) = 1
= P(E) − P(E ∩ F) + P(F) − P(E ∩ F) 1
⇒ P(A) =
3
= 0.05 − 0.02 + 0.10 − 0.02 = 0.11
38. Ans (A)
34. Ans (D)
P(A ∪ B ′ ) = P(A) + P(B ′ ) − P(A)P(B ′ )
Given, A and B are mutually exclusive events.
∴ 0.8 = 0.3 + P(B ′ ) − 0.3P(B ′ )
⇒ P(A ∩ B) = 0
MATHEMATICS Page | 9A. 5
PROBABILITY

⇒ 0.5 = P(B ′ )(0.7) 5 biscuits can be distributed away 3 Children in


5 5 2
⇒ P(B ′ ) = ∴ P(B) = 1 − = the following ways :
7 7 7
(2,2,1), (2,1,2), (1,2,2), (3,1,1), (1,3,1), (1,1,3)
39. Ans (D)
No. of ways =6
Given, P(E‾2 ) = 0.6 = P(E1 ∪ E2 )
1 Child will get 2 biscuits
Since, E1 , E2 are mutually exclusive events, then
Possible cases = (2,2,1),(2,1,2) (1,2,2)
P(E1 ∩ E2 ) = 0
i.e., 3 ways
Now, P(E1 ∪ E2 )
3 1
= P(E1 ) + P(E2 ) − P(E1 ∩ E2 ) Required Prob = =
6 2

⇒ 0.6 = P(E1 ) + 1 − P(E‾2 ) − 0 44. Ans(A)


⇒ 0.6 = P(E1 ) + 0.4 n(A) = 15
⇒ P(E1 ) = 0.2 n(S) = 36
40. Ans (A) n(A) 15 5
P(A) = = =
P(A‾ ∩ B) = P(B) − P(A ∩ B) n(S) 36 12
45. Ans(D)
= 0.2 − 0.1 = 0.1
Let A, B and C be the three 6-faced dice.
Then, according to the question,
41. Ans(B)
Since two dices has to be equal, that value can be
Out of 10 ,1 is right and 9 is wrong.
6
WWWWR any of the 6 faces, i.e., 𝐶1 cases.
9 8 7 6 1 1 Now for each case, 2 equal dices can be selected
× × × × =
10 9 8 7 6 10
3
42. Ans(D) from 3 dices in 𝐶2 i.e., 3 ways.

No. of days in a leap =366 And for each of the above, the third dice can have
So, there will be 52 weeks and 2 days any of the 5 remaining faces
So, every leap year has 52 Tuesday The possible outcomes are P(A)=1/6, P(B)=1/6,
Now, the probability depends on P(C)=5/6, P(A)=1/6, P(B)=5/6, P(C)=1/6
remaining 2 days. The possible pairing of days and P(A)=5/6, P(B)=1/6, P(C)=1/6. Hence the

are Sunday-Monday required probability


1 1 5 90
Monday-Tuesday = ∗ ∗ *6*3=
6 6 6 216
Tuesday-Wednesday 46. Ans(C)
Wednesday-Thursday Four people are chosen,
Thursday-Friday If no people have same birthdays, then everyone
Friday-Saturday has a different birthday.
Saturday-Sunday So, the probability that no two people have same
There are total 7 pairs and out of 7 pairs, 7 6 5 4 120
birthday is = × × × =
7 7 7 7 343
only 2 pairs have Tuesday. The remaining 5 pairs
47. Ans(C)
does not include Tuesday.
Total 2 digit numbers that can be formed by using
5
⇒ Req. Prob = 1,2,3&4 is equal to 4*4=16
7
43. Ans(A) The number of numbers of two digits 12,24,32,44

No. of biscuits=5 & No. of Children=3 Which are divisible by 4


4 1
probability = =
16 4

MATHEMATICS Page | 9A. 6


PROBABILITY

48. Ans(C) 10
X ∈N, Total ways = 𝐶1 =100
"Since " A" and " B" are mutually exclusive"
⇒n(S)=100
∴ P(A ∩ B) = 0
(x−30)(x−20)
<0
∴ P(A ∪ B) = P(A) + P(B) (x−40)

P(S) = P(A) + P(B)[∵ s = A ∪ B] ∴x∈ (−∞,20) ∪ (30,40)


3x + 1 1 − x ∴E= {1,2, 3,...19,31,32,33,...,39}
1= +
3 4 ⇒n(E)=28
12x + 4 + 3 − 3x 28 7
1= = 9x + 7 ∴ Required probability = =
12 100 25

5 55. Ans(A)
9x = 5 ⇒ x =
9 The sample space for given condition is
1 5
∴ x ∈ [− , ] (1,8),(2,8),(3,8),(4,8),(5,8),(6,8),(7,8)
3 9
(1,7),(2,7),(3,7),(4,7),(5,7),(6,7)
49. Ans(A)
40 (1,6),(2,6),(3,6),(4,6),(5,6)
C2 + 60 C2 17
Prob = 100 C
= (1,5),(2,5),(3,5),(4,5)
2 33
50. Ans(C) (1,4),(2,4),(3,4)

n(S) = 100
C1 = 100 (1,3),(2,3)

E = {5,15,25,35,45,50 … . ,59,65,75,85,95} (1,2)


19 Number of sample space is 28.
n(E) = 19, P(E) =
100 Number of sample space where the sum of two
51. Ans(B) numbers is more than 13 is 2(i.e.,(6,8),(7,8)).
Since only 15 places are occupied, 10 cars have 2
Therefore, the probability is
left. Now, if both the neighbouring places are to 28

be empty, then 8 of the remaining 22 cars 56. Ans(C)

12
1
(excluding his own car) have left. n(S) = C4 , n(E) = 6 C2 , P(E) =
33
22
𝐶8
Required probability = 24 57. Ans(B)
𝐶10
Since, if is an eight-storey building. So, there are 7
22! 14! ×10! 15
= × = possible options for them in 7 floors in total if
14! × 8! 24! 92

52. Ans(A) ground floor is not considered.


We know that number of ways n people can be Hence, total possible outcomes
seated around a round table=(n-1)! =7×7×7×7×7=75
7 people can be seated at a round table in 6! ways. Thus, number of ways in which 5 persons can
The number of ways in which two distinguished 7
leave from seven floors differently = 𝑃5
persons will be next to each other=2× (5)! Hence,
7
𝑃5
5!
the required probability is =2× ∴ Required probability =
6! 75

=2×
5×4×3×2×1
=
1 58. Ans(C)
6×5×4×3×2×1 3
8
53. Ans(D) C2= all possible outcomes smallest of the 2 has
20
C2 19 is < 4.
P(E) = 25 C
=
2 30 If the small no is 1,then {2,3….,...8} can be the
54. Ans(C) others no. So, for 1,there will be 7 outcomes
similarly, for 2, there will be 6 outcomes
MATHEMATICS Page | 9A. 7
PROBABILITY

for 3, there will be 5 outcomes February has 29 days in a leap year out of
𝟕+𝟔+𝟓 9 which 28 days comprise 4 complete weeks,
P(A)= 8 =
𝐂2 14
thereby 4 Saturdays.
59. Ans(A)
Remaining one day can be any of the 7 days.
15 c × 5 c
2 1
Required Probability = 20 c ∴n(S)=7
3

60. Ans(A) For February to have 5 Saturdays, the remaining


2 points for Head and 1 point for Tail. day should be Saturday.
1
Cases of getting a total of odd no. of points is: ∴ Required probability =
7
Sum of three number will be odd when all three 64. Ans(C)
are odd or one is odd and two are even. 1
Probability to select a calendar month =
12
Case 1: OOO= when all are 1
And Probability to select a calendar day=
tails =TTT=1 possibility. 7

1 1 1 1 1 1 1
Probability when all are tails = × × = Req. prob = × =
2 2 2 8 12 7 84
Case 2: OEE= When one is tails and two are 65. Ans(C)
head =THH=3 possibility. Non-leap year has 365 days
Probability when one is tails and two are 1 year =52 weeks=52×7days
1 1 1 3
head=3× × × = =364 days [It contains 52 Sundays, Mondays and so
2 2 2 8

Probability of getting a total of odd no. of on]. ∴1 day will be left.


1 3 1 sample space for this 1 day
points is=( )+( )=
8 8 2
={Mon, Tue, Wed, Thurs, Fri, Sat, Sun}
61. Ans(C) 1 1
P[53 Sundays]= ,P[53 Tuesdays]=
Six faces are marked as 1, -1, 0, -2, 2, 3 7 7
1
Die is thrown thrice. & P[53 Thursdays]=
7

The following are possibilities for the sum to be 6 P(53 Sundays or 53 Tuesdays or 53 Thursdays)
1+2+3→ can be interchanged in 3!=6 ways =P(53 Sun)+P(53 Tue)+P(53 Thu)
0+3+3→ can be interchanged in 3!/2!=3 ways 1 1 1 3
= + + =
7 7 7 7
2+2+2→ = 1 way
66. Ans(B)
So, number of ways = 10 ways 13 c × 13 c × 13 c × 13 c
1 1 1 1
Required probability = 52 c
Total number of possibilities = 6*6*6 4

= 216 67. Ans(A)


10 52 cards (13 cards of hearts + 4 cards of 10’s)
Probability =
216
52!
Total no. of outcomes= 52 c2 = =26×51
62. Ans(C) 2!.50!

Leap year contains 366 days = Favourable outcomes = 3 c1 × 13


c1
3 c × 13 c 1
1 1
52 weeks +2 days P(E) = =
26×51 34
S={(Su, M), (M, Tu), (Tu, W), (W, Th ), 68. Ans(B)
( Th,Fri ), ( Fri,Sat ), (Sa, Su)} n(S) = 9 C3 ; n(E) = 3 C1 ⋅ 4 C1 ⋅ 2 C1
∴ E: 53 Sundays and 53 Mondays, n(E) 2
P(E) = =
n(E) 1 n(S) 7
P(E) = =
n(S) 7 69. Ans(C)
63. Ans(C) n(S) = 12
c3

MATHEMATICS Page | 9A. 8


PROBABILITY
3 2 3 18
n(E) = 4 c2 × 8 c1 + 3 c2 × 9 c1 + 5 c2 × 7 c1 (i) A - girl, B - boy, C - boy: × × =
4 3 4 64
1 2 3 6
(ii) A - boy, B - girl, C - boy: × × =
4 4 4 64
1 2 1 2
(iii) A - boy, B - boy, C - girl: × × =
4 4 4 64

Hence, required probability


18 6 2 13
= + + =
64 64 64 32

75. Ans(C)
In 'ASSISTANT' and 'STATISTICS', the same letters
are A,I,S,T
n(E) 145 19
P(E) = = = 2c 1c 1
n(S) 220 44 Prob. of choosing A= 9 1 × 10 1 =
c1 c1 45
70. Ans(C) 1c 2c 1
Prob. of choosing I= 9 1 × 1
10 c =
c1 1 45
Selection of 6 months from the 12 months and
3c 3c 1
then deciding the order of the birthdays of the six Prob. of choosing S= 9 1 × 10 1 =
c1 c1 10
12 2c 3c 1
girls can be done in 𝐶6∗6! ways. Prob. of choosing T= 9 1 × 10 1 =
c1 c1 15
12 12
𝐶6 ×6! 𝑝6 So, total probability
The required probability = =
126 126 1 1 1 1 19
= + + + =
71. Ans(C) 45 45 10 15 90

There are 4 even numbers and 5 odd numbers 76. Ans(D)

Let A = the event of choosing odd numbers Only 4 Favourable cases for G.P are

B = The event of getting the sum an even number. (1,2,4), (2,4,8), (1,3,9), (4,6,9)
4
Then A∩B = The event of choosing odd numbers Required probability =11
C3
whose sum is even
77. Ans(B)
∴n(B)= 4 c2 + 5 c2 =16 20
n(S) = C2 = 190
5
And n(A∩B)= c2 =10
Sum odd = one even and one odd
n(A∩B) 10 5
∴ Required probability = = = n(A) = 10
C1 × 10
C1 = 100
n(B) 16 8

72. Ans(C) 100 10


P(A) = =
Since there are 10 digits (0 to 9) 190 19
10 78. Ans(D)
n(S) = P2 = 90, n(E) = 1
1
1 1
and P(E)= P(A) = , P(B) =
90 2 3
73. Ans(C) P( Solving the problem )

Probability of correct key lost=


2 = A solved & B doesnt or A doesnt & B solved
6
2 2 or Both A &B solved
Req. Prob=1− =
6 3
=P(A) ⋅ P(B‾) + P(A‾) ⋅ P(B) + P(A) ⋅ P(B)
74. Ans(C) 1 2 1 1 1 1 2+1+1
= ⋅ + ⋅ + ⋅ =
2 3 2 3 2 3 6
Let: Group A: 3 girls, 1 boy
4 2
Group B: 2 girls, 2 boys = =
6 3

Group C: 1 girl, 3 boys 79. Ans(A)


The combination of 1 girl and 2 boys can be 3 4
P(A) = , P(B) =
4 5
selected in:

MATHEMATICS Page | 9A. 9


PROBABILITY

Contradict means A speak truth &B lies or A lies B E2 = Divisible by 8


speak truth = {8,16,24,32,40,48, … … }
i.e.,P(E) = P(A) ⋅ P(B‾) + P(A‾) ⋅ P(B) n(E2 ) = 25
3 1 1 4 7 Now
total 200
gives the value nearly 8 so we
= × + × = L.C.M of 6 & 8
4 5 4 5 20
conclude 𝑛(E1 ∩ E2 ) = 8
80. Ans(A)
n(E1 ) + n(E2 ) − n(E1 ∩ E2 )
Now,P(A ∪ B ∪ C) = P(E1 ∪ E2 ) =
𝑛(𝑆)
=P(A) + P(B) + P(C) − P(A ∩ B) − P(B ∩ C) −
33 + 25 − 8 50 1
P(C ∩ A) + P(A ∩ B ∩ C) = = =
200 200 4
A, B, C are mutually independent 85. Ans(A)
∴ P(A ∪ B ∪ C) Required probability
1 1 1 1 1 1 1 3 100 C + 100 C + 100 C +⋯….+ 100 C
1 3 5 99
= + + − − − + = =
2 3 4 6 12 8 24 4 2100
299 1
81. Ans(A) = =
2100 2
S = {1,2,3,4,5,6} 86. Ans(B)
A = {4,5,6} ⇒ n(A) = 3 52
n(S) = C2
B = {1,2,3,4} ⇒ n(B) = 4
13 13
A ∩ B = {4} ⇒ n(A ∩ B) = 1 n(E) = C1 × C1

3 1 4 2 13 × 13 13
P(A) = = , P(B) = = , P(E) = ×2×1 =
6 2 6 3 52 × 51 102
1 87. Ans(C)
P(A ∩ B) =
6 Total 12 persons it can be arranged in 12 !
P(A ∪ B) = P(A) + P(B) − P(A ∩ B) Ways i.e., n(s) = 12 !
1 2 1 We have 2 cases
= + −
2 3 6
(i)TSTSTSTSTSTS (i.e., 6!×6! = (6!)2 Ways)
3+4−1 6
= = =1 (ii)STSTSTSTSTST(i.e., 6!×6! = (6!)2 Ways)
6 6
82. Ans(D) Teacher sit 1st or Student sit 1st
P(E) =
total possible arrangements
P(A‾) + P(B‾)
(6!)2 + (6!)2 2 × (6!)2
= 1 − P(A) + 1 − P(B) = =
(12)! (12)!
= 2 − {(P(A ∪ B) + P(A ∩ B))} 2×6×5×4×3×2×1 1
= =
= 2 − {1.1} = 0.9 12×11×10×9×8×7 462

83. Ans(C) 88. Ans(C)

n(S) = 22 = 4 Since A,B &C are Mutually exclusive and

E = {(H, H)} Exhaustive events,

n(E) = 1 P(A)+P(B)+P(C)=1………………….(1)
2 1
1 P(B) + P(B) + P(B) = 1
3 2
P(E) =
4 4P(B) + 6P(B) + 3P(B)
84. Ans(A) =1
6
Given, n(S) = 200 6
P(B) =
E1 = Divisible by 6 13
3
= {6,12,18,24,30,36,42,48,54, … … } Given P(B) = P(A)
2

n(E1 ) = 33 4
So, P(A) =
13
MATHEMATICS Page | 9A. 10
PROBABILITY
1 6 1 2 1
Also P(C) = × Choosing a white ball from bag ' X ′ = ⋅ =
2 13 2 5 5

3 Choosing a white ball from bag ' Y = ⋅ = ′ 1 4 1


P(C) = 2 6 3
13
1 1 8
P(A ∪ C) = P(A) + P(C) − P(A ∩ B) Probability = + =
5 3 15

4 3 7 95. Ans(C)
= + −0=
13 13 13 1 1
P(H) = , P(W) = ; Since events are independent
7 5
89. Ans(B)
1 1 1 here
P(A) = , P(B) = , P(C) = 1
3 4 5 P(H ∩ W) = P(H) × P(W) =
35
2 3 4
P(A‾) = P(B‾) = , P(C‾) = 96. Ans(D)
3 4 5
100
n(S) = c3
P(A ∪ B ∪ C) = 1 − P(A‾)P(B‾)P(C‾)
2 3 4 L.C.M of 2 & 3 gives the numbers which are
P(A ∪ B ∪ C) = 1 − ⋅ ⋅
3 4 5 divisible by both 2 & 3
60 − 24 36 E = {6,12,18,24,30,36,42,48,54,
= =
60 60
60,66,72,78,84,90,96}
90. Ans(C) 16
n(E) = c3
n(S) = 90 16
c3 16 × 15 × 14
L.C.M of 6 & 8 gives the numbers which are P(E) = 100 c
=
3 100 × 99 × 98
divisible by both 6 & 8 4 4
= =
i.e., E = {24,48,72} 35 × 33 115
n(E) = 3 97. Ans(B)
3 1 n(S) = 4 × 4 × 3(ignore 0 in Hundreds place)
P(E) = =
90 30 if A be the event of getting 3-digit even numbers
91. Ans(D) =(unit place ends with 0 )+( unit place ends with
P(A ∩ B) = P(A) + P(B) − P(A ∪ B) 2 or 4)
= 0.4 + 0.3 − 0.5 = 0.2 =(4 × 3 × 1)+(3× 3 × 2)
P(B ∣ ∩ A) = P(A) − P(A ∩ B) =12+18
1 n(A) 30 5
= 0.4 − 0.2 = 0.2 = P(A) = = =
5 n(S) 4 × 4 × 3 8
92. Ans(A)
98. Ans(B)
n(S) = 8 C2
n(S) = 9 C3
3 2
n(E) = C2 × C1
n(E) = 3 C1 × 4 C1 × 2 C1
3 2
n(E) C2 × C1 3
P(E) = = 8C
n(E) C1 × 4 C1 × 2 C1 2
n(S) 2 P(E) = = 9C
=
n(S) 3 7
3×2 6 3
= = = 99. Ans(B)
8 × 7 56 28
93. Ans(A) In a non-leap year' there are 365 days which have

Total 3 persons can be arranged in 33 52 weeks and 1 day. If this day is a Tuesday or

i.e., n(S) = 33 Wednesday, then the year will have 53 Tuesday or

n(E) = 3 C1 = 3 53 Wednesday.

3 1 1 1 2
P(E) = = ∴ Required probability = + =
27 9 7 7 7
94. Ans(C) 100.Ans(B)

MATHEMATICS Page | 9A. 11


PROBABILITY

Since, the set of three consecutive numbers from 1 P(A ∪ B) = P(A) + P(B) − P(A ∩ B)
to 20 are (1,2,3), P(A ∪ B) = P(A) + P(B)
(2,3,4), (3,4,5), … , (18,19,20), i.e., 18 . P(A) + P(B) ≤ 1
18
P( numbers are consecutive) = P(A) ≤ 1 − P(B)
20 C
3

18 3 ∴ P(A) ≤ P(B‾)
= =
20 × 19 × 18 190 105.Ans(A)
3!
We have, P(A ∪ B) = P(A ∩ B)
P( three number are not consecutive )
P(A) + P(B) − P(A ∩ B) = P(A ∩ B)
3 187
=1− = [P(A) − P(A ∩ B)] + [P(B) − P(A ∩ B)] = 0
190 190
101.Ans(C) But P(A) − P(A ∩ B) ≥ 0 and

In a back of 52 cards 26 are red colour and 26 are P(B) − P(A ∩ B) ≥ 0

black colour. [∵ P(A ∩ B) ≤ P(A) or P(B)]

∴ P( both cards of opposite colour ) P(A) − P(A ∩ B) = 0 & P(B) − P(A ∩ B) = 0


26
C1 × 26 C1 26 × 26 26 ⇒ P(A) = P(A ∩ B)& P(B) = P(A ∩ B)
= = =
52 C
2 52 × 51 51 So, P(A) = P(B)
2
106.Ans(C)
102.Ans(C)
If all the girls sit together, then considered it as 1
If two persons sit next to each other, then consider
group.
these two people as 1 group.
Number of arrangements of 6 + 1 = 7 persons in
Now, we have to arrange 6 persons.
a row is 7 ! and the girls interchanges their seats
∴ Number of arrangements = 2! × 6 !
in 6 ! ways.
Total number of arrangements of 7 persons
6!7! 1
= 7! ∴ Required probability = =
12! 132

∴ Required probability =
2!6!
=
2 107.Ans(B)
7! 7
Total number of alphabets in the word probability
103.Ans (D)
= 11
We have digits 0,2,3,5.
Number of vowels = 4(O, A, I, I)
Number of divisible by 5 if unit place digit is ' 0 '
4
or ' 5 ' P( letter is vowel ) =
11

If unit place is ' 0 ' then first three places can be 108.Ans(C)
filled in 3 ! Ways Given, P(A fail ) = 0.2 and P(B fail ) = 0.3
If unit place is ' 5 ' then first place can be filled in ∴ P( either A or B fail ) ≤ P(A fail ) + P(B fail )
two ways and second and third place can be filled ≤ 0.2 + 0.3 ≤ 0.5
in 2 ! ways. 109.Ans(C)
So, number of numbers ending with digit ' 5 ' is We have, P(A ∪ B) = 0.6 and P(A ∩ B) = 0.2
2×2!= 4 P(A ∪ B) = P(A) + P(B) − P(A ∩ B)
∴ Total number of numbers divisible by 5 = 3! + 0.6 = P(A) + P(B) − 0.2
4 = 10 = n(E) P(A) + P(B) = 0.8
Also, total number of numbers = 3 × 3! = 18 P(A‾) + P(B‾) = 1 − P(A) + 1 − P(B)

∴ Required probability =
10
=
5 = 2 − [P(A) + P(B)] = 2 − 0.8 = 1.2
18 9
110.Ans (B)
104.Ans(A)
If M and N are any two events.
For mutually exclusive events, P(A ∩ B) = 0
MATHEMATICS Page | 9A. 12
PROBABILITY

∴ P(M ∪ N) = P(M) + P(N) − P(M ∩ N). Here n(S) = (6 x 6) = 36


111.Ans(B) Let E = event of getting a total more than 7
2 ={(2,6),(3,5),(3,6),(4,4),(4,5),(4,6),(5,3),
P(A′ ) =
3 (5,4),(5,5),(5,6),(6,2),(6,3),(6,4),(6,5),(6,6)}
2 1
P(A) = 1 − = So, n(E) = 15
3 3
15 5
2 Therefore, P(E) = =
P(B ′ ) = 36 12
7
117.Ans(A)
2 5
P(B) = 1 − = A∩B=ϕ
7 7
So, P(A∩ B)=P(A).P(B) P(A ∩ B) = 0
1 5 5 P(A ∩ B) = P(A) + P(B)
= . =
3 7 21
3 1 4
112.Ans(A) = + = = 0.8
5 5 5
n(s) = 62 118.Ans(A)
E = {(1,4), (4,1), (2,3), (3,2)} 8 7 7
R. P = × =
4 1 17 16 34
P(E) = =
36 9 119.Ans(A)
113.Ans(D) 4C
3
R. P = 10 C
3
Let E be the event that the sum of the numbers on
4×3×2 1
both the dice is more than 5 = =
10 × 9 × 8 30
So E′ will be the event that sum on the dices is less
than or equal to 5 120.Ans(A)
So E′={(1,1),(1,2),(2,1),(2,2),(2,3),(3,2),(4,1)(1,4 Vowels = E,U,A,I,O
),(3,1),(1,3)} Consonants = Q, T, N, S
⇒n(E′)=10 5c × 4c
1 1 5×4×2 5
R. P = 9c = =
9×8 9
Also n(S)=6×6=36 2

121.Ans (A)
So P(E′)= n(E′)/ n(S)=10/36=5/18
1 1 1
Hence P(E)=1−P(E′)=1−5/18=13/18 We have, 𝑃(𝐴) = , 𝑃(𝐵) = and 𝑃(𝐶) =
2 4 3

Now, required probability = 𝑃(𝐴 ∩ 𝐵 ∩ 𝐶 ′ ) +


Alternatively:E = { Sum more than 5} 𝑃(𝐴 ∩ 𝐵′ ∩ C) + 𝑃(𝐴′ ∩ 𝐵 ∩ C)
n(E) = 5 + 6 + 5 + 4 + 3 + 2 + 1 = 26
= 𝑃(𝐴) ⋅ 𝑃(𝐵) ⋅ 𝑃(𝐶 ′ ) + 𝑃(𝐴) ⋅ P(B′) ⋅ P(𝐶) +
26 13
P(E) = = 𝑃(𝐴′) ⋅ P(B) ⋅ P(C)
36 18
114.Ans(B) (∵ 𝐴, 𝐵, 𝐶 are independent +𝑃(𝐴′ ) ⋅ 𝑃(𝐵) ⋅ 𝑃(𝐶)
Total =10 and White with odd=2(i.e, 13 &15)
1 1 2 1 3 1 1 1 1 2+3+1 1
= ⋅ ⋅ + ⋅ ⋅ + ⋅ ⋅ = =
Red White 2 4 3 2 4 3 2 4 3 24 4

1,2,3,4,5,6 12, 𝟏𝟑, 14, 𝟏𝟓 122.Ans (B)


We have, 𝐴, 𝐵, 𝐶 are mutually exclusive and
2 1
Required probability= = exhaustive events.
10 5

115.Ans(D) ∴ 𝐴∩𝐵 =𝐵∩𝐶 =𝐶∩𝐴 =𝐴∩𝐵∩𝐶 =𝜙


4C 1
2
R. P = 52 C = ⇒ 𝑃(𝐴 ∩ 𝐵) = 𝑃(𝐵 ∩ 𝐶) = 𝑃(𝐶 ∩ 𝐴)
2 221

116.Ans(C)
MATHEMATICS Page | 9A. 13
PROBABILITY

= 𝑃(𝐴 ∩ 𝐵 ∩ 𝐶) = 0 3. Ans (C)


26C × 26C
3 3
Now, P(𝐴) = 2𝑃(𝐵) = 3𝑃(𝐶) Required probability = 52C
6

2 4. Ans (B)
⇒ 𝑃(𝐴) = 2𝑃(𝐵) and 𝑃(𝐶) = 𝑃(𝐵)
3
We have been given that, a committee has to be
∴ 𝑃(𝐴 ∪ 𝐵 ∪ 𝐶) = 𝑃(𝐴) + 𝑃(𝐵) + 𝑃(𝐶) made of 5 members from 6 men and 4 women.
We need to find probability that at least one
2
⇒ 1 = 2𝑃(𝐵) + 𝑃(𝐵) + 𝑃(𝐵) woman is present in the committee.
3
Let S be the total outcomes and A be the possible
11 3
⇒1= 𝑃(𝐵) ⇒ 𝑃(𝐵) = outcomes.
3 11
As a committee has to be made of 5 members
123.Ans (C)
from 6 men and 4 women, total cases would be,
P(A1 ∩ B1 ) = 1 − P(A ∪ B)
= 4C1 × 6C4 + 4C2 × 6C3 + 4C3 × 6C2 + 4C4 × 6C1
= 1 − [0.59 + 0.3 − 0.21]
+ 6 C5
= 0.32
= 60 + 120 + 60 + 6 + 6 = 252
124.Ans (B)
No. of ways in which at least one woman exist are
1 3 13
P(A) = , P(B) = , P(A ∪ B) = = 4C1 × 6C4 + 4C2 × 6C3 + 4C3 × 6C2 + 4C4 × 6C1 =
4 4 20
1 1 13 246
+𝑥− ⋅𝑥 =
4 4 20 246 41
Hence required probability = = .
252 42
3 13 5 8
𝑥− − = 5. Ans (A)
4 20 20 20
8 4 8 Mohan can gets one prize, 2 prizes or 3 prizes
𝑥= × =
20 3 15 and his chance of failure means he get no
125.Ans (B)
prize.
Use ∑ 𝑃(𝑤𝑖 ) = 1 12
Number of total ways = C3 = 220
Favourable number of ways to be failure
KCET MOCKTEST SOLUTIONS: = 9C3 = 84
1. Ans (B) 84 34
Hence required probability = 1 − = .
220 55
Total number of outcomes = 36
6. Ans (A)
Favourable number of outcomes = 6
(i) This question can also be solved by one student
i.e., (1, 1), (2, 2), (3, 3), (4, 4), (5, 5), (6, 6),
6 1
(ii) This question can be solved by two students
 Required probability = = .
36 6 simultaneously
2. Ans (B) (ii) This question can be solved by three students
The second ball can be red in two different all together.
ways 1 1 1
P(A) = , P(B) = , P(C) =
(i)First is white and second red 2 4 6
3 2 6 P(A ∪ B ∪ C) = P(A) + P(B) + P(C)
P(A) = × =
5 4 20 −[P(A). P(B) + P(B). P(C) + P(C). P(A)]
(ii) First is red and second is also red + [P(A). P(B). P(C
2 1 2 1 1 1 1 1 1 1 1 1
P(B) = × = = + + −[ × + × + × ]
5 4 20 2 4 6 2 4 4 6 6 2
6 2 2
Req. Prob= + = 1 1 1 33
20 20 5
+[ × × ]=
2 4 6 48
MATHEMATICS Page | 9A. 14
PROBABILITY

7. Ans (A) Since the drawn balls can’t be of red colour, as


3 1
P(A ∪ B) = , P(A ∩ B) = these are 2 in numbers.
4 4
2 1 Therefore favourable number of ways
P(Ā) = ⇒ P(A) =
3 3
= 3 C3 + 4 C3 .
∴ P(A ∩ B) = P(A) + P(B) − P(A ∪ B) 5
1 1 3 2
Hence the required probability = .
84
= + P(B) −  P(B) =
4 3 4 3
16. Ans (C)
2 1
P(Ā ∩ B) = P(B) − P(A ∩ B) = −
3 4 Total number of ways = 5!
8−3 5 Favourable number of ways 2.4!
= =
12 12 2.4! 2
Hence required probability = =
8. Ans (C) 5! 5

Number of tickets numbered such that it is 17. Ans (C)


10000 We are given that P(A ∪ B) = 0.6 and
divisible by 20 are = 500.
20
500 1
P(A ∩ B) = 0.2.
Hence required probability = = .
10000 20 We know that if A and B are any two events, then
9. Ans (C) P(A ∪ B) = P(A) + P(B) − P(A ∩ B)
Required probability is 1 − P (they go in concerned ⇒ 0.6 = 1 − P(Ā) + 1 − P(B̄) − 0.2
1 23
envelopes) = 1 − = . ⇒ P(Ā) + P(B̄) = 2 − 0.8 = 1.2.
4! 24

10. Ans (B) 18. Ans (B)


Numbers multiple of 5= (5,10,15,20,25,30) 1 1 9
P(A ∪ B) = P(A) + P(B) = + = .
4 5 20
Numbers multiple of 7 = (7,14,21,28)
{Since events are mutually exclusive, so
Multiple of both = (0);
6 4 10 1
P(A ∩ B) = 0}
P (5 or 7)= + = =
30 30 30 3 19. Ans(C)
11. Ans (A)
S={aaa,aab,aba,baa,abb,bab,bba,bbb}
Total number of ways = 9C4 , 2 children are chosen
Two coin shows same letter
in 4C2 ways and other 2 persons are chosen in 5C2 ={aab,aba,baa,abb,bab,bba}
ways. 6 3
Req.Prob = =
4C × 5C 8 4
2 2 10
Hence required probability = = .
9C
4 21 20. Ans(B)
12. Ans (D) P(6) = 2P(1)
8C
Required probability = 4
. P(6) = 3P(2) = 3P(3) = 3P(4) = 3P(5)
28
Since sum of prob=1
13. Ans (A)
P(1) + P(2) + P(3) + P(4) + P(5) + P(6) = 1
Total 15 balls ,Not Red means, the ball should be
3
from (5 White+4 Black balls=9 balls ) Required ∴ P(1) =
17
9C 9×8 12
2
probability = 15C = = . 21. Ans(C)
2 15×14 35

14. Ans (B) Since Sample Space = getting total of


3C × 3 C
1 1 3×3 3 2,3,4,5,6,7,8,9,10,11,12
Required probability = 6C = = .
2 15 5
and maximum occurrence is of total 7 which is
15. Ans (C)
equal to 6 times
Total number of ways in which 3 balls can be
(i. e. , {(1,6)(2,5)(3,4)(4,3)(5,2)(6,1)})
drawn are 9C3 = 84.
n(S) = 62 =36

MATHEMATICS Page | 9A. 15


PROBABILITY
6
Req. Prob= So,n(E) = 53
36
53
22. Ans(B) Hence P(E)=
900

A.P. with common difference one are =4


i.e., (123),(234),(345),(456)
A.P. with common difference two are =2
i.e., (135),(246)
A.P. with common difference zero are =6
i.e., (111), (222), (333), (444), (555), (666)
Let P be the probability of that they show the
number in A.P.
Favourable ways=2+4+6=12
n(S) = 63
12 12 1
Required probability is = = =
63 216 18

23. Ans(C)
4 13 39
c1 × c9 × c4
P(E) = 52 c
13

24. Ans(A)
10
n(S) = c2 = 45
A={(5,6)(5,7)(5,8)(5,9)(5,10)}
n(A) = 5
5 1
so, P(A)= =
45 9

25. Ans (C)


7
C1 × 3C1
R. P = 15C
2

26. Ans(C)
2c 1 5
2
Required probability 1 − 4c =1− =
2 6 6

27. Ans(B)
20 10
n(S) = c3 , n(E) = c3
28. Ans(B)
If two persons sit together consider them as single
object,
thus n(E) = no. of objects = m − 1
m
n(S) = c2
𝑚−1
hence P(E)= m
c2

29. Ans(A)
n(S) = 100, n(E) = 74
30. Ans(C)
n(S) = 900
E={102,119……986}

MATHEMATICS Page | 9A. 16


SEQUENCES AND SERIES

HINTS AND SOLUTIONS

1. Ans (B) 1 1
a5 = , a9 =
S2n = 3Sn 9 5
1 1
2n 3n a + 4d = → (1), a + 8d = → (2)
[2a + (2n − 1)d] = [2a + (n − 1)d] 9 5
2 2
4
4a + 4nd − 2 d = 6a + 3nd − 3 d (2) − (1) ⇒ d =
45
nd + d = 2a 4
from (1), a = ⇒a=d
3n 45
S3n [2a + (3n − 1)d]
= 2n 7. Ans (D)
Sn [2a + (n − 1)d]
2 t10 from beginning = 7 + 9 d
nd + A + 3nd − A
= 3[ ] t10 from end = t n−9 from beginning
nd + A + nd − d̸
4nd = 7 + (n − 10)d
= 3. = 3. (2) = 6
2nd But 55 = 7 + (n − 1)d
2. Ans (D) (n-1) d = 48
n = 1 ⇒ S1 = 5 therefore t10 + t n−9
n = 2 ⇒ S2 = 3 ⋅ 2 + 2 ⋅ 22 = 14 = 7 + 9d + 7 + (n − 10)d
T2 = S2 − S1 = 14 − 5 = 9 = 14 + (n − 1)d = 14 + 48
T1 = 5 8. Ans (A)
d = T2 − T1 = 4 5(a + 4d) = a(a + 8d)
3. Ans (D) a + 13d = 0
Given, T14 = a + 13d = 0
a1 + a1 + 4d + a1 + 9d + a1 + 14d + a1 + 19d 9. Ans (C)
+ a1 + 23d = 225 T4 + T8 = 24, T6 + T10 = 34
6a1 + 69d = 225 a + 3d + a + 7d = 24, a + 5d + a + 9d = 34
a1 + a2 + a 3 + ⋯ … + a 24 = 24a1 + 276d 2a + 10d = 24,2a + 14d = 34
= 4(6a1 + 69d) = 4 × 225 = 900 a + 5d = 12 → (1), a + 7d = 17 → (2)
4. Ans (A) 5
(2) − (1) ⇒ 2d = 5 ⇒ d =
T7 = 40 2
10. Ans (C)
a + 6d = 40
13 d = −5, a = 253
s13 = [2a + 12d] = 13[a + 6d]
2 T12 = a + 11d
= 13 × 40 = 520 = 253 − 55 = 198
5. Ans (A) 11. Ans (B)
1 1 a + 7d = 22
T9 = ,T =
7 7 9
a + 19d = 46
1
a=d= ⇒a = 8d= 2
63
63 2 62 64 t18 = 8 + 17(2) = 42
S63 = [ + ]= = 32
2 63 63 2 12. Ans (C)
6. Ans (B)
MATHEMATICS Page |10A. 1
SEQUENCES AND SERIES

n n−1
[2a + (n − 1)d] 5n + 4 a+( )d 3n + 8
2 = 2
n =
[2a′ + (n − 1)d′ ] 9n + 16 a1 + (
n − 1
) d1 7n + 15
2 2
(n − 1) n−1
a+ d 5n + 4
2 = = 11
(n − 1) ′ 9n + 16 2
a′ + d
2 n = 23
n−1
18th element, = 17 ⇒ n = 35 T12 77 7
2
1 = =
T12 176 16

5(35) + 4 175 + 4 179 18. Ans (B)


ratio = = =
9(35) + 16 315 + 16 331 a − d + a + a + d = 18
13. Ans (B) 3a = 18
2 a=6
Sn = 3n + 5n
t n = Sn − Sn−1 (a − d)2 + a2 + (a + d)2 = 158
t n = 6n + 2 2(a2 + d2 ) = 158 − 36
a m = 6m + 2 a2 + d2 = 61
164 = 6m + 2 d2 = 61 − 36 = 25
m = 27 d = 5 ,Greatest number = a + d = 6 + 5 = 11
14. Ans (B) 19. Ans (D)
a = 11 n=1
d = L.C.MofC. Doftheseries = L. C. Mof 4,5 = 20 S1 = P = T1
T12 = a + 11d n=2
= 11 + 220 = 231 S2 = 2P + Q
15. Ans(C) T2 = S2 − S1 = P + Q
a = 21 d = T2 − T1 = Q
d = L ⋅ C ⋅ M of 4,5 = 20 20. Ans (B)
Tn = a + (n − 1)d ≤ 417 A = Set of int egers divisible by 2,
= 21 + (n − 1)20 ≤ 417 SA = 2 + 4 + 6 + ⋯ … + 100
(n − 1)20 ≤ 396 = 50(51) = 2550
396 B = Set of integers divisible by 5
n−1≤
20 = {5,10,15,20 … … … .100}
n ≤ 20.9
SB = 5 + 10 + 15 + ⋯ … + 100
n = 20 20 × 21
=5× = 1050
16. Ans (B) 2
an + bn a+b C = Set of integers divisible by 2 and 5
n−1 n−1
=
a +b 2 = {10,20,30, … .100}
n=1
SC = 10 + 20 + 30 + ⋯ … + 100
17. Ans (B) 10 × 11
Sn 3n + 8 = 10 × = 550
2
1
=
Sn 7n + 15 Sum of integers divisible by 2 (or) 5
n
[2a + (n − 1)d] 3n + 8 = 2550 + 1050 − 550 = 3050
2 =
n
[2a1 + (n − 1)d1 ] 7n + 15 21. Ans (D)
2

MATHEMATICS Page |10A. 2


SEQUENCES AND SERIES

Let n be the number of terms needed. Given that ⇒ x 2 + 5x + 4 = 0


1 3069
a = 3, r = and Sn = ⇒ x = −1, −4
2 512
a(1−rn ) So, the GP is −4, −6, −9, …
Now, Sn =
1−r
(Considering x = −4, as for
1
3069 3 (1 − 2n ) 1 x = −12x + 2 = 0 )
⇒ = = 6 (1 − n )
512 1 2 Hence, fourth term is −9 × 1.5=-13.5
1−
2
3069 1 26. Ans (B)
⇒ =1− n
3072 2 Given, series 1 + 11 + 111 + ⋯ + n terms
1 3069 1 1
⇒ n =1− = = [9 + 99 + 999 + ⋯ + n terms ]
9
2 3072 1024
1
or 2n = 1024 = 210 , = [(10 − 1) + (100 − 1) + (1000 − 1) + ⋯ . +n
9

which gives n = 10. terms ]


22. Ans (A) 1
= [10 + 102 + 103 + ⋯ . +10n − n]
The geometric mean of 2 and 8 is √16 i.e., 4. 9
1 10n − 1
23. Ans (C) = [10. ( ) − n]
9 10 − 1
−3
Given, −1, x, are in GP 1 10
4
= [ (10n − 1) − n]
−3 9 9
∴ x 2 = (−1) ( ) 10 n
4 = (10n − 1) −
81 9
3 √3 27. Ans (B)
⇒x=√ ⇒x=
4 2
ar 2 + ar 3 = 60; (a. ar . ar 2 )=1000, a>0
24. Ans (B) ⇒ ar 2 (1 + r) = 60; a3 r 3 = 1000,a>0
2 2√2
Given, series is √2, , ,… ⇒ ar = 10
3 9

√2 (√2)2 (√2)3 ar 2 (1 + r) 60
or 0, , , …. ⇒ =
3 3 32 ar 10
√2
which is an infinite GP whose, first term a = = ⇒ r(r + 1) = 6
30
⇒r=2⇒a=5
√2 and common ratio
⇒ T7 = ar 6 = (5)(2)6 = 320
(√2)2 1 √2
r= × = 28. Ans (B)
3 √2 3
16 Let first term = a > 0
Let nth term of GP=
2187
Common ratio = r > 0
n−1
16
∴ ar = ar + ar 2 + ar 3 = 3
2187
n−1 ar 5 + ar 6 + ar 7 = 243
√2 16
⇒ √2 ( ) = r 4 (ar + ar 2 + ar 3 ) = 243
3 2187
n−1 7 r 4 (3) = 243
√2 8√2 √2
⇒( ) = =( ) ⇒ r = 3 as r > 0
3 2187 3
On comparing the powers, we get from (1)
1
n−1=7⇒n=8 3a + 9a + 27a = 3 ⇒ a =
13
25. Ans (D)
a(r 50 − 1) 1 50
x, 2x + 2,3x + 3 are in G.P. S50 = = (3 − 1)
(r − 1) 26
⇒ (2x + 2)2 = x(3x + 3) 29. Ans (D)
⇒ 4x 2 + 8x + 4 = 3x 2 + 3x
MATHEMATICS Page |10A. 3
SEQUENCES AND SERIES
a
Let three terms of G.P. are , a, ar we get a = 6
r

product = 27 Hence, a10 = 6(2)9 = 3072

⇒ a3 = 27 ⇒ a = 3 33. Ans (B)

3 Let the sum of n terms of the given series is 5461.


S= + 3r + 3
r Here, a = 1, r = 4 and Sn = 5461.
For r > 0 rn − 1
⇒ a( ) = 5461
3 r−1
+ 3r
r ≥ √32 (ByAM ≥ GM) 4n − 1
2 ⇒ = 5461
3
4−1
⇒ + 3r ≥ 6. ⇒ 4n − 1 = 16383 ⇒ 4n = 16384
r
3
For r < 0 + 3r ≤ −6. ⇒ 4n = 47 ⇒ n = 7
r

From (1) &(2) 34. Ans (D)


a
S ∈ (−∞ − 3] ∪ [9, ∞] S∞ = = 162
1−r
30. Ans (C) a(1 − r n )
1 1 1 1 Sn = = 160
24 ⋅ 416 ⋅ 848 ⋅ 16128 ⋅ … ∞ 1−r
160 80
1 2 3 4 Dividing 1 − r n = =
162 81
= 24 ⋅ 416 ⋅ 848 ⋅ 16128 ⋅ … ∞
80 1 1 n
1 1 1 1 ∴1− = r n or r n = or ( ) = 81
−24 ⋅ 48 ⋅ 816 ⋅ 1632 ⋅ …∞ 81 81 r
1
1 1 1 1 Now it is given that is an integer and n is also an
= 24+8+16+32+⋯∞ r
1
(
1/4
) integer. Hence the relation (1) implies that =
1−1/2 1/2 r
= (2) =2
3,9, or 81 so that hence the relation
31. Ans (D)
32 32 n = 4,2 or 1
a = 4, T3 − T5 = or a(r 2 − r 4 ) =
81 81 1 1
8
∴ a = 162 (1 − ) or 162 (1 − )
4 2 3 9
or r − r + = 0 or
81 1
or 162 (1 − )
81r 4 − 81r 2 + 8 = 0 81

or (9r 2 − 8)(9r 2 − 1) = 0 = 108 or 144 or 160

∴ r 2 = 8/9,1/9 35. Ans (D)


sum of 1st three terms 125
So, the value of r is to be +ve since all the terms Given, =
sum of 1st six terms 152

are +ve. a + ar + ar 2

For r =
1 a + ar + ar 2 + ar 3 + ar 4 + ar 5
3
125
a 4 4⋅3 =
∴ S∞ = = = =6 152
1−r 1−1 2
3 1 + r + r2 125
⇒ =
Similarly, we can find S∞ , 1 + r + r 2 + r 3 + r 4 + r 5 152
1 + r + r2 125
when r = 2√2/3 ⇒ 2 3
=
(1 + r + r )(1 + r ) 152
32. Ans (C)
1 125 152
Here, a3 = ar 2 = 24 …. (i) ⇒ = ⇒ 1 + r3 =
1 + r 3 152 125
and a 6 = ar 5 = 192. …. (ii) 152 27 3 3
⇒ r3 = −1= =( )
Dividing Eq. (ii) by Eq. (i), we get 125 125 5
192 3
r3 = ⇒ r3 = 8 ⇒ r = 2 ⇒r=
24 5
Substituting r = 2 in Eq. (i), 36. Ans (B)

MATHEMATICS Page |10A. 4


SEQUENCES AND SERIES

Let a be the first term and r be the common ratio + ⋯ + 20 terms]


7
of the given G. P. Then = [(1 + 1 + ⋯ + 20 terms )
9
a 4 = 10, a 7 = 80 1 1 1
−( + + + ⋯ + 20 terms)]
3 6 10 102 103
⇒ ar = 10 and ar = 80
1 1 20
ar 6 80 7 10
{1 − ( ) }
10
⇒ 3= ⇒ r3 = 8 ⇒ r = 2 = 20 −
ar 10 9 1
1−
Putting r = 2 in ar 3 = 10, [ 10 ]

we get: a =
10
=
5 7 1 1 20
8 4 = [20 − {1 − ( ) }]
9 9 10
Let there be n terms in the given G.P. then, a n = 20
7 179 1 1
2650 = [ + ( ) ]
9 9 9 10
n−1 7
⇒ ar = 2560
= [179 + (10)−20 ]
10 n−1 81
⇒ (2 ) = 2560 ⇒ 2n−4 = 256
8 41. Ans (D)
⇒ 2n−4 = 28 ⇒ n − 4 = 8 a n = a n−1 + a n−2
⇒ n = 12 a n+1 = a n + a n−1 , n > 1
37. Ans (D) a n+1 a n−1
=1+ ,n > 1
an an
Given geometric series is
a n+1 a2 a3 a4 a5 a6
2 + 6 + 18 + 54 + ⋯ , n = 1,2,3,4,5 = { , , , , }
an a1 a 2 a 3 a 4 a 5
6
Here, a = 2, r = = 3 3 5 8
2
= {1,2, , , }
Sum of first 8 terms 2 3 5
42. Ans (B)
a(r 8 − 1) 2(38 − 1)
= = 1 + 3 + ⋯ … + 2n − 1 = n2
r−1 3−1
= 38 − 1 = 6561 − 1 = 6560 (1 + 3 + 5 + 7 + ⋯ … + 29) + (30 + 31 + 32
38. Ans (A) + ⋯ … . +60)
Let 131072 be the nth term of the given G. P. 31
= (15)2 + [60 + 30]
2
Here a = 2 and r = 4.
= 225 + 31 × 45
∴ 131072 = a n = 2(4)n−1
= 225 + 1395 = 1620
⇒ 65536 = 4n−1
43. Ans (B)
⇒ 48 = 4n−1 ⇒ n − 1 = 8 ⇒ n = 9.
100T100 = 50T50
Hence, 131072 is the 9th term of the G.P.
2[a + 99d] = a + 49d
39. Ans (B)
a+149d=0
We have: 61/2 ⋅ 61/4 ⋅ 61/8 …
T150 =0
= 6[1/2+1/4+1/8+⋯ up to ∞]
44. Ans (A)
= 6[(1/2)/(1−1/2)] = 61 = 6
A1 , A2 , … … … An are n AM's inserted between a, b
40. Ans (C)
⇒ a, A1 , A2 , … … . An , b are in A.P
Let S = 0.7 + 0.77 + 0.777 + ⋯
⇒ A1 + A2 + ⋯ … … + An = n(A. M of a, b)
7 77 777
= + 2 + 3 + ⋯ 20 terms a+b
10 10 10
= n( )
1 11 111 2
=7 [ + + + ⋯ 20 terms ]
10 102 103
Explanation
7 9 99 999
= [ + + + ⋯ 20 terms ] a, A1 , A2 , … An , b are in A.P
9 10 100 1000
7 1 1 1
= [(1 − ) + (1 − ) + (1 − ) A1 = a + d, b = a + (n + 1)d
9 10 102 103
MATHEMATICS Page |10A. 5
SEQUENCES AND SERIES

A2 = a + 2d n[−8 + n − 1] = 52
An = a + nd n(n − 9) = 13 × 4
A1 + A2 + ⋯ + An = na + (1 + 2 + 3 + ⋯ + n)d n = 13
n(n + 1) 50. Ans (A)
= na + d
2 (a − d) + (a) + (a + d) = 27
b−a
= n [a + ] a=9
2
a+b (a − d)(a + d) = 77
= n[ ]
2 a2 − d2 = 77
45. Ans (B) 81 − 77 = d2
Sn = 2n2 + n d=2
Tn = Sn − Sn−1 51. Ans (C)
2 2
Tn = 2n + n − (2(n − 1) + (n − 1)) T1 − T2 + T3 − T4 + ⋯ … − T20 + T21
= 2n2 − 2(n2 − 2n + 1) + 1 = (T1 + T21 ) − (T2 + T20 ) + (T3 + T19 ) …
= 4n − 1 − (T10 + T12 ) + T11
T8 = 32 − 1 = 31 = T11 = a + 10d(∵ a1 + a n = a 2 + a n + ⋯ . )
46. Ans (C) 52. Ans (C)
a1 , a 2 , a 3 , … … a n a = 56
are in A.P d=7
⇒ a1 + a n = a 2 + a n−1 = a 3 + a n−3 = ⋯.. I = 497
a1 + a5 + a10 + a15 + a 20 + a 24 = 450 a + (n − 1)7 = 497
3(a1 + a 24 ) = 450 n − 1 = 71 − 8
a1 + a24 = 150 n = 64
a1 + a8 + a17 + a 24 = 2(a1 + a 24 ) n
S = [2a + (n − 1)d] = 32[56 + 497] = 17,696
2
= 2(150)
53. Ans (A)
= 300
a = 1000 , d = 40
47. Ans (B)
a+b
Sum of interior angles of a polygon = (n − 2)π
Sum of n AM's = n ( ) n
2
[2a + (n − 1)d] = (n − 2)π
2 + 38 2
160 = n ( )
2 n[100 + (n − 1)2] = (n − 2)1800
160 = n(20) n(n + 490 ) = (n − 2)900
n=8 n=5
48. Ans (C) 54. Ans (B)
′ n
α sum of mAM s of a, b
= S1 = [2a + (n − 1)d]
β sum of nAM ′ s of a, b 2
a+b n n(n + 1)
m( ) m = [2 + n − 1] =
= 2 = 2 2
a+b n n n
n( ) S2 = [4 + (n − 1)3] = [3n + 1]
2
2 2
49. Ans (B) n
Sp = [2p + (n − 1)(2p − 1)]
a = −8, d = 2 2
Sn = 52 S1 + S2 + ⋯ . +Sp
n
[2a + (n − 1)d] = 52
2
MATHEMATICS Page |10A. 6
SEQUENCES AND SERIES
n a=q+p−1
= [2(1 + 2 + ⋯ + p) + (n − 1)(1 + 3 + ⋯ .2p
2
∴ Tq = a + (q − 1)d
− 1)]
n =q+p−1−q+1
= [p(p + 1) + (n − 1)p2 ]
2 =p
np 59. Ans (C)
= [np + 1]
2
n=m=q
"Alternative Solution "
Sq = q3
n=1
60. Ans (A)
S1 + S2 + S3 + ⋯ … … . +Sp = 1 + 2 + 3 + ⋯ … + p
p (x12 − x22 ) + (x32 − x42 ) + ⋯
= (p + 1)
2 = −d[x1 + x2 + x3 + ⋯ + x2n ]
np
= (np + 1) = d[n(x1 + x2n )]
2
−(x2n − x1 )(x2n + x1 )(n)
55. Ans (B) =
2n − 1
a=2 x2n − x1 n(x12 − x2n
2 )
where d = =
T1 + T2 + T3 + T4 + T5 2n − 1 2n − 1
1 61. Ans (B)
= [T6 + T7 + T8 + T9 + T10 ]
4 Let first term is a and common difference is d.
5(S5 ) = S10
Then
2
5 × [4 + 4d] = [4 + 9d] T9 = 0

20 + 20d = 8 + 18d ⇒ a + 8d = 0

−12 = 2d ⇒ a = −8d … (i)

d = −6 ⇒ a + 8d = 0

T20 = a + 19d = 2 − 114 = −112 ⇒ a = −8d … (i)


T29 a + 28d
56. Ans (A) Now , =
T19 a + 18d
a = 100, d = 5
−8d + 28d
T30 = a + 29d = [ from eq (i)]
−8d + 18d
= 100 + 29 × 5 = 245 20d 2
= =
57. Ans (A) 10d 1

1 + 3 + 5 + ⋯ … … + 2n − 1 = n2 Hence, ratio of 29th term and 19th terms is 2: 1

1 + 3 + 5 + ⋯ … … + 2n − 1 = n2 62. Ans (C)

1 + 3 + 5 + ⋯ … + 2001 Given, (−8 + 18i), (−6 + 15i), (−4 + 12i) …

= 1 + 3 + 5 + ⋯ … + 2(1001) − 1 Now, T2 − T1 = (−6 + 15i) − (−8 + 18i)

= (1001)2 = −6 + 15i + 8 − 18i = 2 − 3i

= 1002001 and T3 − T2 = (−4 + 12i) − (−6 + 15i)

58. Ans (B) = −4 + 12i + 6 − 15i = 2 − 3i

Tp = q, Tp+q = 0 ∵ T2 − T1 = T3 − T2
So, it is an AP
a + (p − 1)d = q
Let nth of this AP is purely imaginary.
a + (p + q − 1)d = 0
Then, Tn = a + (n − 1)d
⇒ q + qd = 0
= (−8 + 18i) + (n − 1)(2 − 3i)
d = −1
= −8 + 18i + n(2 − 3i) − 2 + 3i
a−p+1=q
MATHEMATICS Page |10A. 7
SEQUENCES AND SERIES

= 10 + 21i + n(2 − 3i) 33 n


Now, S33 = [3 + 99] [∵ Sn = (a + l)]
2 2
= (2n − 10) + (21 − 3n)i 33
= × 102
Since, Tn is purely imaginary term 2
∴ Re (Tn ) = 0 = 33 × 51
⇒ 2n − 10 = 0 ⇒ n = 5 = 1683
Hence, 5th term of the given AP is purely 67. Ans (B)
1
imaginary Given A1 , A2 are two arithmetic means between
3
63. Ans (C) 1
and then
24
Given, 2x, (x + 8), (3x + 1) are in an AP, then
1 1
, A1 , A 2 , are in AP
2(x + 8) = 2x + 3x + 1[∵ a, b, and c are in AP 3 24
1 1
2 b = a + c] Here a = , l = ,n = 4
3 24

⇒ 2x + 16 = 5x + 1 Let d be the common difference


⇒ 3x = 15 ⇒ x = 5 Then, l = a + (n − 1)d
64. Ans (B) 1 1
⇒ = + (4 − 1)d
Given series is 101 + 99 + 97 + ⋯ + 47 24 3
1 1 1−8
Here a = 101, d = 99 − 101 = 97 − 99 = −2 ⇒ − = 3d ⇒ = 3d
24 3 24
Let n be the number of terms in the given series −7 7
⇒ = 3d ⇒ d = −
Then, 47 = 101 + (n − 1)(−2) 24 72
[∵ l = a + (n − 1)d] 1 7 24 − 7 17
∴ A1 = a + d = − = =
3 72 72 72
⇒ 47 − 101 = −2n + 2
1 14 24−14 10 5
and A2 = a + 2d = − = = =
⇒ −54 = −2n + 2 ⇒ 2n = 56 3 72 72 72 36

n = 28 68. Ans (B)

65. Ans (C) AM of first n natural numbers

Given, first term (a) = 10, last term (l) = 50 1 + 2+ 3 + 4+ ⋯+ n


=
n
and sum of all term = 300 n(n+1) n+1
= =
Let n be the number of terms in AP 2n 2

n 69. Ans (A)


∴ Sn = [a + l]
2 Let three numbers are (a − d), a and (a + d)
n
⇒ 300 = [10 + 50] Then, (a − d) + a + (a + d) = 33
2
n ⇒ 3a = 33
⇒ 300 = × 60
2 ⇒ a = 11 and (a − d)a(a + d) = 792
∴ n = 10 792
⇒ a(a2 − d2 ) = 792 ⇒ 112 − d2 =
66. Ans (D) 11
All natural numbers between 1 and 100, which are 792 1331 − 792
⇒ d2 = 121 − ⇒ d2 =
11 11
multiple of 3 are 3,9,12, … . ,99
539
Here a = 3, d = 3 ⇒ d2 = ⇒ d2 = 49 ⇒ d = 7
11
Let n be the number of terms in above series ∴ Smallest number = a − d = 11 − 7 = 4
∴ l = 9 + (n − 1)d 70. Ans (A)
⇒ 99 = 3 + (n − 1)3 3 + 5 + 7 + ⋯ … + n terms
Given, =7
⇒ 99 = 3n 5 + 8 + 11 + ⋯ … + to 10 terms
⇒ n = 33

MATHEMATICS Page |10A. 8


SEQUENCES AND SERIES

n Sum of 20 terms of common terms of the above


[3 × 2 + (n − 1)2]
⇒2 =7
10 series
[2 × 5 + 9 × 3]
2 n
n = [2a + (n − 1)d]
[6 + 2n − 2] 2
⇒2 =7
5[10 + 27] = 10[22 + 19 × 20] = 4020
n 73. Ans (C)
⇒ (2n + 4) = 7 × 5 × 37
2
Given 17,21,25, … … ,417 … . . (i)
⇒ n(n + 2) = 35 × 37
and 16,21,26, … 466 … (ii)
⇒ n(n + 2) = 35(35 + 2)
Let pth term, of the first sequence is equal to the
∴ n = 35
qth term of the second sequence, then
71. Ans (C)
17 + 4(p − 1) = 16 + 5(q − 1)
Given a1 , a 2 , a 3 , … a n be an AP such that
⇒ 4p + 13 = 5q + 11
a1 +a2 +⋯.+ap p3
= ;p ≠ q
a1 +a2 +⋯…+aq q3 ⇒ 4p + 2 = 5q
Let d be the common difference of above AP 4p + 2 p−2
⇒q= =P−( )
p 5 5
[2a1 + (p − 1)d] p3
⇒2q = 3 Since q is an integer, p-2 must be a multiple of 5
[2a1 + (q − 1)d] q
2 i.e.; p must be of the form 5λ + 2 with λ ⩾ 0
2a1 + (p − 1)d p2 p = 4λ + 2
⇒ =
2a1 + (q − 1)d q2
Since, the first sequence has 101 terms and
p−1
a1 + ( ) d p2 second sequence has 91 terms
⇒ 2 = 2
(q − 1) q ∴ 0 ⩽ 5λ + 2 ⩽ 101
a1 + d
2
a6 p−1 and 0 ⩽ 4λ + 2 ⩽ 91
To find ,put = 5 in the numerator of
a121 2
⇒ 0 ⩽ λ ⩽ 19
q−1
equation (i) and = 20 in the denominator of Given sequence have 20 common terms
2
p−1 q−1
equation (i), we get = 5 ⇒ p = 11 and = But it is given that, common terms are 4k
2 2

20 ⇒ q = 41 4k = 20 ⇒ k = 5

a6 (11)2 121 74. Ans(A)


now, = = a
a21 (41)2 1681 , a, ar are the edges
r
72. Ans (B) Volume of the block = 216
n
Sum of 'n' terms Sn = [2a + (n − 1)d] a3 = 216 = 63 ⇒ a = 6
2

Given series Total surface area of rectangular stick = 252


S1 = 3 + 7 + 11 + 15 + ⋯ a a
2 ( a + a ⋅ ar + ar ⋅ ) = 252
r r
common difference d1 = 4
1
S2 = 1 + 6 + 11 + 16 + ⋯ a2 (r + + 1) = 126
r
common difference d2 = 5 1 252 7
r+ +1= =
First common term in a above series r 36 2
1 5 1
a = 11 r+ = =2+ ⇒r=2
r 2 2
Common difference in a common term of the
The sides of rectangular block are 3,6,12
above series d = L.C.M of {d1 , d2 }
75. Ans (B)
= 20 a
= 2 Where a = 1
1−r

MATHEMATICS Page |10A. 9


SEQUENCES AND SERIES

1 1 80. Ans (B)


=2⇒r=
1−r 2
n]
GM of a, b is √ab
a[1 − r 2n − 1
∴ Sn = = n−1 (a + 1)(a − 2) = (a − 1)2
1−r 2
76. Ans (C) −a − 2 = −2a + 1

T2 = a + d a=3

T3 = a + 2d 81. Ans (D)

T6 = a + 5d a = 3,

(a + 2d)2 = (a + d)(a + 5d) 3 3 1


T6 = ⇒ ar 2 = ⇒r=
32 32 2
a2 + 4d2 + 4ad = a2 + 6ad + 5d2
P = a ⋅ ar ⋅ ar 2 ⋅ ar 3 ⋅ ar 4 ⋅ ar 5 = a6 × r15
2ad + d2 = 0
1 15 9 3 9 6
d = −2a = 36 ( ) = ( ) ⇒ P 2 = ( )
2 32 32
T2 = −a 82. Ans (C)
T3 = −3a an + bn
= √ab
T6 = −9a an−1 + b n−1
−3a an + bn = an−1 √a√b + bn−1 √a√b
Common ratio = =3
−a
an−1 √a(√a − √b) = bn−1 √b(√a − √b)
77. Ans (D)
1 1 1 1
ar 3 1 an−2 = b n−2 ⇒ n − =0⇒n=
= 2 2
ar 5 4
83. Ans (D)
r 2 = 4 ⇒ r = ±2
T1 = a
ar + ar 4 = 216
3
r=2 T2 = , r = r
4
18a = 216 3
ar =
216 4
a= = 12
18 a + ar + ⋯ … = 4
r = −2 a
=4
1−r
14a = 216
a2
216 108 =4
a= = a − ar
14 7
3
78. Ans (B) a2 = 4 [a − ]
4
T4 = 8 = 4a − 3
ar 3 = 8 a2 − 4a + 3 = 0
a. ar ⋅ ar 2 ⋅ ar 3 ⋅ ar 4 ⋅ ar 5 ⋅ ar 6 = a7 ⋅ r 21 a = 1,3
= (a ⋅ r 3 )7 = 87 3
Case (i) : a = 1, r =
79. Ans (A) 4
b2 =ac 1
Case (ii) : a = 3, r =
4
2logb = loga + logc
84. Ans (C)
loga + logc
logb = a
2 s∞ = , |r| < 1
1−r
nlog a , nlog b , nlog c are in A.P x
5= ⇒ x = 5 − 5r
In general, 1−r
log an , log bn , log c n are in A.P 5−x
⇒r=
5
MATHEMATICS Page |10A. 10
SEQUENCES AND SERIES

Since |r| < 1 7 1 1 1


= [(1 − ) + (1 − 2 ) + (1 − 3 )
5−x 9 10 10 10
⇒ −1 < < 1 ⇒ x ∈ (0,10) 1
5 + ⋯ … . + (1 − 20 )]
85. Ans (C) 10
1 1
Sn = 2n+1 + n − 2 7 (1 − 20 )
10 10
= [20 − ]
t n = Sn − S(n−1) 9 1
1−
10
= (2n+1 + n − 2) − (2n + n − 3) 7 1
n n = [20 − [1 − 10−20 ]]
= 2.2 + n − 2 − 2 − n + 3 9 9
= 2n + 1 7
= [179 + 10−20 ]
86. Ans (C) 81

6 90. Ans (B)


[9 + 99 + 999 + ⋯ … . terms ]
9 a, ar, ar 2 are in A.P
6 a, 2ar, ar 2 are in A.P
= [(10 − 1) + (102 − 1) + (103 − 1)
9
⇒ 4ar = a + ar 2
+ ⋯ … . +(10n − 1)]
4r = 1 + r 2
6 10(10n − 1)
= [ − n] r 2 − 4r + 1 = 0
9 9
2 4 ± √16 − 4
= [10n+1 − 10 − 9n] r= = 2 ± √3
27 2
87. Ans (B) r>1

ar 4 = 2 ∴ r = 2 + √3

a. ar ⋅ ar 2 ⋅ ar 3 ⋅ ar 4 ⋅ ar 5 ⋅ ar 6 ⋅ ar 7 ⋅ ar 8 91. Ans (D)

= (ar 4 )8 ⋅ (ar 4 ) Given ar n−1 = ar n + ar n+1

= 29 = 512 ⇒ 1 = r + r2

88. Ans (B) √5 − 1


∴r=
In an A.P T2 = a + d 2
92. Ans (C)
T5 = a + 4d
Suppose the terms in G.P. is 2
T9 = a + 8d are in A.P
Given that a + ar = 5a
(a + 4d)2 = (a + d)(a + 8d)
⇒ ar = 4a
a2 + 16d2 + 8ad = a2 + 9ad + 8d2
⇒r=4
−ad + 8d2 = 0
93. Ans (B)
ad = 8d2
x = ay = ar z = ar 2
a = 8d
a+4d 12d 4
x, 2y, 3z are in AP
Common ration (r)= = =
a+d 9d 3 4y = x + 3z
89. Ans (A) 4(ar) = a + 3ar 2
7 7 10 − 1 7 1
0.7 = = ( ) = (1 − ) 4r = 3r 2 + 1
10 9 10 9 10
3r 2 − 4r + 1 = 0
77 7 102 − 1 7 1
0.77 = 2 = ( ) = (1 − 2 ) 1
10 9 102 9 10 r=
3
777 7 103 − 1 7 1
0.777 = = ( ) = (1 − 3 ) 94. Ans (C)
103 9 103 9 10
2n − 1
S20 = 0.7 + 0.77 + 0.777 + ⋯ … ..20terms 1 + 2 + 22 + ⋯ … … + 2n−1 =
2−1

MATHEMATICS Page |10A. 11


SEQUENCES AND SERIES
1
95. Ans (C)
= 4 × [(729)2 ] = 4 × 27 = 108
1
r= 101. Ans (C)
3
1 3 3 3069
a + ar + ⋯ … = 3 + + + ⋯ … n terms =
2 4 512
2 n
a 1 3a 1 1 1
( ) −1 3069
= ⇒
1−r 2 2
= ⇒a=
2 3 3[ 2 ]=
1 512
−1
96. Ans (B) 2
a = 3, a ⋅ r n−1 = 96 1 3069
6 (1 − n
)=
2 512
r n−1 = 32 = 26−1 → (1)
1 1023
Sn = 189 1− n =
2 1024
a(r n − 1) 1 1023 1
= 189 =1− =
r−1 n
(2 ) 1024 1024
rn − 1 26 − 1 1
= 63 = → (2) = .So n=10
210
r−1 2−1
From (1) & (2) r = 2, n = 6 102. Ans (A)

97. Ans (D) T1 = 2, T2 = 4, T3 = 8, … ….

G1 , G2 , … G8 a=2

G.M's insertions between 2 & 3 T11 = a ⋅ r10 = 2.210 = 211 = 2046

⇒ 2, G, G2 , G3 , … . G8 , 3 are in G.P 103. Ans (A)


a
⇒ G1 ⋅ G2 ⋅ G3 … … G8 = (√ab)n , a, ar are three terms in G.P
r

= (√2 × 3)8 = 64 a 39
+ a + ar =
r 10
98. Ans (B) a
⋅ a ⋅ ar = 1
1 + 3 + 9 + ⋯ … terms = 364 r
3n − 1 a=1
= 364 ⇒ 3n − 1 = 728
3−1 1 39
n 6 ∴ +1+r=
3 = 729 = 3 ⇒ n = 6 r 10
99. Ans (B) 1 29 5 2 5
⇒r+ = = + ⇒r=
r 10 2 5 2
Given,
104. Ans (A)
2 3
1 − y6
4 5
1 + 2x + (2x) + (2x) + (2x) + (2x) = 9 99 999 9999
1−y + + + +⋯
19 192 193 194
Here a=1, r=2x & n=6, then sum of n-terms of G.P 10−1 102 −1 103 −1 104 −1
= + + + +⋯
1((2x)6 −1) −(y6 −1) 19 192 193 194
is =
2x−1 −(y−1) 10 10 2 10 3 1 1 2
y
=( + ( ) + ( ) + ⋯ ∞) − ( +( ) +
19 19 19 19 19
Solve and take take, 2x = y ⇒ = 2
x
1 3
100. Ans (C) ( ) + ⋯ + ∞)
19

a = 4, b = 2916 10 1 10 1
= 19 − 19 = 19 − 19
a, G1 , G2 , … G2n+1 , b are in G.P 10 1 9 18
1− 1−
b = a ⋅ r 2n+2 19 19 19 19
10 1 20 − 1 19
2916 = 4r 2n+2 ⇒ 729 = r 2n+2 = − = =
9 18 18 18
Rise power to 2n+2 on both sides,
105. Ans (B)
1
n+1
r= (729)2n+2 ⇒ Gn+1 = a ⋅ r Let α be the 1st term and d be the common
difference, then a= α + 10d,b= α + 14d
MATHEMATICS Page |10A. 12
SEQUENCES AND SERIES

& c= α + 16d ∴ r = ±2
Since a, b &c are in G.P 109. Ans (A)
(α + 14d)2 = (α + 10d)(α + 16d) Let the numbers are a, ar, ar 2
⇒ α2 + 196d2 + 28αd = α2 + 26αd + 160d2 a + ar + ar 2 = 38
⇒ 36d2 = −2αd a(1 + r + r 2 ) = 38 and a. ar ⋅ ar 2 = 1728
⇒ α = −18d ⇒ a3 r 3 = 1728
a+b ⇒ (ar)3 = 1728
∴ =6
c 1
⇒ ar = (1728)3
106. Ans (C)
S5 ⇒ ar = 12 … (ii)
⇒ = 49 (Here, S5 = sum of first 5 terms
s′5
a(1 + r + r 2 ) = 38 … (i)
and s5′ = sum of their reciprocals) From equations (i) and (ii)
5
a1 (r1 − 1) 12
(r − 1) (1 + r + r 2 ) = 38
⇒ −1 1 −5 = 49 r
a (r − 1)
(r −1 − 1) ⇒ 12(1 + r + r 2 ) = 38r

a(r 5 − 1) × (r −1 − 1) ⇒ 6(1 + r + r 2 ) = 19r


⇒ −1 −5 = 49
a (r − 1) × (r − 1) ⇒ 6r 2 + 6r + 6 − 19r = 0
a2 (1−r5 )×(1−r)×r5 ⇒ 6r 2 − 13r + 6 = 0
or (1−r5 )×(1−r)×r
= 49
⇒ 6r 2 − 9r − 4r + 6 = 0
⇒ a2 r 4 = 49 ⇒ a2 r 4 = 72
⇒ (2r − 3)(3r − 2) = 0
⇒ ar 2 = 7 → (1)
3 2
a + ar 2 = 35 → (2) r= ,
2 3
Now substituting the value of equation (1) in 3
when r = , then a = 8
2
equation (2),a + 7 = 35 ⇒ a = 28
So, the series 8,12,18
107. Ans (B) 2
when r = , then a = 18
Given a = 5, r = −5 3

Let nth term of the GP be 3125 Series 18,12,8

Then, a n = ar n−1 = 3125 110. Ans (B)

⇒ 5(−5)n−1 = 3125 Given, a1 + a 2 = 1 and a n = 2. a n−1 , a ⩾ 2

⇒ (−1)n−1 5n = (5)5 let a be the first term and r be the common ratio,
then
⇒ (−1)n−1 5n = (−1)5−1 (5)5
a + ar = 1 … (i)
∴n=5
and ar = 2a[∵ a 2 = 2a1 ]
108. Ans (B)
⇒ r = 2 …. (ii)
Given 6th term, a 6 = 32 and 8th term,
from equation (i) and (ii)
a 8 = 128
3a = 1
Let a be the first term and r be the common ratio
1
of the GP, then, a 6 = ar 5 = 32 ∴a=
3
[∵ a n = ar n−1 ]
111. Ans (C)
and a 8 = ar 7 = 128 Since, a1 , a 2 , a 3 , … … a 50 are in GP
ar 7 128
= =4 Let a1 = a, a 2 = ar, a 3 = ar 2
ar 5 32
a 4 = ar 3 , a 49 = ar 48 , a 50 = ar 49
⇒ r2 = 4
Now, using the above terms
MATHEMATICS Page |10A. 13
SEQUENCES AND SERIES

a1 − a 3 + a 5 … a 49 a 7 ± 3√5
a 2 − a4 + a 6 + ⋯ a 50 =
b 2
a − ar 2 + ar 4 … ar 48 a
Since, > 0
= b
ar − ar 3 + ar 5 + ⋯ + ar 49
a 7 + 3√5
a(1 − r 2 + r 4 … … r 48 ) 1 =
= = b 2
ar(1 − r 2 + r 4 … r 48 ) r
114. Ans (B)
a1 a 1
Also, = = If a, b, c are in G.P. Then b2 = ac
a 2 ar r
a1 − a3 + a 5 + ⋯ + a 49 a1 ⇒ b2 (a − c) = ac(a − c)
=
a 2 − a4 + a 6 + ⋯ + a 50 a 2 ⇒ b2 a − b2 c = a2 c − ac 2
112. Ans (B) ⇒ a(b2 + c 2 ) = c(a2 + b2 ).
Let five GM's be G1 , G2 , G3 , G4 , G5 between 486 and Trick: Put a = 1, b = 2, c = 4 and check the
2
3
alternates.
2
So, 486, G1 , G2 , G3 , G4 , G5 , are in GP 115. Ans (D)
3
Given sequence is √2, √10, √50 … ….. Common
Here, a = 486, n = 7
th
2 ratio r = √5, first term a = √2, then 7 term t 7 =
and l =
3
√2(√5)7−1
Let r be the common ratio of the given GP
2 = √2(√5)6 = √2(5)3 = 125√2.
∴ = ar n−1 [∵ l = ar n−1 ]
3 116. Ans (C)
2 Let first term of G.P. = A and common ratio = r
⇒ = 486. r 6
3
We know that nth term of G.P.
2 1
⇒ r6 = = = Ar n−1
3 × 486 729
1 1/6 1 1/6 1 Now t 4 = a = Ar 3 , t 7 = b = Ar 6 and t10 = c = Ar 9
⇒r=( ) = ( 6) =
729 3 3 Relation b2 = ac is true because b2 = (Ar 6 )2 =
∴ G4 = ar 4 A2 r12 and ac = (Ar 3 )(Ar 9 ) = A2 r12
1 4 486 Aliter : As we know, if p, q, r in A.P., then
= 486 × ( ) = =6
3 81 pth , qth , r th terms of a G.P. are always in G.P.,
113. Ans (B)
therefore, a, b, c will be in G.P. i.e. b2 = ac.
AM 3
= ⇒ a + b = 3√ab 117. Ans (B)
GM 2
Given that first term a = 5 and common ratio r =
a b
√ +√ =3 −5 . Suppose that nth term is 3125,
b a
then ar n−1 = 3125
a
put, √ = t ⇒ 5(−5)n−1 = 3125 ⇒ (−5)n−1 = 54
b
1 Hence n = 5.
t+ =3
t 118. Ans (B)
2
t − 3t + 1 = 0 Suppose that the added number be x then x +
−b ± √b 2 − 4ac 2, x + 14, x + 62 be in G.P.
t=
2a Therefore (x + 14)2 = (x + 2)(x + 62)
3 ± √9 − 4 ⇒ x 2 + 196 + 28x = x 2 + 64x + 124
=
2
⇒ 36x = 72 ⇒ x = 2.
a 3 ± √5 a 14 ± 6√5
√ = ⇒ =
b 2 b 4

MATHEMATICS Page |10A. 14


SEQUENCES AND SERIES

Trick: (a) Let 1 is added, then the numbers will be 124. Ans (A)
3,15,63 which are obviously not in G.P. a = ARp−1 , b = ARq−1 , c = ARr−1
(b) Let 2 is added, then the numbers will be 4,16, c p b r a q
∴( ) ( ) ( )
64 which are obviously in G.P. b a c
p r q
ARr−1 ARq−1 ARp−1
119. Ans (A) =( ) ( ) ( )
ARq−1 ARp−1 ARr−1
Under condition Tn = Tn+1 + Tn+2 = R(r−q)p+(q−p)r+(p−r)q = R0 = 1
n−1 n n+1
⇒ ar = ar + ar 125. Ans (A)
n−1 n 2
⇒r = r (1 + r) ⇒ r + r − 1 = 0 Accordingly, ar 9 = 9 and ar 3 = 4
−1 ± √1 + 4 −1 ± √5 ⇒ r 3 = and a = .
3 8
⇒r= = 2 3
2 2
8 3 2
Since, each term is +ve. ∴ 7th term i.e. ar 6 = ( ) = 6.
3 2
√5−1
Hence common ratio is . Trick: 7th term is equidistant from 10th and 4th so
2

120. Ans (D) it will be √9 × 4 = 6.


Given that x, 2x + 2,3x + 3 are in G.P. 126. Ans (B)
Therefore, (2x + 2) = x(3x + 3) 2 T6 = 32 and T8 = 128
⇒ x 2 + 5x + 4 = 0 ⇒ ar 5 = 32 and ar 7 = 128
⇒ (x + 4)(x + 1) = 0 ⇒ x = −1, −4 Dividing (ii) by (i), r 2 = 4 ⇒ r = 2.
Now first term a = x 127. Ans (A)

Second term ar = 2(x + 1) ⇒ r =


2(x+1) 5 −1 n−1
x Tn = ar n−1 ⇒ = 5⋅( )
1024 2
2(x+1) 3
then 4 term = ar 3 = x [
th

x
] −1 10 −1 n−1
⇒( ) =( ) ⇒ 10 = n − 1
8 2 2
= (x + 1)3
x2 ⇒ n = 11.
Putting x = −4 128. Ans (C)
8 27
We get T4 = 3
(−3) = − = −13.5. a2 = ar 2 ⇒ a = r 2 . Also ar = 8 ⇒ r = 2 and a = 4.
16 2

121. Ans (A) ∴ T6 = ar 5 = 4 × 25 = 128.

S3 125 a(r3 −1)/(r−1) 125 129. Ans (B)


Here = ⇒ =
S6 152 a(r6 −1)/(r−1) 152
Let the 9 terms of a G.P. is
⇒ (r 3 − 1)152 = 125(r 6 − 1) a a a a
, , , , a, ar, ar 2 , ar 3 , ar 4 .
27 3 r4 r3 r2 r
⇒ r3 = ⇒r= .
125 5 Given, fifth term a = 2
122. Ans (C) Hence product of 9 terms is a9
Given that ar 2 = 4 = (2)9 = 512.
Then product of first 5 terms 130. Ans (D)
2 )(ar 3 )(ar 4 )
= a(ar)(ar Infinite series is a, ar, ar 2 ∞
5 10 [ar 2 ]5 5
=a r = =4 . Sum S =
a
=9
1−r
123. Ans (B)
⇒ a = 9(1 − r) and a + ar = 5
1 16
T5 = ar 4 = and T9 = ar 8 =
3 243 ⇒ a(1 + r) = 5
2
Solving (i) and (ii), we get r = ⇒ 9(1 − r)(1 + r) = 5 [By equation (i)]
3
5 2
and a =
27
. 1 − r2 = ⇒ r = ± .
16 9 3

33 23 1 131. Ans (A)


Now 4th term = ar 3 = ⋅ = .
24 33 2
MATHEMATICS Page |10A. 15
SEQUENCES AND SERIES
1 3
Given series is 3 + 4 + 6 + ⋯ … .. Putting n = 8 in equation (iv),
2 4
9 27 we have a ⋅ 27 = 128 = 27 or a = 1.
=3+ + + ⋯.
2 4
135. Ans (B)
32 33 34 35
=3+ + + + + ⋯.. (in G.P.) Under given conditions, we get
2 4 8 16
3
Here a = 3, r = , then sum of the five terms a(r 6 − 1) a(r 3 − 1)
2 ⇒ =9⋅ (∵ r > 1)
(r − 1) (r − 1)
3 5
3 [( ) − 1] ⇒ r 6 − 1 = 9r 3 − 9
a(r n − 1) 2
S5 = =
r−1 3 ⇒ (r 3 )2 − 9(r 3 ) + 8 = 0
−1
2
5
⇒ (r 3 − 1)(r 3 − 8) = 0
3
1 [ − 1] 243 − 32 211 × 3 ⇒ r = 1, ω, ω2 and r = 2. But r = 1, ω, ω2 cannot
32
= = 6[ ]=
1 32 16 satisfy the given condition.
2
633 9 Hence r = 2.
= = 39 .
16 16 136. Ans (C)
132. Ans (A) S = 1 + 10 + 102 + ⋯ . . . +1090
9
Series is a G.P. with a = 0.9 = and (91 terms)
10

1 1. (1091 − 1)
r= = 0.1 =
10 10 − 1
1 (1013 )7 − 1 1013 − 1
1 − r100 9 1 − 10100 = ×
∴ S100 = a( )= ( ) 1013 − 1 10 − 1
1−r 10 1 − 1
10 = [(1013 )6 + (1013 )5 + (1013 )4 (1012 +

=1−
1 1011 +……+1)
10100
It is the product of two integers and hence not
133. Ans (D)
prime.
Let first term and common ratio of G.P. are
137. Ans (B)
respectively a and r, then under condition,
1
Tn = Tn−1 + Tn−2 The sequence is a G.P. with common ratio .
3

⇒ ar n−1
= ar n−2
+ ar n−3 a(1−rn ) 2[1−(1/3)20 ] 1
Now from , = 3 [1 − ].
1−r 1−(1/3) 320
n−1 n−1 −1 n−1 −2
⇒ ar = ar r + ar r 138. Ans (B)
1 1
⇒ 1 = + 2 ⇒ r2 − r − 1 = 0 a1 = 3, a n = 96 ⇒ a1 r n−1 = 96
r r
1 ± √1 + 4 1 + √5 ⇒ r n−1 = 32 ⇒ r n = 32r
⇒r= =
2 2 a1 (r n − 1)
Taking only (+) sign (∵ r > 1). Sn = = 189
r−1
134. Ans (A) 3(32r − 1)
⇒ = 189
a(rn −1) r−1
Given that = 255 (∵ r > 1)
r−1
Hence r = 2 and n = 6.
ar n−1 = 128
139. Ans (A)
and common ratio r = 2
Let three number of G.P. are a, ar, ar 2
n−1
we get a(2) = 128
a + ar + ar 2 = 38
a(2n −1)
and = 255 a(1 + r + r 2 ) = 38 … (i)
2−1

divide both the equations a3 r 3 = 1728…(ii)


2n −1 255 255 12
we get = ⇒ 2 − 2−n+1 = From equation (ii) ar = 12 ⇒ a =
2n−1 128 128 r
−n −8 12
⇒2 =2 ⇒n=8 From equation (i) (1 + r + r 2 ) = 38
r

MATHEMATICS Page |10A. 16


SEQUENCES AND SERIES
12×3 45
on solving r = 2/3 and a =
2
= 18 3 + 3α + 3α2 + 3α3 + ⋯ =
8

Required number 18, 12, 8 1 45


⇒ 3[ ]= ⇒ 8 = 15(1 − α)
Maximum number i.e., 18. 1−α 8
7
140. Ans (D) ⇒α= .
15

a = 7 and ar n−1 = 448 147. Ans (C)


a(rn −1) a
Now, Sn = = 889 We have ar = 2 and S∞ = 8 =
r−1 1−r

(ar n−1 ⋅ r − a) 2 2
⇒ = 889 ⇒8= (∵ a = )
r−1 r(1 − r) r


448r−7
= 889 ⇒ r = 2. ⇒ 4r(1 − r) = 1 ⇒ 4r − 4r 2 − 1 = 0
r−1
⇒ 4r 2 − 4r + 1 = 0
141. Ans (A)
1 1
arn −a arn−1 ⋅r−a ⇒ (r − ) (4r − 2) = 0 ⇒ r =
= 364 ⇒ = 364 2 2
r−1 r−1
3 × 243 − a So, first term a = 4.
⇒ = 364 ⇒ a = 1
2 148. Ans (D)
Now, putting this in (i), n = 6. x x
y= ⇒y=
1−(−x) 1+x
142. Ans (A)
y
⇒ y + yx = x ⇒ x = .
Let α and β be the roots of equation 1−y

x 2 − 18x + 9 = 0 149. Ans (B)

∴ G.M. of α and β = √αβ = √9 = 3. Let the first series be a + ar + ar 2 + ⋯ ∞ = 3


then the second series is a2 + a2 r 2 + a2 r 4 +
143. Ans (D)
1 ⋯∞ = 3
We have 4, g1 , g 2 , g 3 , is a G.P.
4 a
= 3 or a = 3(1 − r)
1−r
Here a = 4,
a2
g1 = ar = 4 × r, g 2 = ar 2 , g 3 = ar 3 and g 4 = and = 3 or a2 = 3(1 − r 2 )
1−r2
1
ar 4 = 4 × r 4 = Eliminating a, {3(1 − r)}2 = 3(1 − r 2 )
4
⇒ 3(1 − r) = (1 + r), {∵: r ≠ 1}
1 1 4 1
⇒ r4 = =( ) ⇒r= 1
16 2 2 ⇒ 4r = 2 or r = .
2
Now product of three G.M. g1 ⋅ g 2 ⋅ g 3
150. Ans (B)
= ar. ar 2 ⋅ ar 3 Let r be the common ratio of the G.P. Then
1 6 43 a a
= a 3 r 6 = 43 × ( ) = = 1. S= ⇒r=1−
2 43
1−r S
Note: The product of ' n ' geometric means
Now Sn = Sum of n terms
1
between ' a ' and is always equal to 1. 1 − rn a
a
= a( )= (1 − r n )
144. Ans (B) 1−r 1−r
a n
Let 1, a, b, 64 ⇒ a2 = b and b2 = 64a = S [1 − (1 − ) ].
S
⇒ a = 4 and b = 16 151. Ans (A)
145. Ans (A) 41/3 ⋅ 41/9 ⋅ 41/27 … . . . ∞
∵ a, b, c are in G.P. ∴ S = 41/3+1/9+1/27 … . . . ∞
2 2
b c b c 1 1
∴ = = r ⇒ 2 = 2 = r2 ( 31 ) 3
a b a b 1−
2 1
⇒S= 4 3 = 43 ⇒ S = 42
⇒ a2 , b2 , c 2 are in G.P.
⇒ S = 2.
146. Ans (B)
152. Ans (A)
MATHEMATICS Page |10A. 17
SEQUENCES AND SERIES

√2+1
,
1 1
, , … …. S1 = 3(1) + 2(1)2 = 5
√2−1 √2(√2−1) 2
S2 = 3(2) + 2(4) = 14
1
Common ratio of the series =
√2(√2+1) S1 = a1 = 5
a
∴ sum = S2 − S1 = a 2 = 14 − 5 = 9
1−r
∴ Common difference d = a 2 − a1
√2 + 1 1
=( ) / (1 − ) =9−5=4
√2 − 1 √2(√2 + 1)
(√2+1) √2(√2+1)
159. Ans(C)
= ⋅ = √2(√2 + 1)2 .
(√2−1) (1+√2) Given that T3 = 4
153. Ans (A) ⇒ ar 3−1 = 4
2
Common ratio r =
x
⇒ ar 2 = 4

For sum to be finite r < 1 ⇒ < 1


2 [∵ Tn = ar n−1 ]
x
Product of first 5 terms
⇒ 2 < x ⇒ x > 2.
= a ⋅ ar ⋅ ar 2 ⋅ ar 3 ⋅ ar 4
154. Ans (C)
= a5 r10 = (ar 2 )5 = (4)5
Given series 0.5737373 …..
160. Ans(A)
= 0.5 + 0.073 + 0.00073
73 73 Tn = a + (n − 1)d
= 0.5 + + + ⋯.
1000 100000
T9 = a + 8d
1 1
= 0.5 + 73 [ + + ⋯..] T13 = a + 12d
1000 100000
1 ∴ T9 = a + 8d
1000 73 100
= 0.5 + 73 [ ] = 0.5 + ⋅ As per the given condition
1 1000 99
1− 9[a + 8d] = 13[a + 12d]
100
5 73 495+73 568
= + = = . 9a + 72d = 13a + 156d
10 990 990 990

155. Ans (A) ⇒ −4a = 84d

3 + x, 9 + x, 21 + x are in G.P. A= −21d ⇒ T22 = a + 21d

⇒ (9 + x)2 = (3 + x)(21 + x) = −21d + 21d = 0

⇒ 81 + x 2 + 18x = x 2 + 24x + 63 161. Ans(B)

⇒ 6x = 18 or x = 3. Since x, 2y, 3z are in A.P.

Trick: Check for (A), 3 + 3,9 + 3,21 + 3 are in G.P. 2y − x = 3z − 2y

156. Ans (C) 4y= x + 3z


1 Now x, y, z are in G.P.
Infinite series 9 − 3 + 1 − … . . ∞ is a
3 y z
−1
Common ratio, r = =
x y
G.P. with a = 9, r =
3
y 2 = xz
a 9 9×3 27
∴ S∞ = = 1 = = .
1−r 1+( )
3
4 4 putting the value of x from eq. (i), we get
157. Ans (C) y 2 = (4y − 3z)z
1 1 ⇒ y 2 = 4yz − 3z 2
(32)(32)6 (32)36 … ∞
1 1 ⇒ 3z 2 − 4yz + y 2 = 0
1 5 6
)
= (32)1−(6 = (32) 6 = (32)5 ⇒ 3z 2 − 3yz − yz + y 2 = 0
= 26 = 64. ⇒ 3z(z − y) − y(z − y) = 0
158. Ans(D) ⇒ (3z − y)(z − y) = 0
2 ⇒ 3z − y = 0 and z − y = 0
Given that Sn = 3n + 2n

MATHEMATICS Page |10A. 18


SEQUENCES AND SERIES

⇒ 3z = y and z ≠ y 3n n
= [2a + (3n − 1)d]: [2a + (n − 1)d]
z 1 1 2 2
⇒ = ⇒ r= 3n
y 3 3 [2a + (3n − 1)d]
= 2n
[∵ z and y are distinct numbers ] [2a + (n − 1)d]
2
162. Ans(C)
3[2a + (3n − 1)d]
The given series is A.P. whose first term is a and =
2a + (n − 1)d
common difference is d 3[(n + 1)d + (3n − 1)d]
=
n (n + 1)d + (n − 1)d
∴ Sn = [2a + (n − 1)d] = qn2
2 3d[n + 1 + 3n − 1] 3[4n]
= 2a + (n − 1)d = 2qn = = =6
d(n + 1 + n − 1) 2n
m
Sm = [2a + (m − 1)d] = qm2 164. Ans(B)
2
We know that AM ≥ GM
⇒ 2a + (m − 1)d = 2qm
4x + 41−x
Solving eq. ( i ) and eq. (ii) we get d=2q ∴ ≥ √4x ⋅ 41−x
2
Putting the value of d in eq. (ii) we get
⇒ 4x + 41−x ≥ 2√4x+1−x
2a + (m − 1) ⋅ 2q = 2qm
⇒ 4x + 41−x ≥ 2 ⋅ 2 ⇒ 4x + 41−x ≥ 4
⇒ 2a = 2qm − (m − 1)2q
165. Ans(A)
⇒ 2a = 2q(m − m + 1)
Sr S1 S2 S3 Sn
Given that ∑ni=1 = + + + ⋯+
⇒ 2a = 2q ⇒ a = q sr s1 s2 s3 sn

q Let Tn be the nth term of the above series


∴ Sq = [2a + (q − 1)d]
2
n(n + 1) 2
q Sn [ ]
= [2q + (q − 1)2q] 2
2 ∴ Tn = =
Sn n(n + 1)
q q 2
= [2q + 2q2 − 2q] = × 2q2 = q3
2 2 n(n + 1) n2 + n
163. Ans(B) = =
2 2
n Now sum of the given series
Sn = [2a + (n − 1)d]
2
1
2n ∑ Tn = ∑ [n2 + n]
S2n = [2a + (2n − 1)d] 2
2
1
3n = [∑ n2 + ∑ n]
S3n = [2a + (3n − 1)d] 2
2
1 n(n + 1)(2n + 1) n(n + 1)
As per the condition of the question, we have = [ + ]
2 6 2
S2n = 3 ⋅ Sn 1 n(n + 1) 2n + 1
= ⋅ [ + 1]
2n 2 2 3
⇒ [2a + (2n − 1)d]
2 n(n + 1) 2n + 1 + 3
n = [ ]
4 3
= 3 ⋅ [2a + (n − 1)d]
2 n(n + 1) (2n + 4)
⇒ 2[2a + (2n − 1)d] = 3[2a + (n − 1)d] = ⋅
4 3
⇒ 4a + (4n − 2)d = 6a + (3n − 3)d n(n + 1)(n + 2)
=
⇒ 6a + (3n − 3)d − 4a − (4n − 2)d = 0 6
166. Ans(D)
⇒ 2a + (3n − 3 − 4n + 2)d = 0
Using method of difference, we get
⇒ 2a + (−n − 1)d = 0
Sn = 2 + 3 + 6 + 11 + 18 + ⋯ + t 50
⇒ 2a − (n + 1)d = 0 ⇒ 2a = (n + 1)d
Sn = 0 + 2 + 3 + 6 + 11 + ⋯ + t 49 + t 50
Now S3n : Sn
0= 2 + 1 + 3 + 5 + 7 + ⋯ − t 50 terms

MATHEMATICS Page |10A. 19


SEQUENCES AND SERIES

t 50 = 2 + (1 + 3 + 5 + 7 + ⋯ upto 49 terms ) a = 4 − 4r
49 3
t 50 = 2 + [2 × 1 + (49 − 1)2] = 4r − 4r 2
2 4
49 16r 2 − 16r + 3 = 0
=2+ [2 + 96]
2 1 3
49 r = ( or )r =
=2+ × 98 = 2 + 49 × 49 = 492 + 2 4 4
2 1
167. Ans(A) r= ⇒a=3
4
Let the length, breadth and height of a rectangular 169. Ans(A)
a
block be , a and ar. ar = 24, ar 4 = 3
r

V=l×b×h ar 4 3 1
r3 = = =
a ar 24 8
= × a × ar 1
r r = ⇒ a = 48
2
a3 = 216
⇒ a + ar + ar 2 + ⋯ … … + ar 5
⇒a=6
1
Now total surface area = 2[lb + bh + lh] a(1 − r 6 ) 48 [1 − 64]
= =
a a 1−r 1
252 = 2 [ ⋅ a + a ⋅ ar + ⋅ ar] 1−
2
r r
63 3 × 63 189
a2 = 48 × ×2 = =
⇒ 252 = 2 [ + a2 r + a2 ] 64 2 2
r
170. Ans(D)
1 2
⇒ 252 = 2a [ + r + 1] a=a
r
1 + r2 + r b=a+d
⇒ 252 = 2 × (6)2 [ ]
r c = a + 2 d & x=x
2
1+r +r y = xr, z = xr 2
⇒ 252 = 72 [ ]
r ∴ x b−c ⋅ y c−a ⋅ z a−b
2
252 1 + r + r = r c−a ⋅ r 2a−2b = r c+a−2b = r 0 = 1
⇒ =
72 r
171. Ans (B)
7 1 + r + r2
= ar 2 = 9
2 r
⇒ 2 + 2r + 2r 2 = 7r a ⋅ ar ⋅ ar 2 ⋅ ar 3 ⋅ ar 4 = a5 ⋅ r10

2r 2 − 5r + 2 = 0 = (ar 2 )5 = 95 = 310

2r(r − 2) − 1(r − 2) = 0 172. Ans(B)


1 We have, Sn = n2 + n
r = 2,
2 ⇒ Sn−1 = (n − 1)2 + (n − 1)
Therefore, the three edge are: = (n − 1)[n − 1 + 1]
1
If r = 2 then edges are 3,6,12 If r = then edges = n(n − 1) = n2 − n
2

are 12,6,3 ∴ Tn = Sn − Sn−1 = n2 + n − (n2 − n)


So, the length of the longest edge = 12 = 2n
168. Ans(D) ⇒ Tn−1 = 2(n − 1) = 2n − 2
Let a, r are first term and common ratio. ∴ common difference= Tn − Tn−1
ar = 3/4 = 2n − 2n + 2 = 2
a + ar + ar 3 + ⋯ … = 4 173. Ans (A)
a midterm is T26 = 300
=4
1−r
MATHEMATICS Page |10A. 20
SEQUENCES AND SERIES

T1 = 300 − 25 d; T51 = 300 + 25 d n(n + 1) 1 n2 (n + 1)2


⇒ = ×
51 2 78 4
S= [300 − 25 d + 300 + 25 d]
2 ⇒ n2 + n − 156 = 0
51 ⇒ (n + 13)(n − 12) = 0
[600] = 15,300
2
⇒ n = 12 [∵ n ≠ −13]
174. Ans (C)
3. Ans (D)
a1 , a 2 , … … , a10 are in G.P.,
Given that, sum of geometric series = 4 second
Let the common ratio be r 3
term =
a3 a1 r 2 4
= 25 ⇒ = 25 a 3 3
a1 a1 ⇒ = 4, ar = ⇒ r =
1−r 4 4a
⇒ r 2 = 25 a 4a2
∴ =4 ⇒ =4
a 9 a1 r 8 3 4a − 3
1−
= = r 4 = 54 4a
a 5 a1 r 4
⇒ (a − 1)(a − 3) = 0 ⇒ a = 1 or 3
175. Ans (A) 3
When a = 1, r =
For p = 1, q = 2, r = 3 we get A.P. 4
1
176. Ans (B) When a = 3, r =
4

177. Ans (D) 4. Ans (C)


Use TSR : If a and b are two positive numbers Required sum
1
with arithmetic mean A and geometric = [(1 − x) + (1 − x 2 ) + (1 − x 3 )+. .. up to n
1−x
mean G. Then these numbers are terms]
𝑎 = A + √(A + G)(A − G) and 1
= [n − (x + x 2 + x 3 + ⋯ upto n terms)]
1−x
b = A − √(A + G)(A − G)
1 (1 − x n )
178. Ans (C) = [n − x ]
1−x 1−x
n(1 − x) − x(1 − x n )
=
(1 − x)2
5. Ans (C)
MOCK TEST SOLUTION Since, (2x + 2)2 = x × (3x + 3) [∵ b2 = ac]
1. Ans (C)
−5 ± √25 − 16
Let a be the first term and r be the common ration. ⇒ x 2 + 5x + 4 = 0 ⇒ x =
2
We have, −5 ± 3
⇒x=
a + ar = 1 ….(i) 2
8 2
and, ⇒ x = − = −4 and x = − = −1
2 2
n−1 n n+1 At x = −1, second term become zero, so we
ar = 2(ar + ar + ⋯)
n
ar neglect that.
⇒ ar n−1 = 2
1−r 3
At x = −4, a = −4, r =
n−1 n n 2
⇒r − r = 2r
3
3 27
⇒ r n−1 = 3 r n ∴ T4 = −4 × ( ) = − = −13.5
2 2
⇒ 3 r = 1 ⇒ r = 1/3
6. Ans (D)
Putting r = 1/3 in (i), we get a = 3/4
Since, a, b, c are in AP.
2. Ans (B)
∴ b = a + d, c = a + 2d,
1
Since, Σn = Σn3 Where d is a common difference, d > 0
78

MATHEMATICS Page |10A. 21


SEQUENCES AND SERIES

Again, since a2 , b2 , c 2 are in GP. Then, it is given that


∴ a2 , (a + d)2 , (a + 2d)2 are in GP (a − s)2 , a2 , (a + d)2 are in GP.
⇒ (a + d)4 = a2 (a + 2d)2 ⇒ a4 = (a − d)2 (a + d)2
or (a + d)2 = ±a(a + 2d) ⇒ a4 − 2 a2 d2 = 0 ⇒ d = 0, ±√2 a
⇒ a2 + d2 + 2ad = ±(a2 + 2ad) Hence, d has three values
Taking (+) sign, d = 0 10. Ans (B)
(not possible as a < b < c) Let there be n sides of the polygon. Then, the sum
Taking (−)sign of all interior angles is (2n − 4) right angles
2
2a + 4ad + d = 0 2 n
∴ {240 + (n − 1)5} = (2n − 4 × 90)
1 1 2 2
⇒ 2a2 + 4a ( − a) + ( − a) = 0
2 2 ⇒ n2 − 25 n + 144 = 0 ⇒ n − 9, 16
3 1 But, for n = 16, the largest angle = 120 +
(∴ a + b + c = ⇒a+d= )
2 2
(16 − 1)5 = 195°; which is impossible
⇒ 4a2 − 4a − 1 = 0
Hence, n = 9
1 1
∴ a= ± 11. Ans (C)
2 √2
1 Let the two numbers be a and b
Here, d = − a > 0
2 a+b
1 ∴ AM = =A …(i)
2
So, a <
2
1 1
and GM = √ab ⇒ G2 = ab …(ii)
Hence, a = − 2 2
2 √2 Now, (a − b) = (a + b) − 4ab
7. Ans (A) = (2A)2 − 4G2
Since, a1 , a 2 , a 3 , … , a n form an AP. = 4(A2 − G2 )
∴ a 2 − a1 = a 4 − a 3 =. . . = a 2n − a 2n−1 = d
⇒ a − b = ±2√(A + G)(A − G) ...(iii)
Let S = a21 − a22 + a23 − a24 +. . . +a22n−1 − a22n
On solving Eqs. (i) and (iii), we get
= (a1 − a 2 )(a1 + a 2 ) + (a 3 − a 4 )(a 3 + a 4 )
a = A ± √(A + G)(A − G)
+. . . +(a 2n−1 − a 2n )(a 2n−1 + a 2n )
and b = A ± √(A + G)(A − G)
= −d(a1 + a 2 + ⋯ + a 2n )
2n
12. Ans (B)
= −d ( (a1 + a 2n )) …(i)
2 Let a1 , a 2 , a 3 and d1 , d2 , d3 are the first term and
Also, we know a 2n = a1 + (2n − 1)d common difference of the three AP’s respectively.
a 2n − a a1 − a 2n
⇒ d= ⇒ −d = We have, a1 = a 2 = a 3 = 1 and d1 = 1,
2n − 1 2n − 1
d2 = 2, d3 = 3
On putting the value of d in Eq, (i), we get
Therefore,
n(a1 − a 2n )(a1 + a 2n )
S= n
2n − 1 S1 = (n + 1) …(i)
2
n n
= (a2 − a22n ) S2 = (2n) …(ii)
2n − 1 1 2
n
8. Ans (D) S3 = (3n − 1) …(iii)
2
a+2
= √2a + 1 On adding Eqs. (i) and (iii), we get
2
n
a a2 S1 + S3 = [(n + 1) + (3n − 1)]
⇒ = √2a ⇒ = 2a ⇒ a = 8 2
2 4 n
= 2 [ (2n)] = 2S2
9. Ans (C) 2
Let the numbers be a − d, a, a + d. Hence, correct relation is S1 + S3 = 2 S2

MATHEMATICS Page |10A. 22


SEQUENCES AND SERIES

13. Ans (C) ⇒ 2 n + 1 = 15 ⇒ n = 7


Since, 2T11 = 7T21 19. Ans (C)
p
⇒ 2(a + 10d) = 7(a + 20d) 2
[2a1 +(p−1)d] p2
Given, q =
[2a1 +(q−1)d] q2
⇒ 2a + 20d = 7a + 140d 2

(2a1 − d) + pd p
⇒ a = −24d ⇒ =
(2a1 − d) + qd q
∴ T25 = −24d + 24d = 0
⇒ (2a1 − d)(p − q) = 0
14. Ans (C)
d
Let Sn denote the sum of n terms. Then, ⇒ a1 =
2
Sn = 3n2 + 5 Now,
Now, a6 a1 + 5d
=
a n = Sn − Sn−1 a 21 a 2 + 20d
⇒ a n = (3 n2 + 5) − (3(n − 1)2 + 5) = 6n − 3 d
+ 5d 11
= 2 =
∴ a n = 159 ⇒ 6n − 3 = 159 d
+ 20d 41
2
⇒ 6n = 162 ⇒ n = 27
20. Ans (A)
15. Ans (C)
Let α and β are the roots of the equation
Let Sn denote the sum of n terms of the given
x 2 − 2ax + a2 = 0
series.
∴ α + β = 2a and αβ = a2 …(i)
Then,
α+β
Since, A = and G = √αβ
Sn = 5n2 + 2n 2

Clearly, ⇒ A = a and G2 = a2 [from Eq. (i)]

Second term = S2 − S1 ⇒ G2 = A2 ⇒ G = A

⇒ Second term 21. Ans (C)

= (5 × 22 + 2 × 2) − (5 × 12 + 2 × 1) Given, 1 + sin x + sin2 x +. . . ∞ = 4 + 2√3


= 24 − 7 = 17 1
⇒ = 4 + 2√3
1 − sin x
16. Ans (A)
1 4 − 2√3
Given sequence is ⇒ 1 − sin x = ×
4 + 2√3 4 − 2√3
(−8 + 18i), (−6 + 15i), (−4 + 12i),
4 − 2√3
(−2 + 9i), (0 + 6i), … ⇒ 1 − sin x =
4
Hence ,5th term is purely imaginary. 2√3 √3
⇒ sin x = =
17. Ans (D) 4 2
Since, Tp = q = a + (p − 1)d …(i) π 2π
⇒ x= ,
3 3
and Tq = p = a + (q − 1)d …(ii)
22. Ans (D)
On solving Eqs. (i) and (ii), we get
We have,
d = −1 and a = p + q − 1
1 + cos α + cos 2 α+. . . to ∞ = 2 − √2
∴ T10 = a + (10 − 1)d = p + q − 10
1
18. Ans (C) ⇒ = 2 − √2
1 − cos α
We have, 1 1
⇒ 1 − cos α = =1+
1 2 − √2 √2
∑ n = ( ) ∑ n2
5 1 3π
⇒ cos α = − ⇒ α=
(n + 1) 1 n(n + 1)(2 n + 1) √2 4
⇒n = { }
2 5 6 23. Ans (C)
MATHEMATICS Page |10A. 23
SEQUENCES AND SERIES

Let the three numbers in AP are a − d, a, a + d a 1


S∞ = 1−r , Where a = 4 and r=− 2
Since, a − d + a + a + d = 15 ⇒ a = 5 4 4 8 2
S∞ = 1 = =3=2 3
1
Since, (a − d + 1), (a + 4), (a + d + 19) 1−(− )
2
1+
2

are in GP. 30. Ans (B)


2
∴ (a + 4) = (a − d + 1)(a + d + 19) Consider
⇒ 92 = (6 − d)(24 + d) [∵ a = 5]
a = A, b = Ar, c = Ar2 and d = Ar3
⇒ d2 + 18d − 63 = 0 ⇒ d = 3, −21
a2 + b2 + c2 = A2 (1 + r2 + r4)
∴ Required series are 2, 5, 8 and 26, 5, −16
ab + bc + cd = A2 r ( 1 + r2 + r4)
24. Ans (D)
a(rn −1) b2 + c2 + d2 + A2 r2 ( 1 + r2 + r4)
Given, a = 7, ar n−1 = 448 and Sn = = 889
r−1
Those are in G. P
n−1
ar r−a 448r − 7
⇒ = 889 ⇒ = 889
r−1 r−1
⇒ 448r − 7 = 889(r − 1)
⇒r = 2
25. Ans (A)
a(rn −1) (a.rn −a)
Given, Sn = =
r−1 r−1

(a. r n−1
. r − a)
Sn =
r−1
Given, Sn = 364
(a.rn−1 .r−a)
So, = 364
r−1

Since, last term l=ar n−1 =243


3 × 243 − a
⇒ = 364 ⇒ a = 1
2
Again, l = ar n−1 ⇒ 243 = 1(3)n−1
⇒ 35 = 3n−1 ⇒ n = 6
26. Ans (B)
Let a be the first term and r be the common ratio
of the G.P. a1 , a 2 , a 3 , …
We have, a 5 = 5 ⇒ ar 4 = 2
Now, a1 a 2 a3 … a 9 = a ar ar 2 … ar 8
⇒ a1 a2 a 3 … a 9 = a9 r1+2+⋯+8
⇒ a1 a 2 a 3 … a 9 = a9 r 36 = (ar 4 )9 = 29 = 512
27. Ans (B)
n
n+1
G. M = √3. 32 33 . . . . 3n = 3 2

28. Ans (B)


2, 4, 6, 8,….. is an A. P
1 1
1, 3 , 32 , . . .. s a G. P
12
6th term in A. G. P is T6 = 35

29. Ans (A)


MATHEMATICS Page |10A. 24
PERMUTATIONS AND COMBINATIONS

HINTS AND SOLUTIONS

1. Ans (B) 5. Ans (A)


I. In the first throw of coin we have 2 Since repetition of digits not allowed,
outcomes (Head or Tail). Similarly, in second the required 4-digit numbers
and third throw of coin, we have 2 outcomes. 9! 9!
= 9 P4 = =
∴ Number of possible outcomes (9 − 4)! 5!

=2×2×2=8 = 9 × 8 × 7 × 6 = 3024.

(multiplication principle) 6. Ans (D)


I. Number of 3-digit numbers (repetition
II. Number of 2 flag signals = Number of ways
allowed)
of filling 2 vacant places in succession on by
= 5 × 5 × 5 = 125
the 5 flags available
(as unit's, ten's and hundred's places can be
= 5 × 4 = 20 (multiplication principle)
filled in 5 ways)
2. Ans (C)
we start filling in unit's place, because the II. Number of 3-digit numbers (repetition is

options for this place are 2 and 4 only and not allowed)

this can be done in 2 ways; following which = 5 × 4 × 3 = 60

the ten's place can be filled by any of the 5 (as hundred's, ten's and unit's places can be

digits in 5 different ways as the digits can be filled in 5, 4 and 3 ways respectively)

repeated. Therefore, by the multiplication 7. Ans (A)


Required number of ways = 4 != 24
principle, the required number of two digit
8. Ans (B)
even numbers is 2 × 5, i.e., 10. Sum of unit's place = 4 ! (1 + 2 + 3 + 4 +
3. Ans (C) 5) = 24 × 15 = 360
I. Number of 4 letter words that can be
[Since 1 will come 24 times in unit place and
formed from alphabets of the word 'PART' =
so the other digits if 5-digit number is formed
4
P4 = 4! = 24
with digits 1,2,3,4,5 taken all at a time]
II. Number of 4 letter words that can be
So, the sum in tens, hundreds, thousands and
formed when repetition is allowed
ten thousands places is
= 44 = 256
= 360 × 10000 + 360 × 1000 + 360 × 100
4. Ans (D)
There are 10 questions with options of false/ + 360 × 10 + 360

true. It means each question has two options. = 3999960.

Thus, the number of ways that these Alternatively, use T.S.R

questions can be answered 9. Ans (A)


These are 10 letters in the word IRRATIONAL
= 210 = 1024 ways
in which there are 2 I, 2 R and 2 A '.
MATHEMATICS Page | 11A. 1
PERMUTATIONS AND COMBINATIONS

∴ The total number of words 9! 9!


= (10 − 1) = (9)
10! 10! 5! (9 − 4)!
= =
2! 2! 2! (2!)3 = 9 × 9 P4
10. Ans (D) 15. Ans (C)
Vowels are O. I. E. { In the required word } (i) We consider the arrangements by taking 2
No. of odd place = 4 particular children together as one and hence
∴ No. of ways = 4 P3 × 4! = 576 the 4 children can be arranged in 4! = 24
11. Ans (C) ways. Again, two particular children taken
Since, out of eleven members, two members together can be arranged in two ways.
sit together, then the number of Therefore, there are 24 × 2 = 48 total ways
arrangements = 9! × 2 of arrangement.
(∵ two members can sit in two ways.) (ii) Among the 5! = 120 permutations of 5
12. Ans (A) children, there are 48 in which two children
We have n P5 = 42n P3
are together. In the remaining 120 − 48 = 72
⇒ n(n − 1)(n − 2)(n − 3)(n − 4)
permutations, two particular children are
= 42n(n − 1)(n − 2)
never together.
⇒ (n − 3)(n − 4) = 42
16. Ans (C)
[Since n > 4, so n(n − 1)(n − 2) ≠ 0 ] We have 54 Pr = 65 Pr−1
⇒ n2 − 7n − 30 = 0 4! 5!
2 ⇒5× =6×
⇒ n − 10n + 3n − 30 = 0 (4 − r)! (5 − r + 1)!
⇒ n = 10 or n = −3 ⇒ r 2 − 11r + 24 = 0
As n cannot be negative, so n = 10 ⇒ r = 8 or r = 3.
13. Ans (C) But r ≤ 4 ⇒ r = 3
E,E, N, Q,U 17. Ans (C)
Number of words starting with There are 9 objects (letters) of which there
(i) E … … … . . = 24 are 4A′ s, 2L′ s and rest all are different.
4! Therefore, the required number of
(ii) N … … … . . = 2
= 12
arrangements
(iii) Q E....... = 3! = 6
3!
9!
(iv) Q N ..... = 2! = 3 =
4! 2!
(v) QUEEN = 1 5×6×7×8×9
= = 7560
Total = (i) + (ii) + (iii) + (iv) + (v) 2
18. Ans (D)
= 46th
Given ALLMSA
14. Ans (B)
words start from
Total number of 5 -digit numbers having all
4! 24
10
the digits distinct = P5 − 9 P4 A(LLMS) → = = 12
2! 2
10! 9! 10 × 9! 9! L(ALMS) → 4! = 24
= − = −
5! 5! 5! 5!

MATHEMATICS Page | 11A. 2


PERMUTATIONS AND COMBINATIONS

n−1 P
4! 24 3 1
M(ALLS) → = = 12 I. Here, nP =9
2! 2 4

3! (n − 1)(n − 2)(n − 3) 1
SA(MLL) → = 3 ⇒ =
2! n(n − 1)(n − 2)(n − 3) 9
SL(ALM) → 3! = 6 1 1
⇒ = ⇒n=9
Total words = 12 + 24 + 12 + 3 + 6 n 9
5!
5P
= 57 II. Here, r
=2⇒ (5−r)!
6! =2
6P
r=1
[6−(r−1)!
∴ the position of the word SMALL is 58th
5! (7 − r)!
19. Ans (A) ⇒ × =2
(5 − r)! 6!
As shown in figure 1, 2 and X are the three
5! (7 − r)(6 − r)(5 − r)!
boys and 3, 4 and Y are three girls, Boy X will ⇒ × =2
(5 − r)! 6 × 5!
have neighbours as boys 1 and 2 and the girl (7 − r)(6 − r)
Y will have neighbours as girls 3 and 4 ⇒ =2
6
1 and 2 can be arranged in → 42 − 13r + r 2 − 12
= 2! = 2 × 1 = 2 ways ⇒ r 2 − 13r + 30 = 0
Also, 3 and 4 can be arranged in = 2! = ⇒ (r − 10)(r − 3) = 0
2 × 1 = 2 ways ⇒ r = 10,3
Hence, required no. of permutations ( r = 10 rejected as 5 P10 and 6 P9 are not
defined) ∴ r = 3
22. Ans (D)
In the word MONDAY, all letters are different.
I. Out of 6 different letters, 4 letters can be
selected in 6 P4 ways.
∴ Required number of words = 6 P4
6! 6!
= =
=2×2=4 (6 − 4)! 2!
20. Ans (A) 6 × 5 × 4 × 3 × 2!
= = 360
The number of ways in which 5 beads of 2!
different colors can be arranged in a circle to II. The word 'MONDAY' has 6 different letters.

form a necklace are Number of ways of arranging 6 letters at a

(5 − 1)! = 4 ! time = 6 P6

But the clock wise and anticlockwise ∴ Required number of words = 6 P6

arrangement are not different (because when 6! 6×5×4×3×2×1


= = = 720
(6 − 6)! 0!
the necklace is turned over one gives rise to
III. First, we will fix the vowels.
another)
In the word 'MONDAY', there are two vowels
Hence the total number of ways of arranging
1
O and A.
the bead = 2 (4!) = 12
∴ First letter can be chosen by 2 ways.
21. Ans (C)
MATHEMATICS Page | 11A. 3
PERMUTATIONS AND COMBINATIONS

Number of ways taking 5 different letters 11 × 10 × 9 × 8 × 7 × 6 × 5 × 4!


=
from remaining 5 letters = 5 P5 4! × 4 × 3 × 2 × 1 × 2 × 1
8 × 7 × 6 × 5 × 4!
∴ Required number of words = 5 P5 −
4! × 2 × 1
5! 5! = 34650 − 840 = 33810
= =
(5 − 5)! 0!
24. Ans (C)
= 5 × 4 × 3 × 2 × 1 = 120 1 1 1 1 x
+ = + =
Hence, by multiplication rule, the total 6! 7! 6! 7.6! 8.7.6!
number of ways = 2 × 120 = 240 1 1 x
⇒ (1 + ) =
6! 7 8.7.6!
23. Ans (C)
8 x
I. The word 'EQUATION' has 8 different ⇒ =
7 8.7
letters.
⇒ x = 64
Number of ways of arranging 8 different
25. Ans (A)
letters at a time= 8 P8 Since, 3 red balls out of 6 red balls can be
8! 8! selected in 6 C3 ways, 3 white balls out of 5
= =
(8 − 8)! 0!
white balls can be selected in 5 C3 ways and 3
8×7×6×5×4×3×2×1
= blue balls out of 5 blue balls can be selected in
1
5
= 40320(∵ 0! = 1) C3 ways.

II. The word 'MISSISSIPPI' has 11 letters in Hence, by FPC total number of ways selecting

which 9 balls ( 3 balls of each colour) is = 6 C3 ×


5
M → 1 times C3 × 5 C3

I → 4 times 6×5×4 5×4 5×4


= × ×
6 2 2
S → 4 times
= 20 × 10 × 10 = 2000
P → 2 times
26. Ans (D)
The number of permutations of the word There are 13 cards in each suit.
'MISSISSIPPI' in which 4 I's, 4 S's and 2 P's are ∴ Required number of ways
11! 13 13 13 13
alike=
4!4!2!
. = C1 × C1 × C1 × C1 = 13 4
If all the l's are together, then it will be 27. Ans (D)
16
Cr = 16 Cr+1
considered as one letter together with
⇒ 16 = r + r + 1
remaining 7 letters (including 4 ∣ 's)
⇒ 2r = 15
will be considered as 8 letters. So, the number
8! ⇒ r = 7.5
of permutations is 4!2!
.
Which is not possible, since r should be an
( ∵ P is repeated twice and S is repeated four
integer.
times)
28. Ans (B)
Hence, total number of arrangements the required number of ways of selecting 3
11! 8! persons from 2men and 3 women
= −
4! 4! 2! 4! 2!
MATHEMATICS Page | 11A. 4
PERMUTATIONS AND COMBINATIONS

5! 4×5 There are 26 red cards and 26 black cards.


= 5 C3 = 3!2! = 2
= 10.
Therefore, the required number of ways =
Now, 1 man can be selected from 2 men in
26 26
2 C2 × C2
C1 ways and 2 women can be selected from
3 women in 3 C2 ways. 26! 2
=( ) = (325)2
2! 24!
Therefore, the required number of
= 105625
committees
34. Ans (C)
2
2!3
3! n!
= C1 × C2 = × =6 Given (r−1)!(n−r+1)! = 36, … (1)
1! 1! 2! 1!
n!
29. Ans (C) = 84, … (2)
r!(n−r)!
Required number of ways
n!
= 12
C4 × 8 C4 × 4 C4 And (r+1)!(n−r−1)! = 126. … (3)

12! 8! 12! r 36
= × ×1= (1) ÷ (2) gives = 84
n−r+1
8! × 4! 4! × 4! (4!)3
⇒ 84r = 36n − 36r + 36
30. Ans (B)
n
Cr n−r+1 15
Cr 16 − r ⇒ 120r = 36n + 36 … (4)
nC
= ⇒ 15 C
= r+1 84
r−1 r r−1 r (2) ÷ (3) gives = 126
n−r
16 − r
∴ ∑15 2
r=1 r ( ) = ∑15 2
r=1 (16r − r )
⇒ 126r + 126 = 84n − 84r
r
15×16 15×16×31 ⇒ 210r = 84n − 126 … (5)
= 16 × 2
− 6
[∵ ∑nr=1 r 2 =
Solving (4) and (5) n = 9, r = 3
n(n+1)(2n+1)
6 So n C8 = 9 C8 = 9
= 680 35. Ans (B)
31. Ans (B) Total number of players = 16
We have, x C15 = x C14 ∴ No. of ways of selecting 11 players out of 16
16
⇒ x = 14 + 15 players= C11 .
⇒ x = 29 (i) If 2 particular players are included, then
[∴ n Cz = n Cy number of ways
16−2 14
⇒ z = y or n = z + y] = C2 = C9
So, x C29 = 29
C29 = 1 (ii) If 2 particular players are excluded, then
32. Ans (A) number of ways
n
C3 + n C4 > n+1
C3 = 16−2
C11 = 14
C11
n+1 n+1
⇒ C4 > C3 36. Ans (D)
(∵ n Cr + n Cr+1 = n+1
Cr+1 ) Given, 32 P6 = k( 32 C6 )
n+1 32! 32! n!
C4 ⇒ (32−6)! = k ⋅ 6!(32−6)! [∵ n Pr = (n−r)! and
⇒ n+1 > 1
C3
n n!
n−2 Cr = r!(n−r)!]
⇒ >1⇒n>6
4 k
⇒ 1 = 6! ⇒ k = 6 !
33. Ans (C)
⇒ k = 6 × 5 × 4 × 3 × 2 × 1 = 720

MATHEMATICS Page | 11A. 5


PERMUTATIONS AND COMBINATIONS

37. Ans (A) II. Let the number of sides of a polygon = n


Given, 2n C2 : n C2 = 9: 2 Number of diagonal = Number of line
2n
C2 9 segment joining any two vertices of polygon-
⇒ nC
=
2 2
Number of sides
(2n)! 2! (n − 2)! 9
⇒ × = = n C2 − n
2! (2n − 2)! n! 2
n(n − 1) n(n − 3)
(2n)(2n − 1)(2n − 2)! 2(n − 2)! = −n=
⇒ × 2 2
2(2n − 2)! n(n − 1)(n − 2)!
n(n−3)
9 Now, = 44
2
=
2 ⇒ n2 − 3n − 88 = 0
2(2n − 1) 9
⇒ = ⇒ (n − 11)(n + 8) = 0
n−1 2
⇒ n = 11
⇒ 4(2n − 1) = 9(n − 1)
or n = −8 rejected.
⇒ 8n − 4 = 9n − 9
41. Ans (C)
⇒ 9n − 8n = −4 + 9
Number of chords that can be drawn.
⇒n=5
through 21 points on circle = Number of
38. Ans (B)
ways of selecting 2 points from 21 points on
Out of 7 consonants, the number of ways of
21 21×20
selecting 3 consonants = 7 C3 circle = C2 = 2×1
= 210

Similarly, number of ways of selecting 42. Ans (B)


12
Required number ways = C2 − 3 C2
2 vowels out of 4 vowels = 4 C2
12 × 11
∴ Total number of words formed = − 3 = 6 × 11 − 3
1×2
= 7 C3 × 4 C2 × 5 P5 = 66 − 3 = 63
7×6×5 4×3
= 3×2×1
× 2×1 × 5! 43. Ans (D)
= 7 × 5 × 2 × 3 × 120 = 25200 Number of diagonals in a polygon of n sides

39. Ans (D) = n C2 − n


11 Here, n = 20
Required number of ways = C8 = 165
{ As captain already be chosen, now from 11 ∴ Required number of diagonals
players 8 are to be chosen } 20
20 × 19
= C2 − 20 = − 20 = 170
2×1
40. Ans (A)
I. In n-sided polygon, the number of vertices 44. Ans (D)
Number of triangles that can be formed using
=n
12
12 non-collinear points = C3
∴ Number of lines that can be formed using n
Number of triangles that can be formed using
points
7 non-collinear points = 7 C3
= n C2 .
But, 7 points are collinear.
Out of these, n C2 lines, n lines form the
12
∴ Number of triangles = C3 − 7 C3
polygon.
12 × 11 × 10 7 × 6 × 5
∴ Number of diagonals = n C2 − n = −
3×2×1 3×2×1
MATHEMATICS Page | 11A. 6
PERMUTATIONS AND COMBINATIONS

= 220 − 35 = 185 A1 , A2 … … are arranged in 10! ways in which


half of arrangements A1 always above A2
10!
45. Ans(C) R⋅A= = 5 × 9!
DRAUGHT 2!
53. Ans(B)
AU → 1 unit nP

D, R, G, H, T → 5 units
r
= n Cr
r!
Total = 6 units ∑nr=1 n Cr = n C1 + n C2 + ⋯ n Cn
n
R. A = 6! × 2 ! =2 −1
= 720 × 2 = 1440 54. Ans(D)
46. Ans(A) Given word is ARTICLE
X−X−X−X−X R. A = (7−1) C(4−1) × 4!
' X ' for boys, ' - ' for girls = 6 C3 × 4 !
R. A = 5! × 4 P3 = 20 × 24
The number of ways = 4(3! 5!) = 480
= 4(720) = 2880 55. Ans(B)
47. Ans(D) Given digits 0,2,4,5
×B×B×B×B×B×B×B× The number of four-digit numbers
× represents position for girls = 3 × 3! = 18
R.A = 7! × 8 P3 The number of four-digit number which are
= 42 × 8! divisible by 5 is 10
48. Ans(B)
R. A = 6 P1 + 6 P2 + 6 P3 + 6 P4 + 6 P5 + 6 P6
= 6 + 30 + 120 + 360 + 720 + 720 = 1956
49. Ans(D)
R. A = 4 P2 × 6 P3
50. Ans(D) ∴ R. A = 18 − 10 = 8
A, C, H, I, N, S 56. Ans(A)
Number of words start with A is 5! 3,4,5,6 → 18
Number of words start with C is 5! 2,4,5,6 → 17
Number of words start with H is 5! 2,3,5,6 → 16
Number of words start with i is 5! 2,3,4,6 → 15
Number of words start with N is 5! 2,3,4,5 → 14
⇒ total number of words hence formed Number of way = 2(4!) = 48
= 5(5!) = 600 57. Ans(A)
12
∴ 601th word start with S Pr = 11 P6 + 6. 11 P5
i.e., SACHIN r = 6 (∵ (n−1) Pr + r ⋅ (n−1) Pr−1 = n Pr )
51. Ans(C) Alternatively, verify the options
Given digits are 0,1,2,3,4,5,6 58. Ans(B)
R. A = 5 P4 (Repetations are not allowed)
59. Ans(A)
Given digits 3,4,5,6,7
Required sum
= (n − 1) ! (sum of digits) (11111)
= 4! (25) (1111) = 6666600
60. Ans(B)
R ⋅ A = 300 + 120 = 420
Now, we have been given 15 stations in a
52. Ans(D)
railway line and we need to find the total
MATHEMATICS Page | 11A. 7
PERMUTATIONS AND COMBINATIONS

number of different tickets so that a HOSTEL= 1630


passenger can travel to any other station Vowels = O, E
from any one station. Consonants = H, S, T, L
Now, since there are a total of 15 stations, VCCCCV
each station should contain a ticket of all the R. A = 2! × 4! = 48
other stations. Hence, each station should 66. Ans(A)
have a ticket for 15−1=14 stations. f(x) = 7−x Px−3
Thus, each station should have a total of 14 7−x⩾0⇒x⩽7
tickets. x−3⩾0⇒x⩾3
Now, since there are a total of 15 stations, the and 7 − x ⩾ x − 3 ⇒ x ⩽ 5
total number of tickets will be given as: ⇒ 3 ⩽ x ⩽ 5 ⇒ x = 3,4,5
15×14=210 ⇒ Range is {1,2,3)
Thus, each station should have a total of 210 67. Ans(B)
tickets In each selection 2 twos are fixed.
61. Ans(A) It can be done in 7 C2 ways.
BGBGBGBGBG → 5! 5! (or) Remaining 5 places can be filled using two
GBGBGBGBGB → 5! 5! digits in 25 ways.
The row starts with either boy (or) girl ∴ Total number = 7 C2 ⋅ 25
R. A = 2. (5!)2 68. Ans(D)
62. Ans(B) Let us say three boxes A , B & C
×G×G×G×G×G× Each ball can be placed in 3 Ways.
R. A = 5! × 6 P5 Number of ways in which n distinct balls can
63. Ans(A) be put into three boxes = 3ⁿ
R. A = 5 ×8 P3 = 5 × 8 × 7 × 6 Now take the case when two boxes remain
= 1680 empty
2 boxes out of 3 can be empty
in ³C₂ = 3 Ways ( AB , BC , AC)
when these boxes are empty then n Balls can
be put in 1 Way only.
Number of ways in which n distinct balls can
be put into three boxes so that no two boxes
64. Ans(C)
remain empty = 3ⁿ - 3
Given digits 0,1,2,3,4,5
69. Ans(A)
R. A = nr − n Pr
= 105 − 10 P5
70. Ans(D)
Total number of attempts = 4.4.4.4.4
= 45 = 1024
The number of unsuccessful attempts =
1024 − 1
= 1023
71. Ans(D)
we have the numbers 0,1,2,3,5,7
Now the digit should be odd and hence last
Total numbers = digit should be filled with an odd number
5 + 25 + 100 + 300 + 1200 ⇒ Number of way to fill last number =4 (i.e.
65. Ans(A) 1,3,5,7)
MATHEMATICS Page | 11A. 8
PERMUTATIONS AND COMBINATIONS

⇒ Number of way to fill third digit =6 (i.e. 78. Ans(B)


0,1,2,3,5,7) Given word
⇒ Number of way to fille second digit =6 (i.e. E N D E AN O E L
0,1,2,3,5,7) √− √− √− √− √
⇒ Number of way to fille first digit =5 5! 4!
(i.e.1,2,3,5,7) R. A = ×
3! 2!
72. Total 4digit numbers = 2 × 5!
=4×6×6×5=720Ans(D) 79. Ans(D)
R. A = 64 − 54 1C, 1H, 1S, 3E
= 1296 − 625 6!
R. A = 3! = 120
= 671 80. Ans(D)
73. Ans(A)
5,6,6,7,9
In DELHI 5! 120
R. A = 2! = = 60
Vowels = E, I 2
Consonants = D, L, H 81. Ans(A)
R. A = (18 − 1)! × 18 P2
= 17! × 18 × 17
= 17 × 18!
82. Ans(B)
Total no, of persons = 20 + 1
R. A = 53 × 32 = 1125 = 21 (Including host)
74. Ans(C) Two particular persons can sit either side of
Vowels = E, E, A the host R. A = (19 − 1)! × 2 !
Consonants = M, C, T = 2! × 18!
Consonants can be arranged in 3 ! ways 83. Ans(D)
There are four places for vowels 6 men can be arranged around a table in
4P
3
R. A = 3! × =72 5 ! ways. There are 6 places for 5 women
2!
75. Ans(A) 5 women can be arranged in 6 places in 6 P5
Total no. of books = 12 ways. R. A = 5! × 6 P5 = 5! × 6!
12! 84. Ans(C)
∴ No.of arrangements = ( ) There are two cases for each bulb (say ON
3! 3! 3! 3!
= 369600 and OFF)
76. Ans(C) There will be light in the room when at least
Given number one bulb is ON
223355888 − √− √−√− √− No. of ways of selecting ON or OFF are 2 for
odd digits occupy even places each i.e., 212 for 12 bulbs.
4! 5! But in the above ways of selection there will
R. A = × = 60 be a case in which all the bulbs are OFF which
2! 2! 2! 3!
77. Ans(A) leaves the room without light
After excluding N’s, remaining letters ∴ The number of ways to light the room with
4! different amounts of illumination is 212−1
B, A, A, A are arranged in
3!
85. Ans(C)
There are five places for 2 N 's n
5P C0 + n C1 + n C2 + ⋯ . + n Cn = 2n
2
N's can be arranged in 2!
10
C0 + 10 C1 + 10 C2 + ⋯ . + 10 C10 = 210
4! 5 P2 10
2 = 1024
∴ R. A = ×
3! 2! 86. Ans(B)
= 2 × 20 = 40 n
C2 = 66

MATHEMATICS Page | 11A. 9


PERMUTATIONS AND COMBINATIONS

n(n − 1)
⇒ = 66
2 Number of
⇒ n(n − 1) = 12 × 11 3B 2G 4C
Selection
So, n = 12
87. Ans(B) 0 2 2 3
c0 × 2 c2 × 4 c2
Total no.f ways
= 5 C4 × 8 C6 + 5 C5 × 8 C5 2 0 2 3
c2 × 2 c0 × 4 c2
= 140 + 56 = 196
3
1 1 2 c1 × 2 c1 × 4 c2
5 8 13

5 8 10 94. Ans(A)
C4 C6
The number of ways 52 cards can be divided
5 8 10
C5 C5 among 4 players so that each have 13 cards =
(52)!
4!(13!)4
×4!
88. Ans(C) 95. Ans(A)
Let the number of teams = n Required number of ways = 3!(2!)3 ⋅1!
7!
n
C2 = 153
7×6×5×4
n(n − 1) = 51 × 3 × 2 =
8
= 17 × 18,n = 18
= 105
89. Ans(D)
96. Ans(D)
Required number of selections
n = 12
= (12−1) C(5−1)
P = 5 (collinear points).
= 11 C4 The no. of straight lines = n C2 − P C2 + 1
90. Ans(B) = 12 C2 − 5 C2 + 1
C1 C2 Number of selection
8
= 66 − 10 + 1 = 57
4 4 C4 × 4 C4
8
97. Ans(D)
5 3 C5 × 3 C3
The no. of triangles = 12 C3 − 7 C3
Total number of ways = 8 C4 × 8 C5
= 220 − 35 = 185
= 70 + 56
98. Ans(C)
= 126
Number of positive numbers
91. Ans(C)
= (5 + 1)(6 + 1)(3 + 1)
Required number of selections
= 6.7.4 = 168
= n C2 − n = 12 C2 − 12
99. Ans(A)
= 66 − 12 = 54 2n C
3 44
92. Ans(D) nC = 3
⇒n=6
2
Given there are three boxes, each containing Required: nPn=n!=6!=720
10 balls labelled 1, 2, 3, …, 10. Now, one ball is 100. Ans(A)
randomly drawn from each box, and ni denote Given that n C12 = n C8
the label of the ball drawn from the ith box, (i ⇒ n C12 = n Cn−8
= 1, 2, 3). Then, the number of ways in which [∵ n Cr = n Cn−r ]
the balls can be chosen such that n1 < n2 < n3 ∴ n − 8 = 12
is same as selection of 3 different numbers ⇒ n = 12 + 8 = 20
from numbers {1, 2, 3, …, 10} = 10C3 = 120. 101. Ans(A)
93. Ans(A) We know that a coin has Head and Tail (H, T)
60 60 10 ∴ When a coin is tossed 6 times, the number
P(E) = 9 = =
C4 9 × 7 × 3 21 of Possible outcomes = 26 = 64

MATHEMATICS Page | 11A. 10


PERMUTATIONS AND COMBINATIONS

102. Ans(C) Total number of ways = 96 + 120 = 216


Four-digit numbers are to be formed from the 106. Ans(B)
digits 2,3,4,7 without repetition Let the total number of persons in a room be
So, the required 4-digit numbers n since, two persons make 1 hand shake
= 4 P4 = 4! = 4 × 3 × 2 × 1 = 24 ∴ The number of hand shakes = n C2
103. Ans(B) So, n C2 = 66
If we fix 3 at unit place, then the total possible n!
⇒ = 66
numbers = 3! If we fix 4,5 and 6 at unit place, 2! (n − 2)!
in each case, total possible numbers are3! n(n − 1)(n − 2)!
⇒ = 66
Required sum of unit digits of all such 2 × 1 × (n − 2)!
numbers is n(n − 1)
⇒ = 66 ⇒ n2 − n = 132
= 3 × 3! + 4 × 3! + 5 × 3! + 6 × 3! 2
= (3 + 4 + 5 + 6) × 3! ⇒ n2 − n − 132 = 0
= 18 × 3! = 18 × 3 × 2 × 1 = 108 ⇒ (n − 12)(n + 11) = 0
104. Ans (C) ⇒ n − 12 = 0, n + 11 = 0
Given that total numbers of vowels = 4 and ⇒ n = 12, n = −11
total numbers of consonants = 5 ∴ n = 12(∵ n ≠ −11)
Total number of words formed by 2 vowels 107. Ans(D)
and 3 consonants Total number of triangles formed from 12
4! 5! points taking 3 at a time = 12 C3
= 4 C2 × 5 C3 = × But given that out of 12 points, 7 are
2! 2! 3! 2!
4 × 3 × 2! 5 × 4 × 3! collinear. So, these seven points will form no
= ×
2 × 1 × 2! 3! × 2 triangle.
=6 × 10 = 60 ∴ The required number of triangles
Now permutation of 2 vowels and 3 = 12 C3 − 7 C3
consonants = 5 ! 12! 7!
= −
=5 × 4 × 3 × 2 × 1 = 120 3! ⋅ 9! 3! 4!
So, the total number of words = 60 × 120 = 12 × 11 × 10 × 9! 7 × 6 × 5 × 4!
= −
7200. 3 × 2 × 1 × 9! 3 × 2 × 1 × 4!
105. Ans(A) 12 × 11 × 10 7 × 6 × 5
= −
We know that a number is divisible by 3 3×2 3×2
=220 − 35 = 185
when the sum of its digits is divisible by 3.
108. Ans(B)
If we take the digits 0,1,2,4,5, then the sum of
We know that to form a parallelogram, we
the digits
require a pair of lines from a set of 4 lines and
= 0 + 1 + 2 + 4 + 5 = 12,which is divisible
another pair of lines from another set of 3
by 3
lines
So, the 5 digit numbers using the digits
∴ Required numbers of parallelograms
0,1,2,4, and 5
TTh Th H T O = 4 C2 × 3 C2 = 6 × 3 = 18
4 4 3 2 1 109. Ans(C)
Total number of players = 22
4 × 4 × 3 × 2 × 1 = 96 2 players are always included and 4 are
and if we take the digits 1,2,3,4,5, then their always excluding or never included = 22 −
sum= 1 + 2 + 3 + 4 + 5 2 − 4 = 16
=15 divisible by 3 ∴ Required number of selection = 16 C9
So, five digit numbers can be formed using 110. Ans(D)
the digits 1,2,3,4, 5 is 5 ! ways Total number of 5-digit telephone number if
= 5 × 4 × 3 × 2 × 1 = 120 ways all the digits are repeated = (10)5
MATHEMATICS Page | 11A. 11
PERMUTATIONS AND COMBINATIONS

If digits are not repeated, then 5-digit Possible number of choosing 4 blue dyes = 24
telephones, can be formed in 10 P5 ways and possible number of choosing 3 red dyes
∴ Required number of ways = 23
= (10)5 − 10 P5 If at least one blue and one green dyes are
10! selected then the total number of selection
= 100000 −
(10 − 5)! =(25 − 1) × (24 − 1) × 23
10 × 9 × 8 × 7 × 6 × 5! = 31 × 15 × 8 = 3720
= 100000 −
5! 115. Ans(B)
= 100000 − 30240 = 69760 R. A = 1 × 1 × 8 P3 = 8 × 7 × 6
111. Ans(A) 116. Ans(C)
Number of men = 4 n(A) = 4, n(B) = 5
Number of women = 6 The no. of injective mappings from A to B
We are given that the committee includes 2 n(B)
men and exactly twice as many women as Pn(A) = 5 P4 = 120
men. 117. Ans(D)
Thus, the possible selection can be 5G, 3 B
2 men and 4 women and 3 men and 6 women. ×G×G×G×G×G×
So, the number of committees Arrangement of 5girls is 5!
= 4 C2 × 6 C4 + 4 C3 × 6 C6 Arrangement of 3 boys in cross marked
=6 × 15 + 4 × 1 = 90 + 4 = 94 places is 6P3
112. Ans(C) R. A = 5 ! × 6P3 = 120 × 6 × 5 × 4
We have to form 9-digit numbers from = 14400
0,1,2,3,4,5,6,7,8,9 and we know that 0 can not 118. Ans(B)
be put on extremely left place. The number of persons = n
So, first place from the left can be filled in 9 nc2 = 45
ways. n(n − 1) = 10 × 9
Now repetition is not allowed. So, the n = 10
remaining 8 places can be filled in 9! 119. Ans(D)
∴ The required number of ways = 9 × 9 ! odd = 1,3,5,7
113. Ans(B) even = 2,4,6
Total number of letters in the 'ARTICLE' is 7 R. A = 3 C2 × 4 C2 × 4!
out which A, E, I are vowels and R, T, C, L are = 3 × 6 × 24 = 432
consonants 120. Ans (B)
Given that vowels occupy even place n(A × B) = 35 = 7 × 5, 7C5 = 7C2 = 21
∴ possible arrangement can be shown as 121. Ans (D)
below C, V, C, V, C, V, C i.e. on 2nd , 4th and 6th AKMS
places Number of words start with A → 3!
Therefore, number of arrangements = 3 P3 = Number of words start with K → 3!
3! = 6 ways Number of words start with M → 3!
Now consonants can be placed at 1,3,5 and 7 ∴ 19th word start with S i. e. , SAKM
th place 122. Ans (𝐂) or (Grace)
∴ Number of arrangements = 4 P4 = 4! = 24 6c3 × 4c2 or 6c3 × 7c2
So, the total number of arrangements
= 6 × 24 = 144
123. Ans (𝐀)
114. Ans(B)
124. Ans (𝐂)
Possible number of choosing 5 different
125. Ans (𝐂)
green dyes = 25

MATHEMATICS Page | 11A. 12


PERMUTATIONS AND COMBINATIONS

No. ways of selecting the correct answer = 1 


MOCK TEST SOLUTIONS: 11=1
 The no. ways a student can fail to get the
1. Ans (D) correct answer = 64 – 1 = 63
n.n! = [(n + 1) –1] n! = (n + 1)! - n! 8. Ans (C)
=[2! + 3! + 4! … . +(n + 1)!] − [1! + 2!+. . . n!] R, D can be arranged in 2! Ways. Arrange two
= (n + 1)! letters from 4 letters in 4P2 ways and let total
2. Ans (A) of this is one unit with this and remaining are
1!+2!+3!+4!+5!+6!+7!+8!+9!+10!+...+29! 3! Ways
Since,1!+2!+3!+4!=33  Total = 2!. 4P2. 3! Ways
After this for 5!,6!,7! all ones place will come 0 9. Ans (C)
Two specified books = 1 objects; they are
Hence ones place of 1!+2!+....29! is 3
internally arranged in 2!
But for tens place, the tens place will become 0 So, remaining 8 books + 1 object = 9 objects
for factorial of numbers above 10. Hence, we required number of arrangements= 9! 2!
10. Ans (C)
have to add all the tens places for no.s below
8P3 5
10, which is, 11. Ans (B)
3(till4!)+2(for5!)+2(for6!)+4(for7!)+ 6 × 6 = 36
2(for8!)+8(for9!)=21 12. Ans (B)
Two nonzero digits can be selected in 9C2 =
Hence, the tens digit will be 1 and 2 will be 36
carry. ⇒ total = 36 × 4 = 144,
3. Ans (A) Eg :{112, 122, 211, 221}
Take r=1 and verify options One zero and one non zero in 9 ways
4. Ans (A) = 9  2 = 18 Eg : 0. 1 {110, 100}
(2n + 1)! Total = 144 + 18 = 162
(n + 2)! 3 13. Ans (C)
= ⇒n=4 7!
(2n − 1)! 5 TRAC ; E E N N; 2! × 1
(n − 1)!
5. Ans (A) 14. Ans (D)
The number of ways of selecting either boy or 11
P6
11
P5
girl is 10C1+ 8C1 =18 NNN GGR IIEEE = 3!2!
= 2
,
6. Ans (A)
(3! & 2! in denominator for 3N′s and 2G′s)
The first ring can be worn in any of the 4
fingers. So, there are 4 ways of wearing it. 15. Ans (A)
Number of 5 letter words with the condition
Similarly, each of the other rings can be worn that a letter can be repeated is 105.
in 4 ways.
The number of words using 5 different letters
So, the required number of ways is 10P5 .
=(4×4×4×4×4×4×4)=47  required number = 105 − 10P5
7. Ans (D)
(i. e., total number words – number of words
There are 3 questions and 4 options for each 10!
in which no letter is repeated)= 105 −
questions 5!
The total number of possible answers = 4  4 =100000 – 30240 = 69760
 4 = 64
MATHEMATICS Page | 11A. 13
PERMUTATIONS AND COMBINATIONS

16. Ans (B) 24. Ans (C)


(m − 1)! mPn = (5 − 1)! 5P5 Maximum number of points
8×7
= 5! 4! = 8C2 = 1×2 = 28
17. Ans (A) 25. Ans (C)
(m−1)!
mC × nC
2 2 2

7!
= 2520 26. Ans(C)
2 n
C2 − n = 44
18. Ans (A)
n(n − 1)
Reqd. no. of ways ⇒ − n = 44
2
=(4 − 1)! 4P3 = 6 × 4! ⇒ n2 − 3n − 88 = 0
= 6  24 = 144 ⇒ n = 8 or n = 11
19. Ans (A) 27. Ans(D)
(6−1)! 6P3
Reqd. no. of ways = For each of the things there are two
2

5! 6 × 5 × 4 possibilities, it will be selected or rejected.


= 120 × 60 = 7200
2 So, each of the things will have two choices.
20. Ans (D) Therefore, total number of possibilities =27
n−1
C6 + n−1C7 = nC7 = ∴ nC1 > nC6 =128.
n! n! And there will be only one case when all
⇒ >
7! n − 7! 6! n − 6! things will be rejected.
1 1
⇒ > Hence, number of ways of selecting one or
7.6! n − 7! 6! (n − 6)(n − 7)!
more out of 7 things is =27−1=127
 n – 6 > 7  n > 13
28. Ans(D)
21. Ans (D) Required:
8
Reqd. no. of st. lines = 15
C2 − 6C2 + 1 C1 + 8 C2 + 8 C3 + ⋯ … … + 8 C8

[∵join of two points from a straight line and 6 = 28 − 1 = 256 − 1 = 255


points which are collinear given only straight
29. Ans (D)
line in place of 6C2
Synopsis: (9C2 ×9 C2 ) − ∑(8)2
22. Ans (D) 30. Ans (B)
10C2 = 45
nCr n−r+1
Use nC = r
r−1
23. Ans (D)
10C3 − 6C3 = 100

MATHEMATICS Page | 11A. 14


BINOMIAL THEOREM

1
HINTS AND SOLUTIONS

1. Ans (B) = 2[x 5 + 10x 3 (x 3 − 1) + 5x 4 (x 6 − 2x 3 + 1]


[(x + 4y)3 (x − 4y)3 ]2 = 10x 7 + 20x 6 + 2x 5 − 20x 4 − 20x 3 + 10x
= [{x 2 − (4y)2 }]6 = (x 2 − 16y 2 )6 ∴ polynomial has degree 7
∴ No. of terms in the expansion = 7 7. Ans (D)
2. Ans (A) (2x 2 − x − 1)5 = a 0 + a1 x + a 2 x 2 + ⋯ + a10 x10
In the expansion of (a + b)n , the sum of the Put x = 1, we get, 0 = a 0 + a1 + a 2 + ⋯ +
indices of a and b is n + 0 = n in the first term, a10 … (i)
(n − 1) + 1 = n Put x = −1, we get, 32 = a 0 − a1 + a 2 − ⋯ + a10
in the second term and so on. Adding ( i ) and (ii), we get
Thus, it can be seen that the sum of the indices of a 2 + a 4 + ⋯ + a10 = (32/2) + 1
a and b is n in every term of the expansion. = 16 + 1 = 17(∵ a 0 = −1)
3. Ans (A) 8. Ans (C)
Given that, Put √1 − x 2 = y,
(1 + ax)n = 1 + 8x + 24x 2 + ⋯ The given expression becomes
n n(n − 1) 2 2 = (x 2 − y)4 + (x 2 + y)4
⇒ 1 + ax + a x +⋯
1 1.2
= 2[x 8 + 4 C2 x 4 y 2 + 4 C4 y 4 ]
= 1 + 8x + 24x 2 + ⋯
4×3 4
On comparing the coefficient of x, x 2 , we get = 2 [x 8 + x ⋅ (1 − x 2 ) + (1 − x 2 )2 ]
2×1
n(n − 1) 2 = 2[x 8 + 6x 4 (1 − x 2 ) + (1 − 2x 2 + x 4 )]
na = 8, a = 24
2
= 2x 8 − 12x 6 + 14x 4 − 4x 2 + 2
⇒ na(na − a) = 48
9. Ans (B)
⇒ 8(8 − a) = 48
2 x 5
⇒8−a=6⇒a=2 Expand ( − ) using Binomial Theorem
x 2

∴n×2=8⇒n=4 32 40 20 5x 3 x 5
= 5
− 3+ − 5x + −
4. Ans (C) x x x 8 32
10. Ans (A)
√5[(√5 + 1)50 − (√5 − 1)50 ]
We know that,
= 2√5[ 50 C1 (√5)49 + 50
C3 (√5)47 + ⋯ ]
2n−1 = n C0 + n C2 + n C4 + ⋯
50 50 50 48
= 2[ C1 (√5) + C3 (√5) + ⋯] = n C1 + n C3 + n C5 + ⋯
is a natural number ⇒ 10
C1 + 10
C3 + 10
C5 + ⋯ + 10
C9
5. Ans (B) =2 10−1
=2 9

12 terms. 11. Ans (C)


n
[∵ No. of terms in (x + a) = n + 1)] We know that,
6. Ans (A) (1 + x)n = n C0 + n C1 x + n C2 x 2 + ⋯ + n Cn x n ..(i)
5 5
(x + √x 3 − 1) + (x − √x 3 − 1) and (1 − x)n = n C0 − n C1 x + n C2 x 2
+ ⋯ + (−1)n Cn x n
= 2[x 5 + 5 C2 x 3 (x 3 − 1) + 5 C4 x(x 3 − 1)2 ]
On adding Eqs. (i) and (ii),
MATHEMATICS Page |12A. 1
BINOMIAL THEOREM

We get, (1 + x)n + (1 − x)n = 2[8 + 60 + 30 + 1] = 2[99] = 198


= 2[ n C0 + n C2 x 2 + ⋯ + n Cn x n ] So, only Statement II is true.
2n 16. Ans (A)
= n C0 + n C2 + ⋯ + n Cn =
2 50
C4 + 55
C3 + 54
C3 + 53
C3 + 52
C3 + 51
C3
n−1
=2 50
+ C3
12. Ans (C) 50 50 51 52 53 54
= C3 + C4 + C3 + C3 + C3 + C3
5 5]
Since, [(a + b) + (a − b) 55
+ C3
= 2[a5 + 10a3 b2 + 5ab4 ] 51 51 52 53 54 55
= C4 + C3 + C3 + C3 + C3 + C3
5 5
∴ (√3 + 1) + (√3 − 1)
(∵ n Cr + n Cr−1 = n+1
Cr )
5 3
= [(√3) + 10(√3) + 5√3] = 52
C4 + 52
C3 + 53
C3 + 54
C3 + 55
C3
4 2 53 53 54 55
= (√3) [(√3) + 10(√3) + 5] = C4 + C3 + C3 + C3
54 54 55
= C4 + C3 + C3
= 88√3
55 55 56
= C4 + C3 = C4
13. Ans (C)
17. Ans (A)
(1 + 100)100
100 ⋅ 99 We have, (√3 + 1)4 + (√3 − 1)4
= 1 + 100 ⋅ 100 + ⋅ (100)2 + ⋯
1⋅2 = 2( 4 C0 (√3)4 + 4 C2 (√3)2 + 4 C4 (√3)0 )
⇒ (101)100 − 1
= 2(9 + 18 + 1) = 56.
100 ⋅ 99
= 10,000 [1 + + ⋯] 18. Ans (D)
(1 ⋅ 2)(1)
14 14 14 14
C1 + C3 + C5 + ⋯ + C11
So, it is clear that (101)100 − 1 is divisible by
= ( 14 C1 + 14
C3 + 14
C5 + ⋯ + 14
C13 ) − 14
C13
(100)2 = 10000
= 214−1 − 14
C1 = 213 − 14.
14. Ans (C)
19. Ans (C)
(1 + x)n = n C0 + n C1 x + n C2 x 2 + n C3 x 3 + ⋯ ….
If n is odd, then the expansion of
= 1 + nx + x 2 ( n C2 + n C3 x + ⋯ … … )
n+1
⇒ (1 + x)n − nx − 1 = x 2 ( n C2 + n C3 x + ⋯ … . ) (x + a)n − (x − a)n contains ( ) terms
2

Hence, it is divisible by x. 20. Ans (B)


15. Ans (D) We know that,
Now, expand (x + 1)6 using Binomial Theorem (1 + x)20 = 20
C0 + 20
C1 x + ⋯ + 20
C10 x10 + ⋯
20
⇒ (x + 1)6 = x 6 + 6x 5 + 15x 4 + 20x 3 + 15x 2 + C20 x 20
+ 6x + 1 … On putting x = −1, we get
20 20 20 20 20
Similarly, 0= C0 − C1 + ⋯ − C9 + C10 − C11
6 6 5 4 3 2 20
(x − 1) = x − 6x + 15x − 20x + 15x − 6x + ⋯+ C20
20 20 20 20 20
+1 ⇒0= C0 − C1 + ⋯ − C9 + C10 − C9
20 20
Now, adding Eqs. (i) and (ii), we get + C8 − ⋯ + C0
n
(∵ Cr = Cn−r ) n
(x + 1)6 + (x − 1)6
= 2[x 6 + 15x 4 + 15x 2 + 1] ⇒ 0 = 2( 20 C0 − 20
C1 + ⋯ − 20
C9 ) + 20
C10
20
Now, put x = √2 Adding C10 both sides
20
⇒ (√2 + 1)6 + (√2 − 1)6 ⇒ C10 = 2( 20 C0 − 20
C1 + ⋯ + 20
C10 )
1
= 2[(√2)6 + 15(√2)4 + 15(√2)2 + 1] ⇒ 20
C0 − 20
C1 + ⋯ + 20
C10 = 20
C10
2
= 2[23 + 15 × 22 + 15 × 2 + 1] 21. Ans (D)
= 2[8 + 15 × 4 + 30 + 1
MATHEMATICS Page |12A. 2
BINOMIAL THEOREM

If n is odd, then the expansion of = 225K, where K is an integer.

(x + a)n + (x − a)n contains (


n+1
) terms. So, the Hence, 24n − 15n − 1 is divisible by 225 .
2

expansion of
9+1
26. Ans(C)
(1 + 5√2x)9 + (1 − 5√2x)9 has ( ) = 5 terms
2
We have, (a2 x 2 − 2ax + 1)51
22. Ans (C)
The sum of the binomial coefficient of the
Let the consecutive coefficient of
expansion is (a2 − 2a + 1)51
n n n n
(1 + x) are Cr−1 , Cr and Cr+1
The sum of the coefficient of the expansion is
From the given condition, n Cr−1 : n Cr : n Cr+1 =
vanishes, if a2 − 2a + 1 = 0
6: 33: 110
⇒ (a − 1)2 = 0 ⇒ a = 1
n n
Now Cr−1 : Cr = 6: 33
27. Ans(C)
n! r! (n − r)! C1 C2 Cn n
⇒ × C0 + x + x2 + ⋯ + x
(r − 1)! (n − r + 1)! n! 2 3 n+1
6 (1 + x)n+1 − 1
= =
33 (n + 1)x
r 2
⇒ = put x = 1
n − r + 1 11
⇒ 11r = 2n − 2r + 2 C1 C2 Cn 2n+1 − 1
⇒ C0 + + + ⋯+ =
2 3 n+1 n+1
⇒ 2n − 13r + 2 = 0
Alternatively, verify options for n=1, 2
and n Cr : n Cr+1 = 33: 110
28. Ans(D)
n! (r + 1)! (n − r − 1)!
n
⇒ × Cr + 2n Cr−1 + n Cr−2
r! (n − r)! n!
33 3 = n Cr + n Cr−1 + n Cr−1 + n Cr−2
= = n+1 n+1
110 10 = Cr + Cr−1
(r + 1) 3 = n+1+1
Cr = n+2
Cr
⇒ =
n−r 10
29. Ans(A)
⇒ 3n − 13r − 10 = 0
C02 + C12 + C22 + ⋯ + Cn2 = 2nCn (n = 50)
Solving (i) & (ii), we get n = 12
2 2 2 2
∴ (50C0 ) + (50C1 ) + (50C2 ) + ⋯ … + (50C50 )
23. Ans (A)
Using T.S.R = 100C50

24. Ans (A) 30. Ans(D)


n n nCr = nCs ⇒ n = r + s(or)r = s
∑ ∑ (Cr + Cs )
r=0 s=0 n2 − n = 2 + 10
= ∑nr=0 ∑ns=0 Cr + ∑nr=0 ∑ns=0 Cs n2 − n − 12 = 0
= ∑ns=0 (∑nr=0 Cr ) + ∑nr=0 (∑ns=0 Cs ) n = 4 or − 3
= ∑ns=0 2n + ∑nr=0 2n 31. Ans(B)
= (n + 1)2n + (n + 1)2n put x = 1
= (n + 1)2n+1 a = The sum of the coefficients in
25. Ans (A) (1 − 3x + 10x 2 )n
4n (24 )n n
We have, 2 = = (16) = (1 − 3 + 10)n = 8n
= (1 + 15)n b = The sum of the coefficients in
∴ 24n = 1 + n C1 ⋅ 15 + n C2 152 + n C3 153 + ⋯ (1 + x 2 )n = 2n
4n 2( n n
⇒2 − 1 − 15n = 15 C2 + C3 ⋅ 15 + ⋯ ) b3 = (2n )3 = 8n = a

MATHEMATICS Page |12A. 3


BINOMIAL THEOREM

32. Ans(A) ∴ na = 8 ⇒ n = 4
( 21 C1 + 21
C2 … + 21
C10 ) 39. Ans (C)
− ( 10 C1 + 10
C2 … . 10
C10 ) Coefficient of x17 = −
n ×(n+1)
2
1
= [( 21 C1 + ⋯ + 21
C10 ) + ( 21
C11 + ⋯ + 21
C20 )] 18 × 19
2 =− = −171
2
− (210 − 1)
40. Ans (C)
1
= [221 − 2] − (210 − 1) (1 + x 2 )5 (1 + x)4
2
= (1 + 5x 2 + 10x 4 + 10x 6 + 5x 8 + 10)(1 + 4x
= (220 − 1) − (210 − 1) = 220 − 210
+ 6x 2 + 4x 3 + x 4 )
33. Ans(A)
Coefficient of x 5 = 20 + 40 = 60 (use Pascal
We have, (2x 2 − 3x + 1)
Sum of all coefficient for the given expansion, we triangle)

put x = 1 41. Ans (B)

∴ (2 − 3 + 1)11 = 0 (1 − x)5 (1 + x + x 2 + x 3 )4

34. Ans(A) = (1 − x)5 (1 + x)4 (1 + x 2 )4

It is a T.S.R =(1 − x)(1 − x 2 )4 (1 + x 2 )4

35. Ans(C) =(1 − x)(1 − x 4 )4

put x = 1 =(1 − x)[1 − 4x 4 + 6x 8 − 4x12 + x16 ]

The sum of the coefficients ∴ Coefficient of x13 = 4

= (1 + 1 − 3)3148 = 1 42. Ans (D)

36. Ans(A) The number of terms is given by


(n + 2)(n + 1)
put x = 1 and x = −1 (n+3−1)
C3−1 = (n+2)
C2 =
2
1 + a1 + a 2 + ⋯ … + a12 = 0
43. Ans (B)
1 − a1 + a 2 − a 3 + ⋯ … a12 = 26
1 n 1
adding (1 + x)n (1 + ) = n (1 + x)2n
x x
2[1 + a 2 + a 4 + ⋯ . +a12 ] = 26 1 1
(1 + x)2n = n (1 + 2n C1 x + 2n
C2 ⋅ x 2
1 + a 2 + a 4 + ⋯ … + a12 = 25 = 32 xn x
+ ⋯ . . +x 2n )
a1 + a4 + ⋯ . +a12 = 31
1
37. Ans(C) Clearly, Coefficient Of x 𝑛 = 1
put x = y = z = 1 44. Ans (A)
The sum of the coefficients We have,(1 + 3x + 3x 2 + x 3 )20
= (1 + 2 + 1)10 = 410 = [(1 + x)3 ]20 = (1 + x)60
38. Ans (A) Coefficient of x 20 in (1 + x)60
60 60
(1 + ax)n = 1 + 8x + 24x 2 + ⋯ = C20 = C40

1 + n. (ax) +
n(n−1) 2 2
a x + ⋯ . = 1 + 8x + 24x 2 + 45. Ans (D)
2
Standard result

n(n−1) 2
C1 C3 C5 2n − 1
⇒ na = 8 and a = 24 + + +⋯ =
2 2 4 6 n+1
n(n − 1) 2 8 8 Alternatively, verify the options for n=1, 2
a = 24 ⇒ ( − 1) a2 = 48
2 a a 46. Ans (B)
⇒ (8 − a) = 6
Sum of the even binomial coefficients
⇒a=2
= 2n-1 ,Here n = 9,
MATHEMATICS Page |12A. 4
BINOMIAL THEOREM

∴ C0 + C2 + ⋯ + C8 = 28 = 256 53. Ans (B)


47. Ans (B) ∵ (0.99)15 = (1 − 0.01)15
(1 + x)(1 − x)n = (1 + x)(1 − n C1 ⋅ x + =1− 15
C1 (0.01) + 15
C2 (0.01)2
n
C2 x 2 − ⋯ + (−1)n n Cr ⋅ x r + ⋯ … ) − 15
C3 (0.01)3 +. ..
Coff. of x n = 1 − 0.15 + 0.0105 − 0.000455+. ..
n n n−1 n
= (−1) ⋅ Cn + (−1) ⋅ Cn−1 = 1.0105 − 0.150455 = 0.8601
= (−1)n + (−1)n−1 + n 54. Ans (D)
n
(∵ Cn−1 = C1 = n) n
Put x = 1, we get
n
= (−1) (1 − n) n(n + 1) 2
2
(1 + 2 + 3 + ⋯ + n) = ( ) = ∑ n3
48. Ans (D) 2
= ( n C0 )2 + ( n C1 )2 + ( n C2 )2 + ⋯ + ( n C5 )2 55. Ans (C)

= ( 5 C0 )2 + ( 5 C1 )2 + ( 5 C2 )2 +( 5 C3 )2 + ( 5 C4 )2 + We have, 49n + 16n − 1 = (1 + 48)n + 16n − 1

( 5 C5 ) 2 = 1 + n C1 (48) + n C2 (48)2 +. . . + n Cn (48)n + 16n

= 1 + 25 + 100 + 100 + 25 + 1 = 252 −1

49. Ans (A) = (48n + 16n) + n C2 (48)2

We know, + n C3 (48)3 +. . . + n Cn (48)n

(1 + x)n = C0 + C1 x + C2 x 2 +. . . +Cr x r +. .. …(i) = 64n + 82 [ n C2 62 + n C3 63 8 + n C4 64 82 + ⋯


1 n 1 1 1 + n Cn 6n 8n−2 ]
and (1 + ) = C0 + C1 + C2 2 +. . . +Cr
x x x xr
Hence, 49n + 16n − 1 is divisible by 64
1 1 1
+Cr+1 + Cr+2 . . . Cn …(ii) 56. Ans (C)
xr+1 xr+2 xn

On multiplying Eqs. (i) and (ii), equation Number of terms =


(n+1)(n+2)
2
1
coefficient of x r in n (1 + x)
2n
or the coefficient of 11 × 12
x
= = 66
x n+r 2n
in (1 + x) , we get the value of required 2

expression which is 57. Ans (C)

(2n)! Given that, T1 = n C0 = 1 …(i)


2n
Cn+r =
(n − r)! (n + r)! T2 = n C1 ax = 6x
n!
50. Ans (B) ⇒ a = 6 ⇒ na = 6 …(ii)
(n−1!)
101 101
In (x + a) + (x − a) , n is odd
and T3 = n C2 (ax)2 = 6x 2
n+1
∴ Total number of terms = n(n−1) 2
2 ⇒ a = 16 ...(iii)
2
101 + 1
= = 51 Only option (C) is satisfying Eqs. (ii) and (iii)
2
51. Ans (B) 58. Ans (A)
n(n+1) Given, (1 + x)n = C0 + C1 x + C2 x 2 + ⋯ + Cn x n
Coefficient of x19 = −
2
Also, (x + 1)n = Cn + Cn−1 x + Cn−2 x 2 + ⋯ + C0 x n
20
= − [ (1 + 20)] = −10 × 21 = −210 On multiplying both equations and comparing
2
52. Ans (C) coefficient of x n−1 on both sides, we get
2n
We know that, C0 C2 + C1 C2 + C2 C3 + ⋯ + Cn−1 Cn = Cn−1
15
C0 + 15
C1 + 15
C2 + ⋯ + 15
C15 = 215 (2n)!
=
(n − 1)! (n + 1)!
⟹ 2( 15 C8 + 15
C9 + ⋯ + 15
C15 ) = 215
Alternatively, verify options for n=1, 2
[∵ n Cr = n Cn−r ]
15 15 15
59. Ans (A)
⟹ C8 + C9 + ⋯ + C15 = 214

MATHEMATICS Page |12A. 5


BINOMIAL THEOREM

∑10
k=0
20
Ck = 20
C0 + 20
C1 + 20
C2 + ⋯ + 20
C10 Given 1! + 2! + 3!+. . . +11!
We have = (1! + 2! + 3!) + (4! + 5!. . . +11!)
220 = 2( 20 C0 + 20
C1 + 20
C2 + ⋯ + 20
C9 ) = 9 + (4! + 5!. . . +11!)
20 Note that 4!, 5!, ..., 11! are divisible by 12
+ C10
⟹ 219 = ( 20 C0 + 20
C1 + 20
C2 + ⋯ + 20
C9 ) ∴ Remainder = 9
1 20
65. Ans(D)
+ C10
2 1 14

⟹2 19
= 20
C0 + 20
C1 + 20
C2 + ⋯ + 20
C10 (x + )
√x
1 10
− 20
C10 14
1
2 T11 = C10 (x)4 ( )
20 20 20
√x
⟹ C0 + C1 + ⋯ + C10
1 14 × 13 × 12 × 11 1001
1 = =
= 219 + 20
C10 x 4×3×2×1 x
2
66. Ans(A)
60. Ans (D)
(x + a)47 − (x − a)47
5
x = (√3 + 1) n+1 48
The no. of terms = = = 24
2 2
5 4 3
5 5
= (√3) + C1 (√3) + C2 (√3) 67. Ans(C)
2 n+1
+ 5 C3 (√3) + 5 C4 (√3) + 5 C5 Number of terms = = 13
2
= 9√3 + 45 + 30√3 + 30 + 5√3 + 1 68. Ans(D)
16 16 16 16
= 76 + 44√3 We have, C9 + C10 − C6 − C7
5 17 17 n n n+1
∴ [x] = [(√3 + 1) ] = [76 + 44√3] = C10 − C7 (∵ Cr + Cr−1 = Cr )
17 17
= C10 − C10 = 0
= [76] + [44 × 1.732]
n
(∵ Cr = Cn−r ) n
= 76 + [76.2]
69. Ans(D)
= 76 + 76 = 152
(n+1)(n+2) 11×12
61. Ans (B) Required number of terms= = =
2 2

Since, total number of terms 66


= 59 + 1 = 60 70. Ans (A)
We have sum of all binomial coefficients=259 nCn/2 = 924 ⇒ n = 12
259
∴ Required sum = = 258 71. Ans (A)
2
𝑛
Cr 𝑛−𝑟+1
Use TSR: 𝑛C =
r−1 𝑟
62. Ans (A)
72. Ans (D)
The coefficient of x in the expansion of
(1 + x)(1 + 2x)(1 + 3x)…(1 + 100x)
MOCK TEST SOLUTIONS:
= 1 + 2 + 3 +….+100
1. Ans(C)
100(100 + 1)
= 50 × 101 = 5050 The number of terms in
2
63. Ans(C) n
(x + a)n + (x − a)n = + 1,
2
Number of terms in the expansion of
when " n" is even N=100=even
𝑛+2
(x + a)100 + (x − a)100 =
2 The number of terms = 51
Therefore, the number of terms = 51 2. Ans(B)
64. Ans(A) The number of terms in (a + x)n = n + 1
MATHEMATICS Page |12A. 6
BINOMIAL THEOREM

(1 + 5x + 10x 2 + 10x 3 + 5x 4 + x 5 )20 (1 + x)n = 1 + nx +


n(n−1) 2
x +
n(n−1)(n−2) 3
x + ⋯.
1.2 1.2.3
5 20 100
= ((1 + x) ) = (1 + x)
∴ (1 + x)n − nx − 1
The number of terms = 100 + 1 = 101 n(n − 1) n(n − 1)(n − 2)
= x2 [ + x + ⋯.]
3. Ans(C) 1.2 1.2.3
T5 = 24T3 From above it is clear that (1 + x)n − nx − 1 is
⇒ 11C4 x 4 = 24 × 11C2 x 2 divisible by x 2 .
11 × 10 10. Ans (B)
⇒ 11C4 . x 2 = 24 ×
2×1 From the given condition, replacing a by ai and
⇒ x2 = 4 −ai respectively, we get
⇒ x = ±2 (x + ai)n = (T0 − T2 + T4 − ⋯ . )
4. Ans(C) + i(T1 − T3 + T5 − ⋯ . )
(x + a)n + (x − a)n =2[ n Co x n + n C2 x n−2 a2 + n
and (x − ai) = (T0 − T2 + T4 − ⋯ . ) − i(T1 − T3 +
n n−4 4
C4 x a + ⋯.] T5 − ⋯ . )
(x 3 1/2 5 (x 3 1/2 5
So, [x + − 1) ] + [x − − 1) ] Multiplying (i) and (ii) we get required result
5 5 5 3 (x 3 5 3 2]
=2[ C0 x + C2 x − 1) + C4 x(x − 1) i.e. (x 2 + a2 )n = (T0 − T2 + T4 − ⋯ . )2 + (T1 −
So, degree = 7 T3 + T5 − ⋯ . . )2
5. Ans(C) 11. Ans (C)
(1 + x + x 2 )(1 − x)15 Given expression
2 )(15C 14
= (1 + x + x 0 − 15C1 x − ⋯ … + 15C14 x = 2[1 + 9 C2 (3√2x)2 + 9 C4 (3√2x)4
− 15C15 x15 ) + 9 C6 (3√2x)6 + 9 C8 (3√2x)8 ]
16 15 15
Coefficient x = − C15 + C14 ∴ The number of non-zero terms is 5 .
= −1 + 15 = 14 12. Ans (A)
6. Ans (C) We know that
(x + a)n − (x − a)n 2n−1 = n C0 + n C2 + n C4 + ⋯ .
= 2[ n C1 x n−1 a + n C3 x n−3 a3 + n C5 x n−5 a5 + ⋯ . . ] = n C1 + n C3 + n C5 + ⋯ ..
∴ (√2 + 1)6 − (√2 − 1)6 So, 10
C1 + 10
C3 + 10
C5 + ⋯ . . + 10 C9 = 210−1 =
= 2[ 6 C1 (√2)5 (1)1 + 6 C3 (√2)3 (1)3 29
+ 6 C5 (√2)1 (1)5 ] 13. Ans (A)

∵ (√2 + 1)6 − (√2 − 1)6 (1 + x)n = C0 + C1 x + C2 x 2 + ⋯ . +Cr x r + ⋯.


1 n 1 1 1
= 2[6 × 4√2 + 20 × 2√2 + 6√2] (1 + ) = C0 + C1 + C2 2 + ⋯ . . +Cr r … (i)
x x x x
= 2[24√2 + 40√2 + 6√2] = 140√2. Multiplying both sides and equating coefficient of
7. Ans (C) 1
x r in (1 + x)2n or the coefficient of x n+r in (1 +
n n n n−1 n n−2 2 xn
(x + a) = C0 + C1 x a + C2 x a + ⋯..
x)2n we get the value of required expression
So, (x + 2a)5 = x 5 + 10x 4 a + 40x 3 a2 + 80x 2 a3 +
2n
(2n)!
80xa4 + 32a5 . = Cn+r =
(n − r)! (n + r)!
8. Ans (B) Trick: Solving conversely.
5 5
(√5 + 1) − (√5 − 1) Put n = 1 and r = 0 in first term, (given
5 4 5 2 5
= 2{ C1 (√5) + C3 (√5) + C5 ⋅ 1} condition)
= 352 (i) 1 C0 1 C0 + 1 C1 1 C1 = 1 + 1 = 2 ,
9. Ans (B) (∵ r ≤ n)
MATHEMATICS Page |12A. 7
BINOMIAL THEOREM

Put n = 2, r = 1, then We have (1 + x)15 = C0 + C1 x +


(ii) 2 C0 2 C1 + 2 C1 2 C2 = 2 + 2 = 4 C2 x 2 . + ⋯ . +C15 x15
Now check the options (1 + x)15 − 1
⇒ = C1 + C2 x + ⋯ . +C15 x14
2! x
(a) (i) Put n = 1, r = 0, we get =2
(1)!(1)!
Differentiating both sides with respect to x, we get
4!
(ii) Put n = 2, r = 1, we get =4 x. 15(1 + x)14 − (1 + x)15 + 1
(1)!(3)!
=
Note: Students should remember this question as x2
= C2 + 2C3 x + ⋯ . . + 14 C15 x13
an identity.
Putting x = 1, we get
14. Ans (C)
C2 + 2C3 + ⋯ . +14C15 = 15.214 − 215 + 1 =
Trick: Put n = 1,2
1 1 1 1 13.214 + 1.
At n = 1, 1 C0 − C1 =1− =
2 2 2
17. Ans (C)
2 1 2 1 2
At n = 2, C0 − C1 + C2 Putting x = 1 in (x 2 + x − 3)319
2 3

1 1 We get the sum of coefficient


=1−1+ =
3 3
= (1 + 1 − 3)319 = −1
which is given by option (C).
18. Ans (D)
15. Ans (B)
(1 + x − 2x 2 )6 = 1 + a1 x + a 2 x 2 + ⋯ . +a12 x12 .
(1 + x)n = C0 + C1 x + C2 x 2 + ⋯ . . +Cn x n
Putting x = 1 and x = −1 and adding the results
1 n 1 1 2 1 n
and (1 + ) = C0 + C1 + C2 ( ) + ⋯ . . +Cn ( ) 64 = 2(1 + a 2 + a 4 + ⋯ )
x x x x

If we multiply (i) and (ii), we get ∴ a 2 + a 4 + a 6 + ⋯ . +a12 = 31.


C02 + C12 + C22 + ⋯ . . +Cn2 19. Ans (C)
is the term independent of x and hence it is equal (1 + x + x 2 + x 3 )5 = (1 + x)5 (1 + x 2 )5
to the term independent of x in the product (1 + = (1 + 5x + 10x 2 + 10x 3 + 5x 4 + x 5 )
1 n × (1 + 5x 2 + 10x 4 + 10x 6 + 5x 8 + x10 )
x)n (1 + ) or
x Therefore, the required sum of coefficients =
1 2n n 2n
in (1 + x) or term containing x in (1 + x) . (1 + 10 + 5) ⋅ 25
xn

Clearly the coefficient of x n in (1 + x)2n is Tn+1 = 16 × 32 = 512


(2n)!
and equal to 2n
Cn = Note: 2n = 25 = Sum of all the binomial
n!n!

Trick: Solving conversely. coefficients in the 2nd bracket in which all the

Put n = 1, n = 2, ….. then we get S1 = 1 C02 + powers of x are even.


1 2
C1 = 2, S2 = 2 C02 + 2 C12 + 2 C22 = 12 + 22 + 12 = 20. Ans (C)

6 The sum of the coefficients of the polynomial

Now check the options (α2 x 2 − 2αx + 1)51 is obtained by putting x = 1

(A) Does not hold given condition, in


2! (α2 x 2 − 2αx + 1)51 .
(B) (i) Put n = 1, then =2
1!1!
Therefore, by hypothesis
4! 4×3×2×1
(ii) Put n = 2, then = =6
2!2! 2×1×2×1 (α2 − 2α + 1)51 = 0 ⇒ α = 1
Note: Students should remember this question as 21. Ans (C)
an identity. We have 15
C0 + 15
C1 + ⋯ + 15
C15 = 215
16. Ans (B) ⇒ 2( 15 C8 + 15
C9 + ⋯ + 15
C15 )
= 215 (∵ n Cr = n Cn−r )

MATHEMATICS Page |12A. 8


BINOMIAL THEOREM

⇒ 15
C8 + 15
C9 + ⋯ + 15
C15 = 214 We have, (1 + x)50 = ∑50
r=0
50
Cr x r .

22. Ans (C) Therefore, sum of coefficients of odd power of x =


50 50 50
As we know that C1 + C3 + ⋯ + C49
n
C0 − n C12 + n C22 − n C32 + ⋯ + (−1)n . n Cn2 = 0, 1 1
= [ 50 C0 + 50
C1 + ⋯ + 50
C50 ] = [250 ]
2 2
(if n is odd)
= 249
and in the question n = 15 (odd).
26. Ans (B)
23. Ans (A)
For, the sum of coefficients, put x = 1, to obtain
(1 + x)n = C0 + C1 x + C2 x 2 + ⋯ . +Cn x n
the sum as (1 + 1 − 3)2134 = 1.
Putting x = 1, we get
27. Ans (B)
⇒ 2n = C0 + C1 + C2 + ⋯ . . +Cn
We can obtain sum of coefficients by putting x = 1
or C1 + C2 + C3 + ⋯ . +Cn = 2n − 1
in polynomial.
[∵ C0 = n C0 = 1]
⇒ (1 + x + x 2 )n = (1 + 1 + 1)n = 3n
Again, putting x = −1,
28. Ans (C)
we get
Sum of the coefficients = (1 + 1)5 = 25
0 = C0 − C1 + C2 − C3 + ⋯
= 32.
or C0 + C2 + C4 + ⋯ . = C1 + C3 + C5 + ⋯. i.e. A =
29. Ans (A)
B
∑10
K=0
20
Ck i.e., 20
C0 + 20
C1 + ⋯ . . + 20 C10
n
Also, from (i), A + B = 2 or
We know that, (1 + x)n = n C0 + n C1 x1 +
A = 2n−1 = B n
C2 x 2 + ⋯ . + n Cn ⋅ x n
n−1
Hence, C0 + C2 + C4 + ⋯ . = 2 .
Put x = 1; 2n = n C0 + n C1 + n C2 + ⋯ . . + n Cn
24. Ans (B)
Put n = 20; 2 20 = 20
C0 + 20
C1 + 20
C2 +
(1 + x)n = C0 + C1 x + C2 x 2 + C3 x 3 + ⋯ . . +Cn x n
⋯ … . . + 20 C20
(1 − x)n = C0 − C1 x + C2 x 2 − C3 x 3
220 + 20
C10 = 2[ 20 C0 + 20
C1 + ⋯ … . + 20 C10 ]
+ ⋯ . . +(−1)n Cn x n
1
[(1 + x)n − (1 − x)n ] = 2[C1 x + C3 x 3 + C5 x 5 + ⋯ ] [ 20 C0 + 20
C1 + ⋯ . . . + 20 C10 ] = 219 + 20
C10
2
1 1 20
[(1 + x)n − (1 − x)n ] = C1 x + C3 x 3 + C5 x 5 + ∑10
k=0
20
Ck = 219 + C10 .
2 2

⋯ …. 30. Ans (C)


3 5
Put x = 2,2. C1 + 2 . C3 + 2 . C5 + ⋯ .. Putting x = 1 in (x 2 − x − 1)99
3n − (−1)n we get the sum of the coefficient of (x 2 − x − 1)99
=
2
= (12 − 1 − 1)99 = −1.
25. Ans (B)

MATHEMATICS Page |12A. 9


L LIMITS AND DERIVATIVES

HINTS AND SOLUTIONS

1. Ans (C) 1 1
= =
We know that greatest integer function is not 4+2+1 7
7. Ans (B)
continuous at integer. So, limx→n [x] does not exist.
sin x(2 cos x − a)
Alternatively, at all integral points LHL≠RHL limx→0
x ⋅ x2
So, limit does not exist. sin⁡ x 2cos⁡ x − a
= limx→0 ( )( )
2. Ans (C) x x2
x For this limit to exists finitely,
[ ]
∴ limx→π 3 =0 2cos⁡ x−a
2 ln⁡(1 + cot⁡ x) limx→0 = finite
x2
π π 0
Here [ ] = 0, Because⁡ < 1 ∴ It must be form
6 6 0

3. Ans (D) ∴ 2cos⁡(0) − a = 0 ⇒ a = 2


xn −3n 8. Ans (C)
We have, limx→3 = n(3)n−1
x−3
1+2+3+⋯+n
Therefore, n(3)n−1 = 108 As limn→∞
n2

= 4(27) = 4(3)4−1 n(n + 1)


= limn→∞
2n2
On comparing, we get n = 4
1 1 1
4. Ans (D) = limn→∞ (1 + ) =
2 n 2
|x+2|
We have, limx→−2 9. Ans (C)
x+2

L.H.L = limx→−2−
|x+2| limx→3 [x(x + 1)] = 3(3 + 1) = 3(4) = 12
x+2
10. Ans (B)
−(x + 2)
= limx→−2 = −1 Put y = 1 + x,
x+2
R.H. L = limx→−2+
|x+2| so that y → 1 as x → 0
x+2
√1+x−1 √y−1
x+2 Then limx→0 = limy→1
= limx→−2 =1 x y−1
x+2 1
Since L. H. L. ≠ R. H. L. y2
−1 1 1 1
= limy→1 = (1)2−1 =
y−1 2 2
∴ Limit does not exist.
11. Ans (C)
5. Ans (B)
(1 − cos⁡ 2x)(3 + cos⁡ x) (2x − 3)(√x − 1)
limx→0 ⁡⁡lim
x tan 4x
x→1 2x 2 + x − 3
(2x − 3)(√x − 1)
(2sin2 x)(3 + cos⁡ x) ⁡⁡= lim
= limx→0 x→1 (2x + 3)(x − 1)
xtan⁡ 4x
(2x − 3)(√x − 1)
sin⁡ x 2 4x 1 ⁡⁡= lim
= limx→0 ( ) ( ) (3 + cos⁡ x) × = 2 x→1 (2x + 3)(√x + 1)(√x − 1)
x tan⁡ 4x 2
(2x − 3)
6. Ans (D) ⁡⁡= lim
x→1 (2x + 3)(√x + 1)
x−2
We have, limx→2 ( ) 2×1−3 1
x3 −x2 −x−2 ⁡⁡= ⁡= −
x−2 (2 × 1 + 3)(√1 + 1) 10
⁡⁡
= limx→2 ( )
(x − 2)(x 2 + x + 1) 12. Ans (D)
1 1 log(1 + sin 10x)
= limx→2 ( 2 ) limx→0
x +x+1 2 tan 3x
MATHEMATICS Page | 13A. 1
L LIMITS AND DERIVATIVES

(⁡ as, 1 + sin⁡ 10x = (sin⁡ 5x + cos⁡ 5x)2 ) x+x2 +⋯+xn −1−1−1−⋯−1


= lim { }
x→1 x−1
1
= limx→0 (sin 10x) (x−1)+(x2 −1)+(x3 −1)+⋯+(xn −1)
log(1 + sin 10x) = lim { }
x→1 x−1
sin 10x 3x 10x = lim {1 + (x + 1) + (x 2 + x + 1) + ⋯ }
⋅( )⋅( )⋅ x→1
10x tan⁡ 3x 3x
1 10 5 = 1+ 2 + 3+ ⋯+ n
= ×1×1× =
2 3 3 n(n + 1)
=
13. Ans (A) 2
limx→0+ xcot⁡ x + limx→0+ xln⁡ x 17. Ans (D)

x ln⁡ x x3 x2
⁡⁡= lim+ + lim+ ⁡⁡ lim ( − )
x→0 tan⁡ x x→0 1 x→∞ 3x 2 − 4 3x + 2
( )
x x 3 (3x + 2) − x 2 (3x 2 − 4)
1 ⁡⁡= lim ( )
x ( ) x→∞ (3x 2 − 4)(3x + 2)
⁡⁡= lim+ + lim+ x = 1 + lim+ (−x) 3x 4 + 2x 3 − 3x 4 + 4x 2
x→0 tan⁡ x x→0 1 x→0
(− 2 ) ⁡⁡= lim ( )
x x→∞ 9x 3 + 6x 2 − 12x − 8
⁡⁡= 1 + 0 = 1
2x 3 + 4x 2
14. Ans (C) ⁡⁡= lim ( 3 )
x→∞ 9x + 6x 2 − 12x − 8
x1/4 − x1/5 4
limx→1 2+
x3 − 1 ⁡⁡= lim ( x )
x→∞ 6 12 8
(x1/4 − 1) − (x1/5 − 1) 9+ − 2 − 3
x x x
= limx→1 2 a
x3 − 1 ⁡⁡= [∵ lim = 0]
1 1 1 1 9 x→∞ x
x 4 − 14 x 5 − x 5 18. Ans (A)
= limx→1 x−1 − x−1
x 3 − 13 x 3 − 1 3 We have, limx→2 [
1

2(2x−3)
]
x−1 x−1 x−2 x3 −3x2 +2x
( ) 1 2(2x − 3)
1 1 1 = limx→2 [ − ]
= − = x − 2 x(x − 1)(x − 2)
12 15 60
x(x − 1) − 2(2x − 3)
15. Ans (D) = limx→2 [ ]
x(x − 1)(x − 2)
(√1 + 3x − √1 − 3x) x 2 − 5x + 6
limx→0 = limx→2 [ ]
x x(x − 1)(x − 2)
(√1 + 3x − √1 − 3x) (x − 2)(x − 3)
= limx→0 = limx→2 [ ]
x x(x − 1)(x − 2)
(√1 + 3x + √1 − 3x) (x − 3) −1
× = limx→2 [ ]=
(√1 + 3x + √1 − 3x) x(x − 1) 2
{(1 + 3x) − (1 − 3x)} 19. Ans (D)
= limx→0
x(√1 + 3x + √1 − 3x) 1 + 2 + 3 + ⋯ . +n n
6x limn→∞ { − }
= limx→0 n+2 2
x(√1 + 3x + √1 − 3x) n(n + 1) 1 n
= limn→∞ { × − }
6 2 n+2 2
= limx→0
(√1 + 3x + √1 − 3x) n(n + 1)
{∵ ⁡∑nr=1 r = }
6 6 2
= = = 3.
(√1+√1) 2 (n + 1) 1
= limn→∞ { − }⋅n
Alternatively, use L-H rule 2(n + 2) 2
16. Ans (C) 1 n+1−n−2
= limn→∞ { }⋅n
x + x2 + ⋯ + xn − n 2 n+2
lim { } n (−1)
x→1 x−1 = limn→∞ ×
2 n+2

MATHEMATICS Page | 13A. 2


L LIMITS AND DERIVATIVES

−1 1 1 1 (1 − cos⁡ 2x)sin⁡ 5x
= limn→∞ ( ) ⋅ =− ×1=− limx→0
2 2 2 2 x 2 sin⁡ 3x
(1 + )
n
2 sin2 x sin⁡ 5x
20. Ans (B) = limx→0
x 2 sin⁡ 3x
√x + 3 sin⁡ 5x
limx→−3 sin2 x ( 5x ) × 5
x+1 = limx→0 2 ( 2 )
x sin⁡ 3x
But √x + 3 is not defined on left hand side of -3. ( )×3
3x
Hence, function is not defined sin 5x
sin x 2 5⁡lim5x→0 ( 5x )
21. Ans (B) = limx→0 ( ) ×
x sin 3x
3⁡lim3x→0 ( )
sin2 ⁡ x + cos⁡ x − 1 3x
limx→0 2 × 5 10
x2 = =
3 3
cos⁡ x − cos 2 ⁡ x
= limx→0 25. Ans (D)
x2
1 − cos⁡ x sin⁡ |x|
= limx→0 ( ) cos⁡ x limx→0
x2 x
x LHL = −1, RHL = 1
2sin2 ⁡
= limx→0 2 cos⁡ x
x2 Limit does not exist.
x 26. Ans (B)
2sin2 ⁡
= limx→0 2 ⋅ limx→0 cos⁡ x cot⁡ 2x−cosec⁡ 2x
x 2 We have, limx→0
( ) ⋅4 x
2
cos⁡ 2x 1
2 1 −
= ×1= = limx→0 sin⁡ 2x sin⁡ 2x
4 2 x
22. Ans (C) cos⁡ 2x − 1
= limx→0
sec⁡ 5x − sec⁡ 3x xsin⁡ 2x
limx→0
sec⁡ 3x − sec⁡ x −(1 − cos⁡ 2x)
= limx→0
cos⁡ 3x − cos⁡ 5x cos⁡ x xsin⁡ 2x
= limx→0 ( )( )
cos⁡ x − cos⁡ 3x cos⁡ 5x 2 sin2 x
= −limx→0
2 sin 4x sin⁡ x cos⁡ x x(2 sin xcos x)
= limx→0 limx→0
2 sin 2x sin⁡ x cos⁡ 5x tan⁡ x
= −limx→0 = −1
2 sin 2x cos⁡ 2x x
= limx→0
sin⁡ 2x 27. Ans (B)
= limx→0 2cos⁡ 2x = 2 (x − 1)(x 2 + 2x + 2)
limx→1
23. Ans (C) sin⁡(x − 1)
tan⁡ x − sin⁡ x x−1
limx→0 = limx→1 limx→1 (x 2 + 2x + 2)
x3 sin⁡(x − 1)
sin⁡ x = 1(1 + 2 + 2) = 5.
− sin⁡ x
= limx→0 cos⁡ x 3
x 28. Ans (B)
sin⁡ x(1 − cos⁡ x) sin⁡ x
= limx→0 We have limx→0
x(1+cos⁡ x)
x 3 cos⁡ x
x x x
sin⁡ x ⋅ 2sin2 ⁡ 2sin⁡ cos⁡ 1
2 = limx→0 2 2
= limx→0
x 3 cos⁡ x 2 x =2
x (2cos ⁡ )
2
x 2 x
2 sin⁡ x sin⁡ 1 tan⁡ 1
2
= limx→0
cos⁡ x
⋅(
x
)⋅( x ) ⋅
4 limx→0 x 2 =
2
2 2
1 1 29. Ans (B)
= 2⋅1⋅1⋅ =
4 2 2sin2 ⁡ x+sin⁡ x−1 0
We have, limx→π ( form)
24. Ans (A) 2
6 2sin ⁡ x−3sin⁡ x+1 0
MATHEMATICS Page | 13A. 3
L LIMITS AND DERIVATIVES

(sin⁡ x + 1)(2sin⁡ x − 1) dy
= limx→π ∴ = 2x + cos⁡ x + (−2)x −3
6 (sin⁡ x − 1)(2sin⁡ x − 1) dx
(sin⁡ x + 1) 2
= limx→π = 2x + cos⁡ x − 3
6 (sin⁡ x − 1) x
1 34. Ans (D)
limx→π/6 (sin⁡ x + 1) (2 + 1) f(x+h)−f(x)
= = = −3 I. f ′ (x) = limh→0
limx→π/6 (sin⁡ x − 1) (1 − 1) h
2
(x + h)3 − x 3
30. Ans (B) = limh→0
h
sin⁡ x n x 3 + h3 + 3xh(x + h) − x 3
limx→0 = limh→0
(sin⁡ x)m h
sin⁡ x n x n x m = limh→0 (h2 + 3x(x + h)) = 3x 2
= limx→0 ( ) ( ) ( )
xn x m sin⁡ x f(x+h)−f(x)
II. f ′ (x) = limh→0
= limx→0 x n−m = 0 [∵ m < n] h
1 1
31. Ans (C) − 1
(x + h)3 x 3
x100 x99 x2
= limh→0 [∵ f(x) = 3 ]
Given, f(x) = + + ⋯+ +x+1 h x
100 99 2
x 3 − (x + h)3
100x 99 99x 98 2x = limh→0
⇒ f ′ (x) = + + ⋯+ + 1+ 0 (x + h)3 x 3 h
100 99 2
x 3 − [x 3 + h3 + 3xh(x + h)]
[∵ f(x) = x n ⇒ f ′ (x) = nx n−1 ] = limh→0
(x + h)3 x 3 h
⇒ f ′ (x) = x 99 + x 98 + ⋯ + x + 1
−h3 − 3xh(x + h)
Putting x = 1, we get = limh→0
(x + h)3 x 3 h
f ′ (1) = ⏟
(1)99 + 198 + ⋯ + 1 + 1 −h[h2 + 3x(x + h)] −3
100 times = limh→0 = 4
(x + h)3 x 3 h x
=⏟
1 + 1 + 1…+ 1 + 1
d
100 times Alternatively, use (x n ) = nx n−1
dx
⇒ f ′ (1) = 100.
35. Ans (A)
Again, putting x = 0, we get f(0 + h) − f(0)
f ′ (0) = 0 + 0 + ⋯ + 0 + 1 f ′ (0) = limh→0
h
⇒ f ′ (0) = 1 … ( iii) 3−3 0
= limh→0 = limh→0 = 0
h h
From Eqs. (ii) and (iii), we get
f(3+h)−f(3)
f ′ (1) = 100f ′ (0) Similarly, f ′ (3) = limh→0
h

Hence, m = 100 3−3


= limh→0 =0
h
32. Ans (D)
36. Ans (B)
We have y = sin2 ⁡ x + cos 4 ⁡ x
If given function is 6x100 − x 55 + x.
dy
∴ = 2 sin x cos⁡ x + 4cos 3 x(−sin⁡ x) Then, the derivative of function is
dx
= sin⁡ 2x − 4 sin x cos⁡ x(cos 2 ⁡ x) 6 ⋅ 100 ⋅ x 99 − 55 ⋅ x 54 + 1
cos⁡ 2x + 1 or 600x 99 − 55x 54 + 1
= sin⁡ 2x − 2 sin 2x ( )
2 37. Ans (C)
= sin⁡ 2x − sin 2x cos⁡ 2x − sin⁡ 2x
We have, f(x) = 2x 2 + 3x − 5
−sin⁡ 4x
= − sin 2x cos⁡ 2x = f(−1 + h) − f(−1)
2 ∴ f ′ (−1) = limh→0
h
33. Ans (C)
2h2 − h
2 1 = limh→0 = limh→0 (2h − 1)
We have, y = x + sin⁡ x + h
x2
= 2(0) − 1 = −1

MATHEMATICS Page | 13A. 4


L LIMITS AND DERIVATIVES

d Differentiating y w.r.t. x, we get


Alternatively, use (x n ) = nx n−1
dx
dy
38. Ans (C) =2×1−0 =2
dx
x
Let y = II. We have, y = (5x 3 + 3x − 1)(x − 1)
1+tan⁡ x

Differentiating y w.r.t. x, we get Differentiating y w.r.t x, we get


d d dy d
dy (1 + tan⁡ x) dx (x) − x dx (1 + tan⁡ x) = (5x 3 + 3x − 1) (x − 1) +
= dx dx
dx (1 + tan⁡ x)2
d
(x − 1) (5x 3 + 3x − 1)
(1 + tan x)(1) − x(0 + sec 2 x) dx
=
(1 + tan⁡ x)2 = (5x 3 + 3x − 1)(1 − 0) + (x − 1)(5 × 3x 2 +
1 + tan⁡ x − xsec 2 ⁡ x 3 × 1 − 0)
=
(1 + tan⁡ x)2 = (5x 3 + 3x − 1) + (x − 1)(15x 2 + 3)
39. Ans (A) = 5x 3 + 3x − 1 + 15x 3 + 3x − 15x 2 − 3
I. Recall the trigonometric rule sin 2x = = 20x 3 − 15x 2 + 6x − 4
2sinx⁡cos⁡ x. 41. Ans (D)
df(x) d
Thus, = (2 sin xcos x) LHL = limx→0− f(x) = limx→0 x = 0
dx dx

d RHL = limx→0+ f(x)


=2
(sin x cos⁡ x)
dx = limx→0 1 + x = 1 + 0 = 1
= 2[(sin x)′ cos⁡ x + sin⁡ x(cos⁡ x)′ ] ⇒ LHL ≠ RHL
= 2[(cos x)cos⁡ x + sin⁡ x(−sin⁡ x)] 42. Ans (D)
2 2
= 2(cos x − sin x) |x| x
Note that limx→0 ⁡and⁡limx→0 ⁡are⁡does not
cos⁡ x x |x|
II. g(x) = cot⁡ x =
sin⁡ x
exist.
d d d cos⁡ x
⇒ (g(x)) = (cot⁡ x) = ( ) (x+1)(x−1) x−1
dx dx dx sin⁡ x Given lim = 2 ⋅ lim
x→1 |x−1| x→1 |x−1|
(cos⁡ x)′ (sin⁡ x) − (cos⁡ x)(sin⁡ x)′ x−1
= = 2 lim = does⁡not⁡exist
(sin⁡ x)2 (x−1)→0 |x − 1|

(−sin⁡ x)(sin⁡ x) − (cos⁡ x)(cos⁡ x) 43. Ans (D)


=
(sin⁡ x)2
limx→k− [x] = k − 1, limx→k+ [x] = k, k is an
−(sin2 ⁡ x + cos 2 ⁡ x)
= = −cosec 2 ⁡ x integer
sin2 ⁡ x
⇒ LHL ≠ RHL
Alternatively, this may be computed by noting that
1 ∴ limx→1 [x − 1] = lim(x−1)→0 [x − 1] = non
cot⁡ x = . Here, we use the fact that the
tan⁡ x
existent
derivative of tan⁡ x is sec 2 ⁡ x and also that the
44. Ans (D)
derivative of the constant function is 0 .
|x|
limx→0 = does not exist
d d d 1 x
(g(x)) = (cot⁡ x) = ( )
dx dx dx tan⁡ x √2sin2 ⁡(x−1)
=⁡lim
(1)′ (tan⁡ x) − (1)(tan⁡ x)′ x→1 x−1
= |sin⁡(x−1)|
(tan⁡ x)2 =√2 lim
(x−1)→0 x−1
(0)(tan⁡ x) − (sec⁡ x)2 − sec 2 x sin⁡(x−1) |x−1|
= = =√2 lim | | = does⁡not⁡exist
(tan⁡ x)2 tan2 x (x−1)→0 x−1 x−1

= −cosec 2 ⁡ x 45. Ans (D)


40. Ans (B) √1 − cos⁡ 2x √2|sin⁡ x|
3
limx→0 = ⁡limx→0
I. We have, y = 2x − √2x √2x
4
L.⁡H. L = −1⁡&⁡R.⁡H. L = 1
MATHEMATICS Page | 13A. 5
L LIMITS AND DERIVATIVES

Limit does not exist xsin⁡(x−1)


=limx→1+ = 1(1) = 1
x−1
46. Ans (A) xsin(x−[x])
=limx→1−
x−1
Since limit exist,LHL = RHL
xsin⁡(x−0)
⇒ ⁡⁡limx→α− f(x) = limx→α+ f(x) = limx→1 = indefinite (−∞)
x−1

limx→α 2x + b = limx→α x + d 55. Ans (C)


2α + b = α + d ⁡limx→1+ f(g(x)) = f(g(1+ )) = f(2+ ) = 22 + 2 = 6
α=d−b and limx→1− f(g(x)) = f(g(1− )) =
limx→α f(x) = limx→α x + d f(3 − 1− ) = f(2+ ) = 22 + 2 = 6
= α + d = 2d − b Hence, limx→1 f(g(x)) = 6
47. Ans (C) 56. Ans (C)

As x → k ⇒ [x − k] = −1 3x 2 + ax + a + 1
f(x) =
limx→3− [x − 3] + |x − 4| = −1 + |3 − 4| = 0 (x + 2)(x − 1)
48. Ans (B) As x → −2, Dr → 0
limx→k− [x] = k − 1 Hence, as x → −2, Nr → 0
limx→k− x − [x] = k − (k − 1) = 1 Therefore 12 − 2a + a + 1 = 0 or a = 13
49. Ans (D) 57. Ans (A)
The Q.E whose roots are α, β is f(−x) = −f(x)
x 2 − (α + β)x + αβ = 0 limx→0− f(x) = limx→0+ f(x)
2 limh→0 f(0 − h) = limh→0 f(0 + h)
α = lim− f(x) = 2 − 1 = 3
x→2

β = lim+ f(x) = 2(2) + 3 = 7 −limh→0 f(h) = limh→0 f(h)


x→2
⇒ 0 = 2limh→0 f(h)
x 2 − (3 + 7)x + 3.7 = 0
∴ limx→0 f(x) = 0
x 2 − 10x + 21 = 0
58. Ans (A)
50. Ans (C)
1
If x is⁡not⁡an⁡integer,⁡then [x] + [−x] = −1 −1 ⩽ sin⁡ ( ) ⩽ 1
x
x→1⇒x≠1 1
limx→0 x ⋅ sin⁡ ( ) = 0
and⁡very⁡close⁡to 1 x
lim 1 − x + [x − 1] + [1 − x] (a finite quantity lies between −1,1 )
x→1
59. Ans (C)
= lim 1 − x + [x − 1] + [−(x − 1)]
x→1
x → 0 ⇒ x ≠ 0 but very close to O
= 1 − 1 − 1 = −1
limx→0 g[f(x)] = limx→0 g(sin⁡ x)
51. Ans (C)
= limx→0 sin2 ⁡ x + 1 = 0 + 1 = 1
Limx→2+ f(x) = Limx→2+ (x − 3)
60. Ans (B)
= 2 − 3 = −1 π
∀x, −1 ⩽ sin⁡ ( ) ⩽ 1
52. Ans (A) x
π
⁡Limx→0 ⁡ f(x) = limx→0 (x or −x) = 0 limx→0 x 2 sin⁡ ( ) = 0
x
53. Ans (B)
(a finite quantity lies between −1,1) = 0
LHL: 1-1=0 and RHL: -1-1=-2
61. Ans (C)
⇒ LHL ≠ RHL
x < 2x < 3x
54. Ans (C)
f(x) < f(2x) < f(3x) [∵ f⁡is⁡increasing
xsin(x − [x])
limx→1+ f(2x) f(3x)
x−1 1< <
f(x) f(x)
MATHEMATICS Page | 13A. 6
L LIMITS AND DERIVATIVES

f(3x) 68. Ans (A)


limx→∞ =1
f(x) Apply L − H rule
f(2x)
⇒ limx→1 =1 log⁡(1 + x) + x 2 − x
f(x) limx→0
x2
∵ f(x) ⩽ g(x) ⩽ h(x) 1
+ 2x − 1
limx→a f(x) = ℓ = limx→a h(x) ⁡= lim 1 + x
x→0 2x
⇒ limx→a g(x) = ℓ 1 + 2x + 2x 2 − 1 − x
= lim
62. Ans (A) x→0 2x(1 + x)
As x → 0 ⇒ cos x → 1 ⇒ cos x < 1 x(1 + 2x) 1
= lim =
⇒ [cos⁡ x] → 0 x→0 2x(1 + x) 2
sin⁡[cos⁡ x] sin⁡ 0 69. Ans (B)
∴ limx→0 = =0
1 + [cos⁡ x] 1 + 0 sin⁡ x − x 0
Lt ( form )
63. Ans (D) x→0 x3 0
L⁡hospital⁡rule
2x − 3 2x 2 + 5x
limx→∞ < limx→∞ f(x) < limx→∞ cos⁡ x − 1
x x2 = ⁡
3
2−x
5
2+x
x→0 3x 2
⇒ limx→∞ < limx→∞ f(x) < limx→∞ −1 1−cos⁡ x 1 1 −1
1 1
= × Lt x2 = − × =
⇒ 2 < limx→∞ f(x) < 2 3 xt−0 3 2 6
70. Ans (A)
∴ limx→∞ f(x) = 2
sin⁡ x < x < tan⁡ x
64. Ans (A)
2
x sin⁡ x
ex −cos x >1⇒ <1
Apply L-Hospital rule limx→0 sin⁡ x x
x2
x2
100x 99sin⁡ x
2xe + sin⁡ x Lt [ ]+[ ] ⁡ = 100 + 98 = 198
= limx→0 x→0 sin⁡ x x
2x
71. Ans (B)
2 sin⁡ x 1 3
= limx→0 ex + =1+ = sin⁡ x < x < tan⁡ x
2x 2 2
65. Ans (C) sin⁡ x tan⁡ x
< 1⁡and⁡ >1
Apply L − Hospital rule x x
nsin⁡ x ntan⁡ x
ex − 1 1 Lt [ ] + ⁡Lt
x→0 [ ]= n−1+n
limx→0 = x→0 x x
2x 2
=2n-1
66. Ans (D)
log(1+x)
72. Ans (B)
We have limx→0 =1 and
x tan⁡ x ⋅ sin⁡ x
Lt >1
ax − 1 x→0 x2
limx→0 = log ae
x 100tan⁡ xsin⁡ x
Lt > 100
log⁡(1+x)
1 x→0 x2
Given lim x
x
= = log e3
x→0 3 −1 log3
e 100tan⁡ x ⋅ sin⁡ x
x ∴ Lt [ ] = 100
x→0 x2
67. Ans (B)
73. Ans (C)
log e ⁡(sin⁡ x)
Lt ⁡x 2 − 2x − 3 = (x + 1)(x − 3)
x→0 1
( )
x (x − 3)(x 2 − 2) 7
Apply L − Hospital rule lim =
x→3 (x + 1)(x − 3) 4
cot⁡ x
= Lt 74. Ans (C)
x→0 −1
x2 x m − am m m−n
limx→a = a
x x n − an n
= ⁡Lt 2 Lt→
x→0 − x cot x = ⁡x→0 (−x) ( ) = 0 = log1e
tan x
MATHEMATICS Page | 13A. 7
L LIMITS AND DERIVATIVES

x18 − 118 18 18−6 1


lim = ⋅1 =3 = = −2
x→1 x 6 − 16 6 −1 1
( − )
4 4
75. Ans (B)
81. Ans (C)
L- Hospital rule
x m − am m m−n
1 1 limx→a = a
× 3x 2 − (−3x 2 ) x n − an n
2√2 + x 3 2√2 − x 3
lim 11 11 11
x→0 3x 2 x2 −22 44
lim 7 2 2
1 1 7 = 7 ⋅2 =
3x 2 [ + ] x→2 7
x 2 − 22 2
2√2 + x 3 2√2 − x 3
= lim
x→0 3x 2 82. Ans (A)
1 1 1 1
= + = cos⁡(√x+h) −0
2√x+h
2√2 2√2 √2 L- Hospital rule limh→0
1
76. Ans (A) cos⁡ √x
=
Apply L-Hospital rule 2 √x
√1 + x − √1 − x Alternatively, from 1st principle of derivative,
limx→0
x cos⁡ √x
1 1 f(x) = sin⁡ √x ⇒ f′(x) =
+ 2 √x
2√1 + x 2√1 − x
= limx→0 83. Ans (A)
1
1 1 ⁡x → tan−1 ⁡ 3 ⇒ tan⁡ x → 3, put tanx = a
= + =1
2 2 a2 − 2a − 3
= Lima→3
77. Ans (A) a2 − 4a + 3
L − H rule (a − 3)(a + 1)
= lim =2
1 a→3 (a − 3)(a − 1)
⁡−⁡0 1
=lim 2√1+x⁡ = 84. Ans (B)
x→0 1 2
1 1
√1+x−1 (1+x)2 −12 L-Hospital rule
Alternatively, lim ⁡= lim
x→0 x (1+x)→1 (1+x)−1 π
cos⁡( 3 −x)(−1)
limx→π
1 1 3 −2sin⁡ x
= ⋅1=
2 2 −1 1
= =
78. Ans (A) √3 √3
−2 ×
L- Hospital rule 2
2x (−2x)
85. Ans (D)
⁡−⁡
2√(1+x2 ) 2√(1−x2 ) 1−cos⁡ ax a2
lim =0+0=0 We have, limx→0 =
x→0 1 x2 2
1−cos nx
79. Ans (C) x2 n2
Thus, given limit: lim 1−cos⁡ mx =
x→0 m2
x2
L- Hospital rule
1 1 86. Ans (C)
+
2√9 + x 2√9 − x tan −1 x sin⁡ x
lim limx→0 ( ⁡) ⁡ = limx→0 (⁡ )=1
x→0 1 x x
1 1 2 1
= + = = tan−1 ⁡(7x)
2√9 2√9 6 3 × 7x 7
lim 7x =
80. Ans (B) x→0 sin⁡ 4x 4
× 4x
4x
L- Hospital rule 87. Ans (C)
1
limx→0 1 − cos⁡ ax a2
−1 1 ⁡limx→0 =
− x2 2
2√4 − x 2√4 + x

MATHEMATICS Page | 13A. 8


L LIMITS AND DERIVATIVES

1 − cos⁡ 7x 72 1 − cos⁡ θ 1
2 49 θ2 1
= lim x = 22 = lim =2=
x→0 1 − cos⁡ 9x 9 81 θ→0 sin⁡ θ 1 2
θ⁡⁡ 2
x2 2 θ
88. Ans (C) 94. Ans (B)
1 − cos⁡ x 1 sin⁡ x From 1st principle of derivative limx→1
g(x)−g(1)
=
limx→0 2
= , limx→0 =1 x−1
x 2 x
1 − cos⁡ 5x g ′ (1)
× 25x 2 1
(5x)2 2x 1
lim = 2 ×0 Where g ′ (x) =
2√25−x2
⇒ g ′ (1) =
√24
x→0 sin⁡ 4x 1
× 4x
4x 95. Ans (C)
89. Ans (D)
L- Hospital rule,
1−cos⁡ ax a2
We have, limx→0
x2
=
2
f(5) − 5f ′ (x)
⁡lim
1−cos⁡ 7x (1−cos⁡ 2x) x→5 1

x2 x2
Given limit: lim sin⁡ 3xTan⁡ 5x = f(5) − 5f ′ (5)
x→0 ×3×5
3x 5x
= 7 − 5(7) = −28
[∵ subtracted⁡and⁡added⁡1
96. Ans (B)
72 22

= 2 2 = 45 = 3 10x − 2x − 5x + 1 = (5x − 1)(2x − 1)
15 30 2 ax −1
90. Ans (B) We have, Lt x→0 ⁡ = log ae
x

1−cos⁡ ax a2 (5x − 1)(2x − 1)


We have,⁡limx→0 = lim =
x2 2 x→0 x tan x
1 − cos⁡ 7x (1 − cos⁡ 9x) 5x − 1 2x − 1
lim − ⋅⁡
x→0 x2 x2 = lim x tan⁡ x x
x→0
72 92 49 − 81 x
= − = = −16
2 2 2 = log⁡ 5 ⋅ log⁡ 2
91. Ans (A)
97. Ans (C)
Apply,a3 − b3 = (a − b)(a2 + a ⋅ b + b2 )
f ′ (x) = 36x 8 − 24x 7 + 14x 6 + 3x 2 + 2x
1 − cos⁡ ax a2
lim = After L-Hospital rule
x→0 x2 2
cos2x − cos3x −f ′ (1 − h) − 0 −f ′ (1)
lim (cos 2 2x + cos2x. cos3x lim =
x→0 x2
x→0 3h2 + 3 3
−(36 − 24 + 14 + 3 + 2) −31
+ cos 2 3x) = =
3 3
1 − cos⁡ 3x (1 − cos⁡ 2x)
= [lim − ] [1 + 1 + 1] 98. Ans (B)
x→0 x2 x2
ax − 1
32 22 5 15 ⁡limx→0 = log ae
= ( − )3 = × 3 = x
2 2 2 2
3x − 1 − 2x + 1
92. Ans (C) = lim
x→0 x
1 − sin⁡ x 3x − 1 2x − 1
limx→π (sec⁡ x − tan⁡ x) = ⁡ limx→π = lim −
2 2 cos⁡ x x→0 x x
Apply L - Hospital rule 3
= log⁡ 3 − log⁡ 2 = log⁡
−cos⁡ x 2
= limx→π =0
2 −sin⁡ x 99. Ans (D)
93. Ans (A) sin x cos x tan x
1 − cos⁡ θ 1 f(x) = | x 3 x2 x |
limθ→0 = 2x 1 1
θ2 2

MATHEMATICS Page | 13A. 9


L LIMITS AND DERIVATIVES

sin x sin2 ⁡ α − sin2 ⁡ β = sin⁡(α + β)sin⁡(α − β)


f(x) cos x tan x
⇒ 2 =| x | sin⁡(α + β) sin⁡(α − β)
x x x 1 lim .
2 1 1 (α−β)→0 α+β α−β
C1 R2 sin⁡ 2β
[∵ C1 → ⁡and⁡R 2 → ⁡⁡ = ×1
x x 2β
f(x) 1 1 0
Lt 2 = |0 0 1| = 1
x→0 x
2 1 1
100. Ans (B) 106. Ans (C)
x2 x2 sin⁡ 2n θ
∫ sec 2 ⁡ tdt ∫ sec 2 ⁡ tdt cos θ ⋅ cos 2θ cos 4θ … … cos 2n−1 θ =
⁡ Lt 0 = Lt 0 2n sin⁡ θ
x→0 xsin⁡ x x→0 x2 x x x
Tan⁡(x 2 ) lim cos⁡
n
cos⁡ n−1 … … cos⁡
= Lt x→0 ⁡ =1
n→∞ 2 2 2
x2 sin⁡ x
lim
n→∞ 2n sin⁡ x
2n
sin⁡ x
101. Ans (B) x λ
= lim x ⁡⁡⁡⁡⁡(as⁡⁡n → ∞, n → 0)
n→∞ sin⁡ n 2
x2
cos(t 2 )dt
∫0 ( x2 )
= Lt 2n
x→0 sin x
x2 ⁡ ∙
x sin⁡ x
4 ). =
cos⁡(x 2x x
= Lt =1
x→0 2x 107. Ans (C)
102. Ans (C) eax − ex − x
⁡Lt ⁡ =b
L- Hospital rule x→0 x2
1 using L- hospital rule
f ′ (x)
2√f(x) a ⋅ eax − ex − 1
lim Lt x→0 ⁡ =b
x→9 1 2x
2√ x
a−1−1 =0
f 1 (9)
= × √9 ⇒a=2
√f(9)
a2 ⋅ eax − ex
4 Lt =b
= ×3=6 x→0 2
√4 a2 − 1
103. Ans (A) =b
2
f(2)−2f1 (x) 3
L − Hospitalrule limx→2 b=
1
2
1
= f(2) − 2f (2) = 4 − 2(1) = 2 108. Ans (C)
104. Ans (B)
Lt 1 − cos2x (3 + cosx)
g(x) − g(a) ( )
x→0 x2 tan4x
limx→a = g1 (a) (
x
)
x−a
g(x) − g(4) 2.4
limx→4 = g1 (4) = =2
x−4 4
−1 × 2x 109. Ans (D)
g ′ (x) = 3
2(9 + x 2 )2 f(1 + h) f ′ (1 + h)
⁡limh→0 = limh→0
h 1
−4 −4
g ′ (4) = 3 = ⇒ 5 = f ′ (1)
125
(9 + 16)2
110. Ans (B)
105. Ans (D)
Here, the doubtful points are 1,2
⁡α → β ⇒ α − β → 0⁡⁡& Apply
MATHEMATICS Page | 13A. 10
L LIMITS AND DERIVATIVES

clearly f is continuous at x=1,2 −2x,⁡⁡⁡⁡⁡⁡⁡⁡⁡⁡⁡⁡ for x ⩽ 1


f(x) = {2,⁡⁡⁡⁡⁡⁡⁡⁡ − 1 ⩽ x ⩽ −1
⇒ f is⁡continuous⁡every⁡where 2x,⁡⁡⁡⁡⁡⁡⁡⁡⁡⁡⁡⁡⁡⁡⁡ for x ⩾ 1
But f ′ (1− ) = 1, f ′ (1+ ) = −1 f is differentiable in (−1,1)
⇒ f is⁡not⁡differentiable⁡at x = 1 116. Ans (A)
′ (2− ) ′ (2+ )
f = −1, f = 3 − 4 = −1 ⁡ ∵ g(x) is continuous at x = 3
f⁡is⁡diff⁡at x = 2
∴ k × √3 + 1 = 3m + 2
111. Ans (C)
2k = 3m + 2 … . . (1)
x
, x>0 for g(x) to be differentiable
f(x) = {1 + x
x
, x⩽0 g ′ (3− ) = g ′ (3+ )
1−x k
⇒ = m ⇒ k = 4m …... (2)
1 4
, x>0
(1 + x)2 from (1) and (2),
f ′ (x) =
1
, x⩽0 8m = 3m + 2 and k = 4 × (2/5) = 8/5
{(1 − x)2
⇒ m = 2/5k + m = 10/5 = 2
f ′ (0+ ) = f ′ (0− ) = 1
117. Ans (B)
∴ f is differentiable ∀x ∈ R
f(x) = x − [x] − cos⁡ x
112. Ans (B)
π+ π−
f(x) = |x − 1| + |x + 1| for x= (or) ⇒ [x] = 1
2 2

−x + 1 − x − 1 = −2x,⁡⁡⁡⁡⁡⁡⁡⁡⁡x ⩽ −1 ⇒ f is continuous
= { 2, ⁡ ⁡⁡⁡⁡⁡⁡⁡⁡⁡⁡⁡⁡⁡⁡⁡⁡⁡⁡⁡⁡⁡⁡⁡⁡⁡⁡⁡⁡⁡⁡⁡⁡⁡⁡⁡⁡⁡⁡ − 1 < x ⩽ 1
Now, f ′ (x) = 1 + sin⁡ x
2x, ⁡⁡⁡⁡⁡⁡⁡⁡⁡⁡⁡⁡⁡⁡⁡⁡⁡⁡⁡⁡⁡⁡⁡⁡⁡⁡⁡⁡⁡⁡⁡⁡⁡⁡⁡⁡⁡⁡⁡⁡⁡⁡⁡⁡⁡⁡⁡⁡⁡⁡⁡x > 1
π
Here, f(−1− ) = 2, f(−1+ ) = 2, f(−1) = 2 ⇒ f′ ( ) = 1 + 1 = 2
2
f(+1− ) = 2, f(1+ ) = 2, f(1) = 2 118. Ans (C)
so⁡f is⁡continuous g(3 − h) − g(3)
′ (−1− ) ′ (−1+ )
g ′ (3− ) = limh→0
f = −2, f =0 −h
a√4−h−(3b+2)
f ′ (1− ) = 0, f 1 (1+ ) = 2 = limh→0 ⁡…(1)
−h
113. Ans (B) for existence of limit, limh→0 = 0
f is⁡differentiable⁡at x = x0 ∴ 2a − 3b = 2 … . . . (2)
⇒ f is⁡continuous⁡at x = x0 b(3+h)+2−(3b+2)
Now, g ′ (3+ ) = limh→0 = b … (3)
h
f(x0− ) = f(x0+ )and f ′ (x0− ) = f ′ (x0+ )
Substituting 3b + 2 = 2a in equation (1), we get
x02 = ax0 + b and 2x0 = a
a√4 − h − 2a
b = −x02 g ′ (3− ) = limh→0
−h
114. Ans (A)
(4 − h) − 4 a
f(3 + h2 ) − f(3 − h2 ) = limh→0 ( )=
(−h)(√4 − h + 2) 4
Lt
h→0 2h2
Hence,g ′ (3− ) = g ′ (3+ )
f(3 + h2 ) − f(3) + f(3) − f(3 − h2 ) a
= Lt = b ⇒ a = 4b … (4)
h→0 2h2 4

1 f(3 + h2 ) − f(3) 1 f(3 − h2 ) − f(3) From equation (2) and (4)


= Lt 2
+ Lt
2 h→0 h 2 h→0 −h2 2 8
8b − 3b = 2 ⇒ b = and a =
1 1 5 5
= f ′ (3) + f ′ (3) = f ′ (3) = 5
2 2 or a + b = 2
115. Ans (C) 119. Ans (D)
L - Hospital rule

MATHEMATICS Page | 13A. 11


L LIMITS AND DERIVATIVES

1 1
1 −(2x − 1) cos x − 2 sin x ,⁡⁡⁡⁡x < ⁡
2√x + h ′ 2
limh→0 = f (x) = {
1 2√ x 1
(2x − 1) cos x + 2 sin x ,⁡⁡⁡⁡⁡x ⩾
Alternatively, from 1st principle of derivative, 2
1
1 L. H. D ≠ R. H. D at x = ⇒ ⁡ f ′ (x) does not exist
2
f(x) = √x ⇒ f′(x) =
2 √x Alternatively,
120. Ans (D) |f(x)|⁡is⁡not⁡differentiable, whenever⁡f(x) = 0
⁡f(x + y) = f(x) ⋅ f(y) 125. Ans (A)
put x = y = 0 x
x ,x > 0
f(0) = [f(0)]2 ⇒ f(0) = 1 f(x) = = {1 +
x
x
1 + |x| ,x ⩽ 0
f(x + h) − f(x) 1−x
f ′ (x) = Lt
h→0 h f(0+ ) = f(0− ) = f(0)
f(x) ⋅ f(h) − f(x)
Lt f is continuous every where
h→0 h
f(x)−f(0)
f(h) − 1 f ′ (0− ) = Lt−
= f(x) Lt x→0 x−0
h→0 h x
f(h) − f(0) −0
= f(x)L⁡ t = Lt 1 − x = 1f ′ (0+ )
h→0 h−0 x→0 x
= f(x) ⋅ f ′ (0) f(x) − f(0)
= Lt+
x→0 x−0
f ′ (5) = f(5) ⋅ f ′ (0) = 3 × 2 = 6 x
−0 1
= Lt 1 + x = =1
x→0 x 1
121. Ans (D)
f ′ (0+ ) = f ′ (0− )
−1
⁡G′ (x) = (−2x) ∴ f is differentiable every where.
2√25 − x 2
G′ (x) − 0 126. Ans (D)
limx→1 = G′ (1) f(x) − f(5)
1
Lt = f 1 (5)
1 1 x→5 x−5
= =
√25 − 1 2√6 = 10a + b
122. Ans (C) 127. Ans (B)
f(x) − f(2) f(x) − f(y)
⁡f ′ (2+ ) = limx→2+ | |≤ x−y
x−2 x−y
3x + 4 − 10 f(x) − f(y)
= limx→2 |Lt x→y ⁡ |≤0
x−2 x−y
3(x − 2) ⇒ |f ′ (y)| ≤ 0 ⇒ f ′ (y) = 0
= lim =3
x→2 x − 2
⇒ f(x) is⁡constant⁡with⁡f(0) = 0
123. Ans (B)
⇒ f(1) = 0
(x − 3) cos x ,⁡⁡⁡⁡⁡⁡⁡⁡⁡⁡⁡⁡x ⩾ 3
f(x) = { 128. Ans (B)
−(x − 3) cos x , x<3
−(x − 3) sin x + cos x , x⩾3 d
f ′ (x) = { cos x 0 =
(x − 3) sin x − cos x ,⁡⁡⁡⁡⁡⁡⁡⁡⁡⁡⁡⁡x < 3 dx
L. H. D ≠ R. H. D at x = 3, ⇒ ⁡ f ′ (x) does not exist d πx πx π
⁡ cos⁡ ( ) = −sin⁡ ( )⋅
dx 180 180 180
Alternatively,
−π
|f(x)|⁡is⁡not⁡differentiable, whenever⁡f(x) = 0 = (sin⁡ x 0 )
180
124. Ans (B) 129. Ans (C)
log e ⁡ x
y = log x5 =
log e ⁡ 5

MATHEMATICS Page | 13A. 12


L LIMITS AND DERIVATIVES

dy 1 1 f(x) = 1 + x 3
= ×
dx log e ⁡ 5 x f ′ (x) = 3x 2
1
=
xlog e ⁡ 5
137. Ans (A)
g1 (x) = x199 + x198 + x197 + ⋯ … + +x + 1
130. Ans (A)
g1 (0) = 1
d af(x) + b (ad − bc) 1
( )= . f (x) 138. Ans (D)
dx cf(x) + d (cf(x) + d)2
d 3x − 5 9 + 10 f ′ (x) = ex g ′ (x) + g(x) ⋅ ex
( )=
dx 2x + 3 (2x + 3)2 f ′ (0) = g ′ (0) + g(0)
131. Ans (B) =4+2=6
Given series is a geometrical series 139. Ans (A)
x
So, f(x) = f(x) = (ax + b)sin⁡ x + (cx + d)cos⁡ x
1+x
1−0 1 f ′ (x) = (ax + b) cos x + (sin x)a
⇒ f ′ (x) = 2
=
(1 + x) (1 + x)2 +(cx + d)(−sin⁡ x) + (cos⁡ x)c
132. Ans (A) = x ⋅ cos⁡ x
du dv b = c = 0, a = d = 1
d u v −u
⁡ ( ) = ( dx 2 dx ) 140. Ans (D)
dx v v
1−cos(x−1)
Lt
f(x) = cosx ⋅ cos2x ⋅ cos4x ⋅ cos8x ⋅ cos16x x 3 + 2x 2 + x + 1 x→1 (x−1)2
Lt ( )
sin⁡ 25 x sin⁡ 32x
x→1 x 2 + 2x + 3
= = [ ∵ using⁡T. S. R
25 ⋅sin⁡ x 25 ⋅sin⁡ x 1−cos(x−1) 1
5 Lt 5 2
(x−1)2
1 sin x⁡⋅cos 32x⋅32−sin⁡ 32x⋅cos⁡ x = ( )x→1 =( ) [ ∵ using⁡T. S. R
f ′ (x) = [ ] 6 6
25 sin2 ⁡ x

1 141. Ans (A)


(32)
π 1 As x → 4+, [x] = 4
f′ ( ) = [ √2 ] = √2
4 32 1
2 (x − 3)(x − 4)
lim+ = lim+ (x − 3) = 1
x→4 x−4 x→4
133. Ans (B)
142. Ans (B)
x
1 − cos⁡ x 2sin2 ⁡ x
= 2 = tan⁡ √3x − a − √x + a
sin⁡ x x x 2 lim
2sin⁡ cos⁡ x→a x−a
2 2
d 1 − cos⁡ x d x 1 x 3 1 3 1
( ) = (tan⁡ ) = sec 2 ⁡ = lim − = −
dx sin⁡ x dx 2 2 2 x→a 2√3x − a 2√x + a 2√2a 2√2a
134. Ans (B) 2 1
= =
f(x) = |x 2 − 5x + 6| 2√2a √2a

= |(x − 2)(x − 3)|


= −(x 2 − 5x + 6), 2 < x < 3 143. Ans (B)
x n − 2n
= 5x − x 2 − 6,2 < x < 3 lim = n. 2n−1
x→2 x − 2
f 1 (x) = 5 − 2x, ⁡2 < x < 3
⇒ n. 2n−1 = 80 ⇒ n = 5.
135. Ans (B)
144. Ans (A)

1
f (x) = 1 + 2 (2x − 3)(√x − 1) × (√x + 1)
x lim
x→1 (x − 1)(2x + 3) × (√x + 1)
f ′ (−1) = 1 + 1 = 2
−1 −1
136. Ans (B) = =
5.2 10

f(4) = 65 = 1 + 43 145. Ans (B)


MATHEMATICS Page | 13A. 13
L LIMITS AND DERIVATIVES

f ′ (x) f ′ (9) 1 − cos 4θ


× 16θ2
√f(x) − 3 2√f(x) √f(9) (4θ)2
lim = lim = Given⁡limit:⁡⁡lim
1 1 θ→0 1 − cos 6θ
x→9 √x − 3 x→9 × 36θ2
(6θ)2
2√ x √9
4 1
× 16 4
= ×3=4
3 =2 =
1
× 36 9
146. Ans (C) 2
sin x 151. Ans (C)
Given, Limx→π
x−π cosec x−cot x
Given Limx→0
sin⁡(π − x) x
= Limx→π ⁡ = −1 1 cos⁡ x
−(π − x) ⁡−⁡
= Limx→0 ⁡ sin⁡ x sin⁡ x
147. Ans (A) x
x2 cos x x
Given Limx→0 1 − cos⁡ x 2sin2
1−cos x = Limx→0 ⁡ = 2
x ∙ sin⁡ x x x
x 2 cos⁡ x x ⋅ 2sin⁡ cos⁡
2 2
= Limx→0 ⁡ x
2sin2 ⁡ x x
2 sin⁡ tan
= Limx→0 ⁡ 2 2
x2 x = Limx→0 ⁡ x
× 4cos⁡ x x ∙ cos⁡
2
= Limx→0 ⁡ 4 x x
2sin2 ⁡ tan⁡ 1 1
2 = Limx→0 ⁡ 2
x =2×1=2
x 2 2×
( ) ⋅ 2cos⁡ x 2
= Limx→0⇒x→0 ⁡ 2 x
2 sin2 ⁡
2
2
x
152. Ans (C)
= Limx→0 ( 2 x ) ⋅ 2 cos x = 2 cos 0
2 sin Given Limx→0 ⁡
sin⁡ x
2 √x+1−√1−x
=2×1=2 sin⁡ x[√x+1+√1−x]
= Limx→0 ⁡
x (√x+1−√1−x)(√x+1+√1−x)
[∵ ⁡Limx→0 ⁡ = 1]
sin⁡ x sin⁡ x[√x + 1 + √1 − x]
= Limx→0 ⁡
148. Ans (A) x+1−1+x
(1+x)n −1 sin⁡ x[√x + 1 + √1 − x]
Given Limx→0 = Limx→0 ⁡
x
2x
(1 + x)n − (1)n
= Limx→0 ⁡ 1 sin⁡ x
(1 + x) − (1) = ⋅ Limx→0 ⁡ [√x + 1 + √1 − x]
2 x
(1 + x)n − (1)n 1
= Lim1+x→1 = × 1 × [√0 + 1 + √1 − 0]
(1 + x) − (1) 2
= n(1)n−1 = n 1
= ×1×2=1
149. Ans (B) 2
xm −1 153. Ans (D)
Given Limx→1
xn −1 sec2 x−2
m m Given, Limx→π
x − (1) 4 tan x−1

= Limx→1 ⁡ nx − 1 n 1 + tan2 ⁡ x − 2
x − (1) = Limx→π ⁡
x−1 4 tan⁡ x − 1
m(1)m−1
m tan2 ⁡ x − 1
= = ⁡ = Limx→π ⁡
n(1)n−1 n 4 tan⁡ x − 1

150. Ans (A) (tan⁡ x + 1)(tan⁡ x − 1)


= Limx→π ⁡
1−cos x 1 4 (tan⁡ x − 1)
We have lim =
x→0 x2 2 = Limx→π ⁡(tan⁡ x + 1)
4

MATHEMATICS Page | 13A. 14


L LIMITS AND DERIVATIVES
π and Limx→2+ f(x) = limx→2+ (2x + 3)
= tan⁡ + 1 = 1 + 1 = 2
4
= 2(2) + 3
154. Ans (B)
Therefore, the quadratic equation whose roots
(√x−1)(2x−3)
Given Limx→1 ⁡ are 3 and 7 is
2x2 +3x−2x−3

(√x − 1)(2x − 3) x 2 − (3 + 7)x + 3 × 7 = 0


= Limx→1 ⁡
x(2x + 3) − 1(2x + 3) i.e., x 2 − 10x + 21 = 0.
(√x − 1)(2x − 3) 158. Ans (B)
= Limx→1 ⁡
(x − 1)(2x + 3) tan 2x−x
Given Limx→0
3x−sin x
(√x − 1)(√x + 1)(2x − 3)
= Limx→1 ⁡ x[
tan⁡ 2x
−1]
(x − 1)(√x + 1)(2x + 3) =Lim x→0 ⁡ x
sin⁡ x
x[3− ]
x
(x − 1)(2x − 3)
= Limx→1 ⁡ tan⁡ 2x
[ 2x ×2−1]
(x − 1)(√x + 1)(2x + 3) 1.2−1 1
=Limx→0 sin⁡ x = ⁡=
[3− ] 3−1 2
2x − 3 x
= Limx→1 ⁡
(√x + 1)(2x + 3) 159. Ans (B)
2(1) − 3 x, x ∉ z
= Given f(x) = x − [x] = {
0, x ∈ z
(√1 + 1)(2 × 1 + 3)
⇒ f ′ (x) = 1, when⁡x ∉ z⁡
−1 −1
= = 1
2 × 5 10 ∴ ⁡f ′ ( ) = 1
2
155. Ans (D)
sin⁡[x]
160. Ans (D)
, [x] ≠ 0 1
Given, f(x) = { [x]
Given that y = √x +
0, [x] = 0 √x

sin[x] sin(−1) dy 1 1
LHL = Limx→0− = ⁡= − 3/2
[x] −1 dx 2√ x 2x
dy 1 1
[∵ ⁡as x → 0− , [x] → −1 ⇒( ) = − =0
dx at x=1 2 2
sin[x] sin[0 + h]
RHL = Limx→0+ = Limh→0 161. Ans (A)
[x] [0 + h]
x−4
sin⁡[h] Given that f(x) =
2√x
= Limh→0 ⁡ =1
[h] 1
x ⋅ 1 − (x − 4) ⋅ ]
LHL ≠ RHL 1 √ 2√ x
∴ ⁡f ′ (x) ⁡ = [ ]
2 x
So, the limit does not exist.
156. Ans (C) 1 2x − x + 4 1 x+4
= [ ]= [ ]
Given Limx→0 ⁡
|sin⁡ x| 2 2 √x ⋅ x 2 2(x)3/2
x
1 1+4 5
− sin x ∴ ⁡f ′ (1) = [ ]=
LHL⁡ = Limx→0− 2 2×1 4
x
sin⁡ x 162. Ans (A)
= −1⁡ [∵ ⁡Limx→0 ⁡ = 1] 1
x 1+ 2
x x2 +1
Given y = 1 ⇒y=
sin⁡ x 1− 2 x2 −1
x
RHL⁡ = Limx→0+ ⁡ =1
x dy (x 2 − 1) ⋅ 2x − (x 2 + 1) ⋅ 2x
LHL ≠ RHL, so⁡the⁡limit⁡does⁡not⁡exist. ∴⁡ =
dx (x 2 − 1)2
157. Ans (D) 2x(x 2 − 1 − x 2 − 1) 2x(−2)
= = 2
x 2 − 1, 0 < x < 2 (x 2 − 1)2 (x − 1)2
Given f(x) = {
2x + 3, 2 ≤ x < 3 −4x
2 =
∴ ⁡ Limx→2− f(x) = Limx→2 − (x − 1) (x 2− 1)2
=4−1=3 163. Ans (A)
MATHEMATICS Page | 13A. 15
L LIMITS AND DERIVATIVES

Given y =
sin⁡(x+9) ⇒ Lt f(x) − 2 = π × Lt x 2 − 1 = 0
cos⁡ x x→1 x→1
dy
=
cos⁡ x⋅cos⁡(x+9)−sin⁡(x+9)(−sin⁡ x) ⇒ Lt f(x) = 2
dx cos2 ⁡ x x→1

cos⁡ xcos⁡(x + 9) + sin⁡ xsin⁡(x + 9) 169. Ans (C)


=
cos 2 ⁡ x Lt
1−cos x
=
1
[∵ using T.S.R
x→0 x2 2
cos⁡(x + 9 − x) cos⁡ 9
= = 170. Ans (A)
cos 2 ⁡ x cos 2 ⁡ x
dy cos⁡ 9 1 − cos⁡ x 1
∴( ) = = cos⁡ 9 lim =
dx at x cos2 ⁡ 0 x→0 x2 2
164. Ans (B) 1 − cos⁡ 4θ
× 16θ2
x2 x100
(4θ)2
Given f(x) = 1 + x + + ⋯+ lim
θ→0 1 − cos⁡ 6θ
2 100 × 36θ2
99 (6θ)2
2x 100x
f ′ (x) ⁡ = 1 + + ⋯+ 1
2 100 × 16 4
=2 =
∴ ⁡f ′ (1) = 1 + 1 + 1 + ⋯ + 1(100 times ) 1
× 36 9
2
= 100
171. Ans (D)
165. Ans (C)
−x
xn −an L.H.L : Lt− = −1
Given f(x) = x→0 x
x−a
x
(x − a)(n ⋅ x n−1 ) − (x n − an ) ⋅ 1 R.H.L : Lt+ = 1
x→0 x
f ′ (x) =
(x − a)2
⇒ L.H.L≠R.H.L
(a − a)(n ⋅ an−1 ) − (an − an )
∴ ⁡f ′ (a) = 172. Ans (C)
(a − a)2
Given ∑nr=1 (2r − 1) = x ⇒x =n2
0
So ⁡f ′ (a) = = does⁡not⁡exist 13 23 33 n3
0 Given sum: limn→∞ [ 2 + + + ⋯+ ]
x x2 x2 x2
166. Ans (A) n(n+1) 2
( ) 1
2
Given, f(x) = x 100
+x 99
+⋯+x+1 = limn→∞ 4 =
n 4

∴ ⁡f ′ (x) = 100x 99 + 99 ⋅ x 98 + ⋯ + 1 173. Ans (C)


So, ⁡f ′ (1) = 100 + 99 + 98 + ⋯ + 1 f ′ (x) = x 99 + x 98 + ⋯ … + x + 1
100 × 101 f ′ (0) = 1
=
2 174. Ans (D)
= 50 × 101 = 5050
y = f(x 2 + 2), f ′ (3) = 5
167. Ans (D)
dy
Given that = f ′ (x 2 + 2). 2x
dx
f(x) = 1 − x + x 2 − x 3 + ⋯ − x 99 + x100 Put x=1,
dy
= f ′ (3). 2 = 10
dx
f ′ (x) =-1+2x-3x 2 …-99x 98 +100x 99
175. Ans (A)
∴ ⁡f ′ (1) ⁡ = −1 + 2 − 3 + ⋯ − 99 + 100
g1 (x) = x199 + x198 + x197 + ⋯ + x + 1
=(-1-3-5⋯-99)+(2+4+6+⋯+100)
g1 (0) = 1
50 50
= [2×-1+(50-1)(-2)]+ [2×2+(50-1)2] 176. Ans (B)
2 2

= 25[−2 − 98] + 25[4 + 98] 1


f ′ (x) = 1 +
= 25 × −100 + 25 × 102 x2
f ′ (−1) = 1 + 1 = 2
= 25[−100 + 102] = 25 × 2 = 50
177. Ans (D)
168. Ans (B)
f(x)−2 We have,
Given limx→1 =π
x2 −1 tan⁡ x 0
limx→0 ⁡ ( form )
√2x+4−2 0
MATHEMATICS Page | 13A. 16
L LIMITS AND DERIVATIVES

Applying L' Hospital's Rule, we get After rationalisation,


2
sec x 3 + y3 − 3 1
limx→0 limy→0 =
1 2 y(√3 + y 3 + √3) 2√3
×
2 √2x + 4
184. Ans (C)
= limx→0 (√2x + 4 ⋅ sec 2 ⁡ x)
⁡Nr = cos⁡(2 + x) + cos⁡(2 − x)
=2⋅1=2
= 2cos⁡ 2 for x = 0
178. Ans (A)
185. Ans (C)
e1/x −1
R.H.L. = limx→0+ 186. Ans (C)
e1/x +1
1/h −1/h
e −1 1−e
= limh→0 1/h
= limh→0
e +1 1 + e−1/h MOCK TEST SOLUTIONS:
1−0
= =1 1. Ans (B)
1+0
Given limit
1.H.L.
(2x − 3)(√x − 1)
e1/x − 1 e−1/h − 1 lim
= limx→0− 1/x
= limh→0 −1/h x→1 (√x − 1)(√x + 1)(2x + 3)
e +1 e +1
0−1 (−1) −1
= = −1 = =
0+1 2(5) 10
179. Ans (B) 2. Ans (C)
Statement 1: 1 2 1 2
Lt (1 + sin x)−3 cos x + (1 − sin x)−3 cos x
x→0 3 3
ax 2 + bx + c a + b + c
limx→1 = =1 1 1 2
cx 2 + bx + a a + b + c = + =
3 3 3
Statement 2:
3. Ans (A)
1 1
+ 2+x
limx→−2 x 2 = limx→−2 lim
√x 2 − 1 + √x − 1
= lim ⁡1 +
√x − 1
x+2 2x(x + 2) x→1 √x 2 −1 x→1 √x 2 − 1
1 1 1 1
= =− = lim ⁡1 + =1+
2(−2) 4 x→1 √x+1 √2

4. Ans (D)
180. Ans (A) Using L − Hospital rule
2
f(x) = 4cos 3 ⁡ x − 3cos⁡ x = cos⁡ 3x Given limit:
1 1
− (8 + 3x)−3 = 0
2√4+x 3
Thus, limx→π cos⁡ 3x = cos⁡ 3π
5. Ans (B)
= −1
On rationalizing given limit
181. Ans (A)
x3 1 1 1
a+b+c = lim [sin−1(x3)] . =1. =
statement 1 is true[i. e. , = 1] x→0 1+√1−x2 2 2
a+b+c
6. Ans (C)
Statement 2 is false
1 1 Since given limit is finite and as x → 0,
lim x+2 1
Because is − denominator → 0
x → −2 x+2 4

182. Ans (B) ∴ numerator also → 0


2 ⇒1+a+b=0
α = limx→2− f(x) = limx→2 x − 1 = 3
β = limx→2+ f(x) = limx→2 2x + 3 = 7 ⇒ option (C) satisfies this condition
2 7. Ans (D)
x − (α + β)x + αβ = 0
x 2 − 10x + 21 = 0 (∑200 k
K=1 x )−200
 Given Limit Lim
x→1 x−1
183. Ans (A)

MATHEMATICS Page | 13A. 17


L LIMITS AND DERIVATIVES

(x + x 2 +. . . . . +x 200 ) − 200
= lim sin θ 1 − cos θ
x→1 x−1 = lim ( )( )
θ→0 cos θ θ2
θ

tan θ 2 sin 2
= lim .
θ→0 θ θ2
By L − Hospital rule 15. Ans (D)
Given limit is
(1 + 2x + 3x 2 +. . . . . +200. x190 ) 200 × 201
= lim = 2 sin x (cos x − 1) + 2(1 − cos 2 x)
x→1 1 2 =4
cos x (1 − cos x)
= 20100
16. Ans (B)
8. Ans (D)
4 sin3 x 4
sin x lim 2
= =1
lim = does not exist x→0 x tan 2 x 4
x→0 |x|
17. Ans (A)
9. Ans (C)
cos x (1 − cos x)
For x ∈ (2,3), [x] = 2; Lt
x→0 x 2 (1 + cos x)2
x 2 x
for⁡ ∈ ( , 1) cos 2 x (2 sin2 ) 1
3 3 = Lt 2 =
x x→0 x 2 8
⇒[ ]=0 x 2 (2 cos 2 )
2
3
18. Ans (C)
[x]3 x3
∴ lim+ ( −[ ] ) Given limit is
x→2 3 3
8 x2 x2
1 8 lim (1 − cos ) (1 − cos )
= (2)3 − (0)3 = x→0 x8 2 4
3 3
1 1 1
10. Ans (B) = 32 ( ) ( ) =
16 64 32
lim[cos x] = [1− ] = 0 19. Ans (B)
x→0
− −)
(∵ As x → 0 , cos x → 1 2x + 23−x − 6
lim
sin[cos x] 0 x→2 √2−x − 21−x
∴ lim⁡ = =0
x→0 1 + [cos x] 1+0 (2x )2 − 6. 2x + 23
= lim
11. Ans (B) x→2 √2x − 2
π [Multiplying Nr and Dr by 2x ]
lim⁡x 2 sin = 0 × a finite number between
x→0 x
lim(2x − 2)(√2x + 2)
-1 and +1=0 x→2

12. Ans (B) = (22 − 2)(2 + 2) = 8


esin x (ex−sin x −1) 1 ex −1
lim = [∵ lim⁡ =1
x→0 2(x−sin x) 2 x→0 x

13. Ans (B)


Apply L − Hospital rule
14. Ans (B)
20. Ans (A)
π
Put x − = θ 1−rn
2 The sum of n terms in G.P is a ( )
π π 1−r
cot( 2 −θ)−cos( 2 −θ)
Given limit lim (if r < 1)
θ→0 θ3

tan θ−sin θ 1
sin θ
−sin θ After applying the limit
= lim = lim cos θ 3
θ→0 θ3 2 θ→0 θ

MATHEMATICS Page | 13A. 18


L LIMITS AND DERIVATIVES

1 2
2 1
1 1 1 n (√1 + 2 + 1)
1 + + +. . . +∞ 1− n (1 + 1)2
s = lim ( 2 4 )= 2 =4 Lt = =4
n→∞ 1 1 1 3 n→∞ 3 1 1
1 + + +. . . +∞ 1 n2 . √1 +
3 9 n6
1−
3
28. Ans (A)
21. Ans (A)
2
Divide by x Lim⁡x 3 cos = 0 × finite number between −
x→0 x

1 1 1 and + 1
x sin ( ) − x sin ( ) − 1
Lim ( x ) = Lim ( x )=0
x→∞ 1 − |x| x→∞ 1 − |x| =0
x 29. Ans (B)
22. Ans (C) sin|x|
π π
As x → 0, |x|
→ 1⁡⁡
x.2 sin(8x) cos(8x)
lim
x→∞ 2 [i. e. , close⁡to⁡1⁡from⁡left⁡side
π π sin|x|
x. sin ( ) ( ) i.e., <1
= lim 4x = 4 |x|
x→∞ 2 2
sin|x|
23. Ans (A) [ ]=0
|x|
Divide with ax and apply limit
30. Ans (C)
24. Ans (B) 12 +22 +32 +.....+n2 Σn2
Given limit = lim = lim
Divide with x10 n→∞ 1+n3 n→∞ 1+n3

1 1 n(n + 1)(2n + 1)
as x → ∞, → 0 = lim
x n→∞ 6 1 + n3
25. Ans (B) 1 1
1 (1 + n) (2 + n)
Given limit = lim
n→∞ 6 1
1 1 1 1 1 1 ( 3 + 1)
n
= lim ( − + − + ⋯ . +
2 n→∞ 1 3 3 5 (2n − 1) 1 2 1
= . 1. =( )
1 1 6 (1) 3
− )=
(2n + 1) 2
26. Ans (C)
1
limn→∞ [12 + (12 + 22 ) + (12 + 22 + 32 ) + ⋯ +
n4

(12 + 22 +…⁡+⁡n2 )
1 n
= limn→∞ ∑ (12 + 22 + 32 + ⋯ . r 2 )
n4 r=1
1 r(r + 1)(2r + 1)
= limn→∞ 4 ∑nr=1
n 6
1 n 2r 3 + 3r 2 + r
= limn→∞ ∑
n4 r=1 6
1 1
limn→∞ 4 [ n2 (n + 1)2 ⁡ +
n 12
1 1
n(n + 1)(2n + 1) + n(n + 1)
12 12
1 n+1 2
= limn→∞ ( ) +
12 n
1 1 1 1 1 1 1 1
⋅ (1 + ) (2 + ) + ⋅ (1 + ) =
12 n n n 12 n2 n 12
27. Ans (D)

MATHEMATICS Page | 13A. 19


STATISTICS

HINTS AND SOLUTIONS

1. Ans (D) which is discarded as n cannot be a fraction.


Mean (arithmetic mean), median and mode are ∴ n − 11 = 0 ⇒ n = 11
three measures of central tendency. A measure of 6. Ans (C)
central tendency gives us a rough idea, where data First, we write all the observation as
points are centred. 10,18,21,35,42.
2. Ans (D) Since, number of observations = 5 (odd)
Sum of 20 observations = 20 × 15.5 = 310 n+1 th
∴ Median = ( ) observation
2
Corrected sum = 310 − 42 + 24 = 292
6
292 = ( ) = 3rd observation = 21
So, corrected Mean = = 14.6 2
20
7. Ans (C)
3. Ans (B)
As given: marks of 10 students out of 15 in the
Mean of four observations = 20
ascending order are 40,50,60,70,70,75,80,80,90,95
⇒ mean of observations = 20 × 4 = 80
Total number of terms = 15 and 5 students who
When c is added in each observation, sum of
failed are below 40 marks,
observations will be 80 + 4c,
n+1
Given thar, the new mean = 22 median = ( ) th term
2

∴ According to the question, 15+1 th


=( ) term= 8th term = 60
2
80 + 4c = 22 × 4 ⇒ 80 + 4c = 88
8. Ans (C)
⇒ 4c = 8 ⇒ c = 2
Sum of 6 numbers = 30 × 6 = 180
4. Ans (D)
Sum of remaining 5 numbers= 29 × 5 = 145
For the second set of observations
∴ Excluded number = 180 − 145 = 35.
N = 150 (even), so median is the A.M. of 75th and
9. Ans (C)
76th observation first set is not sorted so we have
While dividing each entry in a data by a nonzero
to arrange them in increasing or decreasing order
number ′a′, the arithmetic mean of the new data is
5. Ans (A)
divided by ‘a′.
Mean of n observations is
10. Ans (C)
12 + 22 + 32 + ⋯ + n2 n(n + 1)(2n + 1)
= Let x be a set of observations given as
n 6n
x = a1 , a 2 , … … … , a n
From the description of the problem:
a1 +a2 +⋯…+an
(n + 1)(2n + 1) 46n Then x‾ =
n
=
6 11 If now each observation is divided by α, then
⇒ 11 × (2n2 + 3n + 1) = 6 × 46n a1 a2 a
+ +⋯……+ n +10n
α α α x‾
= + 10
⇒ 22n2 + 33n + 11 = 276n n α

⇒ 22n2 + 243n + 11 = 0 x‾ + 10α


=
α
⇒ 22n2 − 242n − n + 11 = 0
11. Ans (B)
⇒ 22n(n − 11) − 1(n − 11) = 0
Given observations are 29,32,48,50, x, x +
⇒ (n − 11)(22n − 1) = 0
2,72,78,84,95.
1
Now, 22n − 1 = 0 ⇒ n =
22 Number of observations = 10
MATHEMATICS Page | 14A. 1
STATISTICS

Median 18. Ans (A)


value of 5th term + value of 6th term 2+9+9+3+6+9+4 42
= Mean (X‾) = = =6
7 7
2
∑|xi − x‾|
x+x+2 MD(X‾) =
= = x+1 n
2
4 + 3 + 3 + 3 + 0 + 3 + 2 18
But Median = 63, is given. = = = 2.57
7 7
So, 63 = x + 1 ⇒ x = 62
19. Ans (D)
12. Ans (C) h n
Median = l + ( − C) and then find the Mean
f 2
Total sum of 13 observations
Deviation
= 14 × 13 = 182
20. Ans (D)
Sum of 14 observation
∑fi xi
= 7 × 12 + 7 × 16 = 84 × 112 = 196 Use x‾ = and Mean deviation from the mean =
∑fi

So, the 7th observation= 196 − 182 = 14 ∑fi |xi −x‾|


∑fi
13. Ans (A)
21. Ans (D)
Let a1 , a2 , a 3 , a 4 and a 5 be five quantities
−1 + 0 + 4
Then a1 + a 2 + a 3 + a 4 + a 5 = 30 (given) Mean (x‾) = =1
3
Also given that a1 + a 2 + a 3 = 12 ∑|xi − x‾|
∴ MD =
Now a 4 + a 5 = 18 n
| − 1 − 1| + |0 − 1| + |4 − 1|
Thus, the average of a 4 and a 5 will be = =2
3
a4 +a5 18
= = 9. 22. Ans (C)
2 2

14. Ans (B) Here, N = Σf = 29 (odd)


Sum of all numbers: 19.3 × 20 = 386 N+1 th
∴ Median M = ( ) observation
2
New mean = 19.3 − 0.5 = 18.8
29+1 th
New sum= 21 × 18.8 = 394.8 =( ) observation = 15th observation
2

Thus, new number added 394.8 − 386 = 8.8 ⇒ M = 30].


15. Ans (C) Then find Mean deviation about median using the
Find mean of the given data using formula and also formula =
Σfi |xi −M|
Σfi
median is obtained by first arranging the data in
23. Ans (A)
ascending or descending order and applying the
Arranging the given data in ascending order, we
rules.
get 2,3,5,7,8,9,10,11,14, Here N = 9, which is odd.
16. Ans (D)
1
∴ Median = (n + 1)th observation
The measure of dispersion is mean deviation, 2
1
standard deviation and quartile deviation. = (9 + 1)th observation = 5th observation, Thus,
2
17. Ans (A) M = 8].
Arithmetic mean x‾ of 4,7,8,9,10,12,13,17 is Then find Mean deviation about median using the
4 + 7 + 8 + 9 + 10 + 12 + 13 + 17 Σfi |xi −M|
x‾ = formula =
8 Σfi

80 24. Ans (C)


= = 10
8 11+1 th
Now N=11, So median = ( ) or 6th
Σ|xi − x‾| =6+3+2+1+0+2+3+7=24 2

∴ Mean deviation about mean observation = 9 ,Thus, M = 9.


∑|xi −x‾| 24
= M.D. (x‾) = = =3
n 8
MATHEMATICS Page | 14A. 2
STATISTICS

Then find Mean deviation about median using the S. D. of a series unaltered if each item is raised
Σfi |xi −M| (reduced) by some scalar quantity. So S. D. is
formula =
Σfi
independent of change of origin.
25. Ans (B)
∑xi
∴ S. D. = 30
i.e. x‾ = and Mean deviation from the mean =
n 29. Ans (D)
∑|xi −x‾|
n variance = 42 = 16
26. Ans (A) Σxi2
∴ − ( mean )2 = 16
Now, Σ(x − 5)2 = 43 18
Σxi2
⇒ Σx 2 − Σ10x + Σ25 = 43 ⇒ = 16 + 72 = 65
18
⇒ Σx 2 − 10{Σ(x − 5 + 5)} + (18 × 25) = 43
⇒ Incorrect Σxi2 = 1170
⇒ Σx 2 − 10Σ(x − 5) − 10Σ5 + 450 = 43
∴ Correct Σxi2 = 1170 − 212 + 122 = 873
2
⇒ Σx − 10(3) − (10 × 18 × 5) + 450 = 43
Hence, correct variance
⇒ Σx 2 − 30 − 900 + 450 = 43
correct Σxi2
2
⇒ Σx = 523 and Σ(x − 5) = 3 = − ( correct mean )2
18
⇒ Σx − Σ5 = 3 873
= − (6.5)2 = 48.5 − 42.25 = 6.25
⇒ Σx = 3 + (5 × 18) = 93 18
∴ Correct S. D. = √ correct varience
Σx2 Σx 2
Now, σ = √ −( )
n n = √6.25 = 2.5

523 93 2 30. Ans (D)


=√ − ( ) = √29.06 − (5.17)2
18 18 Data is 6,7,10,12,13,4,8,12

= √29.06 − 26.73 = √2.33 = 1.53 n=8


6+7+10+12+13+4+8+12
27. Ans (D) ∴ Mean = x‾ =
8

Statement-2 is true 72
= =9
8
1
Statement-1: Now, Variance = ∑xi2 − (x‾)2
n

Sum of n even natural numbers = n(n + 1) 1


= [(6)2 + (7)2 + (10)2 + (12)2 + (13)2 + (4)2
n(n+1) 8
Mean (x‾) = =n+1
n + (8)2 + (12)2 ] − 81
1
Variance = [ ∑(xi )2 ] − (x‾)2 1
n = [722] − 81 = 90.25 − 81 = 9.25
8
1 2
= [2 + 42 + ⋯ . +(2n)2 ] − (n + 1)2 31. Ans (C)
n
1 Given that number of observations
= 22 [12 + 22 + ⋯ . +n2 ] − (n + 1)2
n (n) = 100
4 n(n + 1)(2n + 1)
= − (n + 1)2 Incorrect mean (x‾) = 40
n 6 1
(n + 1)[2(2n + 1) − 3(n + 1)] We know that x‾ = ∑ni=1 xi
n
=
3 i.e. 40 =
1
∑100 100
100 i=1 x i or ∑i=1 x i = 4000
(n + 1)[4n + 2 − 3n − 3]
= i.e. Incorrect sum of observations = 4000
3
(n + 1)(n − 1) n2 − 1 Thus, the correct sum of observations
= =
3 3 = 4000 − 50 + 40 = 3990
∴ Statement 1 is false. 3990
Hence Correct Mean = = 39.9
100
28. Ans (D)
MATHEMATICS Page | 14A. 3
STATISTICS

32. Ans (D)


2x1 +2x2 +⋯.+2xn Σx2 Σx 2
A.M. of 2x1 , 2x2 … … 2xn is But we know that, variance = −( )
n n n
x1 + x2 + ⋯ … + xn 134+x2 16+x 2 49
=( ) = 2x‾ ⇒ −( ) = (given)
n 4 4 4

So statement-2 is false 134 + x 2 (256 + x 2 + 32x) 49


⇒ − =
variance (2xi ) = 22 variance (xi ) = 4σ2 4 16 4
so statement-1 is true. 3x 2 − 32x + 280 49
⇒ =
16 4
33. Ans (A)
⇒ 3x 2 − 32x + 280 = 196
Let x‾ be the mean of x1 , x2 , … , xn , Then the varience
⇒ 3x 2 − 32x + 84 = 0
1
is given by σ12 = ∑n (x )2
− x‾ If ' a ' is added to
n i=1 i ⇒ (x − 6)(3x − 14) = 0
each observation, the new observations will be 14
⇒ x = 6, x =
yi = xi + a … (i) Let the mean of the new 3
14
observations be y‾. then Therefore, the values of x are 6 and .
3
1 n 1 35. Ans (B)
y‾ = ∑ y = ∑n (x + a)
n i=1 i n i=1 i
First n natural numbers are 1,2,3,4, n.
1
= [∑ni=1 xi + ∑ni=1 a]
n xi xi2
1 na
= ∑ni=1 xi + = x‾ + a
n n 1 12
i.e. y‾ = x‾ + a
2 22
Thus, the variance of the new observations
1 n 3 32
σ22 = ∑ (y − y‾)2
n i=1 i
1 n 4 42
= ∑ (x + a − x‾ − a)2
n i=1 i
[Using (i) and (ii)] ⋮ ⋮
1 n
σ22 = ∑ (x − x‾)2 = σ12 n n2
n i=1 i
Thus, the variance of the new observations is same n(n + 1)(2n + 1)
n(n+1)
Total =
as that of the original observations. 2 6
Alternatively, use T.S.R
34. Ans (A) ∑xi
∴ Mean =
n
From given data, we make the following table.
n(n + 1) n + 1
∴ x‾ = =
x 𝐱𝟐 2n 2
∑x2
i Σxi 2
Variance = −( )
2 4 n n

n(n + 1)(2n + 1) n(n + 1) 2


3 9 = −[ ]
6n 2n
(n + 1)(2n + 1) (n + 1)2
11 121 = −
6 4
x x2 (n + 1) 2n + 1 n + 1
= [ − ]
2 3 2
(n + 1) 4n + 2 − 3n − 3
Σx=16+x Σx 2 =134+x 2 = [ ]
2 6

MATHEMATICS Page | 14A. 4


STATISTICS

n+1 n−1 n2 − 1 Variance = σ2 =


1
∑(xi − x‾)2
=( )[ ]= 2n+1
2 6 12
2
II. First 10 positive multiples of 3 are = [n2 + (n − 1)2 + ⋯ … … + (1)2 ]
2n + 1
3,6,9,12,15,18,21,24,27,30. 2 2 n(n + 1)(2n + 1)
= ∑n2 = ⋅
2n + 1 2n + 1 6
xi xi2
n(n + 1)
=
3 9 3
n(n+1)
S. D. = √ Varience = √ .
6 36 3

38. Ans (A)


9 81
If each observation is increased by a constant k,

12 144 then standard deviation is unchanged.


39. Ans (B)
15 225
mean of the squares of the deviations from mean is

18 324 called the variance and is denoted by σ2


40. Ans (B)
21 441 Σx2
Variance = i
− (x‾)2
N
24 576 22 + 42 + ⋯ + 1002 2 + 4 + ⋯ + 100 2
σ2 = −( )
50 50
27 729
4(12 + 22 + 32 + ⋯ . +502 )
= − (51)2
30 900 50
50 × 51 × 101
= 4( ) − (51)2
Total = 165 3465 50 × 6
= 3434 − 2601 ⇒ σ2 = 833
Alternatively, use T.S.R
Σx 165
Mean (x‾) = = = 16.5 41. Ans (A)
n 10
Σx2 Σx 2 3465 165 2 Standard deviation, σ = √74.25 = 8.61.
Variance = −( ) = −( )
n n 10 10
42. Ans (A)
= 346.5 − (16.5)2 = 346.5 − 272.25
Let the other two observations be x and y.
= 74.25
Therefore, the series is 1,2,6, x, y.
36. Ans (B) 1+2+6+x+y
Now, Mean (x‾) = 4.4 =
5
∑d2 ∑d 2
S. D. = √ − ( ) = 3.5
N N or 22 = 9 + x + y
2
4 + 9 + a2 + 121 2 + 3 + a + 11 49 Therefore, x + y = 13
⇒ −( ) =
4 4 4 1
Also, variance = 8.24 = ∑5i=1 (xi − x‾)2
n
⇒ (4a2 + 536) − (a2 + 32a + 256) = 196
1
i.e. 8.24 = [(3.4)2 + (2.4)2 + (1.6)2 + x 2 + y 2 −
⇒ 3a2 − 32a + 84 = 0 5

37. Ans (C) 2 × 4.4(x + y) + 2 × (4.4)2 ]

x‾ = mean =
1+2+3+⋯+(2n+1) or 41.20 = 11.56 + 5.76 + 2.56 + x 2 + y 2 −
(2n+1)
8.8 × 13 + 38.72
∑(2n + 1)
= Therefore x 2 + y 2 = 97
(2n + 1)
(2n + 1)(2n + 2) But from (i), we have x 2 + y 2 + 2xy = 169
= =n+1
2(2n + 1) From (ii) and (iii), we have 2xy = 72 … (iv)

MATHEMATICS Page | 14A. 5


STATISTICS

Subtracting (iv) from (ii), we get ⇒ 3a2 − 32a + 84 = 0


x 2 + y 2 − 2xy = 97 − 72 i.e. (x − y)2 = 25 49. Ans(C)
x x x x
or x − y = ±5 … (v) , , , , x ascending order
5 4 3 2
So, from (i) and (v), we get n = 5 ⇒median = 3rd observation
x = 9, y = 4 when x − 7 = 5 or x = 4, y = 9 when x
Given = 8 ⇒ x = 24
3
x − y = −5
50. Ans(A)
So, the remaining observations are 4 and 9. 3
√x × 16 × 50 = 20
43. Ans (A)
1 10
x × 16 × 50 = 20 × 20 × 20
Variance (σ2 ) = ∑ i=1
(xi − x‾)2
n x = 10
1
= × 330 = 33 51. Ans(C)
10
a+b+c 3
∴ Standard deviation (σ) = √33 = 5.74 =x ⇒ √abc=x
3

44. Ans (D) ⇒a=b=c=x


Standard deviation 52. Ans(B)
2 2 w
‾ = 60, n1 = no. of boys
∑18
j=1 (x j − 8) ∑18
j=1 (x k − 8)
=√ −( ) n2 = no. of girls
n n
n1 + n2 = 150
2
45 9 w1 = 70, ̅̅̅̅
̅̅̅̅ w2 = 55
=√ −( )
18 18 n1 ̅̅̅̅
w1 + n2 ̅̅̅̅
w2
w
‾ =
n1 + n2
45 1 81 9 3
=√ − =√ = = n1 70 + (150 − n1 )55
18 4 36 6 2 60 =
150
45. Ans (C) 60 × 150 = 70n1 + 150 × 55 − 55n1
We have, n = 100, x‾ = 50, σ = 5, σ2 = 25 5 × 150 = 15n1
We know that, n1 = 50 & So, n2 = 100
2
Σxi2 1 53. Ans(B)
σ2 = − ( Σxi )
n n
x1 + x2 + ⋯ … + x9 + 9 = 60
Σxi2
⇒ 25 = − (50)2 x1 + x2 + ⋯ … + x9 = 51
100
x1 + x2 + ⋯ … + x9 + t
⇒ 2500 = Σxi2 − 250000 =7
10
⇒ Σxi2 = 252500
Where t is required number
51 + t = 70 ⇒ t = 19
46. Ans(A)
54. Ans (B)
σ(x) = 6 ⇒ σ(x − 1) = σ(x) = 6
AM of xi = x̄
47. Ans(A) x1 + x2 + x3 +. . . . . . +xn
x̄ =
SD will remain same = variance same n
48. Ans(B) AM of axi + b = ax̄ + b
2 + 3 + a + 11 a AM of ax1 + b
x‾ = = + 4,
4 4 2(x1 + x2 +. . . +xn ) + 3(1 + 1+. . . +1)
=
xi2 49 n
σ2 = ∑ − (x‾) =
n 4 3n
= 2(x̄ ) + = 2x̄ + 3
134 + a2 a 2 49 n
− ( + 4) =
4 4 4 55. Ans (B)

MATHEMATICS Page | 14A. 6


STATISTICS

1 + 3 + 5 + 7+. . . . (2n − 1) 62. Ans (B)


=
n For first set, mean(xi )= 10 and no. of observation
n(n + 1)
∑ 2n − 1 2 2
− n n2 is 12, So xi fi = 120
= = =n
n n n & for second set, mean (yi )= 8 and no. of
56. Ans (A)
observation is 8,So yifi = 12  8 = 96
27+x+31+x+89+x+107+x+156+x
82 = ax̄ +bȳ
5 WKT, = AM
a+b
410 =410 + 5x 120+96 216
So, Mean of combine = = = 10.8
0 = 5x  x = 0 20 20

130+x+126+x+68+x+50+x+1+x 63. Ans (C)


x̄ = = 75
5
Mean = 38
57. Ans (D)
Sum of observations = 38  25= 950
12 + 22 + 32 + ….+ n2 n(n+1)(2n+1)
Mean= = Correct sum of observations = 950 – 2 + 2
n 6n

1 Correct mean =
950
= 38
= (n + 1)(2n + 1) 25
6
64. Ans (D)
58. Ans (B)
Cannot find Mean from only one data out of 8 data.
Sum of 50 observations = 50  36 = 1800
65. Ans (B)
30 and 42 observations are deleted
x 2 3 4 5 6 7
Now, sum of observations
f 4 4 2 3 0 P
= 1800 – 42 – 30= 1728
1728 1728 fx 8 12 8 15 0 7p
New mean = = = 36
50−2 48 ∑ fx 43+7p
Mean = =4=
59. Ans (D) ∑f 13+p

GM of (x1 , x2 , x3 , . . . . . . . . . . xn ) 52 + 4p = 43 + 7p⇒3p = 9⇒p = 3

= n√x1 . x2 . x3 . . . . . . . . xn 66. Ans (D)


1
n Gm = √ab=√x 6 × 6=1
= √21 . 22 . 23 . . . . . 2n x

n 67. Ans (D)


= √21+2+3+...+n
Sum of 10 observations = 10× 6 =60
n(n+1) 1/n n+1
= [2 2 ] = 2 2 Sum of 4 observations =4 × 7.5= 30
ax̄ +bȳ
60. Ans (C) WKT, AM =
a+b
n 60−30 30
Gm = √a. ar. ar 2 . . . . ar n−1 Mean = = =5
10−4 6
n (n−1)n 68. Ans (C)
= √ an . r 2
n
Mean = x̄
√an . r1+2+3+ …….+(n−1) x1 x2 x3 xn
( + + . . . . . . ) + 10 + 10 + 10+. . . ..
n(n−1) 1/n α α α α
= (an )1/n . [(r) 2 ]
1 [x1 + x2 + x3 . . . . . xn ] 10(n) x̄
= + = + 10
n−1 α n n α
= a. r 2
69. Ans (B)
61. Ans (C)
(a + d) + (a + 3d) + (a + 5d) ……
0.nc0 +1.nc1 +2.nc2 +....n.ncn
Mean = a+d+a+3d+a+5d.....a+(2n−1)d
nc0 +nc1 +nc2 +....+ncn A. M =
n
n. 2n−1
= = n. 2n−1−n
2n
n
= n . 2–1=
2

MATHEMATICS Page | 14A. 7


STATISTICS

(a + a + a+. . . . . . +a)) 75. Ans (C)


=
n n2 (n + 1)2
d + 3d + 5d+. . . . (2n − 1)d ∑ n3 =
+ 4
n 102 (10+1)2 100(121)
Mean = =
na d[1 + 3 + 5+. . . . (2n − 1)] 4.10 4.10
= +
n n 605
= = 302.5
n2 2
= a + d ( ) = a + nd
n
76. Ans (A)
70. Ans (C) 100×2000−30×4000
Remaining mean =
x1 + x2 + x3 . . . . +xn 70
x̄ = 2,00,000 + 20000 80000
n
= = = 1143
x1 +2+x2 +2+x3 +2....xn +2 70 70
Mean =
n
77. Ans (A)
x1 +x2 +...xn 2n
= + = x̄ + 2
n n

71. Ans (C)


n(n + 1)(2n + 1)
= 11
6n
2n2 + 3n + 1 = 66 ∑ xifi 525
Mean = = = 15
N 35
2n2 + 3n – 65 = 0
78. Ans (B)
2n2 + 13n – 10n – 65 = 0
n (2n + 13) – 5 (2n + 13) = 0
−13
n = 5 or n =
2

72. Ans (B)


a+(a+d)+(a+2d)+(a+3d)+.....a+(n−1)d ∑ xi 535
= Mean = = = 21.4
n N 25

na [d + 2d + 3d+. . . . +(n − 1)d] 79. Ans (C)


= +
n n 5
G. m = √12 × 18 × 25 × 60 × 75
(n − 1)(n) 5 5
=a+d = √25 × 35 × 55 = √(2 × 3 × 5)5 = 30
2n
(n − 1) 80. Ans (B)
=a+ d
2 9
G. M = √21 × 22 × 23 . . . . .× 29
73. Ans (C) 9
= √21+2+3+...+9
G.M of 2,8,16,128,512
9 1
5 (9+1)
= √29 2 = [29 ] = 25 = 32
(5) 9
= √2 × 8 × 16 × 128 × 512
5
= √21 × 23 × 24 × 26 × 29 81. Ans (C)
5 a+b
= √223 = 34&√ab = 30
2
23/5
=2
WKT,a − b = √(a + b)2 − 4(ab)
74. Ans (B)
Given a + b = 68 & ab = 90
x1 +1+x2 +3+x3 +5+.....xn +(2n−1)
n So a-b= √(68)2 − 3600
x1 + x2 + x3 +. . . . . xn
= = √4624 − 3600 = √1024=32
n
1 + 3 + 5 + 7+. . . . (2n − 1) Add a+b=68 & a – b = 32
+
n We get 2a = 100, so a=50 & b =18
2
n 82. Ans (A)
= x̄ + = x̄ + n
n

MATHEMATICS Page | 14A. 8


STATISTICS

Use TSR (i.e., For each data if a constant is added Corrected ∑ x 2 = 170 − 20 + 30 = 180
or subtracted then variance remains same.) 3330 180 2
 correct variance = −( ) = 78
15 15
83. Ans (C)
89. Ans (C)
If terms are in AP; 101
( )[1+1+100d]
2
n2 −1 Now, x̄ = = 1 + 50d
101
S.D=√ × (Common difference)
12 100
1
31, 32, 33…..47 are in A.P with (d) = 1 M. D = {∑|(1 + rd) − (1 + 50d)|
101
r=0
n2 − 1 (17)2 − 1 100
S. D = √ ×d=√ ×1 1
12 12 = ∑|r − 50| d
101
r=0
289−1
=√ = √24 = 2√6 2d 50 × 51 50 × 51
12 = . = d
101 2 101
Where, tn=47a + (n – 1) d
By data, M. D = 255
=31+ (n – 1) (1) = 47
50 × 51
=31 + n – 1 = 47 ∴ 255 = d
101
n = 47 – 30⇒n = 17 101 × 255
⇒d= = 10.1
84. Ans (C) 50 × 51
90. Ans (C)
1
S. D = √ (x1 − x̄ )2 σ If terms are in AP;
10
n2 −1
S.D=√ × (common difference)
12
1
S. D = √ (x1 − x̄ )2 = σ 2, 4, 6….2n, are in A.P with common difference, d
10
=2
85. Ans (A)
n2 −1 n2 −1
We know that when each observation is increased S. D = ×4=
12 3

by a constant, the standard deviation is 91. Ans (D)


unchanged Use TSR (i.e., For each data if a constant is added or
 required standard deviation =  subtracted then SD remains same.)
86. Ans (D) 92. Ans (A)
5+5++ …..+13
Mean =
8
=6 Let x1 , x2 , … , xn be n values of X. Then,
n
1 12 2
S. D = √ ∑(xi − x̄ )2 σ = ∑(xi − X) … (i)
n n
i=1

The variable a X + b takes values


1
= √ (1 + 1 + 1 + 1 + 1 + 1 + 1 + 49)
8 a x1 + b, a x2 + b, … , a xn + b with mean a X +
56 b
=√ = √7
8
∴ Var(aX + b)
87. Ans (C) n
1 2
When each observation is multiplied by 2, then = ∑{(axi + b) − (aX + b)}
n
the variance is also multiplied by 2. i=1
1 2
 required variance = 23.33  2 = 46.66 = a2 n ∑ni=1(xi − X)
88. Ans (B)
Corrected ∑ x 2 = 2830 − 202 + 302 = 3330

MATHEMATICS Page | 14A. 9


STATISTICS

n Let Y be the new mean when x2 is replaced by


1 2
⇒ (S. D. of a X + b) = √a2 ∑(xi − X) λ. Then,
n
i=1
x1 + λ + x3 + ⋯ + xn−1 + xn
Y=
= |a|σ n
93. Ans (C)
(x1 + x2 + ⋯ + xn ) − x2 + λ
⇒Y=
n
Total weight of 9 items = 15 × 9 = 135
n X − x2 + λ
And total weight of 10 items = 16 × 10 = 160 ⇒Y=
n
∴ weight of 10th item = 160 − 135 = 25 100. Ans (D)
94. Ans (D) 1
Mean = [(x1 + x2 +. . . +x10 ) + (4 +
10
−1 + 0 + 4
x̅ = =1 8+. . . +40)]
3
Σ|xi − x̅ 1
∴ MD = = (x + x2 + ⋯ + x10 )
n 10 1
|−1 − 1| + |0 − 1| + |4 − 1| 4
= =2 + (1 + 2 + ⋯ + 10)
3 10
95. Ans (C) 4 × 10 × 11
= 20 + = 42
In ascending order 6,8,9,10,11,12,14
10 × 2
101. Ans (B)
n+1 th
Median=( ) value 99 33
2 2
Given, σ10 = =
12 4
i.e., 4th value=10
96. Ans (C) √33
⇒ σ10 =
2
n2 −1 102 −1
σ=√ =√ =√33=5.74 3√33
3 3 SD of required series = 3σ10 = 2
97. Ans (B) 102. Ans (B)
The required AM is Given series in A.P and no. of terms =2n+1
1 + 2 + 22 + 23 +. . . +2n Now, x
̅=
a+a+2nd
= a + nd
̅=
X 2
n+1
∑(x− x̅)2
1(2n+1 − 1) 1 2n+1 − 1 Thus variance,σ2 = 2n+1
= . =
(2 − 1) (n + 1) n+1
2d2 (12 + 22 + … … + n2 )
98. Ans (C) ⇒σ=√
2n + 1
∑ xi wi
Req. Weighted Mean = ∑ wi
n(n+1)(2n+1)
1.1+2.2+3.3+ ……..+n.n ∑ n2 2d2 ×
=
1+2+3+ ……+n
=∑
n
⇒ σ =√ 2n+1
6

n(n + 1)(2n + 1)
( ) 2n + 1
6 n(n+1)
= = ⇒ σ=√ d
n(n + 1) 3 3
( )
2
103. Ans (C)
99. Ans (D) n+1
Mean of 1st n- natural no. is
2
We have,
And in a moderately symmetric distribution,
x1 + x2 + ⋯ + xn−1 + xn
X= QD=MD=SD
n
104. Ans (A)
⇒ n X = x1 + x2 + ⋯ + xn−1 + xn

MATHEMATICS Page | 14A. 10


STATISTICS

Let the mean of the remaining 4 observations Given series is 148, 146, 144, 142,... whose
be X1 . Then, first term and common difference is

a + 4 X1 nM−a a = 148, d = (146 − 148) = −2


M= ⇒ X1 = n
(n − 4) + 4 4 Sn = 2 [2a + (n + 1)d] = 125 (given)
105. Ans (B) n
⇒ 125n = [2 × 148 + (n − 1) × (−2)]
n1 x1 + n2 x2 2
X=
n1 + n2 ⇒ n2 − 24n = 0 ⇒ n(n − 24) = 0
n1 x1 + 10n1 y ⇒ n = 24 (n ≠ 0)
=
n1 + 10n2
110. Ans (A)
x + 10y
= Median of 33, 28, 20, 25, 34, x is 29.
11
Now, arranging in ascending order
106. Ans (B)
20, 25, 28, x, 33, 34
Let n be the required number of observations.
Here n = 6
The sum of deviations of n observations
∴ Median =12[3rd term + 4th term]
about 25 is 25.
29 = 12[28+x]
⇒ ∑(x−25)=25
58 = 28 + x ⇒ x = 58 - 28 = 30
⇒ ∑x−∑25=25 ⇒ ∑x−25n=25 ....(1) ∴ Possible value of x = 30
The sum of deviations of n observations 111. Ans (C)
about 35 is -25. The new observations are obtained by adding
⇒ ∑(x−35)=−25 ⇒ ∑x−∑35=−25 20 to each. Hence, σ does not change.
⇒ ∑x−35n=−25 ...(2) 112. Ans (C)
After solving above equations (1) &(2) we get Let x1 , x2 , … , xn be n values of variable X.
n=5. Then,
From equation (1), 1
X = ∑ xi
⇒ ∑x−25n=25 ⇒ ∑x−25(5)=25 n
Let y1 = x1 + 1, y2 = x2 + 2, y3 = x3 +
⇒ ∑x−125=25 ⇒ ∑x=150
∑x 3, … , yn = xn + n. Then, the mean of the new
Mean (X ) = n =150/5=30
series is given by
107. Ans (B)
1
n=7 X′ = ∑ yi
n
∑x
Mean (X ) = n =7 1
⇒ X′ = ∑(xi + i)
n
∑|x−X| 4+3+3+3+0+3+2 18 i
M.D= = = = 2.57
n 7 7 1 1
108. Ans (D)
⇒ X′ = ∑ xi + (1 + 2 + 3 + ⋯ + n)
n n
i
∑xi2
σ = 2
− (x‾)2 1 n(n + 1) n+1
n ⇒ X′ = X + ∙ =X+
n 2 2
18000 960
= −
60 60
=300-256=44
109. Ans (B) 113. Ans (C)

MATHEMATICS Page | 14A. 11


STATISTICS

Assuming original nine elements are Consider assumed mean A=25 (Somewhere near

x1,x2,x3,.........x9 and new element is x mid-point in the interval).

Sum of original nine elements is


∑9i=1 xi =9×15=135
Given new mean after addition of x is 16.
⇒10i=∑9i=1(xi + x) = 16
135+x
⇒ = 16⇒x=160−135=25
10
Σfi d2i Σfi di 2
114. Ans (B) ∴ σ2 = −( )
Σfi Σfi
Arranging the terms in increasing order
900 −30 2
= −( ) = 90 − 9
10 10
⇒ σ2 = 81 ⇒ σ = 9
117. Ans (C)
Let the first natural number be x
According to the question,
x+x+1+x+2+x+3+x+4+x+5+x
+6 + x + 7 + x + 8 + x + 9 + x + 10 = 2761
⇒ 11x + 55 = 2761
∵ N = 22 2761 − 55
⇒ x= = 246
N+1 11
∵ Median number = = 11.5
2 ∴ Middle number = x + 5 = 246 + 5 = 251
Which comes under the cumulative frequency 118. Ans (B)
corresponding value of x will be the median Let the n-numbers be x1 , x2 , … , xn . Then,
ie, Median=10 n
1
115. Ans (D) X = ∑ xi
n
i=1
2n+1 2n+1 2n+1
C0 + C1 + C2 +. . . + 2n+1 C2n
x1 + x2 + ⋯ + xn−1 + xn
2n+1 ⇒X=
+ C2n+1 = 22n+1 n
Now, 2n+1
C0 = 2n+1
C2n+1 , k + xn
⇒X= [∵ x1 + x2 + ⋯ xn−1 = k]
2n+1 2n+1 2n+1 2n+1 n
C1 = C2n . . . Cr = C2n−r+1
⇒ xn = n X − k
So, sum of first (n + 1) terms= sum of last
119. Ans (C)
(n + 1) terms
2n+1 2n+1 2n+1 Total of corrected observations
⇒ C0 + C1 + C2 +. . . + 2n+1 Cn
= 4500 − (91 + 13) + (19 + 31)
= 22n
2n+1 2n+1
= 4446
C0 + C1 + 2n+1 C2 + ⋯ + 2n+1
Cn
⇒ 4446
n+1 ∴ Mean = = 44.46
100
22n
= 120. Ans (C)
(n + 1)
n
116. Ans (C) ∑nr=0 Cr 2n
X= =
n+1 n+1
MATHEMATICS Page | 14A. 12
STATISTICS

(∵ no. of terms = n + 1) Let us write in ascending order


121. Ans(B) 20,33,39,40,50,53,59,65,69.

Observations are given by 3, 10, 10, 4, 7,10 Here n = 9


9+1
and 5 ∴ Median = th term = 5 th term i.e. 50
2
3+10+10+4+7+10+5 49
∴ x‾ = = =7 ∴ Median = 50
7 7
Now

xi di = |xi − Med|

20 30

33 17

39 11

40 10

∑ di 18
MD = = = 2.57 50 0
n 7
122. Ans(B) 53 3
1
MD = ∑ni=1 |xi − x‾| 59 9
n
123. Ans(A)
65 15
The lines of 5 bulbs are given by
1357,1090,1666,1494,1623 69 19
1357+1090+1666+1494+1623
∴ MEAN =
5 Total ∑di = 114
7230
⇒ x‾ = = 1446
5

xi di = |xi − x‾| n = 9 and ∑ di = 114

∑ di 114
1357 89 ∴ MD = = = 12.67
n 9

1090 356 125. Ans(A)


Given data are 6,5,9,13,12,8 and 10 ∴ n = 7
1666 220

1494 48

1623 177

Total ∑di = 890

∑ di 890
∴ MD = = = 178
n 5
124. Ans(C)
Marks obtained are 50, 69, 20, 33, 53, 39, 40, 65
and 59

MATHEMATICS Page | 14A. 13


STATISTICS

2 2 ∑ (x2 2
∑x ∑x i +K +2xi K)
=√ i − ( i ) =√
N
− (m + K)2
n n

619 63 2 ∑ x2 ∑ K2 2 K ∑ xi
=√ − ( ) =√
N
i
+
N
+
N
− m2 − K 2 − 2m K
7 7

619 ∑ x2
=√ − (9)2 =√ i
+ K 2 + 2 Km − m2 − K 2 − 2m K
7 N

619−567 52 ∑ x2 ∑ xi
=√ − 81 = √ =√ i
− m2 [∵ = m]=S
7 7 N N

126. Ans(C) 129. Ans(C)


The formula for ∑xi ∑x2 ∑xi 2
Here m = ,S = √ i
−( )
N 5 5
∑(xi −x‾)2
S.D = σ = √
n 2
K 2 ∑ xi2 K ∑ xi
127. Ans(C) ∴ SD = √ −( )
5 5
∑xi
Here x‾ = 2
n
K 2 ∑ xi2 ∑ xi
∑ xi =√ − K2 ( )
50 = ⇒ ∑ xi = 5000 5 5
100
2
∑ xi2 ∑ xi
2 ∑ xi2 ∑ xi
∴ SD = √ −( ) = K√ −( )
n n 5 5

=K⋅S
∑ xi2 5000 2
5=√ −( ) 130. Ans(A)
100 100
Given that wi = xi + k, x‾i = 48,
∑ xi2
⇒ 25 = − 2500 SD(xi ) = 12,wi = 55 and SD (wi ) = 15
100
then w
‾ i = x‾i + k
∑ xi2
⇒ = 2500 + 25 ⇒ 55 = 48 + k
100
∑ xi2 ‾ i = mean of wi 's and x‾i is the mean of xi′ 's
w
⇒ = 2525
100 SD of wi = SD of xi
∴ ∑ xi2 = 2525 × 100 = 252500 15= l × 12
15
128. Ans(A) ⇒ l= = 1.25
12
Given observation are a, b, c, d and e
from eq. (i) and (ii) we have
a+b+c+d+e
∴ Mean = m = k=w
‾ i − x‾ i = 55 − 1.25 × 48
5
= 55 − 60 = −5
∴ ∑ xi = 5 m
131. Ans(D)
Now mean of the data,
n2 −1
We know that SD of first n natural numbers √
a + K, b + K, c + K, d + K and e + K is 12

a+K+b+K+c+K+d+K+e+K Here n = 10
=
5
(a + b + c + d + e) + 5 K (10)2 − 1
= ∴ SD = √
5 12
5m + 5 K
= = m+K 99
5 =√ = √8.25 = 2.87
12
∑ (xi +K)2 ∑ xi +K 2
∴ SD = √ −[ ] 132. Ans(D)
N N

Given numbers are 1,2,3,4,5,6,7,8,9,10

MATHEMATICS Page | 14A. 14


STATISTICS

Numbers obtained when 1 is added to the above 18000 960 2


= −( ) = 300 − 256 = 44
numbers is 2,3,4,5,6,7,8,9,10 and 11 60 60
∑ xi =2+3+4+5+6+7+8+9+10+11 135. Ans(A)

10 Here, we have CV1 = 50, CV2 = 60


= [2 × 2 + (10 − 1) ⋅ 1]
2 x‾1 = 30 and x‾ 2 = 25
= 5[4 + 9] = 5 × 13 = 65 σ1 σ1
∴ CV1 = × 100 ⇒ 50 = × 100
x‾1 30
Now ∑ xi2 = 22 + 32 + 42 + ⋯ + 112
50 × 30
⇒ σ1 = = 15
= (12 + 22 + 32 + 42 + ⋯ + 112 ) − (1)2 100
σ2 σ2
n(n + 1)(2n + 1) 11 × 12 × 23 and CV2 = × 100 ⇒ 60 = × 100
= −1= −1 x‾2 25
6 6
60 × 25
= 22 × 23 − 1 = 506 − 1 = 505 ⇒ σ2 = = 15
100
2
∑ xi2 ∑ xi ∴ Difference σ1 − σ2 = 15 − 15 = 0
∴ Variance (σ2 ) = −( )
N N 136. Ans(A)
2
505 65 Given that σC = 5
= −( )
10 10 5
2
We know that C = (F − 32)
9
505 65
= − ( ) = 50.5 − (6.5)2 9C
10 10 ⇒F= + 32
5
= 50.5 − 42.25 = 8.25
9 9
133. Ans(A) ∴ σF = σC = × 5 = 9
5 5
First 10 positive integers are 1,2,3,4,5,6,7,8,9,10 ∴ σ2F = (9)2 = 81
on multiplying each number by −1, we get 137. Ans(C)
−1, −2, −3, −4, −5, −6, −7, −8, −9, −10 σ = 21, x‾ = ?
on adding 1 to each of the number, we σ
C. V = × 100
get0, −1, −2, −3, −4, −5, −6, −7, −8, −9 x‾
21
∴ ∑ xi = 0 − 1 − 2 − 3 − 4 − 5 − 6 − 7 − 8 60 = × 100
x
− 9 = −45 x‾ = 35
138. Ans(D)
and ∑ xi2 = 02 + (−1)2 + (−2)2 + (−3)2
X = {3,10,10,4,7,10,5}
+ (−4)2 + ⋯ + (−9)2 3 + 10 + 10 + 4 + 7 + 10 + 5
9 × 10 × 19 X‾ = =7
7
= = 285
6 xi − X‾ = {−4,3,3, −3,0, +3, −2}
n(n + 1)(2n + 1)
[∵ ∑ n2 = ] ∑ |xi − x‾| 18
6 M ⋅ D(X‾) = = = 2.57
n 7
∑ x2 ∑ xi 2 285 −45 2
SD= √ i
−( ) =√ −( ) 139. Ans(D)
N N 10 10
x‾ = 50, σ = 4, n = 100
285 2025 2850 − 2025 ∑ xi2
=√ − −√ − (x‾)2 = 16
10 100 100 n
= √8.25 ∑ xi2 = (16 + 2500)100 = 251600
∴ Variance = (SD)2 = (√8.25)2 = 8.25 140. Ans(D)
134. Ans(D) ‾ ) = 6+7+8+9+10 = 8
Mean (x
2 5
∑x2 ∑xi
We know that variance (σ2 ) = i
−( ) Now, ∑ xi = 6 + 72 + 82 + 92 + 102 = 330
2 2
N N

MATHEMATICS Page | 14A. 15


STATISTICS

∑ xi 2 52
∴ Standard deviation, σ = √ − (x‾)2 S. D = √
7
n

2. Ans(C)
330
=√ − (8)2 = √2 x̅1 = 52, ̅̅̅
x2 = 42
5
x‾ = 50
141. Ans(C)
n1 x‾1 + n2 ̅̅̅
x2
X = {−1,0,1, K} x‾ =
n1 + n2
k
X‾ = n1 52 + (100 − n1 )42
4 50 =
100
σ = √5 5000 = 52n1 + 4200 − 42n1
2
∑ xi2 ∑ xi 800 = 10n1
⇒ −( ) =5
n n
n1 = 80
2 2
2+k k
− =5 3. Ans(A)
4 16
Average of three subjects
8 + 4k 2 − k 2
=5 x1 + x2 + x3 75 + 80 + 85
16 = = 80
3 3
8 + 3k 2 = 80
x1 + x2 + x3 = 3 × 80
3k 2 = 72
One more subject x4
3k 2 = 72
will be added
k = √24
x1 + x2 + x3 + x4
Avg =
= 2√6 4
142. Ans (B) 3 × 80 + x4
=
4
n2 −1
S.D. = √ (n = 17) x4
12 = 60 + ( ) ⩾ 60%
4
172 − 1 4. Ans(B)
=√ = 2√6
12 Gives series 1,2, 22 , 23 , 24 , … … 2n
143. Ans (D) 1+2+22 +⋯…+2n
A.M of the series =
n+1
∑xi2
2525 = 2n+1 − 1
100 =
n+1
⇒ ∑xi2 = 252500
5. Ans(B)
144. Ans (D) 1(12 )+2(22 )+⋯+n(n2 )
mean (x‾) =
145. Ans (C) 12 +22 +⋯+n2

n (n + 1)2
2
3 n(n + 1)
= =
4n(n + 1)(2n + 1) 2 (2n + 1)
6
MOCK TEST SOLUTIONS 6. Ans(C)
1. Ans(A) x̅1 = 52, ̅̅̅
x2 = 42
X = {5,6,8,9,10,12,13} x‾ = 50
X ′ = X − 9 = {−4, −3, −1,0,1,3,4} n1 x‾1 + n2 ̅̅̅
x2
x‾ =
n1 + n2
V(X ′ ) = V(X − 9)
n1 52 + (100 − n1 )42
∑(xi′ )2
′)
∑xi 2 50 =
V(X = −( ) 100
n n
5000 = 52n1 + 4200 − 42n1
2(26) 52
= −0 = 800 = 10n1
7 7
MATHEMATICS Page | 14A. 16
STATISTICS

n1 = 80 The mean of new set increased by λ


7. Ans(B) 13. Ans(A)
∑xi fi x̅1 = 70, n1 = 100
mean =
∑fi
x2 = 20, n2 = 100
̅̅̅
∑nr=0 r n Cr
= x3 =? n3 = 100
̅̅̅
∑n n C r
∑nr=0 n ⋅ ( n−1 Cr−1 ) x‾ = 45
=
2n n1 x̅1 + n2 ̅̅̅
x2 + n3 ̅̅̅
x3
x‾ =
n⋅2 n−1
n 300
= = 70 + 20 + ̅̅̅ x3
2n 2 45 =
8. Ans(C) 3

∑xifi 135 − 90 = ̅̅̅


x3 ⇒ ̅̅̅
x3 = 45
x‾ =
∑fi 14. Ans(C)
12 + 22 + ⋯ . . +n2 7 5 1 1
= α − , α − 3, α − , α − 2, α − , α + , α + 4, α
1 + 2 + ⋯…+ n 2 2 2 2
1+2+1)(n+n
2n+1
+5
1n+n
= n(n+1) =
6
2
3 WKT, Median
9. Ans(B) n th n th
= Avg. of ( ) & ( + 1) observation
2 2
∑fi =
th th
= Avg. of 4 &5 observation
qn + n c1 qn−1 + n c2 qn−2 p2 + ⋯ … + pn
1
= (q + p)n = 1 α−2+α−
= 2
∑ xi fi 2
mean x‾ = 5
∑ fi 2α −
= 2=α−5
= 0. qn + 1. n c1 qn−1 p + 2. n c2 qn−2 p2 + ⋯ … + n 2 4
⋅ pn 15. Ans(A)

= np[qn−1 + n−1
C1 , qn−2 p + ⋯ … + pn−1 ] The change in the maximum and minimum values

= np[q + p]n−1 =np in the series does not affect the median of that

10. Ans(C) series new median = 52

Correct mean is 50 × 100 − 100 + 110


Correct mean =
x1 + x2 + ⋯ … + x100 100
100 = 50 + 0.1 = 50.1

=
49×100−40−20−50+60+70+80 16. Ans(A)
100
Data : 2,3,4,6,7,7,8
50 × 100
= = 50 Median = 6
100
11. Ans(C) Data : 2, 3,4, 6, 9, 9, 9, 6
x = {x1 , x2 , … … xn } Mode = 9
6+9
μ(x) = μ(xi) = x‾, i = 1,2 … … n Average = = 14.5
2
x1 = {x1 + 1, x2 + 2, … … , xn + n} 17. Ans(C)
μ(x1 ) = μ(xi + i), When i = 1,2 … . . n
The G.M of x1 , x2 , … … xn is n√x1 ⋅ x2 ⋅ x3 … … xn
1 + 2 + ⋯…+ n 1
= μ(xi) +
n = (2 ⋅ 22 ⋅ 23 … … 2n )n
n+1 1
= x‾ + = [21+2+3+⋯……+n ]n
2
1
12. Ans(B) n(n+1) n n+1
= [2 2 ] =2 2

MATHEMATICS Page | 14A. 17


STATISTICS

18. Ans(A) = 44.46


n n n n
C0 + C1 + C2 + ⋯ + Cn 24. Ans(B)
n+1 1
2n
(x1 , x2 , x3 … x9 ⋅ (12.9))10 = 16.2
=
n+1 x1 x2 x3 … … x9 (12.9) = (16.2)10
19. Ans(B)
x1 x2 … … x9 ⋅ (21.9)
x1 + x2 + ⋯ … + x50 = 38 × 50
(16.2)10 × (21.9)
x1 + x2 + ⋯ … + x48 =
12.9
48
Correct mean is
38×50−55−45
= =37.5 1
48
[X1 ⋅ x2 … … x9 ⋅ (21.9)]10
20. Ans(D) 1
(16.2)10 × 21.9 10
Let {x1 , x2 … … x10 } =[ ]
12.9
Group of items
25. Ans(C)
x1 + x2 + ⋯ … + x10
=6 ∑(xi − m)
10
x1 + x2 + x3 + x4
and = 7.5 ∑ xi − nm = 0
4 =nm
x1 + x2 + x3 + x4 = 30 From (1)
26. Ans(C)
x1 + x2 + x3 + x4 + x5 + x6 + ⋯ … + x10 x = {−1,0,4}
= 60 −1 + 0 + 4
x‾ = =1
x5 + x6 + ⋯ … + x10 = 60 − 30 3
x5 + x6 + ⋯ … + x10 30 ∑ |xi − x‾|
= =5 M. D(x‾) =
6 6 n
21. Ans(A) 2+1+3
= =2
n1 x̅1 + n2 ̅̅̅
x2 3
x‾ = 27. Ans(D)
n1 + n2
49.96 × 100 + 52.32 × 200 x = {1,3,5}
=
300 median x‾ = 3
49.96+104.60 154.60
= = = 51.53 ∑ |xi − x‾|
3 3 M. D(x‾) =
n
22. Ans(B)
2+0+2 4
{x1 , x2 , … … xn } = =
3 3
Let x‾ is mean of x1 , x2 , x3 , … … xn 28. Ans(B)
each observation increased by ' 5 ' x = {39,40,40,41,41,42,42,43,43,44,44,45}
New observations are median m = 42
x1 + 5, x2 + 5, … … xn + 5 ∑ |xi − m|
M. D(m) =
New mean X‾ n
3+2+2+1+1+0+0+1+1+2+2+3
x1 + x2 + ⋯ … … + xn + n(5) =
= 12
n 18
= x‾ + 5 = = 1.5
12
23. Ans(B) 29. Ans(D)
correct mean is X = {a, a + d, a + 2d, … … , a + 2nd}
x1 + x2 + ⋯ … … + x100 a + a + d + ⋯ … + a + 2nd
100 mean X‾ =
2n + 1
45 × 100 + 19 + 31 − 91 − 13
=
100
MATHEMATICS Page | 14A. 18
STATISTICS

2n(2n + 1) mean
(2n + 1)a + d
= 2 = a + nd
2n + 1 3 + 5 + 11 + 13 + 17 + 23 + 29
X‾ =
∑ |xi − x‾| 8
M. D (X‾) = = 15
2n + 1
∑|x −x‾|
2[nd + (n − 1)d + ⋯ … + d] M.D (X‾) = i
= 8
2n + 1
12 + 10 + 4 + 2 + 2 + 4 + 8 + 14
2n(n + 1) =
= d 8
2(2n + 1)
56
30. Ans(D) = =7
8
X = {3,5,11,13,17,19,23,29}

MATHEMATICS Page | 14A. 19

You might also like